You are on page 1of 472

sftE3

^,*r4g*

. Revision Exercises
- Mid-Year/Final Specimen Papers
t
. Answers with Contplete Worke{
yTspi (5

. S'lid-Yexrr$imal Speeimen Fap*rs


I Mensuration of Pyramids, Cones and Spheres... . " 1

Tutorial 8.................. . .... . .. .... . 14


Thinking Skills Corner......... . ...... . .... ..... ... 41

9 Graphs of Linear Equations in Two Unknowns . 44


Tutorial 9.................. . .. . 55
Thinking Skills Corner.... . . ..... . .. .. " "" " " 72

Revision Exercise 4 73

10 Graphs of Ouadratic Equations. .. . .. . 84


Tutorial 10......... 97
Thinking SkillsCorner...... . .. .. . .. 109

l1 Set Language and Notaiion.. .. .. .. 111


124
154

Revision Exercise 5 156

12 Statistics 168
194

13 Probabi1ity................... 240
255
Thinking Skills Corner.............. . .. . 275

277
Revision Exercise 6

Final Examination Specimen Paper 1 ..... . 296


Final Examination Specimen Paper 2 .

Answers with Complete Worked Solutions. . .. .. ' 321


Mensuration of Pyramids,
Gones and Spheres

/EF Pvramids
ptramia is a solid with a polygon-shaped base and the remaining triangular
I
faces (lateral faces) meeting at a vertex. A pynmid is named accordinpito
the ;hape

lriar]gle-based square-based rectangle-based hexagon based


pyramid pyramlcl pyranid pyramid
(also called a
tetrahedfon)

2. In a dght pyramid, the vertex is directly above the centre of tle base. Otherwise. lr
is called an oblique Dvramid.

right pyramid oblique pyramids

will be used
In a pyra.mid.
. rhe Frlrendicular height of height is the peeendicular distance from the
vertex to the base of the pyramid.
. the slanl height is the height of each of the triangular faces.
. fte slant edge is the edge where the ftiangular faces meet each other.

4. Volume of p)'ramid =; Base area \ Heighl


Total surface area of pyramid = Total area of all its faces
= Base area + Area of all lateral faces

(t> WORKED EXAMPLE 1:


The diagram shows a pyramid with a horizontal rectan-
gular base ABCD. yN is perpendicular to ARCD. Given
that yN= 12 cm,.4-B = 8 cm and BC = 5 cm, calculaie the
volume of the pyramid.

SOLUTION:

Volume of pyramid = x Base area x Height


I
]r1s,.s;,.rz
160 cml

:i
(D WORKED E){AMPLE 2:
The diagram shows a tetrahedron VABC.
Given that,4-8 = 15 m, CD = 10 m and the volume of the
letraledron is 225 m3, find rhe heighr of the pymmid.

soLuTtoN:
Let the height of the p)Tamid be h m.
Volume of pyramid = 225 m3 (civen)
l X Base area X Height = 225
3

5
x[],.rs'ro) ,.r
225x3x2
h
15x10

.. The height of the pyramid is 9 m.

1D WORKED EXAMPLE 3:
The diagram shows a pyramid with a square base of sides l0
and height 12 cm.
(a) Find the slant height of the pFamid.
(b) Draw a net of the pyramid.
(c) Find the total surface area of the p'.ramid.

soLufloN:
(a) AB=10+2=5cm
U.ing Pyrhagora\' theorem on Ay,4-8.
VB'=12'+5'
= 169
VB = ,\a
=13cm
.. The slant hei8hr of the pyramid ir ll cm.

cbaprer 8: Mensuradon oflymd! CltE od 5.6-.: :


(c) Area of sqlare base
=t0x l0
= 100 cnf

Area of each triangular face


l
=;XBasexHeight
l
=;,<10,<13
= 65 cm'?

Tolal sudace area of pyramid


= Base area + 4 X Area of triangular face
=100+4x65
= 360 cm'z

@ cott".
1. A cone is a solid that has a circular bas€ and a vertex, In other words. a cone is a
Dvramid wiih a circular base.

2. In a right circular cone, the vertex is directly above the cente ofthe circular base.
Otherwise. it is called an oblioue circular cone.

&
right-circular cone oblique-circular cone

|[;}.,----,--*
3. ln a cone,
. the perpendicular height or height is the peq)endiculat distanc€ ftom tbe
vertex to the base of the cone.
. the slant height is the distance from the vertex to the circumference of the
cone.
. the radius of the cone is the mdius of the base.

heiCht

I
4. Volume of cone = : \ Base area x Heisht

t-
= atrth
Curved surface area of cone = .rnl

Total surface area of cone = Curved surface area + Base area


=nrl+nf

Chaprer 8 Vers@uon oI hEmJ& r^.s rd Secs


.-
.D WORKED EXAMPLE 1:
A dght circular cone has a base ndius of 8 cm and a height of 15 cm.

Flnd
(a) lhe slanl heiSht of lhe cone
(b) the volume of the cone,
(c) the total surface area of the cone.
lTake n = 3.142.1

SOLUTION:
(a) Irt the slant height of the cone be I cm.
Ushg Pythagoras' theorem,
l'? = 15']+ 8'

I = "rr.B9

.. The slant height of the cone is 17 cm

t,_
(b) volume ot cone = -,?',
t-
=rx3.142x8',x15
= 1005.44 cm3

(c) Total surface area of cone

=3.142x8x17 + 3.142 x 8'


= 628.1 cm'

(E> WORKED EXAMPLE 2:


The volume of a cone with a circular base is 216t mm3. If the base ndius of the cone is
9 mm, find the height of the cone.

SOLUTIONT
Volume of cone = 216r mmr (Given)
t-
I-
nlh = 2l6n
:

t^ .
- n 9' / h =216n
i
2\@x3
'' ){ 9' I

-8mm
._. The heighl of rhe cone is 8 mm
(D WORKED EXAMPLE 3:
The diagram shows sector AOB of a circle cut from a piece of cadboard. The mms i-e
sector is 21 cm and ZAOB is 150'. The edges OA and.OB are joined together ro male.
hollow cone.

Find
(a) the base mdius,
(b) the height,
(c) the volume
of the cone formed.
22_

SOLUTION:
t5n" ,,)
(a) Lenpth of arcAB
- = := \
Jrr(r' 2 \ \ 2l
=I
=55cm
Let r cm be the base mdius of the hollow cone formed.

2x+x
2x22

. . The base radius of the cone formed is 8.75 cm.

(b) Let the height of the cone formed be l, cm.


Using Pythagoras' theorem,
h'+8.75'=21'
h" = 21" -8.75"
h =.lzt, - t.ts,
= 19.09
* l9.l cm (correct to 3 sig. fig.)
.. The height of the cone formed is 19.1 cm.

(cl t_
Volume of cone formed = ,-ay'lr

=;t22
" ; " 8.75, x le.oe
-1530 cmr (correct ro 3 sig. fig.)

Chdpter 8: Mensuntion of Py.*tal -U fl


"on". "00".",
(D WORKED EXAMPLE 4:
A solid cone has a circular base of radius 10.5 m and a total surface area of 940 m'?. Find
the slant height of the cone.
[Take z= 3.142.]

soLuT|oN:
Total surface arca = 940 m? (Given)
tul + ,rf = 940
3.142 x 10.5 x I + 3.142 x lO.5' = 940
32.9911 + 346.4055 = 940
32.9911 = 593.5945
I - 18.0 m (conect to 3 sig. fig.)

.. The slant height of the cone is 18.0 m.

(E> WORKED EXAMPLE 5:


A solid cone of height 12 cm is placed into a cyiinder so that their bases and heights are the
same. 2750 cm3 of water is then poued into the cylinder such that the waterjust covers the
vertex of the corle,
(a) Calculate the radius of the cone.
(b) The cone is then removed from the cylinder Find the drop in the water level.
SOLUTION:

12cm
(a) Volume of cylinder Volume of cone = Volume of water
^t^
n'h -nr'h = 2'750
,)

-nr'h = 2750
2"
axnxr'><12=2l5O
2?50 x 3
-z- 2t|tx12
= 109.42
.= . t00.42
= 10.46
- 10.5 cm (correct ro 3 sig. fig_)

.. The radius of the cone is 10.5 cm.

(b) Let the new height of the water level in the cylinder be Il cm.

nr'H = 2'750
nx 10.46'zx H =2'750
2150
'' - n x 10462
- 8.00 cm (conecr ro 3 sig. fig.)

Drop in water level = 12 - 8.00


= 4.00 cm (correct to 3 sig. fig.)

.. The drop in the water level is 4.00 cm.

ffl spne"es
t. A spher€ is a solid in which all points on the surface are the same distance from the
centre. This distance is the radius of the sphere.

2. A hemisphere is half of a sphere with a circular base.

sphere hemisphere

chrfrer b. VeF'u ar'nr c'ry" r"'..-. ." r***


@
Volume of sphere = Jzr' A
Surface area of sphere = 4rr'?

Volume of hemisohere
' = IJ -zr -ffi- //q"t '.da'e

Sudace area of hemisphere = Curved surface + Flat surface


= 2nr2 + ar2
= 3fir

(E> WORKED EXAMPLE 1:


Find
(a) the volume,
(b) the surface area
of a sphere ofradius 21 cm.
77
ITake ,r = J.l
I-

SOLUTION:

4-
(a) Volume of sphere = aff'
=ar..]'zt'
= 38 808 cm3

(b) Surface area of spherc = 4nr2

-a'!"1'
= 5544 cm,

I
fr) **..,u
(-> WORKED EXAMPLE 2:
(a) The volume of a spherc is 2800 nt'
Find its radius.
(b) The total surface area of a hemisphere is 65 mm:. Find its radius

SOLUTION:
(a) Volume of spherc = 2800 mi (Given)

-rr.r'= 2800

j 2800 x 3

- 8.74 m (corect to 3 sig. fig.)

. . The radius of the spherc is 8.74 m.

(b) Total surface arca of hemisphere = 65 mm' (Given)

,65
jo)
'{ 3"7

- 2.63 mm (correct to 3 sig. fig.)

.. The radiu. of lhe hemisphere is 2.63 mm.

t hapFr R Vehiar.on.r P\rmr(L Cob d \rtFs


D WORKED EXAMPLE 3:
A solid is made up of a cone and a hemisphere as shown.
Calculate
(a) the volume,
(b) the total surface area
of the solid in tems of z

SOLUTION:
(^) Radius of cone =12-2=6at
Height of cone =14 6=8cm
volume oI^t,
cone = J)tr -/?

= ! x xx 6'x
a
= 962 cm'

2
volume oI hemlspnele = '
J,7tr
2-
=_X7rX6,
= 14'1. cm3

.. Volume of solid = Volume of cone + Volume of hemisphere


= 96n + l44n
= 240r cml
(b) Let the slant heighr of rhe cone be I cm.
I' =6'+82
= 100
I = nrl00
= 10 cm

Curved surface area of cone = ttl


=1tx6x 10
= 602 cm'z

Curved surface area of hemisphere = 2zr':


=2xEX 6'
= 72x cmz

. . lolJl surlace arcd of 'race]+


solid =/Cuned rur' (uned.urlace
| \ tu-ea ot cone , /
ldreJ ol hemisphere/
l

= 60tt + 728
= ljlrrcm

. : Uct?lis Turor 28
(D WORKED EXAMPLE 4.
Forty solid metal balls of diameter 1 .2 cm each were melted and recast to form a solid cfie
of height 6 cm. The density of the metal used is 8.4 g/cm3.
Find
(a) the radius,
(b) the mass
ol rhe cone formed.
t)
lTai<e ,r = t'
=.1
soLuTroN:
(a) Let the mdius of the cone formed be / cm.
Volume of cone = Volume of 40 metal balls
I . 4 ,r rrl
;<n r'r6=40.1 /'l;l
/1 .\3
^ 40 .4 . l:l
6
r'z= 5.76
r =
"E:ta

.. The mdius of the cone formed is 2.4 cm.

(b) Mass of cone = Density X Volume


t-
= 8.4 x ^ttr'h
1)1 ::
=8.4\: ' 2.4'<6
- 304.128 g

Chapler 8: Mensuration of Pylmids- Cones md Spbse:


A. Pyramids
t- Calculate the volume of each of the following pyramids.

(a) (b)

(c) (d)

(e) (f)

rol----,*-
Calculate the total surfaee area of the following square / rectangulat plramids.

(a) (b)

(c) (d)

OPOR is a tetrahedron. Gived that the area of its base is 12 m': and its height is 4 m, find the
volume of the tetrahedron.

chapter 8: Mesuntion ol Brumidl colos ud sphees al)


The diagnm shows a closed glass container in the shape of a pymmid. The height of the
ionainer is 15 cm. It is filled with water to a depth of 8 cm. Given that the hodzontal
rectangular sudace arca of the water level is 46.2 cm'z. find the volume of the Dvramid not
occupied by water

5. Py.amid VABCD has a square base of sides 16 cm and a height of 15 cm. Find
(a) the volume,
v
(b) the slant height,
(c) the total surface area
of the pylamid.

6. Pyramid yPORS has a square base of sides 6 cm. The volume ofthe pymmid is 48 cmr. Find
(a) lhe height,
(b) the slant height,
(c) the total surface area
of the pyramid.
?. (a) The base area ofapFamid is 21 cm'z and its volume is 112 cm3. Calculate the height of
the pyramid.
,t

O) The height of a pymmid with a square base is 15 m and its volume is 405 m3. Find rhe
length of a side of the square base.

(c) The volume of a pyramid is 880 mmr. If the base of the p).ramid is a thombus whose
diagonals are 10 mm and 16 mm, find rhe heighr of the pyramid.

(d) The base of a plramid is a pentagon of area 64 m'? and its height is 6 m. Find the volume
of the Dvramid.

(e) A pyramid has a squarc base of sides 9 cm and a slant heighr of l2 cm. Fhd the rotal
surface area of the ovmmid

Chaptd 8: Mensuradon of Pymmids. Cms d,j Sehercs


@
(0 The slant height of a pyramid with a square base is 5 cm and the total surface area of the
pyramid is 96 cm':. Find the length of a side of the squarc base.

(B) The total sudace area of a square pylamid with base of sides 8 m is 200 nf. Find the slant
height of the pyrrmid.

(h) Each of the faces of a regular tetrahedron is an equilateral triangle af sides 6 cm. Find the
total surface area of the tetrahedron,

(i) A solid iron cube of sides 8 Inm is melted and rccast into a soljd pyramid with a square
base. If the height of the pyramid is 6 mm, find the length of a side of the square base.

0) A solid is fomed by fixing 6 identical square-based p).ramids exactly onto the 6 faces of
a solid cube. If the length ofa side of the cube is 3 cm and the volume of the solid formed
is 117 cmr, find the height of each pyramid.

I
8. Sketch the ner of the following pyramids and label rhe didensions.

chapter 8: Mensuation of Pyrmids, conob ud sphercs al;)


9- G) Sketch and name the solid formed for each of the following nets'
G) Find the total surface area of each solid

O-'t*-
10. A solid metal p'.ramid with a rectangular base measuring 9 cm by 8 cm is melGd and Ecast
into a solid square pyramid. Given that the height of the rectangular pyramid is twice tbe
height of the square pyramid, find the length of a side of the bale of the square pyramid

11. A solid metal pFamid of height 15 cm has a square base of arca 144 cnf. The pyramid is
placed into a cubical tank df sides 30 cm. Water is then poured into the tank until itjust covers
the pyramid. Find the height of the waier level after the pyramid is removed.

12. The diagram shows the net of a pyramid. It is made up of a square ABCD of sides 14 cm and
four identical isosceles triangles. The perpendicular height ftom V to CD is 25 cm. The net is
folded to form a right pyramid. Calculatb
(a) the total surface arca,
(b) the volume
of the pyramid formed.

chapter 8: Mensnntio. of Pymids. cdes dd sehees


a2)
htu dn4gam, OABCD is a rectangular pyramid. AB = 42 cm, BC =30 cm, OX = 25 cm and
OY:29 cm.
{a) Draw a net of the p}Tamid and label its dimensions.
O) Find
(i) the total surface area of the pyramid,
(ii) the height of the pymmid,
(iii) the volume of the pyramid.

14. OABCDEF is a ight pymmid with a regular hexagon as its base. Given that AB = l0 cm, the
area of its base is 260 cm? and its total suface area is 965 cnf, find the slant height of tle
pymmlcl.

15. The diagmm shows a metal solid made up of a pyramid and a cuboid. If the density of $e
metal used is 9.8 g/cm', find the total mass of the solid.
--l
t6. The metal solid is made up of a pyramid and a prism. 1
i
Find
(a) the v6lume of the solid,
O) the density of the solid if its mass
is 56 kg, giving your answer in g/cmr.

17. The diagam shows a regular tetrahedron y,4BC with each edge of length .I cm. The height of
the tetmhedron is 10 cm.
(a) Find the base area of the tehahedron, givirg your
answer in terms of r.
O) Given that the volume of the tehahedron is 60 cmr,
find the value of r.

18. The diagram shows a tower in the shape of a tetrahedron standing on top of a pdsm. The crcss-
section of the prism is an equilateral triangle of sides 8 m. The slant edge of the tetrahedron
is 14 m and the height of tho pdsm is 20 m.
Find
(a) the length of YM,
(b) the length ofDN,
(c) the total surface area of the solid formed.

Chapter 8i M€nsuation ofPya-l*, Co*s m sO,"er


@
L Con€s
19- - Fitrd (i) the vol@e and (iD the total surface area of each of the following cones
1J
Lr Ke rE=
n.l
(a) (b)

(c) (d)

(e, (f)

@.,".

I ei._i-r!.".2'
[-
20. (a) The volume of a cone is 675 cm3. Given that the base :uea of the cone is 150 cmr, Etrd
the height of the cone.

(b) Find the base radius of a cone if its height is 14 cm and its volum e is
2112 cm1

t'take n = +.j

(c) The base diameter of a cone is 24 m and its volume is 376g ml


Find fte height of rhe

Aake n= 3.142.1

(d) Find the base radius of a cone if its slant height is 12.5 cm and its curved surface area is
2t) cm-.
)1
' 7'

(e) A cone with a circular base ofradius 9 m has a cLrrved surface area of l l?.fi mr. Filld the
slant height of the cone.

chaprer 8: Memualion of Ba'nds, cds dd sphers


C25)
o TIE base diameter of a cone is 14 mm and its height is 24 Inm. Find the total surface area
of the cone,
tt^ke n = 3.142.1

(g) The total suaface area of a cone of base radius 28 cm is 5500 cmz. Find the slanr height
of the cone.
lTake ,t = 3.142.]

(h) Find the volume of a cone with base diameter 18 m and slant height 15 m.
lTake J'= 3.14.1

(i) Find the total surface area of a solid cone if its base circumference is 88 cm and its slant
height is 18 cm.
,')
ITake r=
=.1

(j) The volume of a cone is 24xr cmr and irs base ,-rea js 8r cm':. Find t])e heighr of the cone.

i 5.1
ea 21. Draw a net of the following open cones and label its dimensions.

iht

22. A \olid cone has a ba<e area ol 385cm dnd a slanr heighr of 12.5 cm. Find
(a) ihe total surface area of the cone,
(b) the volume of the cone.
22_
ttaxez=
-,1
I

Lnt

23. The top part of a cone is removed, leaving the solid known as the ftustum of a cone. Find the
volume of the frustum, giying your answer in terms of t

Ch"pre, 8. Ven.udriol ol hmid, c,iF d' rnr


I{. Actlitr&ical wax candle offidius 6 om and height 24 cm ismelted and rcshaped into a solid
cme ofradius 9 c4. Find the height of the cone formed.

25. A solid rcctangular metal pyramid of base measuring 27 cm by 16 cm and height 33 cm is


melted and rccast into a solid cone of height 14 cm.
Find
(a) the base radius,
(b) the total surface area
of the cone formed.
22.
lTake z=
i.)

26. A conical container of mdius 90 cm md height 3.5 m is completely fiIled with water. All the
water in the conical container is poued into a cylindrical tant with base Gdius 1.5 m. Find the
depth, in metres, of water iir the cylinddcal ta.k.

(O.-r-,,
2tl- The diagram shows a canvas tent nade up of a cylinder and a cone. The dirn .- o{,ft
cylindrical part is 24 m aIId its height is I 1 m. If the vertex of the cone is 16 m high, fud tb
total Burtace area of the tent.

16m

llm

2J. The solid shown is made up of two identical cones and a cylindei
Calculate
(a) the total surface area of the solid,
(b) the volume of the solid.
22_
t laKe r= 7.i

29. The sides OA and OC of a semicircular sheet of cardboard OABC, cente O, of diameter
36 cm are joined to form a conicat party hat.
Find 36cn
(a) the slant height,
(b) the curved sudace area,
(c) the base radius
of the cone fomed.

Chapt€r 8: MenluEtion ot P}1mids Co6 ald Soh-6


I L lv{ris a $cnor of a circle, centre O^with radius 12 cm. The
length of the arc pC is 16, cm.
Ibc,sibs Op ad Oq. are joined to form a right circular cone.
End
(a) the base radius,
(b) dre height,
(c) lhe volume
of the cone formed.

31. MaIc used a rectangular sheet ofart napermeasuring


65 cm by 4l cm to make a cone olheight
40 cm. The diagram below shows tire arnount
of paper left.
Find
(a) the area of the circle used to form the
base of the cone,
(b) the amount of paper left.

32.
srant heisht orthe cone is 15
ffi:'#":f":'i; ;l ::itrfi,YjiJ'i#il;T cm rhe heisht
(a) Calculate
(i) the base mdius of the cone,
(ii) the volume of rhe solid,
(iii) the total sudace area of the solid
(b) Given that the metal used to make the sotid has
a d€nsity of 7.5 g/cm3, calculate the mass,
in kilograms, of the sotid.

lTake n= !7' t

@*'"*o
N
{
c Spheres I

:la Find (i) the volume and (ii) the surface area of the following spheres and hemispher€s. Gi*-e
your answers corect Lo I decimal place. lTake,r = 3.1421.

(al (b)

(c) (d)

(e) (0

chapter 8: Mensua$pn of B,-idl cds dd seheds


a3i)
t l{ F t l* ded d the following spheres, given irs volume.
(a) 96O cm3 | G) ?32S mm' (c) 4500n m3

35. Find the radius of each of the following spheres, given its surface area.

{a} 530 cm': | {br 6648 mr | (c) t6q,rcm:

(a) Find the volume of a sphere whose sudace area is 5544 cm'?.

lTare n= 2i.l

(b) Find the surface area of a sphere whose volurne is 14 130 cmi.
lTake ,7 = 3.14.1

.Q..-"-^,"*,"
(c) A solid metal spherc of radius 6 cm, is melted and recas! to folm a solid cme of r&
8 cm. Find the height of the cone formed.

(d) Twenty-four solid metal spheres, each of diameter 7 cm, are melreal and recast to IoIm a
solid hemisphere. Filld the radius of the hemisphere formed.

(e) The curved surface area of a cone is the same as the surface area of a sphere. Given that
the base mdius ofthe cone is 12 mm and its height is 35 mm, find the raJius of the sphere.

(f) The ratio of the volume of a solid hemisphere to the volume of a solid cylinder is 3 : 4.
Find the total surface area of the hemisphere, given that the base ftdius and height of the
cylinde. are 8 cm and 24 cm respectively.

(g) Find the mass of.200 spherical sreet ball bearings, each of radius 0.8 cm, given that the
density of ste.el is 7.85 g/cm3.

Chaprtr 8 MensuErion of htuds, C]rs lod }!GB


\--
(l) Asp-fre ad a cylinder has,1he,$ame rudius of ,'cm. The heisht of rhe cvlinder is L cm.
Ifthe volume of the cylinder is thdce the volume of rhe sph;. calculate rhe numencat
value of
i

(i) The suface area of a solid sphere is 44lzcm:.It is melted and.ecast to form a cone wtth
a base diameter of 35 cm. Find the height of the cone fomed.

(j) Seventy solid hemispheres, each of radius 1.5 cm are dropped into a cylindrical jar of
radius 5 cm containing water. Calculate the rise in the water level of the cylindrical jar.

(k) The volume of a sphere is equal to the volume of a cylinder. If the ndius of the sphere
is 4 cm and the mdius of the cylinder is 8 cm, find the height of the cylinder.

i
I
(a *._u",,*,u
rm. t- The solid is made up of a hemisphere and a cone. Find
cal (a) the volume,
(b) the total surface area
of the solid.
,)
ITake r= I
=.1

38. A conical portion is removed from the solid hemisphere as sho.,rn below Calculate the volume
of remaining solid, giving your answer correct to the nearest cubic cenlimetres,
22_
traKe z= t.i

39. The diagram shows a solid cylinder with a hemisphere and a cone joined to it at both ends.
Given that the volume of the solid is 3850 cmr. find the heiEht of rhe cvtinder
of 22-
t laKe r = t.i

40. The diagram shows a candle holder in the shape of a hemispherc. Find
(a) the volume,
(b) the total surface area
of the candle holder.

chapter 8. Ver'r'ar:nn ol Pvfud\ { (xF s i(!EF I


I ,l|- Tb diagram shows a hollow metal container maale from a cylinder and two hemispheres.
(a) Calculale, correct to I decimal place,
2.\ nl
(i) the volume,
12.6m

(ii) the external su.face area


of the container

(b) Water is then poured into the container until it is half full. Calculate the horizontal surface
area of the water ir the tark-

42. A hemispherical container is completely filled with 12 { of water The water in the container
is then used to fill as nrany identical conical cups as possible. The top radius of the cup is
3.5 cm and its height is 9 cm.
Calculate
(a) the numtrer of conical cups which can be filled,
(b) the volume of water left in the container, giving your answer in cubic centimetres,
fc) the extemal sudace arca of the container.
nate ,r = 3.1

Ww':

G) *--o'*- ""
43. The diagram shows a solid cone of tiase radius / cm and height 6r cm and a solid h€rniqtE
of mdius / cm.
Find
(a) the volume of lhe cone. given lhaL rhe volume of lhe hemisphere is 354.68 cmr.
(b) the total sudace area of the cone, given that the total surface area of the hemisphere is
180 cm'z.

)e

4. A sphere just fits exacdy inside a cylinder


:r (a) If the volume of the cylinder is 204 m3, ind the volume of the sphete.
is (b) If the surface area of the sphere is 312 m', find ihe total extemal surface arca of the open
cylinder

Chdp tr 8 Men.udrrol of hlfrd! C(]b dl S.m


{L Tbdegram shows a rigfu circular cone of mdius 15 cm inscribed in a sphere of radius 17 cm.
Etrd
(a) rhe height of the cone,
(b) the volume of the cone,
(c) the curved surface area of the cone.

46. A hemispherical container has radius 12 cm. 50 identical sphe.ical marbles are placed in the
container. 2910.78 cmr of water is rhen poued inro the co;hiner ro fill it ro rh;
brim.
(a) Find the radius of each marble.
(b) The cost of painting 1 m'z of the container is g96. Find the cost of painting the extenor of
200 such containers, giving your answer colj|ect !o me nearest cent,
lTake n = 3.14.1

4?. The clay model shown is formed by attaching hemispheres to both ends of a cylhder The
mdius of the cylinder is r mm and its length is 75 nrm. If the clay is remoalelled into a
cone of
height 36 cm and radius r mm, find the value of r.

{e *-*'*.""
m. {& The diagmnrshows a closed glass container.made up of a hemisphere, a cylinder and a c@-
It is exacdy half-filled with water. The radius ofthe cone,is 18 cm and its length is 24 cn The
length of the cylinde! is 52 cm. Find, leaving your answefi in tenns of fi,
(a) the sudace arca of the contarner in contact with water,
fb) the volume oI waler in the conuiner.

52 cm

he

of

49- A solid cone has base diameter 16 cm and height 15 cmr The top part of the cone is cut off. The
rcmaining part of the cone is then attached to a hemisphedcal solid which has the sa.rne base
diameter as the cone as shown in the diasnm below. Find the volume of the solid fomed.
giving your answer corect to 3 significant figures.
[Take n = 3.142.1

he
of

l l.25 cm

Chapter 8: Mensuration ot htuidr C(s ,nl iScE


L Alph.ifuallcadba ofmass 1.2l.kg is dropped into an empty open conical container dfradius
E co- l5 litres of water is then Doured into the container to fiIl it ro the brim.
Fitrd
(e) the volume of the ball,
O) the radius of the ball,
(c) dle height of the conical container
(rle oensrry or reao rs I I g/cm ,)

5l- Diagram I shows the qoss-section ol the symmetiically shaped hourglass made from hollow
hemispher€s, cylindem and cones joined together. It contains sandjust enough to fill the top
conical aDd cylindrical parts.
(a) Calculate the volume of salld in the hourglass.
Diagram II shows all drc sand which has run through and collected ai the bottom of the
hourglass.
(b) Calculate the height of sand, , cm in the hourglass.

a\
| 3cm r

O
3.3 cm

ry 2\
I

ll Diagam I
w Diagrantr

(D *---'"*,"
A rcctangular box is inscribed in a cylinder of height 6 cm
and with a circular base of mdius 2.5 cm. The length of pC
is 4 cm. Find the volume of the box.
"-w

The sudace area of a cube is 384 cmr. The cube just fits exactly
inside a sphere. Find the
surface area of the sphere.

A sphere just fits inside a cube and the cube just fits inside a cylinder. Find
the fraction of the
cylinder that is occupied by the sphere.

Chapie. 8: Mensumion of rymids_ clIF a!.1 slit: l


t Diagrao I shows a closed conical container with a diameter of 16 cm and a height of 12 cm
etid is filled with water to a depth of 6 cm.
(a) Frnd dte volume of water in the container, leaving your answer in tems of r.
(b) The conical container is then inverted as shown iD Diagmm II. Find the value of r, giving
vour answer conect to 2 decimal Dlaces.

---->
12 cm

Diagram I Diagram Il

5. A cylindrical container of radius 2 m is placed on its side up against a wall. A ball just fits in
the gap beiween the container, the wall and the floor Find the volume of the ball, gjvjng your
answer corect to 2 decimal Dlaces,

Q"'-'-'-'*-,"
a
i"tr$?.:.l1"#:ffi 'J::ff %:U:-1'.:1f L:*Tf#jiJX,i j,'il*";fu.-;*J
to the brim. The hemispherical part contaim j of the volume of water in rhe cylindrical parr
of the test tube.
(a) Find the values of ft and r.
(b) The water in the test tube is then poured
into a right citcular cone as shown in
Diagram II. If the diameter of the
surface area of the water level is 5 cm,
find
(i) the heighr of the water level in the

(ii) the su.face area of the cone that is


contact with waier,

Diagmm I Diaelam II

ru

Chapter 8:Medsuralion of qy€nids_ Cor r.rj S:]E-


Graphs of Linear
Equations in Two
Unknowns

EE Graptrs of Linear Equations


Graphs ol line equations are straight linelt.

Slcps lo dra$ the graph of a linear equation:

i.1) Construct a table of values ofr and )..


You will need the coordinates of three points.
(A sffaight line graph can be drawn using on]l'two points but the third
point is needed to chcck.)

O Choose a suitable scale-


The scale used on thc r-axis does not have 1() be the samc as the )-:rxis.
The scale chosen should allow fo. the laryesl possible gmph to be drawn.
The bigger the graph. the more accurate will be the results obtained from it_

r!] Plot the points on the graph paper and draw a sffaighl line rhrough all rtte
points.

@) Label your graph with rhe equarion of rhe line.

Et> WORKED EXAMPLE 'I:


Draw the graph of ) = 2r - 1. From _rour graph, find
(a) the value ofl, i
when = 2.1,
(b) the value ofr when r. = -3.8,
(c) the value ofp given thnt (1.5, p) is a solrtion of ) = 2r I
soLuTtoN:
Constuct a table of values for the equation 1 = 2, - l.

We then plot the points and join them with a straight line to obtain the gaph of y
= 2, t.

ffi

From dre graph,


(a) whenr=2.1,)=3.2.
(b) when ), = -3.8,, = 1.4.
(c) whenr=1.5,p=2.

ChdoLs o. C aph, o, Lma tCdon\ o leo I blr..d


The graph of a horizontal line passing through the
point (0, c) and paraflel to the .r-axis is of the fom:

E.g. The graphs of ), = 2 and ], = -1 are shown below.

tl

l ::i

...1
t .l
I ii -L
l I
il I

The gradient of the gaph of the folm J = c is zero.

4. The graph of a vertical line passing through the


point (1', 0) and parallel to the )-axis is of the folm:

E.g. The graphs of jr = 3 and x = -2 are shown below.

The gradient of the graph of the form jr = d is undefined.

i'a '---o-ri,,* 28
5. The gaph of a straight line passing through the origin, (0, 0) and wirh gradientEb
of the fom:

tl::y I

wheremisaconstant,

If the gradient, m is positive, the line slopes upwards,to the righr.


If the gradient, m is negative, the line slopes upwards to the left.

The bigger the numerical value of m, the steeper the line.

. m is positive . m ls negatlve
. the line slopes upwards to dre right . the line slopes upwards to the left

6. The graph of a straight line that cuts the y-a\is ar rhe point (0, c) and has gradient,
m is of the folm:

lt=.'t;l

When the value of m remains the same with c taking on different values, the graphs
are parallel lines cufting the y-axis at the pohrs (0, c).

chapter 9: craphs of Lined Eq*,.* t *. u"**r^


Gi)
m iS a constant value of 1 while c takes on dillerent values.
2

WORKED EXAMPLE 2:
(a) Draw the graph of each of the following equations on the same axes.
(i) .d+2=0
tll, v= L.x I
2
(iii) 2y+3"r-6=0
(b) Find the arca of the triangle bounded by these three lines.

soLuT|0N:
(a) (i) r+2=0

(a**--'-**,u
t.
'2 -2 0 2

2 -t 0

2y+3x-6=Q ,2
2Y= 3r+6 2

3^
J=--X+J 6 3 0

(b) Area bounded b) the three lines

=lxsx+
2
= 16 units'?

ChapFr'1. Gdol\ or I inear Lqlalrol. In Iso I MqG


iD WORKED EXAMPLE 3:
(a) State whether the points Iie on the given lines.
(i) (4,5),)=2r-3
(ir) (2,7).l,=9-f.{
)
{b) ll ta.a+gtrsa\olurionof}= J.r+5. findrhevalueofd.
soLuTtoN:
(a) (i) )=2r-3
When t = 4,
r= 2(4) 3

The point (4, 5) lies on the line ] = 2r - 3.

(ii) r=9 lx
2

y=s-lrzt
'2

The point (2, ?) does not lie on the line y = '2 9 l;r.
(b) Since (a, d + 9) is a solution of l, = 3r + 5, substitute -I = a and ) = a + 9 into the
equation)=3r+5.
)=3x+5
(a+9)=3(a)+5

@ Sot"ing Si-ultaneous Linear Equations Using the Graphical Method


1. In Book 24, we have leamt to solve a pair of simuhaneous linear equations bl using
either the Elimination method or the Substitution method. We now leam to use the
Graphical method to solve a pair of simultaneous linear equations.

2. Steps to solve a pair of simultaneous linear equations graphically:

@ Draw the graphs of both equations on the same axes.

@ Read off the values df the variables at the point of intersection of the
lines. (The coordinates of the point of inte$ection gives the solution of
the simultaneous equations.)

,9 *r**,u
rD woRKED EXAMPLE 1:
Solve the following simultaneous equatioDs gmphically.

x+2Y=8
3x-Y=3
SOLUTION:
Constluct a table of values for each equation.

x+2i=8 3x-y=3
2t= x+8 Y=3:( 3
I
)=--.rt4
-2

Choose a suitable scale and plot the graphs of r + 2) = 8 and 3r -) = 3 on the same axes.

'I

the

.tl
od
ing
the
The solution is, = 2 andy= 3.

i: 1l

l:
-t

ofLiner Equions in T{. LnIa-E$ i:


(B> WORKED EXAMPLE 2:
Solve the following simultaneous equations graphically.
x+2i=6
2x+bt=4
soLuTtoN:
)c+2Y=6 2x+4y =4
2Y=-x+6 4Y --2r+4
l^
,=-_r+r I
1= -t+l
-2 '2
0 2

4 3 2

I I
T I I
+
I
+
ri -
+4) I l
,l

l i
II
.. The simulianeous equations, + 2), = 6 and 2r + 4) = 4 have no solution.

ri*or zo
C:i rraa*remrics
WORKED EXAMPLE 3:
Solve the following simultaneous equations graphically.
2x-3Y=9
4x=6)+ 18

SOLUTION:
h-3y=9 4r=6y+ l8
3Y=2r-g 6y=hc-18
2^
'3 Y=-x-J

.. The simultaneous equations 2r - 3) = 9 and 4r = 6y + 18 have an infinite number of


of solutions.

Chapter 9: Grapns of Lined Eq*,.^ t *" U".-*^


Crr)
A paL of simultaneous linear equations can have one sohtion, no'Sohitio or an
idlnile number of soludons.

One solution No solution Inffnite number


of solutions

E.g.

2r t =+

b-t-q b t=-z *-t=q


Lx+Y=8 b Y=+ 4x-2Y=g
Solution:r=3and)=2

Mad'emdcsruo.28
a5;)
Tutorial

Graphs of Linear Equations


Complete the table. Then plot the coordinates and draw the graph of each equation on the axes
provided.

(a) y=x+2 (b) Y=2r

(c) )=_r r
ffi:-FT (d) -l
)=J-ax

t-lf ilrl,
ffi l
lfirl'jill
f.lllEF:Iillll'-ll T lTll-I i
hHrttjl1i.
nflil1tit-,li-ll
F,' lir t-ilLlllii,l;i ll ]l_
;t
f-.-' [. ]li ' llliil ll l| ,lt_
'*; i1l:I L ri l-f;llltirrlllrIl.l.fl I
f
l
Fll:tfjfifftlif l

cbaptd 9: Cnphs of Unetr Eo*,t^ t U"*t"t*


"*. @
(e) -t+1=3 (D Lx-y=4
0 2 4 2 0 2

(g) 2t+5]= l0 (h) 2y-3.i+6=0

0 5 0 2 4

u"o..*i* r"m. zo
@
On the axes provided below, druw and label each of the following graphs.

O) r=-t
tl
t
!-

ll :

,irl
:l l

(c) r+4=0 (d) l.s-)=0

(e) r+)=0 , Zx=O

,l'
I
L
I
I

chapter 9r craphs of Lined EqD"u-. t *" u*.*. Crr)


3. Write down ihe equations of the following lines

4. Sketch the graph of l' = mr + c wherc:

(a) m>0,c>0 (b)


'fl<0,c<0

(c) la=0, c>0 (d) n <0.c=0

@ "**.,*" ^', *
Each of the equations in the table represents one of the graphs ftom A to I. Wdte down ihe
letter ol each graph in the correct place in the table below.

y=2r+3
-{+)+5=0
2(r,+1)=l(r-4)

chaprer 9: crsphs of Lined Equliom in Two unkmms


459)
6. The diagram shows the gaph of l = .I + 3 On the same diagmm' sketch and label the graphs
ot

(b) r=r-z
(c) Y=3--r

7. The diagmm shows the lines with equatlons ) = 2r' 1= 4' ] =8 -t and] =

Match th€ equation corresponding to each line.


& ' Y=2r
@. . y=4
@. ' )=o r
@. 't= l^x-z
' '3

@ "",0"*u* ,*- ,"


The diagnm shows three sffaight lines.
(a) Write down the equations of the straight lines,
A, B and C.
(b) Write down the coordinates of the points P and O.

(a) On the axes below draw the graphs of the lines ) = 0, r = 6 and I r+ 1. =
(b) Calculate the numeical value of the area of the triangle bounded bl, the lines ), = 0,
r=6andy= r+1.
rt

ii,
i-ll
,-i i

-.I
:il

Chapter 9: Graphs ofLinea! Equtios in Two U"kNwN


@
10. (a) Given the equation ) = 2r + 5, complete the table below.

(b) Draw the gaph of ) = 2r + 5.


(c) From the graph, find
(i) the value oft when x = -1.5,
(ii) the value ofx when ) = 6 8.
(d) Is (2, 9) a solution to the equation ) = 2r + 5?

ll. (a) Given the equation.r + 3) = 6, complete the table below.

(b) Dmw the gaph of .x + 31 = 6.


(c) From the gaph, find
(i)the value of) when r = 2.4,
(ii) the value of, when y = 2.7.
(d) Given that the point (3.6, ft) is a solution to the equation 'f + 3) = 6, find the value of ft

12. (a) civen the equation 3, 4y - 12 = 0, complete the table below.

(b) Draw the graph of 3t - 4y - 12 = 0.


Given that the point (ft, L8) is a solution to the equation 31 4l' - 12 = 0, find the value
of ft.
(d) The graph 3r - 4) 12 = 0 intersects the x-axis at the point A and the )-axis at the point
B. Find the arca of triangle AOB where O is the odgin.

13. (a) Given the equation 3x + y = 0, complete the table below

(b) Draw the graph of 3.r + ) = 0.


(cl If (-1, p - 1) is a solution to 3, + y = 0, find the value ofP.

14. (a) Draw the graph of each of the following equations on the same axes
(i) Y=6
(ii) Y=141
(111) 2y 3x=6
(b) Find the area of the triangle bounded by these tbrce lines.

t**."u*
@ ^..,,u
l:i. It is given that the points (2, 4) and (-8, l) lie on the line hr + trl' + 6 = 0.
(a) Find the values offt and k.
(b) Draw the above gaph for -2 < i < 6.
(c) Write down the )-intercept of the gaph.
(d) If (/, 5) lie on the graph, find the value of /.
16. (a) Draw the gaph of each ofthe followinB equations on the same axis.
(D I (ii\ x=2
(iii) '=
1= 2r (iv) y=21a9
(b) Name the figure formed b) these four lines.
(c) Find the area of the figure fomed.

17. (a) Dftw the graph of each of the following equatioDs on the same axes.
_.. I (ii) -2
r = -i.r
2

{iii) ')y= 1r+6 3-


-2
(b) Name the figule formed b) these four lines.
(c) Write down the coordinates of the vertices of this figllre.

l& In each of the following, state whether the ordered pak is a solution of the equation.

(a) (-1, 4), x 2! ='7 (b) (10, 3), 3.r + sjy = ls

ue

)mt (c) (-1,2),4r+:|,=o (d) (3,3), ?r 8) + 3 = 0

It- (a) Wdte down the equation of the vertical line that passes through the point (3, -2).
(b) Write down the equation of the horizontal line that passes through the point (ll, 8).

Ctoprei 9: Cnphs of Linear EqEiiom in lvo UnkiowN


@
20. (a) The line -r =3 meets the r-axis at P. Wite down the coordinafes of P'
(b) The line y = 4 meets the ]-axis at 8. Write down the coordinates of C'
i"j te m.s = : and ) = 4, meet at R write do*n the coordinates of R'
(d) Calculate "
(i) the area of APOR,
(ii) the perimeter of AP0R.

21. Final fhe values ofp, 4 and ,'if the following points lie on the line 2t + 3) = 9
(a) ( 3, p) (b) (2q - 3. -1) (c) (4r,5-3r)

22. The points (r?, 0), (0, e) and (4,/) lie on the line 5r - 4l = 8'
(a) Find the values of d, e and f
(b) State whether (-2, -41) fes on the line 5.i - 4] = 8

,1

23. TheequationofasffaightlineIis;x+4)=-ll Ifthepoint(2'i+1)liesonl'findthe

r,aaumati"s rurorze

b
@
The point A is the .r-intercept of the line r + 3), =10. Find the coordinates ofA. Another point
B is the ]-intercept of the line 3r - 5l = 15. Find the coordinates of B. Hence, find the
nllmerical value of the area of triaDgle AOB, where O is the origin.

Solving Simultaneous Linear Equations Graphically


The diagfdm shou rhe graph of .l,r + ) = L
'
(a) On the same axes , draw the line with equation l, = r- 5

chaptef 9r cnphs of Lin@r Eq*,a^ t *" uru-**


@
(b) Use your gaphs to solve the simultaneous equations:

tI+!= t

The diagram shows the gaph of 2r + 3.), = 18.

(a) On the same axes , draw the line with equation y = !1- 3.
3

(b) Solve the simultaneous equations:


2r+3Y-18
x-3Y=9

(J) **".,,o.r*-.u
17. The graph of 1= 2r - 6 is shown. Draw on the same
axes the graph ofr_
graph to solve the simullneous equations:
2_v = 6. Use )our

)=Zr 6
x 2!"=6

, ,l
I
:]
I

-,1

::l ]

,l

:4. (a) Comptele the table of values for thc stlaight lines given.
(i) y=11a7 (ii) )=,r+,+
2

-1 0 4 4

r
(b) On the s:une axes, draw the graph of each line_
l
(c) Use your graph Io solve the sinultaDeous equauons:

t_ /
l= -x+
')

l=--r+'l

Chltier 9: Cr.phs of Lrnear Equat.ns ijr ,Ii\ o L ntiol!.j G.


29. Solve the following simultaneous equations using the graphical method.

(a) y=x+3 (b) v=r s (c) x-2r = 4


y=4x-2 3i+)=5

(d) 2x+]+6=0 (e) x=r 2 (0 3.r-lOy=O


'2
I x 2y+2=0 x+2Y=12

(g) 3-r-2)=9 (h)'7t+4y=28 (i) r+3J+5=0


2Y=x-l Lx 3y + 1=0

Q) 4x+y=12 (k) 3)-r+6=0 (l) 31 81= ll


1^
,r+;)=.1
6y-2,r= 15 t 5)+3=0

(m)Zr+I=4 (n, (o) 4,x + 5) = 20


)=t+5.5 2r+)t+7=0 7x+4y=24

@ ""r"-**... ru
:0- (a) Given the equation ) = 2 r, complete th.3 table below

I 0

J -2
(b) Using a scale of 2 cm to reprcsent I unit on each axis, dlaw and label the gmPh of

From your graph, find


(i) the value of) when.x = 2.5.
(ii) the \,alue of-r when ] 1.8.=
(c) Given the equation ) = -2r + 3, complete the table below

,I. 0 2

,t;: 5

(d) Draw and label the graph of) =


2I+ 3 on the same axes as parl (b.)
(e) Hence, find ihe solution to the simrltaneous equations:

3'I+3]=6
h+l'=3
Answ€r the whole of this question on a sheet of graph paper.
(a) Civen the equatioo 2) - 3r + 6 = 0. complete lhe table below

(b) Draw the graph of 2)' - 3r + 6 = 0, using I cm to reprcsent I unit on each axis, lbr
0<.I<8and 3<)<10.
(c) The glaph 2t, 3r + 6 = 0 cuts the -t axis at the pointA and lhc,! axis at the point B- Find
the ea of tliangie AOB where O is t]e odgin.
(d) Onlhe\tuned\e\.drcu lhcSrirPht,fJ 'r F4 Hen.e.sol\elhe.imultane'u'equcl;or''

2r'-3x+6 =0andl,= -ir


l
+ 4.

Given the equalion,v + 2) = 10, complete the lable bebw

10
1:] -5

r
(Dl Using a scale of 1 cm to represcnt I unit on the -r axis and 2 cm to represent I unit on the
)-axis, draw the graph ofr + 2) = 10.
Given the equalion jr - 3] = 0' complete the table below

(d) Draw and label the graph ofr 3) = 0 on the same axes as part (b)

.l.pe 6 C aol^ l
'e
rLa ,-"
' " o
(e) Find the solution to the simultMeous equations:

';+2Y=10
t-3]=0
(f) Calculate the numedcal value oi the area enclosed by the lines with equations:
r =0,r + 2)- 10 = 0,r- 3),=0

33. The diagram shows the glaph of ) = t + 2


(a) On the same axes, draw the graphs of
(i) ), = 2r l, (ii) 3t, + -r = 24.
(b) Use your graph to solve the simultaneous equations:
(i) J)=x+2 (ii) y=aa2
y=2x I 3!+x=U

@ **."* ru., tu
Paula is n yea$ old and her oldcr brother' Marc is I yean old The sum of iheir ages is 20 ! e ars
and the difference between their ages rs 4 yea|s
(a) Form two equations cornecting t and,
(b) On the same axes, draw the graphs of these two cqualons
(c) Ffom yolll graph, find the lges of Paul' and Marc

The concert tickets for two adults md thfee children cost a tonl
of $i48'
(o) U.ing S-t,o t"p.".ent the cost fof tln adult ticket and $) lhe cost for a child tickel' show
Ihat1I+3)=4ti
Jon."rt tickets for one adult and two children cost a tott of $28 Wdte
down
tfri ifr. ."..
another equalion involvi4g -v and )
(c) Solve the iwo simultaneoris equations graphically md hence
wdte dowD thc cosl for an
ailult tickct and the cost fbr a child ti'rker'

marks were 150 He obtained 20


Jonathan sat fb1 a Mathematics irnd a Science test His total
mirr\s in \4dlherlralr.'' lh 'n In Srience
_Uri*
rnofe
i"i t,o t.pt"."it his Mathematics,ma|ks and ) to represent hir Scjence marks' wrire
d,)wn l$o equ:llion'conlre(llna t an'l
tb) Find his marks for Mathenatics and hrs mrrks lt'r Science
u\ing rr llalhLcal Incthod

Chnprd 9: Grlths ol Lnre.r EO-,,o". Ut tt"t


''not*^ !i
1. The cost, $c, of a chest of drawe6 is calculated using the formula c r + d) where r and arc
= )
constants and I represents the number of drawgrs.
The cost of a chest with 4 drawers is 9160.
The cost of a chest with 5 drawers is 9190.
(a) Use the information to write down two equations in r anal ).
(b) Use a graphical method to solve the equations to find the values of n and Hence find
-|,.
the cost of a chest with l0 drawsrs.

2. Mrs Wong makes clay vases for sale. The fixed cost is 930. The cost of making each vase rs
$8 and \he.ell\ erch \a\e tor gl4.
Using graphical merhod, find
(a) the number of vases Mrs Wong needs to sell to break even,
(b) the least number of vases she needs to selt to make a profit,
(c) Mrs Wong's loss when three vases werc made and sold,
(d) her profit when twelve vases were made and sold.

@ ",,*.,,"",*-'"
Mensuration ol Pyramids, Cones and Spheres
Graphs ot Linear Equations in Two Unknowns

nd ) are Find (i) fte volume, (ii) the total suface area of the following solids.
[Take ,' = 3.142 where
necessarv,l

32 cm

R$hmBercise4
@
(0

l,*
I

2. The diagram shows a glass crystal in the shape of a rectangular pyramid.


(a) Draw a net ofthe pFamid.
(b) Find the total surface area of the pyramid.

3. A solid inetal \reight in the shape of a square based pyramid of sides 6 cm and heighr 12 cm
is placed inside a cylinddcal container of radius 8 cm and height 30 cm. The container is then
filled with water to the b.im. Find the drop in the water level when the weight is rcmoved,
giving your answer coffect to 2 decimal places.

@ "**,'*r*-,u
A right triangnlar pyramid has a base in the shape of an equilateml ftiangle of sides 16 mm-
Ci\en thal rhe roral surface area of lhe pyramid is 780 mm:. find its \lant beiBhl.

VABC is a right p'.ramid ivith a square base of sides l0 cm and slant height 13 cm.
Find
(a) v
the height of fte pyramid,
(b) the volume of the pyramid,
(c) the total surface area of the pyramid-

A wax pymmid with a square base of sides 36 cm and height 60 cm is melted and recast to
form the solid shown below. The solid is made up of a cone and a hemispherc. Find the
maximum number of such solids that can be fomed.
.rake ?2.1
n = 7

J'-

RdisionErerclse4
@
7. A glass cylinder has diameter 12 cm. Water is poued into the cylinder to a depth of 8 cm. Two
metal sphercs of radius 2.5 cm each are then dropped into the cylinder. Find the new depth of
tbe water in the cylinder.

8. The diagram shows a solid formed using a cylitder and a cone. Given that the volume of the
solid is 3465 cmr, find fie base radius of the sotd.
22-
'7-

t-

9. The diagam shows a party hat which is maale up of a hemisphere and a circular base. Find the
total area of matedal used to make the hat, leaving your answer in telms of xt.

@ "*"."0* ru- ru
lof The area of the semicircular sheet ofaluminium is 12+rcm'z. The sides OA and OB arejoined
to forn a risht circular cone so that arc AB becomes the circumference ol the base. Find
(a) the diameter of the semicircle,
O) the base diameter of the cone formed,
(c) the volume of the cone formed.

A solid sphere just fits exactly inside a closed cubical box.


Find
(a) the exterior surface area of the box if the sufface area of the sphere is 346.5 cm'?,
(b) the ndius of the spherc if the volume of the unoccupied space in the box is 35.28 cm'.

-naxe n = 2.1
'7-

The diagram shows a solid sphere ofmdius /cm and a solid cylinder of ndius R cm and height
tR cm.
(a) If the volume of the sphere is twice the volume of the cylinder and r = 15 cm, find the
value of R.
O) ff the total surface area of the cylinder is thrice the surface area of the sphere and
R = 9 cm, find the value of r.

**Oar U*-,*,
@
13. The diagram shows an aluminium pail completely filled with water.
Find
(a) the surface area of the pail in contact with water,
(b) the amount of water in the pail, giving your answer in litres.

14. A solid wooden contaher is made ftom a right circular cone. The top of the cone is removed
and a hemispherical hole is then cafled out to obtain the container shown below. Calculare me
volume of the container,

15. Three tennis balls fit snugly into a cylindrical can. The ballsjust touch the top arld the bottom
of the can. Find the fraction of the can which is taken up by the balls.

Crr) "**"u* ^.. ,"


The diasram shows a cvlinder inscdbed in a sDhere. The base mdius of the cvlinder is 4 cm
and its height is 6 cm.
Find
(a) the radius of the sphere,
(b) the volume of the sphere,
(c) the suface area of the sphere.

A solid cone is cut vertically into half. Half of the cone is then attached to a hemispherc to folm
the solid shown below The height of the solid formed is 63 cm and the mdius of the
hemisphere is 27 cm. Calculate
(a) the volume of the solid fotmed,
(b) the mass, in kg, of the solid fomed, given that it is made of material of density
0.84 g/cmr,
(c) the number of tins of paint needed to paint 500 of these solids, given that each tln of paint
can paint an area of 4 m',

R-tsoiErerci*4
@
18. (a) Draw tlte graph oleach of the following iines on the same axes, using a scale of2 cm to
rcpresent 1 unit on the r-axis for 2 < r < 6 and 1 cm to represent 1 unit on the y,axis for
4<y=12.
(1) 4-t=o (ii) r+)=3 {iiir )=l (iv) }=-2
(b) Name the figule fomed by these four lines. Hence, find the area of this figule.

I 9. (a) Using a scale of 2 cm to represent I unit on the r-axis for 3 < .I < 3 and I cm to rcpresent
I unit on the y axis for -8 < ] < 12, draw the following graphs on the same axes.
tit y=)1a5 tii).r=21 riiir )=-Zr- I

(b) Find the area of the niangle bounded by lhese three lines.

20. (a) Given the equation ) = -1r + 3, complete the table below.

ffi.-trff
ffit--r-l
ftl Drawthegaphof y=-lx- 3.

(c) Given that tlle point (2p,2.1) is a solution to tbe equadon x + 4y = 12, find the value ofp.
(c) On the same axes, dmw the graph of 3x - 2y = L Hence solve the simultaneous equations:

r+4y=12
,,

3x-2!=l

i.

@ "*.'*",,'-,"
mto 21. (^\ The diagram shows the graph of 21 31 = 24. On the same axes, draw the graph of
s for
(b) Use the gmphs to find the solution of the simultaneous equations:
2
2x-3Y=24
x+Y=2

Solve the followng simultaneous linear equations graphically.

(a) r+]=4 (b) l=-2r+0


2x-Y= 1 t,= -!,
'3 z

(c) 3r+1 6=0 (d) 4.i 5) = 10


5r t,+14=0 2x l0Y = 35

RwisionErercisel
@
23. (a) The equation of the line I is 5) + 26 = 8r. Find the value of k, gjven that the point
(1 - 2*, 6) lies on the line l.
(b) Write down the equation of the vertical line that passes through the point (5,
(c) The line r = 8 meets the r a-\is at A. Wrile down the coordinates of A.

24- The diagram shows the lines wjth equations 3) = 21


9,2! + x = 6,! =3,2j =.r 6 and
3) + 2r = 9. Write down the equation co esponding to each line.

25. The cost of a mug and a plate is $r and $], respectively.


(a) The cost of two mugs and a plate is til5 while the cost of one mug and two plates is
$16.50. Folrn two equations connecting ir and I.
(b) Draw the graphs of these two equations on the same axes.
(c) Hence, find the price of a mug and the pr;ce of a plate.

@ "**.*..,'-,u
Marc and Esther order pizza fiom Pete's Pizza regularly. Esther lives 5 km further from pete,s
Pizza than Marc does. In a ceftain week, Pete's Pizza alelivered 4 times to Marc and 3 iimes
lo Esther. The total distance travelled by the delivery man to each of their houses for that week
was 36 km.
i
(a) Marc lives kln from Pete's Pizza and Esther lives y km from Pete's pizza. Using the
infomation given, write two equations involving .x and ).
(b) On the same axes, dmw the gaphs of the two equations.
(c) From your graphs, find the distance that Marc and Esther each live from pete's pizza.

Mr Smith bought 210 lighl bulbs consisting two brands, A and B. He found that I ofBrand
/ bulbs and : of Brand B bulbs were faulty. The number of faultj, bulbs for each brand was
lhe same.
(a) Letting r and), be the number ofdefective Brand A and Brand B light bulbs respecuvcry,
write down two equations in tems of r and y.
(b) On the same axes, dmw the graphs of these two equations.
(c) Hence, find the number of Bmnd A and Brand B liehr bulbs Mr Smith bousht.

RwstonErercis4
6i
I

Graphs of
Quadratic Equations

@ craphs of Quadratic Equations


1. Quadratic graphs are graphs whose equations arc of the form 1 = ar2 + ,I + c where
17, D and c are real numbers but i7 cannot be zero.

E.g y=1,'.\=r +Zr-Jcndv=5 lr'are quadratic Eraph\.

2. The graph of a quadmtic equation is a smooth U-shaped or n _shaped cu e called


a parabola.
The examples below show some quadratic gaphs.

J=Lf -1r+9

3. The curve of a quadratic gmph is symmetrical about the line of symmetry.

ru
@ ""*"'"0* ^.,
The curve of a quadratic graph has either a maximum or minimum point depending on the
value of d. i.e. the coefficient ofi'.

The Iine of symmetry is a vertical line that passes through the maximum or the minimum
point of the curve.

to:_::l-*u"
"t::,"*-lc cuehs = +,' + ' are siven berow
1 :r'
(r) If a is positive, i-e. .l > 0, the graph has a minimum point.
. The gmph opens upwards i.e. U shape.
. The vertical line through the minimum point is the line of symmetrl
. The smallef the numerical value ofl1, the wider the graph opens.
. The graph may cut the,v axis at0, I of 2points, and the)-atis at only 1 point.

(Hcrca=1)

Minlmun poinL, (2. -9)

If a is negative, i.e. r < 0, the grzph has a maximum point.


. The graph opens downwards, i.e. n -shape.
. The vertical line through the nraximum point is the line of symmetry.
. The snaller the nurnerical value of a, the wider Ihe graph opens-
. The graph may cut ther-axis at 0, l, or 2 points and the) axis at only I point.

Mr\inrum po nr, ( r .6i)

(Frerea= 2)

chcfle. 0 ctuphs orQuadra'" *---


O
The gr^ph of y = al @,. 0) passes through the origin The line of symmetry is the )-axls

(4>o)

The smaller the numerical value of d, the wider the gmph opens'

8. Steps to dra\t the graph of a quadratic equation:

Construct a table of values oft and y for the quaalratic equation'

Choose a suitable scale.


The scale used on the rr-axis does noi have to be the same as the J-axis The scale
chosen shoulal allow for the largest possible gmPh to be drawn The bigger
the
i
gmph, the morc accumte will be the results obtained from it'
Plot the points on the graph paper and join them up to lblm a smooth curve
Label your gaph with the equation of the cufle.

@ **".** ^.. ru
WORKED E)GMPLE 1:
Dra\ lhe graph of) =,r r 2for-3<x<J.
From your graph, find
(a) the value of y when ,r = 1.7,
(b) the coordinates of the maximum or mhimum point,
(c) the equation of the line of symmetry.

Construct a table for coresponding values of r and 1.

Plot the points and join them to form a smooth curve.

From the gaph,


(a) when.r = 1.7, y - 4.9,
(b) the minimum point is (0, 2),
(c) the equation of the line of symmetry is r = 0.
(The y-axis is the line of symmetry.)

Chapter l0: Graphs ofQuad@tl" Ec*d.^


@
(F WORKED EXAMPLE 2:
Draw the graph of 1 = xz + x2lor -4 < x < 3.
From your graph, find
(a) the value of ) when r = l 6
(b) the values oft when )' = 5'
(c) the smallest value ofY,
(d) the equation of the line of symmetry of ] =f + x 2.

soLuT|0N:
Conshuct a table for conesponding values of .r and l'.

Plot the points and join drem to form a smooth curve.

From the graph,


(a) when.r = 1.6.Y*2.2,
(b) when t' = 5, x - -3 .2 ot x - 22,
(c) the smallest value of ] is -2 3,
(d) the equation of the line of symmetry is x = {5.

r'ratem'rio ruto u e
C8s)
WORKED EXAMPLE 3:
(a) Copy and complete lhe table which gives the values of )=I + Zr _ x2 for
-3 <x<5.

(b) Using a scale of 2 cm to represent I unit on the j-axis and 1 cm to represent I unit
on the.), a-\is, dmw the graph of} = I + 2r
r':.

(c) From your graph, find


(i) the values of) when r = -1.6 and 3.2,
(ii) the values ofx when l' = -9,
(iii) the greatest value ofJand the conesponding value of x,
(iv) the equation of the line of symmefy of the curve.

SOLUTION:
(a)

Chapter l0: G.aphs of Quadm,'. ."r"**


@
t
I
(c) ftom the graph,
(i) whenx=-1.6,y- 4.8,
whenr=3.2,)=-2.8,
(ii) when)= 9, x - 2.3 ar.d t - 4.3,
(iii) the greatest value of :y is 2 and it occuls when,I = I,
(iv) the equation of the line of symmetry is r = L

(E> WORKED EXAMPLE 4:


The diagram shows the gmph of l, = I - 2r - 8. The graph cuts the x-axis at the poitts A
and B, and the )-axis at the point C.
(a) Write down the coodinates ofA, B and c.
(b) D is the minimum point of the gaph
Write down the coordinates of D.
(c) Write down the equation of the line of
symmetry of the curve.

SOLUTION:

'"' '=1,;f;;lor *-ffi


When ], =
(x+2)(r 4)=0
0, --ffi
x+2=0 or x-4=0
x=2 or x=4
..A=(-2,0)andB=(4,0)
0, - ffi
When x =
rv=0'-2(0)-8= 8
... c= (0, 8)

(b)
J+4

Substitute r = I rnto]=x--a{-6,
y=12-2J)-
... D=(t, 9)

(c) Equation of line of symmetry: jr = I

@ ""*.'.o* ^.,,"
@ Sottiog Problems involving Quadratic Graphs
L The following examples show how we solve problems involving quadratic graphs.

(rc WORKED EXAMPLE 1:


The total surface area of a solicl prism, A is given by A =(41 5x + 2) cm'].
(a) Draw the graph of A = 4l - 5x +2forO <r <2.
(b) From the graph, find
Iir rhe minimum lotal surface area of lhe prism.
(ii) the value of-I when the total surface arga is 5 cm:.
SOLUTION:
(a) A=4t 5x+2

i.,li'.il;

chapter ro: cnphs or eudG* ***^ Gi


(b) Frcm the grap.h,
(i) te Jinimiir total suface area ol the prism is 0.45 cm'z
rii) \.rheo A = 5 cm':.-r- 1.7 cm.
.. When the total sudace area is 5 crf, - 1.7 cm.
,T

(-> WORKED EXAMPLE 2:


In the dia$am,,4,BCD is a rectangle.,4'B = (4- i)cm'AD =(Lt +3) cm and CE=rcm'
(a) Show that the are4 A of ABED is Siven bv A = I +l + lr + 6) cm'

rb) Drau rhegraphof A :./ t -t '6for-4 <x<5


= 22
(c) Frcm your graph, find the value of x that will rcsult itr the maximum area of ABdD'

2r+-l D

soLuroN:
(a) BE=BC-EC
=(2r+3)-.t
=(r+3)cm
Area of ABED
-ffi
=lxBasexHeight
=!xnptco
*#ffi
2

=1x(x+:)r(4-r)
=Lg, f*tz zr1
=lgf+x+121
= (-ll + lx + 6) cIIf (shown)

(D ""**o*'"-'u
(b) A= lf+
22
!r+6

ti

"l
'

ft iil .' r.rlrl :::ii.)


ili
.i

fc) from lhe graph.


when A is maximum, r- 0.5 cm.

WORKED EXAMPLE 3:
Wlen an object is thrown upwards fiom the top of a tower, the height, I metres, of the
object fiom the ground after I seconds is given by the formula ft = 30 + I0r - 51.
The table below shows some vallres of l and tbe coresponding values oflr.

(a) Complete the table above.


(b) Using a scale of 4 cm to rcpresent I unit on the t-axis and 2 cm to represent 5 units
on tbe t-axis, draw the graph ofl? = 30 + lOt 5t' for 0 < , < 4.

chaptef I 0: Graphs of Quadhtt. ***..


@
(c) Frcm your graph, find
(i) Ihe height of the tower,
(ii) the time, in seconds, when the object hits the ground,
(iii) when the object will be 25 m from the ground,
(iv) the maximum height of the object above the ground.

SOLUTION:
(a) ft=30+10t-51

(b)
lt' : :: i
,l 1l ll ri;ii,"r, lr
:, I

.l
-.1 -,1 r l :11 fi
,
:i;::
I ]. -l
i::r
,/ \
l I ll \ I
I

ll :.i ::: !r1 :i


;L I

'il it: .ii i' ir,


ri l
lir. I i

.]1 ]i
i l
i tl ::: l::tr
i ,,] l i li
il
i
l
.\ il , .ii L::r,
r.tl il rr::i i: i: .rri li
|)
il \ !.. l; [-l
) .::
5
i"l i' rl,
i.ti.
tl
t" 'i \ ,:lrl 1

iil
: ::::i:|.- --t ii
,i
i:i .
ll \ ir. l I

rio .5 3
.,:t i:l t, rli: :.: i: I : :]:
,,ll .. il: ; I .i .65
lr
i,.i' i I
f,,,
il.
ii:
: :l
i t :I
i: l't
t l i
'iil l,i

@ "**"*, r*- r"


(c) From the graph,
(i) when/=0,ft=30.
.. The height of the tower is 30 m.

(ii) whenr=0,r-3.65.
.. The time when the obiect hits the sround is 3.65 s.

(iii) whenft=25,t-2.4.
.. The object will be 25 m from the ground aller about 2.4 s.

(iv) the maximum height of the object above the ground is 35 m.

(E> WOP669 sx4t"aa o'


The variables ofr and ), are connected by the equation 1= I - 1- 1. 5ome coresponding
values of -r and ) are given in the table below.

(a) Calculate the values of a and ,.


(b) Using a scale of 2 cm to represent I unit on the i-axis and 1 cm to represent I unit
on the y axis, draw the graph ofy = *'z bt
Tfor-2<x<5.
(c) Use your graph to find the solutions of the equation
(t r':-1-l=0, (tt f-2x-1=3.
soLuTtoN:
iar d=(li 2tl) l=-2
b = Q\, 26) r-14

chrpkr rLr. craphs oreurrr'"" *""""^ @


(b)
i ii ,i ;: ii
::.:
riil
|: 1
ill r- .': rt: ri:
:.:: '' 1:: li
'ij i Il, ),L i
ri

i'a r;l l,:,i :l


t,
i., ir 1r al .- ri "
I i:
I

,t
ii /
i.r
li / ii,
L
l,; I:-i 'littl ll
\ rt 3l ,1, /, 'i.':

:.):. .fl t: i:i:


I:.
\ ,i ti,
ii I
ll,
?: i 1\ ''

*
]:::::]
I,a ]::]

l
i li:
'
(c) (i) To solve r' 2i i
I = 0, find the coordinates of the points \there the graph
cuts the r axis, (i.e. ) = 0).
The solutions are ,r = -0.4 or x * 2.4.

(ii) To solve.r'z 2'jr I = I, find the r-coordinaEs of the points of intersection of


thegraphy=l 2ri I andtheline)=3.
The solutions are r * 1.25 or x - 3.25.
i

I
li

t
@ "".n"**.. r*- ,u
t
Tutorial

Each of the equations in the table reprcsents one of the graphs from A to F. Write down the
letter of each graph in the corect place in the table.

Y=x'z-Lx 3

s-,"-l

tl
e, I

!
nof

chapter l0: craphs of Qu.dntic Equtiom


a9;)
2. The curve y = .r2 + 4x - 5 cuts the r-aiis a1 the points P and 0, and the y-axis at the point R.
(a) Write down the coordinates of the points P, O and R.
(b) Given that S is the minimum point of the curve, wdte down the coorahnates of.t.

3. The curve) = -l
-x + 6 cuts the t-axis at the points A and B, and the )-axis at the point C.
D is the maximum point. Wdie down the coordinates of A, B, C and D.

4. The diagram shows the graph of ] = t'


3t. The Sraph passes through the origin and ffosses
the r-axis at the point P.
(a) Find the cooordinates ofP.
(b) Write down the equation of the line of symmefy of the graph.
(c) Find the coordinates of the lowest point on the gmph.
(d) Find the coordinates of the point on the graph, other than the point (0, 0), where the
I
.x- and J coordinates are equal,

@**r*-tu
The diagram shows the graph of] = 2t'+ 3t 20.
(a) The graph cuts the )-axis at A(0, d). Write down the value of d.
(b) The graph cuts the r-axis at B(r, 0) and C(c, 0). Find the values of b and c.
(c.) write down the equation of the line of symmetry of the graph.

The curve y = (:r + 1)(i 3) cuts the i-axis at the points A and B and fhe ]'axis at C.
(a) Write down the coordinates of the points A, B and C.
(b) Find the equation of the line of symmetry of the culve.
l)
(c) The point D(5, lies on the cu e. Find lhe value ofd.

(a) A(1, /)
and B(r, 3) are points on the curve ]= 7 + 3r -t' Find the values of r and r'
(b) The graph y = al passes Ihrough the points (1,5) and (2, &) Find the values of dand,4-

chaptc, l0 crrph\ oi QdJnrJ,. *,",."'


@
8. The diagram shows the graph ofl = 5 + Lr - rz for 2<x<4
From the g{aph, find
(a) the values ofj|, when x = 2.7 and {.9,
(b) the values of.I when ] = 4,
(c) the greatesl value of )',
(d) the equation of the ljne of symmetry of the curve.

il
\
l
.5 2) 1l
/ rl "a\ ::: I
i:i I
tl
/t \: ii r:tl
'I
i,1. l
rl tl
I _l
I

ii ,rl ,\ i 'i':,rli
l
i l
\
rl
lt' : i ,\
l I

\ ]''r "..]
,l l

I \'
:i i
\
,tl ii

t il
1
L.:
t.
ii
:1
,'J il
I
rl
rt: ll
i .li
.il i '1., \
i ,ri rl
l'i
T.:I
i',] |,
i:l
].,-\
Llr
i
l
,i.
ii
lrl'l l ,,i

@ .o,n...u.'.** ,u
I

Thediagramshowsthegaphof)=r':+r-3ior 3 <x<2. Use the diagram to answer rhe I

following:
(a) Estimate the solutions of the equation r'? + r 3=0.
(b) Write down
(i) the smallest value of],
(ii) the equation of the line of synmetry of the curve.

Chapler lO: Craphs of Quadratl" 8q.,,1"", (A


10. (a) Complete the table of values for l = x- 4-r + 4.

I 0 I 2 3 5 6

J 0 I 9

(b) Usjng a scale of2 cm to fepresenl I unit on thei-txis and I cm to represent 1 unit on the
r, axis, dftrw thc gfaph of J= jt: 4t+4for I <r<6
(c) Frcm your graph, find
(i) the value ofl' when r = 3.5,
(ii) the value(t ofr when ) = 10,
(iii) the coordinates of the minimum poinl on the curve.
(d) Draw and libel the iine of symmetry oi the curve.

11. (a) Complete the table of values for li =-rr .t 5.

3 -2 I 0 2 -1 1
,7
v. -l

(b) Using a scale of2 cm to reprcsenl 1 unit on the.t-axis and 1 cm to represent I unit on the
), axis, draw the gmph of l=r'-t 5for 3<-r<'1.
(o From your graph, find
trl .he \ alue. ,,f \ u hcn .r - 1.2 irnd ).4.
(ii) the values of -t when I = 3.
(iii) the smallest value ()1'], and the con€sponding value of -t,
(iv) thc cquation of the line of symmetrf of the cu e.

12. (a) Complete the table ofvalues for-v= 2 2x it'L)t-4<x<2


1 ,2 t 0 I )
-l 2. _l

Using a scale of 2 cm to represent 1 unil on cacb axis, dlaw the graph of.| = 2 2r-r'
for-4<r<2.
(c) From your graph, find
(i) the valucs of,r when )'= 0,
(ii) the value ofl, when r = 1.5,
(iii) the equation of the line ol synxnetry.
U\.r rhe coofLhnates of he Ini\inturn loin..

@.o,n.*,'*r'-.o
(11) Complete fie table of values for ) = 2rr + 3-r - l.
3 2 I 0 2

1 2 t3

Using a scde ()1 2 cm to rcpresenl I unit on the-v'axis and I cln to fepresent I unit on the
-!r-llxis, draw the greph of) = Zr'?+ 3r I for 3 <.! < 2.
(o Draw rd label the line of symmetry of the curve.
Ffom your graph, find
(i) the value of ) when .r = L5.
(ii) the values ofi when ) = 5.
(iii) the smallest vatue of 1rr + 3r L

(a) Conplete the t.$le of vrlues fbr ) = (l r)(-r 3).

I 0 2 3 1 5

3 I

(b) Using a scale of 2 cm to fepresent I unit on each axis. draw the graph of
r = (l -r)(-r 3) for I <.r < 5.
(c) From youl gmph, find
(i) thc values oI i when r, = L,
(ii) thc value ot ) when -r = 4.5,
(iii) the grerlest value of and the corresponding value ofr,
"-
(iv) the equrtid of the line o[ symmetry o] the curve.

The v iables i and l, arc connectcd by thc equation ) = 2r' + 7.r + 4. Some conesponding
_r'
r alues o1'r nncl nre gjven in the following rable.

0.5 I 1.5 2 3 3.5 4 5

) 1 7 10 1 4 0

Calculate the vnlues ol d, and .. l?


ib) Using a scale of2 cm to repfesent I unit on the r-axis rnd I cm to rcprescnt I unit on thc
) axis, draw the graph of ) = -2r' + 7i +,1 fbr the range 0 < i < 5.
Drlrw on thc graph thc line of symmefty of the curve. Wfite down the equrlion o[ thc line
ot synmelry.
From the graph. find thc maximum value of 2r'+ 7.r + 4.
'd)

rhJf( n Cr.,th. fQurdrr,l Elu- -.


16. The variables n and ] are connected by the equation ) = zr'z 5:r 3. Some coffesponding
values ofr ard y are given in the table below.

(a) Calculate the values of d and ,.


(b) Using a scale oi 2 cm to represent I unit on the r-axis and 2 cm to represent 5 units on
the j,-axis, draw ihe graph of) = hr 5r- 3 for 2 <i< 5.
(c) From yow graph, find
(i) the value of) when r = 4.3,
(ii) the values ofr when J, = 8,
(iii) the smallest value ofJ and the corresponding value ofr.

17. The variables r and y are connected by the equarion y = 11Sr , ). Some coresponctmg
2
\alue\ ol y and dre
l, gr\en in lhe rable beloq.

(a) ofa and r.


Calculaie the values
(b) Using a scale of 2 cm to represent 1 unit on each axis, draw the graph of

'r, = 1(5.r
2' -r')
for -t <* < 6.
From your graph, find
(i) the value ofl, when i = ,1.7,
(ii) the values ofi when ) = 2.5,
aiii) lhe grealesr value of).
(iv) the equation of the line of symmetly of the curve.

18. The variables x and 1 are connected by the equation ] = (2r - 3)(5 - ir). Some corresponding
i
values of and ) are given in the table below.

(a) Calculate the values of d and &.


Using a scale of 2 cm to represent 1 unit on the -r-axis and 1 cm to represent 2 units on
the )-axis, draw the graph of ), = (2r
3.)(5 - -v) for 0 < r < 6.
(c) From your graph, find
(i) the value ofr when ] 10, =
(ii) the value ofl, when.x = 2.5.
(d) Draw and label on the graph, the line of symmefy of Lhe curve.

@,o".-.,.,r*-,u
19- The variables ir and y are connected by ahe equation ,! = l8 + 5.1 3r:. Some coresponaling
values of -rr and ! are siven in the table below.

(a) Calculate the values of a and ,.


(b) Using a scale of 2 cm to reprcsent 1 unit on the -t-axis and 2 cm to represent 5 units on
the l, axis, draw the glaph of ) = I 8 + 5ir
3x'for 3<r<4.
(c) Draw and label the line of symme0y of the curve.
(d) From your graph, find
(i) the value of) when -r = -2.5,
{ii) rhe \aluer olr when I - 15.

!O (a) Complete the lable of values for ) = Lrl + 1x 9.

(b) Using a scale ot' 2 cm to represent 1 unit on the r-axis and 2 cm to represent
5 units on the y-a\is, draw the graph of J = 2r': + 4r - 9 lbr -5 < j < 3.
(c) From your graph, find
(i) the .v -coordinates of the points on the curve where ) = 10,
(ii) th€ l, coordinate ofthe point on the curye where -y = 2.5,
(iii) the equation of the line of symmetry,
(iv) the solutions to the equation 2r']+ 4r - 9 = 0.

The table below shows the values of -r and J wbjch are connected by the equation
!=x' Lx 5.

-2 -l -0.5 0 2 3

3 0._25 2 b -3.75 ,2
(a) Calculale the values of d and ,.
(b) Using a scale of 2 cm to represent I unit on each a-\is, draw the graph of] = r2 21 5
for 2<,r<3.
(c) Write down the least value of ) and the corresponding value ofr when it occurs.
(d) Write down lhe equation of the line of symmetry.
(e) Given rhat the points (1.8, 4) and (p, 4) lie on the curve, find the values ofp and ,,.

chafrcr r0 cranh, or Quidra' *-*^


" @
22. (a) Dmw the graph of ) = r' - 3 for -3 <.x < 3
(b) State the equation of the ftne of symmefy of the curve
(c) Use your graph to solve r' 3 = 0.
23. (a) Draw the Braph of) = 3 - 5x x'for-6<x<1'
Use your graph to solve the equations
(i) 3 - 5-r r'= 0,
(ii) 3 - 5.r .r'= 5.

24. The total suface area, A of a solid cone is given by A = (2r2 - 3; + 5) cnf
(a) Draw the graph of A = 2x' - 3x + 5 lor 0 < x < 2.
(b) Fiom the graph, find
(i)the minimum total surface area of the cone,
i
(ii) the value of when tle total surface area is 6 crf.
25. The diagram shows a ffapezium,4-8CD wherc AD = (ir - 2) cm, BC = (11 - Zr) cm and
AE=G+2)cm.
(a) Show tlat the area, A ol the ffapezium is gilen by A = ( f"'+
22 I f, + 91 crn'

rbr Dra$ rhe srroh of e - -Lr+3lr+'lfor


)) 3<x<10.
(c) Use your graph to find the lenglhs ofAr, 8C and At that will rcsult in the maximumarea
of the trapezium.

11 u
26. A barbe,l wire fencing l0 m long is used to create a rectangular enclosure.

(a) If the width of the enclosure is i m, show that the area, A of the enclosure is given by
A = (sr x1 m'?.
(b.) DIaw the graph ofA = 5x - r' for 0 ( -r ! 5-
(c) Use your graph to find
(i) the values of r when the arca of the enclosure is 5 m',
(ii)the maximum area of the enclosure.

r*- ru
@ "",n".,,o,
A steel factory manufacturcs a range of cylindrical oil tanks. Each lank has the same height.
The volume, V m' of a tank with ladius, r metres is given by the fomula y = 4r.'.
(a) Complete the table of values of r from 0 to 4.

tDl Draw the graph of V= 4rr for 0 < / <,t using a scale of 2 cm to represent I unit on the
/-axjs and 2 cm to represeDt 10 units on the y-axis.
(c) Use your graph to estimate
(i) the volume of a tank with a radius ol 2.4 m,
(ii) the radius of a tank with a volume of 60 mr.
:8. Mr Lim orders chairs from a factory to sell. When he orders a batch ofi chairs, the cost ofeach
chair $) is given by ]' = i'1 l4x + 80.
(a) Draw the graph of] =; 14ir + 80 for 0 < i < 14.
(b) Froln your greph, iind
(i) the nunrber ot' chairs ir a batch Mr Lim needs to order so that the cost per chair is
mmlmum)
(ii) the number ofchairs in a balch he needs to order so that the cost per chair is less than
$'ls'

:9. During afestival, fireworks werc launched vefiically upwards at a speed of30 nr,/s. The hcight,
] metres, of the f-ireworks above the gmund after r
seconds is given by the formula
r= 30r 5r2.
(a) Draw the graph ofl= 30-r - 5-rt for 0 (.r< 6, using a scale of2 cm to represenf l uniI
on the.r' axis and 2 cm to represent 5 units on lhe }' L\is.
(b) Use your graph to find
(i) thc hcight reached by the fireworks after 2.5 seconds,
(ii) the sho(est time raken tbr the fireworks to reach a heighl of 20 m,
(iii) the ,rximum hcight reached by the fifeworks above thc grcund.
,1, (a) The per;meter of a rcctangle is 12 cm and its length is y cm. Show that the arca. A cm'
of the rectangle is given by A = (6r
,rr) cm:.
(b) Using a scale of 2 cm to reprcsent each axis, dlaw the gfaph ot' A = 6-r r'for0<-v<6.
(c) From your graph. find
(i) the area of the rectangle when r = 4.8 cm,
(ii) thc dimcnsions of the rectangle when its area is ,1 cln],
nb\ (iii) Ihe maximum area of tbe rectangle,
(i\,) the cofiesponding length and breadth of the rectangle whcn ils area is mr\i um.
(v) the shape of the rectangle when its area is maximlrm.

(hxfLcr l0 Ctuph.,fQudLlnin Et . iuni


@
3I, A shop sells between 5 and 13 bottles of wine daily. The daily profit, $J eamed from the sale
ofr bottles of wine is given by ) = -r'1+ 18r + 168.
(a) Draw the graph of)' = -l + 18'I + 168 for 5 <r < i3.
(b) Frcm your graph, find
(i) the number of bottles of wine the shop needs to sell to maximise its daily profit,
(ii) the daily maximum profit.
32. Marc dives off a diving board at a height of 3 m above the water His height, /? metres after
r seconds is given by l? = (3 + 2t 4f) m.
(a) Using a scale of 2 cm to represenl 0.2 unit on the t-axis ald 2 cm to represent I unit on
the ft a\is, draw the graph ol h=3 + 2t ,lf for 0 <. < 1.4
(b) From your graph, find lhe time when Marc touches the water.

ll. The diagram shows a rectnngular plot of land ABCD where AB = (3,v 2) m ,nd
BC = (5 - 2{) m. The shaded part of the plot of land, A of width r m is lo be cernente.l
(a) Find A, the area of the shaded part, giving your alswer in te.ms o[ r'
(b) DIaw the graph of the above equation for 0 < ,I < 3.
(c) From tbe grnph, find
(i) the value of .x for which the area of lhe cemented part, A is maximum,
(ii) the values ofx fot which the area of the cemenled parl, A is 3 mr.

52t

@ ""*'*..,,,", 'u
..s$o "t-
f'dJ",
:lt,

dtrt The equation of the line of symmehy of the cufle y = x' + pt + 2l is ,r = ,1.
"K
Find the value of

The diagram shows the curve with equation 1 = .rl + ,t + c. Find the values of d, , and c.

chaprer l0: Graphs of Quadra,'. **,t"", @


3. The table gives the n and )-coordinates of some points which lie on a cufle.

(a) Using a scale of2 cm to reprcsent 1 unit on each nxis, plot these poinls and dftw a smooth
curve through them.
(b) Write down thc equation of the line of symmetry.
(c) The points (4.2, ,) and (d, &) lie on the culve. Use your graph to find the values of.r and
b.
(d) The vatues ofr and ), are related by fhe equation J = Pr'+ 0r Use two points on the
curve to obtain a pair of simultaneous equations. Hence calculate the value ofP and the
value of q.

4. A zookeeper wants to build an enclosure having two similar rectangular stalls to keep
animals. He consrucm a bdck wall on one side and uses 150 metrcs of wire fencing to
construct the olher sides of the enclosure. Given that the width of each recta-ngle^ is .)
metres. show that the lotal area, A of the enclosurc is given by A = (1501 3x') m'.
(b) Using a scale of I cm to represent I unit on the .r- axis and 2 cm to rcpresent 10 units on
rhc A a\i\. drau lhe gfaph ofA - l50r h lor20'y'.to.
(c) Use your graph to find the maximum value ofthe total enclosed aJea and Ihe corresponding
dimensions of each stall.

tt*..",n, ,*- tu
@
Set Language and
Notation
oolh

I the
I the

@ s"t Notation
l. A set is a collcction of lvcll-defined objects-

2. Each object in the set is callcd an element or a member of the sel.

-t_ We use bnces, { }, to enclosc the clements of t sel. We use capital leltels. e g. A, B.
C, ... to label a sct and sma1l leuers. e.3. d, r, c. ... to dcnote the elements in a set.
rg Ia
is.r E.g. I The set of vowels in 1he English aiphnbel can be wrrtten as:
A = Id, e, i, (). ul
rs o:
E.g- 2 The set of pime numbers smaller than l0 can be wdttcn as:
rdin: B = {2.3.5.7}

E.g. 3 The set of the days of the week can be written as:
C = {Morday, Tuesday, Wedncsday, ThuNday, Friday, Saturday. Sund.ly}

cr,rorcr ser ri,a,,xrc.N! *. . ' fii'


\-
'.
I
To defme a set, we can

@ List the elements


@ Describe the elements
@ Use set builder notation

A= [2,3,5,11 A is the set of prime A={:r:xisaprime


numbers smaller than 10. number < 101

B=Im,a,t,c,hl B is the set of letters B={r:risaletter


in the word 'match'. in the word 'match'l
c= |t,2,3,4,5....1 C is the set of natural C= {J:risanatural
numbers, numDel

5. If r is an el€m€nt of set A, we write .t € A


If r is not an €lem€nt of set A, we write r e A.
e is an element of
is not an element of

E.g. I A={2,4,6,8}
2 is an element of A. We write 2 e A.
4 is an element ofA. We write 4 € A.
6 is an element ofA. We write 6 € A.
8 is an element ofA. We wriie 8 € A.
5 i\ nol an elemenl ofA. We wrile 5 e A.

F,.8.2 B = {'I : ,r is a rcot of (r lXx - 3) = 0l


(r-lx.v-3)=0
..,v l=0 or .r 3=0
t=I or x=3

1.-R 1G t2 )dR -dt2

@ "*..,.,,**,u
A set must be well-defin€d to avoid any ambiquity about whelher an object i\ or i\
not an element of a set.
E.S. I The set of positive iDtegers is a well-defined set.
(Given any number, $,e can easily tell if it is a posilive inlegel)

E.g. 2 The set of popular mathematics teachers is not n well-defined set.


(We cannot accurutely measure 'populirrily'.)

Thc order in which the elemenls are wfitten makcs no difcrcnce and each elcment
is listed only once.
E.g. A = ll-elteN iD the wo.d 'book'J
A= Ib.o,kl or A={o,r, ll or A=[b.k,o]
A se1 conlaining no elements is called an empty set or a null set- It is denoted by O
or { I.
Z or I j Empty set or Null sct

E.g. P = {All chickens with three Iegsl

O = lMonths with 33 dilysl


..Q=a
R= {r : r is a real number and .r2 = r J
..R-a

The number of elements in set A is denoted by a(A).


L.g. | 4- t. 1. r. il . Ileie ee r ete -en.. in..r 4.

E.g.2 B = {Letters in the wofd 'sphefes J


I - l\. r. r. e. rl . Rcmr
,tR) - 5

E.g.3 C= {i:ris a natural nulnber}


c= 11, 2, 3, 4. s, 6. 1, ...1
rln - in fi n i
E.g.1 D = {r : .r is an integer and 3r = 51

) <
.\=l- J
risnorrrireser
..D=a
n(D) = 0

Chrp{er l1: Sel Langurge lnd \.ratil.n ltl


(F WORKED EXAMPLE 1:
Wdte the following sets using set-builder notation.
(a) {21,22,23,...,991
(b) {8. 16,24,32, ...J
(c) lr,3.s,1,9,...1
SOLUTION:
(.a) {r : r is a natural number and 21 <,I < 99}
(b) {r : x is a multiple of $}
(c) {r : r is an odd natual number}

(R> WORKED EXAMPLE 2.

{x : r is the square of natulal numbers and -! < 50},


{r :-r is a positive integer less than lOJ and
Ix x = 2k + l, 3 < t < 7, t is an integerl,
i
(a) list the elements ofA, B and C,
(b) find r?(A), r(B) and r(C).

SOLUTION:
(a) A = |t,,21,3" 42, 5,, 6,,'7,1
A = { 1, 4, 9, 16,25,36, 49t

B = 1r, 2, 3, 4, 5, 6, 7, 8, 91

12(4\ + r.2(5) + 1, 2(6) + 1)


{9, 11, 131

(b) n(A) = 7
n(B) = 9
n(a=3
10. Two sets are equal if they contain exactly the same elements.
If two sets A and B arc equal, we w.i," fAl?l. H.r..uery element of A is ar
element of B.
If setA aloes not equal set B, we write I AJF].
E.E. I A= {1.2,3.4j
B= 14, t.2.31
Sets A and B are equal.A = E

E.E.2 {Letters in the word 'paper'}


{Letters in the word 'pear'l

i
Sets C and D are equal.

6) ""*."0*'*-,"
I
E.g.3 E= {x:x-l=21
F = {x: xz -9 = 0l
)t-r=2 l-9=0
x =3 x2=9
" =lnE
=3or-3
E= t3l
F= t3, 3]
Sets t and I' are not equal. E + F

E.g.4 G = Ic, a, t, catl


H=Ia,t,cl

Sets G and 11 are not equ'al. G + H

If every element of set A is also an element of set B, then set 4 is a subset of set B.
w" u"" F e?l to d"rote A is a subset of B.

If set A is trot a subset of set B, we write FEFI


E.s.1 A= [1,2]
B = {1,2,3}
A e B since every element ofA is also an element ofB.
I q 4 since nor every elemenr of B i. an elemenl ol A.
E.e.2 A = [a, b, cl
n= {c, a. b\
A e B since every element ofA is also an element ofB.
I E A since every element ofB is also an element ofA.

Cbapter ll:Set Language and


".tr*" @
12. If every element of set A is also an element of set B, but set B has more elements
setA, then set A is a proper subset of set B. We wdte A C B to denote A is a
subset ofB. If setA is not a proper subset of set B, we write, A G B.

E.e. 1 A=[p,q,il
B=Iq,r,pl
C=lp,q,r,s.tl
A C C since every eiement ofA is also an element of C but C has at least
element that is not in ,4.
Similarly, we have B C C.
Since each element ofA is also an element of B and vice versa, we
A E B and B E A. Therefbrc A = B.
C is a subset of
is not a subset of
c is a proper subset of

e is not a prcper subset of

(]w> YlgPKgP EXAMPLE 3:


P= Ia.bl Q=Ia,b,dl R=tb,al s= Ib,eJ
Use e, g, C, q or = to describe the rclation between the following sets.
(a) PandQ (b) PandR (c) Sands

soLuTroN:
(^) PCQ

(b) Since P e R andR e P, wehaveP=R.

(c) sgo

i
I

6i) ""*"*o*^..r"
WORKED EXAMPLE 4r

(a) Ljst the subsets of


(i) {p}, (n) {l,21.
(b) Find the number of subsers of {d, ,, c}.
soLuTtoN:
(a) (i) The subsets of {Z} are O and {pl.
(ii) The subsets of { 1, 2l are A, {1}, {2 } and {1,2}.
(D,) a Ia, bj [a, b, cj

Ibj lb, cj
tcl
.. {a, b, c} has 8 subsets.

13. The Universalset, denoted by €is the set that contains all elements being considered
in a given discussion.
€ | uni\enal set

E.g. t A = {Letters in the word 'mathematics'}


B = {Irtters in the word 'statistics'}

... A = lm, a, t, h, e, i, c, sj
B = Is, t, t1, i, cl
The universal set of the sets A and B could be the set of lette$ of the English
alphabet.

.. t = \a, b, c, d. e, f, g, h, i, j, k, I, n, n, o, p. q, r., r, r. L, r, w, .x, r. ..1

E.g.3 The unive.sal set ofA = { apples, pexrs, orarges} could be E = {All fruits }.

E.g. 4 The universal set of B = {3, 6, 9. 12J could be € = {.r : i is a multiple of 3 }.

choDFr L: s.r Lrnsuase


- and Lr)
^"r,,." \- _/
@ Venn olagrams and Complement of a Set
t. We can also use a Venn diagram to express a set.

2. In a Venn diagram,
. a large rectangle is used to represent the universal set, €.
. circles or ovals are drawn inside the rectangle to represent the subsets of €.

(@ WORKED EXAMPLE l:
GiventhatE={1,2,3,4,5,6.7,81,A=12,4.5landB={1,2,5,71,&awaVenndiagrao
to represent the sets,

soLuTtoN:

(@ WORKED EXAMPLE 2:
Given that €= {p,4, r, s,l, rrl, C = {p, t, &l and D = { r", s}, draw a Venn diagram to represe(
the sets.

soLuTtoN:

t'.)
3. Sets C and D in the example above are called disjoint sets.
Iftwo sets have Ilo elements in common, then ihe two sets are called disjoint sets.

E.g. I IfA = { 1, 3, 5, 7l and B = {2, 4, 6, 81, tlen,4 and B are disjoint sets.

F,.9.2 If H = {Months of the year starting with the lefter '"f I and

tr = { Months of the year starting with the letter 'M' L then


11 and tra are disjoint sets.

,"'. ,"
@ ",*"'"0*
WORKED EXAMPLE 3:
Giventhat E= {5, 10, 15,20,25 J, P = {5, 10} and Q = { 5, r0, 15,201, draw a Venn diagram
to represent the sets.

SOLUTIONT

4. The complement of set A, written A' is the set of all elements in the univemal set €
that are not ir,4. We read A' as 'A complement' or A prime'

A'= {x:i e € ardreAl

E.g l {3, 4, 5, 7, 9l and A = {4, 7, 9}, find A'

A,=t3,5i.-jw

chapter rr, set Lansuaee and *"."


@
lrl W(n|G[' EIAIPLE 4:
$e=Lrr,12,13,74,15;76,11,18], A = {12, 14, 16, 18} and E = { ll, 13, 15, l7},
(a) draw a Venn diagam to reprcsent the sets,
(b, list the elemenr of 4'and B.

soLuTtoN:
(AJ €

@
(b) A' = { 11, 13, 15, t7l
B' = 112, t4, t6, tqj

@ unioti anil Intersection of Sets


t. The union of two sets, ,4 and B is the set of elements which are in A or in , or in both
A and B. It is denoted by A u B.

AUB= {r:r€A orr € B}

uI the union of

WOBKED EXAMPLE 1:
tf € = {t,2,3,4, 5,6,'1, 8}, A = {1, 4, 7l and B = i l, 2, 5, 71, find
(a) Ar,B, (b) ,?(A u B).

SOLUTION:
(a) ,{uB= U,2,4,5,'7J,
(b) n(AuB)=s

6i) '"r"."ri",
ruo. zn
WORKED EXAMPLE 2:
Shade the following regions in the Venn diagram.
(a) A uB (b) (A \J B)' (c) Aw B'
(d) A'uB (e) A'e B'

SOLUTION:
(a) AUB (b)

-x
(A u B)'

Y
il n*# A

(c) Ae B'
Teacher's fips:
Step @: Shade the region A.

Slep @: Shade the regioD B'.

The required region A u B' includes


any shaded parts.

Chaoter I l: Set Lanauase tud


- N",".". f;)
\_/
A'eB

Step @: Shade the region A'.

Step @: Shade the region B.

The required region A' Lr B includes


any shaded par1s.

A'w B'

Step @: Shade the region A'.

Step @: Shade the region B'.

The required rcgion A' u B' includes


any shaded Parts.

i
t (D ",,**o*'*-'u
WORKED EXAi'PLE 3:
lf e = [a, b, c, d, e,f, s\, A = lb, e, dl at\d. B = Ia, d, e, gl, ftnd
(n\ A', (b) B" (c) (A u B)', (d) A'u B'.

SOLUTION:
(a) A'=[a,c,f,stffi
o) B' = [b, c, fl

,ff'
(c)

(Aw B)' = lc,fl


(d) e

A'w B, = Ia, b, c,f, gl

Alt€rnative method:
A' e B'= la, c.f, gl \J lb, c,fl
= Ia, b, c,f, gl

The intersection of two sets, A and B is the set of elements which are common to
both A and B. It is denoted by A B.
^
A^B={x:r€Aandt€Bl

the intersection of
^

Chapter ll: Sel Language and|.I",",t."


@
WORKED EXAMPLE 4:
If € = {2, 4, 6, 8, 10, l2l, A = 14,8, loJ and B = {2, 6, 81, find
(a) A^4, (b) ,(A B).
^
soLuTtoN:
(a) Ar-\B= {8}
(b) n(A r-\ B) = 1
10

WORKED EXAMPLE 5:
Given that € = {r : r is a natuml number smaller than 9}, find A n B in each of
following.
(a) A = {1, 2, 3} and B = {4, 5J.
(b) A = {5,7} and B = {1,5,7J.
SOLUTION:
I
e= {1,2,3,4,5,6,7,8l
(a) At.'B=Q /-]\
(+ )

\_-/

(b) A^B= {5,7J

@ I
234

(,t] **-** r*- *


WORKED EXAMPLE 6:
If€= {8,9, 10, 11, 12, 13, 14, 15, 16}, A = {8, r0, 12, 14, 161, B = {11, 12, 13, 14} and
C= {9, 11, 13, 15}, find
(a) At.'B, (b) n(A^B), (c) A^C, (d) /r(A . C),
(e\ A' (D A^B', (s) A'u B, (h) (A' 0 B)'.
^8,
SOLUTION:
(a) AaB=|2,141
(b\ n(A n B\ =2
(c\ A. C=O
(d) n(A^C)=0
(e) A'^B= 11, 13, 15}
{9, {11, 12, 13, l4J
= {11, 13i ^
(, A .\ B' = {8, 10, 12, 14, 16} {8, 9, 10, 15, 16}
= {8, 10, 161
^
(g) A'v B= {9, lt, 13, 15} u {11, 12, 13, 14}
= {9, 11, 12, 13, 14, 15}

(h) A'uB'={9, 11, 13,15}u {8,9, 10, 15, 16}


= {8,9, 10, 1r, 13, 15, 161
(A' o B')' = {12, t4l

WORKED EXAMPLE 7:
If e = la, b, c, d., e. f. gj, A = la, d, fL znd. B = lb, d, e, fL rtnd
(a) (A^B)', (b) A (c) A' B, (d) A' B'.
^B', ^ ^
soLuT|0N:
(a) (A B)' = la, b, c, e, sl €
^

(b) A^B'={a}

Chapr€r 1l: Sel Language and *r*"


@
(c) A'. A = {l', e}

A'. A'= 1..8]

3. The table bel(nv is e sunmluy of set language and notalion.


e is not an element of
C is a subset of
g is not a subset of
C is e proper subset of
q is not a proper subset of
number oi elements in set A
€ universal set
Aotll emply set of null set
complement of sel A
A\JB union of ,4 md ,B

A.B intenection of A md I
1. The list of somc rcal numbers are given below.
Integ€rs: ..., 3, 2, 1,0, 1,2, 3,...
Negative integers: -1, 2. 3,1, 5, ...
Positive integ€rs: 1, 2, 3, ,t, 5, ...
Whofe numbe.s: 0, 1,2,3, 4, 5, ...
Natural numb€rsr 1, 2. 3, 4. 5, ...

A prime numbe. is a natural number that has exactly 2 different factors, 1 and itself.
Prime numbers: 2, 3, 5, 7. I l, 13, l7, 19,23, ...

A conposite number is a natural number that has more than 2 diflerenl factors.
Composite numbers: 4, 6, 8, 9, 10, 12, ...

A rational number can be written in the fornr


f,
where a and b are intcgers and , + 0.
l'tl
E.s. -l -. --.8.9- are ratronal n mbcrs.

,o*.-.'* r,-.u
@
The shaded regions represent each of the following sets.

(A U B), (A B)'
^

Y x
N- n A
A' 'J B A'^B

A^B'

A'r B' A'^ B'

Chalter ll: Set Languge and *"*t


@
Tutorial

1. (D List the elements ofA.


(ii) Find r(A).

i (a) A = {Letters in the word 'strategy'l tol {Days of the week stafting wilh
the letter 'T'l

(d) A = {x : .{ is a positive odd number


smaller than 101

(e) A = {.r : jr is a prime number (f) A= {n : r is a factor of I 8 }


smaller than 20]

\x : r
= 2k + 1, i is a positjve
integer < 5]

O A = {x rr is a nalural number and


3t- 1<91

(k) A = {.r : x is a root of the equation A = Ix : x = a2 + b', a and b l.re


x' 1=01 natural numbers, I <d<4and
1 < & < 5]

@ "^*.-*.,,*-,u
Write the following using set-builder notation.

(a) A = [2,3,5,1,11, t3, r'7, . (b) B= {t,4,9, t6,251

(c) c = {s, 6, 7, 8, 9} (d) D = {4, 8, t2, 16,20,.

(e) E= [1,2,3,4, s] (f) F= lr,3. s,'7,91

G) G= {a, b, c,d, e,f, ...1 O) A = {January, June, July}

Tick (/) the following sets which are well-defined.


Cross (f ) the following sets which are not well-defined
(a) The set of great novels.
l,- .

(b) The set of pdme numbers greater than 30i


i.. -
(c) The set of smart Mathematics teacher!. I
:

(d) The set of subsets of {d, Dl.

(e) The set {x:r+ 1=0 andiis a natural number}.


t -'*- j l

chapter 11: set Langlaee stl.*"n* (tD


L Tick (/
) fhe pairs of sels that are equal.
Cross (X ) fhe paiN ot' sets that are not equal.

(a) A = {Letters in the word 'algebra'l (b) ?4 = { Letters in the word 'celebrate' }
, = {LetteIS in the word 'beagle'l B = ll-ettels in the word 'bracelet'l

I I

--f- --
(c) A = {Factols of 20J {x : -r is a root ofr': 9 = 0J
B = U,2, 4, 5,20]l
I
{i : -r is an integer and
Lr+1='7]l
i

,j.--,-
(e) A = {1,2,3}u {4, s} I(l) A=la.d,e)^lb,cl
B = {.r : iv is a natural number I B = {,r : r is an integer and
smaller than 6l sr 1=31

5. Which ol the following represents equal sets?

B = [ab, c, d]
C=A
D=TAJ
E= {0}
G = iir : r is a prime number smaller than 10J
lJ = {n : r is a positive integer grealer thar..,
r= l'7,2,5,3t
J = {i : -r is a whole number grcater or equal to 5l

@
A = ipen, pencil, stapler, eraserl
(a) Find n04).
(b) Is ruler € A?
rate'l (c) Is {stapler} € A?
lef l (d) Suggest a universal set lor Set A.
(e) Write down the ser B such rhat B C ,4 ard n(B) 3 and pen
= e B.

P = {Letters in the word .apple'l


0 = {Letters in the word ,leap'J
R = {Letters in the word ,peel,}
(a) List the elements ofp.
(b) Find r(n).
(c) Write down the relation between the sers.
(l\ P and, Q (ii) I and R

FilI in the boxes with € or e to make each statement true.

@) 2) i
{2. s,1t $) o, la
(c) ant j U, n, ,t rai rrr i
'l
lrr.:. sr

Chapter II: set Lansnage and r*,,t


G,)
9. Fill in the boxes with e or C to make each statement ffue'

at ,1, -1,o,r,
o, ul ro t,rrl' jt", r,.r

@ ,1,, . ){apple,
pear, oranse} (d) {s, 18} L ., lftt, tf

10. If A = ic, d, t], state whether each of the following is True (T) or False (F)'
i_ - I--'.
(a) c€A I I (b) {a'rl cA i

G) OEA itr -.-, l


(d) {c}cA i ,

@) lai=o I
---
j
(l AeA I

G) [c,a,t]c tr,r,.t I I

(T) or False
II . If B = { Letters in the word 'statistics' I ' state whether each of the following is True
(F).

(^) la, ci C B I
(b)

@, ACB i' l (d) n16y = 10

(e) {d, t, t, c, si C B il

,"
@ ""*.-",", '*-
P= {.rr: r is arootofl I =0}
o= \rJ
R= {0, l,2}
s= t 1, 1l
Use C, C,
e or G to describe the relation between the following sets.
(a) PndQ (b) PandR
(c) O and R (d) PandS

A = {x : .I is a positive odd integer < 6}


B = {'' :.x is a rcot of (r - lxr- 5) = 0}
C = {.I : -r is a natural number < 6l
, = tr,5l
(a) Find
(i) n(A), (ii) n(B), (jii) (c).
(b) Describe the relation between the following sets using C' = or +
(i) A and B (ii)
A and C (iii) AandD

Chapter ll: set Lan$age and t",",t""


@
14. State whether the following slatements are True (T) or False (F).

h) ae [b,a,tl
(b) 8€i7,8,11]

(c) mile e ls, m, i,I, el

(d) 9 + {1,3,5,7}

(e) 2e lx : r is a prime numberJ

(f) {i,., e} € {Letter in the word 'isosceles'l

(g) 5 € {r : i is the root of (i + 5)(r 6) = 0l

(h) (0.25 x 16) € {r : i is a factor of 12}

(i) {31 € {,I : ,r is a rcot of .r':- 9 = 0J

0) O e {,r:iis a perfect square}

(k) {8,91 c {6, 8, 9, 12}

(1) Z C {}:ris a rational numberl

(m) [p. q, r] Q lq, r, p]

(n) {busJ e Is, u, b, m. a, r, i, n, el

(o) {r} € {?, a, {rl, {s}}

(p) {Fridayl ( iDays of the week}

(q) {71 E {Factors of lzlJ

@,*.,,*,,,.,,u
List all the subsefs of the following sers.

{ant, bee} (d) {1, 2, 3}

A={4,6,81
(a) List all the subsets ofA.
(b) Write down the number of proper subsets ofA.

Given that A is a proper subset of B.


If n(A) = 50, find fte maximlrm number of elements ofA.
(b) Given that C is a subset of D and D is a subset of C.
(i) If n(c) = 20, find
(ii) What other rclation"(D).
exists between C and D?

chapler lrr ser Lansuage lnd ***


@
18. Let € = {I,ettels in the word 'mathematics'} and A = {l,etters in the word 'statistics'}.
(a) List the elements
(i) €, (ii) ,4, (iii) A'.
(b) Draw a Vem diagmm to represent €, A and A'.
(c) Find
(i) (€), (ii) r(A), (iii) ,(A').

19. Let e = {strawberry, apricot, guava, orange, plum}, A = {smwbery, plum} and B = {guava
(a) Draw a Venn diagam to represent the sets above.
O) Lisl lhe elemenls of
(i) A" (ii) R.
(c) Find the relation between A and B.

a= {r:.1 is a positive integer smaller than 16}, A = {x:xis a multiple of 5]


B = {r : r is a factor of 18 }.
(a) List the elements of
I
(i) €, (iD A, (iii) B, (iv) A', (v) B'.
(b) Find
(i) (ii) (iii) n(B').
"(€), "(A),

(,D ***..ru
{-r : r is a positive integer less than 10}
(i) Draw a Venn diagmm for each of the following sets.
(ii) Find A u B andA r'r B.

A = 1r,2,3, 4, 5l (b) A= {1,3,5,7j


B = ].2,4,6,8l B = 12,4,6,8l

(c) A= {r : r is an odd numberl (d) A = {"r : r is a factor of 6}


B= {r : r is a prime numberl B={r:risafactorof2l

(e) A tx : r is a multiple of 4l (t) A= {r: ris a root of.r' 4 = 0}


B {x : .r is a factor of 10} B= {r : r is an even number}

Chapter ll: Set Language dd ***


@
For each of the following, fird A u B and A B.
^
(a) A={7,8,9, l0} (b) A = Ia, b, c, d, el
B = {9, 11, 131 n = {c.f. d, cl

':

(c) A={6, 12} (d) A=Ip,q,t,xl


{3.6,9,12, r5l
.
B= 6 = 1v,w, z)

:
(e) A = {pencil, pen, eraser} (f) A = {Vowels in the word 'balloon'}
i B = {rulel, eraset stapler} B = {Vowels in the word 'laddle'}

t (g) A = {-r i is a factor of (h) A = {,t, e, )}


6 ]
B={.I x is a factor of 5 l B=A

(i) A = {Letters in the word 'lasagne' } (j) A = {r : t is a root of r' 16=0}


d = {Letters in the word 'noodle'l B = {a : r is a factor of 201

@
IfA = {4, 6, 8, 101, B = {3, 6, 9, 11} and C = {4, 8, 12, l6i, find

(b) AUC

(d) A.B

n(A C)
^

E= {r : r is a positive integerJ
A = {r : r is a multiple of 5}
B = {,! : x is a multiple of 2J
(a) List the elements of
(i) e, (ii) A, (iii) B
O) Find A
(c) Desoibe^8.
A B using set builder notarion.
^

chaDkr ll: ser Lmluue lnd 6;


".'.rr". \'--l
r.
i.
25. € = {r : r is a positive integer less than 50}
A = {r : r is divisible by 4}
B = {r : r leaves a remainder of 2 when divided by 14}
(a) List the elements of
(i) €, (ii) A, (iii) B.
(b) Find
(i) A^8, (ii) r-\.8).
'l(.4

26. €= {r : r is a real number}


A = {r : jr is a prime number}
B = {r : .x is a composite number}
(a) List the eloments of,A and B.
(b) Find
(i) A^8, (ri) n(A B).
(c) ^
Draw a Venn diagam to rcpresent the sets €, A and B.

i
(D ***o*'*-,u
A= Ir,2,3,4,5,6,',l,8\
B = 13.5,'/,9, rrl
(a) Find
(i) A^8, (ii) AuB, (iii) n(A B\, (iv) ,(A \JB).
(b) Srate whether True (T) or False (F):
^
(i) 6 €A i- t] (it 8e(AuB)
(iit a cA
l (iv) {s,6,7lcA

A = {-r : r is a natural number < 11 J


d= {x : i is an even number and 1<'r < 21}
C = {i : r is an integer and 15 < r < 25}
(a) Find
(i) A^8. (ji) B C, (iii) ,4 r-\ C,
(iv) ,(A r-\a) ^
(v) (C), (vi) (A n C).
(b) State whelher True (T) or False (F):

ai) ieB (ii) 16eAuB f-'


[_']
(iii) lt(B c) =3 a l (iv) {15, 20, 251 c c i-*'-
^

e= {1, 2, 3, 4, 5, 6, 1, 8, 9, l0l
,4= {Multiples of 2}
A = {Multiples of 3J
C = {Multiples of 4}
(a) List the elements of
(i) A, (1t') B, (iii) C, (iv) A', (:v) B', (vi) C'.
(b) Find
(i) AuB, (ii) BuC, (iii) A . B,
(iv) A^C, (v) B^C, (\l) A'c\B',
(vil) A' e C.

CnaFer ll: Ser Ldglage md|i"*,,""


@
Use the Venn diagram to answer the following questioll.

ta)
"'*m

Find Find
(i) n(Al (it) n(B'\, (1.) n(e), (ii) n(C'),
(iii) A u a, (iv) A B. (iiD n(C e D). (iv) C^D.
^

(c)

Find Find
(i) n('' 4, fi) n(.E e n', (D c" (ii) n(G'J Ir'.
(iii) (E ^4', (iv) E u P, (iii) G' H, (i\) Gv H,
(,",) E ^ It. ^
(v) G'e .
^
State whether true or false: State whether ffue or false:
(vi) 2 € F (vii) 3 e (Eu.iq), (vi) 5CH' $n)2@G^H,
(viii) {4,7l c E (viii) { 1,2,,t, 8} c G

@ *,*".*".r**,"
€= {-r : ris an integerand 1 <']r< 16J
A = {.x :.x is a prime number}
B = {r : x is a factor of 15}
Find
(a) AuB, (b) A^8, (c) A'^ B, (d) A^B'.

l
A=I@,y)t (r, J) liesonthelinel= tt+ lJ
B = {(r, }) : (r, y) ties on the line y = ar + r}
State the possible values of lr and, if
(a) A=8. (b\ A^B=4.

P = {Quadrilatemls with 4 dght angles}


0 = {Quadrilatenls with 4 equal sidesl
R = {Quadrilaterals with 2 pairs of pamllel sides}
(a) LisI the elements of
It P, (.it) Q, (iii) R.
(b) Suggest a universal set for these sets.
(c) Find
(t) P^Q, (ij) I n R.
(d) Describe rhe relation between P and n.

Chapter I l: Set Language md ."t,t"" (+


e = {r:r is a natual number smaller than 81, C = {2,41 ad D = lt.2'4'61.
(a) Draw a Venn diagmm to represent the given sets.
(b) Find
(D (C D)" (ii) n(c \-r D).
^
(c) If CcD?

35. e= {-x:ris a whole number and 3 <.t< 13}


A = {r : -r is a factor of 301
B = {'I : ,r is a not factor of 241
(a) Find
(i) AuB, (ii) Ana, (iii) B" (iv) n(A \,) B)' , (v) n(A'
^.8).
(b) State whether ffue or false:
(i) 12 € B (ii.) 13 e A' (iiD AcB
(iv) {101 cAl-rB ('/\ A^B+A (vi) AUB=€

36. € = lThe lefters of the English alphabetl


A = {Letters in the word'Aus[alia']
B = {Letters in the word'Malaysia'}
C = {Lettels in the word 'Thailand'}
Find
(a) A^8, Q) B.C, (c) A^C,
(d) (A v (A C), (e) n(A^B)' (1) n(B C)'.
^B) ^ ^

,.- r"
@ "."".*",
n e= U,2,3,4,5,61, A = {2,4,6l and B = {1,3,6},
find
(a) A', (b) B" (.c) (A')', (dt Ae B',
(e) A'^ B, (f) BwB" G) B^B'.

If€={5,6,7,8,9, l0l, A = {5, 7, 8} and B = {5, 7, 9, l0l,


find
(a) (A u B)', O) (A r-\ B)', (c) A'w B', (d) A' B'.
^

Ir is given that e = {b,c,d,e,f,gl,M=[b,d,f]andN=[c,d,e,fl.


(a) Find r(M A).
O) ^
List the elements of
(i) (M u 1O', (ii) M'r'\ N, (iii) A4: w N.

fie={2,3,4,5,6,7.8,9, 10, ll, 12 }, R = {.r : r is a factor of 18}andS= {.x:5t 1 > 34},


trDd
(a) R, (b) s, (c) F r-\ S, (d) R u s',
(e) n(R'. S'), (0 /r(R' \J Y).

Chaple.ll: Serl, suge dd *.rt""


@
€= {Letters in the word 'lrigonomehy'}
A = {Letters in the word 'geometry }
B = {Letters in the word 'time'}
(a) List the elements of
ti, rA Br'. rii,4'r8.
' riii) (A \r B)', (iv) AUB'.
(b) Find
(i.) ,r(A' B'), (iD n(A' 'J B').
^

42. €= {t:-risapositive integerandt< l5l, A = {x:4<i< 14}.B cA andB= {t: ! is a pnrne


numberl.
List the elements of
(a) A', (b) B" (c) A^8, (d) A'ua,
(e) {1, 3, 7} r-] B.

(a) On the Venn diagram below, shade the set A' a B

6= {7, 8,9, 10, 11, 12, 13, 14, l5l


P = {x : r is a multiPle of 3l
O = ir : r is an odd number]
R={r:9<ir<l3l
(i) Find n(P u 8).
(ii) List the element ofR'
(iii) Find the element x such that n € (C R') and re P'
^

to,n"."*. ..'* ru
@
A= {squaresl B={rho
B = {rhombuses } C = { parallelograms } D = {rcctangle\}
Simplify each of the followirg.
(a) AUB (b) C\-/D (c) A^B (d) A^,

X= Ib,c,d\ Y= {a,b,c,d,el
(a) Find the least elements in Z such rhat X u Z = y.
(b) Find the least elements in Z sidch that Y Z = X.
^

Given the Venn diagnm below, state which of the following statements are True (T) or Falsc (F)_

oo
(a\ ACB I
(b),,CA
(c) B CA'
L] @) acA
ti
(e\ A^B=O (t) A^B'=A
(g\ A'^B=A' -_r (.h\ AeB'=B'

chaprer li: ser rnguage add ***


@r
In the Venn diagram, e = {A1l triangles], 1= {Isosceles trianglesl and R = {Right-angld
ffianglesl.

(a) Add the set E = {Equilateml tria.ngles} onto the Venn diagram.
(b) A triangle has sides 5 cm, I 2 cm and 1 3 cm. On the Venn diagmm, mark and label a poin
P to reDresent the triansle.

48. (a) If r(A) = 8, n(B) = 5 ^nd A =B A, find z(A u B).


(b) lf n(q = 6, n(D) = 10 and C^ c ,, find
(i) ,(C^r), (ii) u(C u r).

49- Two sets P and C are such that P C O C E.


(a) Draw a Venn diagram and shade the region representing P O.
(b) Write down the set that is equal to P 0. ^
^ P u 8.
(c) Draw a Venll diagmm and shade the region
(d) Write down the set that is equal to P v O.

(,D r"*..***.,t"
State whether the following statements are True (T) or False (F).

(a) IfA = {Natural numbers} and B = {Positive inregers}, then A = B.

(b) ff A = {Prime numbels} and B = {Even integers}, then A B = U. -


^ r -,- l
|"
(c) IfA = { 1, 3, 5}, then lol e A.
I

(d) If 1 € Zand I € R, then Z= R. l


-..1

(e) ffp € A andA E B, thenp € B.


il
tl I

-
(0 IfD C EandE'e F,thenD C F. f*-'. I

(g) IfA = B, then A e ,8.


lt
l
*l
(h) IfA C B, thenA = B. I

(i) If€ = {1,2,3,4} andA = {1,31, thenA'= {2,4}. i

C) IfACB,thenAr,B=8. i
ll
l

"__--l
(k) lfA C BandB^ Q=O,thenA Q= \ t.
^
(l) If A . B = O, then A u B= A.

q I]
(m) If A _C B and B A, thenA = B.

(n) IfA tlenA uB= B.


^B=A,

Ldguage ad Nobrion
Chapter I I : Set
@
L

51. Shade the required regions.

(b) (c)

A\J B A^B

(d) (e) (1)

(())l I t] ll
l\x-ll A'aB (A rr B)'
\-_){_-/

AwB'
|

(h)

I
(A B)' A'vB' A'.8'
^
0) (k) 0)

l((l) tt/|
\)<-4
(A^B)QA' (A^B\'^B
\x-ll
(AuB)^(A^B)'

@
Shade the required regions.

(b)

AQ B A^B @^B)'

(d)

;t

I A'e B
@
A^B'
oo A.\B'

B ABC ABC

A: (AuB)^C (A.B)uC
^C

Blr (A^B)w(B^C) (AeB\'^C (B^C)'^A'

Cbaprer ll:Sel]-anguage and *"*r


@
53- Complete the Venn diagrams below.
A and B are subsets of the universal set €.

(b) A., B =O

54. Complete the Venn diagams below.


A, B and C are subsets of the universal set €.

(a) A^B=A,andAwC=A (b) B c A, n(A O =0 and C.A = c


^

(c) BCA,B^C=@,CqA (d) (AuC) CB,CQAa\d


and'(A^O+0 Q4.^C)+o

@
Describe the shaded regions.

(al (b) (c)

(d) (e.l (f

Y
N--K r* A

t) (k 0)

chapte. I1: ser Laneuage and *.t"r'"" (A


l. Shade the required regions.

(AuB)^C (AUC).(B^C)

---'
(AnB)uc i
B,^C'A
i

(a.)
(b) €

J. I i.t allthe .ubser\ ol q-la.b.'.ll

@ ""*".',o, t*- '"


lf n(A\ = 20 a\d n(B) - 8, find the greatest and least possible values of
(a) n(A r-\ a), (b) n(A u B).

IfM*lr'denotes(Mul/)^(MnM'ande={x:xisapositiveinteger},lisrtheelements
of { 1, 3, 4} * {3, .1, 5, 6}.

A suflery was conducted to find whether a group of pupils like apples, grapes or pears- The
rcsults are given in the table below.
A = {Pupils who like apples}
c = {Pupils who like grapes}
P = {Pupils who like pears}
(a) Express in set notalion using the sets A, G, P.
(i) Pupils who like apples but not grapes.
(ii) Pupils who like grapes but not both apples and pears.
(iii) Pupils who only like two of lhe three fruits.
(b) Given that 8 pupils like only apples and the number of pupils who like only grapes is
equal to the number of pupils who like only pears, calculate the nllmber of pupils who
like all three fiuits.
nlA) t4
n(G) t9
n(.P) 20
ntA n C)
n(G . P)
,1

Chapter ll: Set LangMge and


^",",t* @
Graphs of Quadratic Equations
Set Language and Nolation

L The diagram shows a sketch of the curve ) = (5 - nxl +,). Find


(a) the coordinates ofA. B and C,
(b) the equation of the line of symmetry,
(c) the value of p, if the range of rhe graph is ) < p.

2. The diagram shows the graph of y = I + r1r + D.


Find
(a) the equation of the line of syrnmetry of the culve,
(b) the values of d and ,,
(c) the coordinates of the minimum point of the curve,
(d) the range of values of i for which the gradient of the
curve is positive.

,"'.,u
@ ""*"-",,",
The variables 't and) are comected by the equation ] = {j - 4{ + 3. Some conesponding valDes
of r and ) are siven in the lable below.

(a) Find the values of d alrd b.


(b) Using a scale of 2 crn 1() rcpresent I unif on each axis, draw the gmph of ] = t' 4r + 3'
(c) From your graph, find
(i) the value of l when r = 0.8,
(ii) the values ofr when )'= 2,
(iii) thc coordinates of the minimum point on the curve.
(iv) the equ.rtion of the line of symmetry

The variables r and ) are connected by the equation l' = Zr'1 5;r + 1 and some conesponding
values ofr and ) are Siven in the table below

2 t I 2 _l 4

19 8 l I 1

(a) Find the values of d and ,.


(b) Using a scale of 2 cm to rcpresenl 1 unit on the x-axis and 1 cm to represent I unit on the
],-axis, draw the graph of ] = 1'- 5r + 1 for -2 < t < 4.
(c) From your graph, find
(i) lhe value(s) oft when ) = 10,
(ii) the value(s) of)
when.r = 2 6,
(iii) the minimum point,
(iv) the equation of the line of symmetry of the curve.

The table below gives some values of r and the corresponding values of )' where
l=2-3x x].
1 2 I 0 1 1.5

2 2 1 2

(a) Calculate the values of 4 nnd ,


(b) Using a scale of 2 cm Io rcpresent I unit on each axis. draw the g|aph of ) = 2 3n-rl
for 4.5<r<1.5.
(c) From your graph, find
(i) tle value of) when 1= -4 2,
(ii) the values ofn when ]' = 2 5,
(iij) the gredtest value of ).

RdstonErercfe5
@
6 The variabies x and ] are connected by the equation ) = 3 + 2-v 2l some con espondin'_
values o[.r and ) are given in the Iable bek]w.

3 -2 I 2 -l

-1 3 3 I

(a) Find the value of d-


(b) Using a scale of 2 cm lo reprcseit I unit on the .l-axis and 2 cm to rcprcsent 5 unils o:
the ).axis, dmw the graph ol -"- = 3 + 21 2-I'for 3{-r13.
(c) Frcm your graph, find
(i) . the values ofi
) = 15.
when
(ii) the maxinum value of) and the coresponding value of r,
(iii) thc equttion of the line of symmetry of the cLrrve.

't. The table of values fbr the graph of] = 2-r' 8i + 8 is shown behw
,2 I I 2 3 4 5 6

l8 2

(a) Complcte the table dove.


(b) UsiDg e scale of 2 cm to repfesent I unit on the r-axis and 2 cm to represent 5 units o'
the )-ais, dlaw the graph ofJ = 2i' 8J + 8 for the mnge -2 < i < 6.
(c) Draw and label on the graph the line of symmetry of the curve
(d) Write down the coordinates of the lunfng point
(e) From your gmph, find
(i) the valuesofi when ] = 20,
(ii) the value of] when r = 3.5.

8. The variablesr and) are connected by the equation I = -jrr I llr + ll some ctrrespondin:

values of r and I are given in the table below.

{ 4 -3 : ,l 0 0.5 l 1.-5 2

1.125 0 0.875 2 1.5 0.87-s 1.125

(a) Calcuiale the values ofp and q


(b) Usingascaleot'2cmtorcpresentIunitonlhe''arisand2cmtorcpresent05unilon
rre ) axi-. Jrcw the gruph of \ - r, lrr'\ 'tl lor '1 < ' < 2
I

(cj Write dov'n the greatest value of ) and the conesponding value of .l when it occurs'
(d) Draw and labcl the equation of the line of symmetly.
(e) Given that the pojnts (0.7, D) and (a, D) lie on the cufle. find the values of d and l'

(9
Tbe variables r )
and are connected by the equation y= Ll , 28. Some coresponding
t'alues of r and )are given in the following table.

ia) Complete the table above.


ab) j
Using a scale of 2 cm to represent I unit on the a"{is and 2 cm to represent 5 unirs on
thelaxis, draw the glaph of ] = Zrl - r - 28 for -4 < r < 4.
tc) From your graph, find
(i) the value(!) of -r when ) = -10,
(ii) the value(s) ofl' when x = 1.5,
(iii) the smallest value of Lr'- 28x
{d) Draw and label the line of symmetry of the curve_
te) Use yor $aph to solve the equation 28 + rr = 21.

Ihe base and height of a triangle are (r + 2) cm and (5 .r) cm respectively.

ral Showthrt lhedrea.,4 cm-olrhelrianglei\gi\enb] a = f-1" - tlr, r').r.


\l 2 )
ft) Draw the glaph of A=-.r: + l:.r +5 for-2<r<5.
rc) From your graph, find the base and height of the triangle rhar will give the maximum

-\ ball is thrown up into the air so that its height, ft meffes above lhe starting point atler t
s€conds is given by the formula /? = l6t 4r'.
(a) Using a scale of 2 cm to represent 1 unit on the taxis and 4 cm to represent l0 units on
ihe i-axis, dlaw the graph of, = 16r - 4l for 0 < r < 5.
ftr) From your graph, find
(i) the time when the ball is at its grearest heighr,
(ii) the grealest height reached by the ball,
(iii) the time interval when the ball is ar a height grcater rhan t0 m.

R*hionBrercise5 (o
The diagam shows a rectangular sheet of aluminium of length (t + 8) cm and widrh
5 cm_
12.
rectangular piece with allmensjons (t + 2) cm by r cm is then cut out'
(a) Show that the area, A cm'z of the remaining sheet of aluminium is given by
A = (40 + 3r .r'z) cnf.
(b) Using a scale of 2 cm io rcpresent 0.5 unit on the x-il-\is and a scale of 2 cm to
0.5 u;it on the A axis, dra; the graph of A = 40 + 3r - I for 0 < x
< 3'
(c) From your graph, find
(i) i
the value of for which the area of the remaining sheet of aluminium is
(ii) the values of r for which the area of the remainhg sheet of aluminium is 41

t3. g= lr : r is a natual numberl


A = {r : r is an even number}
B={r:xisamuftipleof3}
(a) List the elements oi
(i) €, (ii) A, (iii) R, (iv) A'
(b) Draw a Venn diagram to represent the relation between E, A and ,8.

14. A = \2, 4,6,8, 10, l2l E = {r : r is an odd numberi


B = 14,8, r2l F = {.r : r is a factor of 6}
c = U,2,3,6l G={r:-tisapdmenunberl
D= {5,7, 1l}
fill in lhe boxe\ wilh lhe .ymbols = +. C or C to describe the relation between the followitrg
sets,

(^) A (b) BA
L"__]t
(d) D IG

i
I

@ "*""'*r*-,u
If A = {r, {r}, }, z}, state whether each of the following is True (T) or False (F).
(a) (b) txte A I]
'€A .l
(c) {}} €A [] (d) {.2,y} cA i__l
(e) ttril cA L l (0 {.r, {r},zl cA i]

Given that P, q and R are tbree sets such thatP c Qa\dQ c R


State whether each of the following statements is True (T) or False (F.).
(a) If "r € -B then r€ O. Ll
(b) If.re Qthenr€P. I
(c) PcR i*l
@) ae P

A= 14, 8, 12, 16,20,24]J


B = {1, 3, 5, 8, 14, 201
(a) Find
(i) Ar.rB, (ii) A u B.
ft) Find the universal set of A and B which will have the least number of elements.

RdisionEreniF5
@
18. P= [2,3,5,'7 | andQ= U,2,3,4,5,6,7.8,9].
(a) Find
(i) P^0, (li\ P u Q,
(|it) n(P Q), (iv) {1,s1.P.
^
(b) State whether the following is True (T) or False (F).

(I) P€Q (\1\ PCQ

(tii) 9 c Q (iv) 3€Pr-\q

19. p= {7,9, 111 o={9, 10, 11} R={7, , 151


(a) Find
(i) PuR, (ii) O. R,
(iii) R), (iv) ,(P u R),
'(O ^
(v) (PuO).)n, (vi) (P C) u R.
^
(b) State whether the lbllowing js True (T) or False (F).

(i) PC0UR
Ql 9eP^Q
(iii)l0e0uR

20. t.et e= {3, 4, 5, 6, 7, 8, 91, A = {3, 7, 9l and B = {4, 6, 9J.


(a) Draw a Venn diagram to represent the given sets.
(b) Find
(i)A" (ii) AuB, (iji) A^8, (iv) r(A u B)'.
(c) IsA e B?

r*-.u
@ "*".",",
Lete= U,2,3.4,5,6,'t, 8,9, 101,A = { 1,2,3,5,8} andB= {3'4,5' 8} ard C= {5' 6, ?' 8}'
find
(^\ B^C. (b) A' t-,B, (c) B'^C,
(d) (B u O', (e) n(A u C).

It is given that € = {x : .x is a natural number < 10], M = { 1, 2,'7, 91, N = {2, s, 6} and
P = {6, 7, 8, t0}.
Find
(a) M ., N, (b) M' N, (c) M rJ (N .\ P\,
(d) u P)'.
^
'1(N

(a) I-ist the elements of


(i) €, (ii) A, (iit B, (i!.) B',
(v)A!,B, (vi) A.B. (vii) n(Ao B)', (viii) ,?(/'^ B).
(b) Is {3, ?l c B?
(c) If C = {.1 : x is a priDe nDmber between 2 and 101, find the rclation between B and C

RwnioiErercise5
@
It is given that € = Ia, b, c, d, e,f, g, hl, A = [c, f, g\ and B = {c, d, g, hl.
(a) Draw a Venn diagam to reprcsent the sets €, A and B.
(b) Fhd
(i) B" (ii) Au -8, (iii) (B u A.,', (iv) n(A . B)'.

25. ta.) If e = {parallelograms }, A = {fi ombuses I and B = {rectangles l,


(i) descdbe A r-r B.
(ii) draw a Venn diagram to show the relation between the sets a, A and B.
(b) If P = {isosceles triangles} and C = {equilateral rriangles}, describe
(1) P^Q, (tt) P w Q.

26. If € = {"x : r j
is an integer and 6 < < 23}, A = {x : x is divisible by 8}, B = {.r : r is a
squarel and C = {.x : r is a pdme number}, find
(a) A^8, (b)
(A O, (c) n(C), C). (d) .
^ '?(A

27. It is given that € = [q, r, s, t, u, r, rr\, J = Iq, \ s, tl, K = {s, ,r,1,, rr] and a = {s, r, v, n}.
(a) Find a(J. a).
(b) List the elements of
(i) (K u a)',
(ii) (J u r) r-\ K

@ r,*,'**r**,"
IfA = {Letters in the word 'atlas'} and B = {Letters in the word ,stall'l and C = {Letters in
the word 'talent'), find
(a) A r-r C,
(b) r(B u O,
(c) the relation between sets A and B.

,4,8and C are subsets of the universal set. e. A a B.A a\C =A. B.\ C+AandCgB.
Complete the Venn diagram below.

(a) A,BandCaresubsetsoftheuniversalset€,AvB=AandA^C=Z.Illustratethison
the Venn diagram below.

(b) e= l.r : r is a positive inIegerl


M={r:jr<l0l
N={i:r>3}
(i) List the elements ofM r') M
(ii) Filtd n(-M).

RdkronErerc$e5
@
31. (a) On the Venn diagram, add the set R which is such that R C M and R N = A.
^

(b) € = {r is an integer, I <n< 12}


P= {r : r is a factor of 12}
Q=lx:2x-7<lj
List the elements of
(D P, (ti) Q, (it1) P^Q'.

fa) On the Venn diagram..hdde lhe:er A ! 8'.

(b) x is a natual number and 0 < -r ( 131


r is an odd numberl
a= {r ,Y is a pdme numberl
c= {r ir is a factor of 15)
(i) List the elements of A C .
(ii) Find r(B' u O.
(iii) Find the element r such that x € (A B') ard .r e C.
^

r,., ru
@ ",*.**,
Shade the required regions.

(a) (b)

A'^B' AvB'

(c) (d)

(A^B)o(AvB)' (A^B)'^(AuB)

(e) (f

oo A'^B
@
B' .4
(h)

@^ByeC (AuB)^C

RdisrciErerche5
@
lffi Dot Diug"u-.
l. A dot diagram provides a quick and sjmple way to organise dala ln a dot diagrrr
no numeical values are lost-

We can spot pattems and summarise data in I dot diagml1l easlly

which each !alue is denotc:


3. A dol cliagram consists ofa horizontal number line on
number liDe value' The numbef ofdots abovc eac:
t u aot i,un" tt,.
"on"sponding
" indicates how many timcs each value occlrrrcd
v;hE

E.g. The spelling marts of l0 sludenls are gilen below'

61

From the dot diagranl. we can see that


6 mlfks
- I sludent scored 3 marks and 2 studenls scored
the lowest scorc $'as 3 marks
- the highesl scorc was 10 marks

- the most common scofe was 7 marks

l6s
,1:
WORKED EXAiIPLE
The lengths, in cm, of 30 fishes in a fish farm are shown below

(a) Represent the data in a dot diagram.


(b) wlat is the most common length of a fish?
SOLUTION:

(a) Step @: Since the lengths vary from 15 cm to 30 cm, draw a number line from 15
to 30.
Step @: Draw a dot for each piece of data above the coreponding number on the
number line.

15 16 t7 18 1.9 20 21 22 23 2+ 25 26 27 28 29 30

Length (cm)

(b) The most common length of a fish is 23 cm.

WORKED EXAMPLE 2:
The dot diagram below shows the time laken, in minutes, by each of a group of students fo
complete a task.

aa aaaaa aa
10 l5 20 25 30

Time (min)

cr."rt". rz,sr"ri.ri".
@
(a) How many students were there in the Broup?
(b.) Find the percentage of students who took more thnn 25 minutes to complete the ta+
(c) Comment bdefly on the data.

SOLUTION:
(a) Number of students in the group = 20

(b) Nrmber of students who took more than 25 minutes to complete the task = 5
Requjred percentage

= _ r00Ea

(c) The data vary between 5 and 30. The shofiest time taken was 5 min and the longes
time taken was 30 min. The most common time taken was 25 min. The data clust*
arcund 25.

@ st"- and Leaf Diagrams


L We can use a st€m and leaf diagram to organise data so that if is easy to spot patterlls
and make comparisons. No original values arc lost in a stem and leaf diagram_

2. In a stem and leaf diagram, each value is splir into two parts, the st€m and the leat
E.g. The stem and leaf diagmm below shows the ages of 20 employees of a banl

Stem Leaf
,7
2 8
l I 3 4
4 0 2 2
5
6

The numbers on the left of the ve(ical line are the stems. Here tle
stems are the tens digits of the ages.

The Dumbers on the right of the vertical line are the leaves. Here the
leaves are the units digits.

In the above example, 2 7 represents 27.

Frcm the stem and leaf diagmm, we can see that


the youngest employee is 27 years old
- the oldest employee is 63 years old.
- the most common age is 42 years,

@ ***.",",'r*- ru
WORKED E)(AMPLE 1:
The masses, measured to the nearest kiloBram, of 20 boys are given below

Represent the data in a stem and leaf diagram-

Srcp @: Determine the stem and teaf units. Shce the data arc 2-digit numbers, we take
the tens digit to be tle stem and the units digit as the leaf-
E.g. For the value 57, 5 is the stem and 7 is the leaf.
For the vatue 49, 4 is the stem and 9 is the leaf.

Step @: The masses mnge frcm 40s to 80s. Here, we take the ten digits of the number
as the stems. Place the stem digirs, i.e. 4, 5, 6, 7 and 8 in a column ftom the least
to the grcatest on the left side of the vertical line.

Stem Leaf
4
5
6
'7

Cr'"lr- rZ,St"ti.rr",
@
The leaves are the units digits. Write each leaf to the right of each stem i
Step O:
ascending order starting fiom tbe lcft
E.g.4l is split into its slem digit '4' and its leafdigil 'l
Stem Leaf
I 2
5 0 I
6 0 0
1 6
t

cI+ WORKED EXAMPLE 2:


students to sohe
The data below rcpresents the time' in mjnutes, taken by a group of 30
-:

mrthemttics question.

Represent the data set in a stcm and leaf diagram

SOLUTION:

Stem Leaf

L 045668
2 011222146 17 899
3 000133579

Tte above stem and lcaf diagram groups lhe data into 3 class intervals When eroupirr
data. try to group then into 5 to ll class intcrvals'

Tbe stem ancl leaf diagram on the fbllowing pag e \ho$\ cach
stcm dr\phled t$ice ie one

stem for the leaves 0-4, and the other stem lor ler\es 5
a The re\ultrngdLiSrurnrscalled
a stem and l€af diagram wjth split stems-

@ "'n.."..,*-,u
Stem Leal

I 04
1 56 68
2 01 l2 22 41
2 6',7 78 89 9
3 00 0l 33
3 5',7 9

WORKED EXAMPLE 3:
The stem and leaf diagram below shows the heights, in cm, of a $oup of children.
Stem Leaf

13 5788
t4 011246
15 0000134579
3345889
17 156
n.) rJ: represenG L' cm r

(a) Find the number of children in the group.


(b) Find the height of the tallest and shofiest child.
Find the percentage of children who are taller than 150 cm.
(d) Descdbe the shape of the distribution.

soLufloN:
(a) Number of children in the group
-4+6+10+'7 +3
=30

(b) Height of tallest child = 176 cm


Height of sho(est child = 135 cm

Number of childrcn who arc taller than 150 cm = 16


Required percentage

= _ 100'70

Cha!'terlr:sramdcs
@
(d) The distdbution is synrmeffical with a peak at the middle. Most of the children
heights between 150 cm and 159 cm. There are about the same number of
with heights above and below the middle values.

(E> WORKED EXAMPLE 4:


The stem and leaf diagram below shows the masses, in kilograms, of 40 parcels

Stem Leaf

22 033
22 7'7 88
23 000r 12 2223 34
23 6671 88 8
ot23 4
24 6788
1i",,,.139'-",,r', 1o G I
(a) Wdte down the mos[ common mass.
(b) 507, of the parcels have a mass of below t kg each. Find the value ofr'
SOLUTION:
(a) The most common mass = 23.2 kg

(b) 50Vo of 40

== 40

=20
.. 20 parcel. have a mas. of below 2J.3 Lg.

... x = 23.3 kc.

3. We use a back-to-back stem and leaf diagram to comparc two sets ol rclated data
To consffuct a back-to-back stem and leaf diagram, we dmw a stem and leaf diagm!
with a colnmon stem in the middle and leaves of each data on both sides of the sten

@ ".*-"0*.,.,'u
WORKED EXAMPLE 5:
The ages of the teachers of two different schools are recorded below.

40 61 52 65 64 54 63 43 59 38
s8 50 37 58 52 41 56 50 63 s3

30 5l 24 37 28 25 30 26 35 2',7

11 29 40 25 30 28 33 2:7 40

(a) Represent the data in a back to-back stem and leaf diagram.
(b) Which school had the oldest Ieacher?
(c) Which school had the youngest teacher?
(d) Compare the distribution of the ages of the teachem of these two schools.

soLuTtoN:

(a) Leaves for School A Stem Leaves for School B


2 455667 7 8 89
87 l 000357
310 001
988643 2200 5 1

5 4331 o

(b) School A had the oldest teacher (65 years old)

(c) School B had the youngest teacher (24 years old)

(d) The ages of ihe teachers in School A clustef around 50 to 60 years old. The ages of
the teachers in School -8 cluster arcund 20 to 30 yean old.
Thus, the average age of the teacheN in School A is more than those in School B.

@
@t uoa"
of central tendency' A measwe df
1. The mean, median and mode are measures
the data :re centred' i-e- ia'
t"na"n"V i, tingle value that descdbes where
""nJ
avemge value.
^

the mode
2. The most common value in a set of data is called

So there ts no mode'
3. ln some distributions' no value appears more than once
th:n one mode'
ln other distributions, therc may be morc

(@ WORKED EXAMPLE 1:
Find the mode(s) of tle following sets of numbe$'
(a) 54, 69' 70, 72, 80
(b) 24,26' 2E 28' 28 ' 29,29
'
(c) 5,6,'7,'7' 9' 12' 12, t5

SOLUTION:
(a) 5,+, 69, 70' 72. 80
There is no mode.

(b) 24, 26, 28, 28, 28, 29


'
29
Mode = 28

(c) ,7, 9. 12,12.


5. 6,1 15
Mode=7and12

@> WORKED EXAMPLE 2:


la) The heights of 9 bols. in cm are:
156 178 175 156
'53. 1ro, 164. 156. 175.
Find fte mode.

the heights of 3 more boys. 178 cm' 175


cm and 180 cm are added to the heigtB
(b) If
of the 9 boys. find the mode'

SOLUTION:
(a) 156, 156, 156, 158, 175, 175, 178. l7q
Mode = 156 cm

(b) 156, 156, 156, 158, 164, 1?5, 175' 175'


I78, 178, 17q. I80
Mode = 156 cm and 175 cm

@ "o*"-",n, ,'- ,"


WORKED EXAMPLE 3:
Find the mode of each of the following.
(a)

7 8 910 11 t2 13
Lensth (cm)

(b) Leaf

3 134 7
4 00? 888
5 022 26'7
6 455 556
7 033 36

(d)

40
30
20
t0
0
Red Blue G€en Sllver
Colour oi ca6

soLufloN:
(a) Mode = 1l cm
(b) Mode = 48 and 65

(c) Mode = $21

(d) Mode = Silver

Chaeter12:stathlics
@
@ u"aiutt
1 The value exactly in the middle of a set of ordered numbers (ascending or descenG
ing) is the median.

2. Steps to find the m€dian of a s€t of tr data:

€) Arrange the numbers in ascending order, i.e. from the leasf to the greatesl-

@ If11 is odd, the median is the middle value.


If n is ev€n. the median is the mean of the two middle values-

(l@ WORKED EXAMPLE 1:


Find fhe median of the following sets of numbers.
(a) 3, 6, 2, 8, 10, 9, s (b) 12, 16, 19, 15, 18, 14
SOLUTION:
Arrange the numbels in ascending order first.

Middle position

(a) 2, 3,5, 6 , 8,9, 10


Median = 6

Middle posillon

(b) 12, 14, 15, 16, 18, 19


l5 + l6
Median = -:-
= 15.5

(@ WORKED EXAMPLE 2:
Find the median of each of the ibllowing.
(a) rhr

-# ---#
21 22 23 24 25 26 5678910
Speed (kn/h) nme (seconds)

m*..*n. t*-.u
@
2 0123315
3 24558
4 013478889
5 0223399
1
r,l.rjo 6;;;,,r. l

(d)

12 003
1225
0466',1
3455889
46 0013
-; ''-:: ^
-

SOLUTION:
(a) Total number of data = 12
1) +
= ----
1

Middle polihon
= 6 5th Position

. . Median = Mean of 6th and 7th values

2
= 24.5 ktn4t

chapd12:srarinrs 6]
(b) Total number of data = 15

15+1
Middle position = ---
= 8th Position
..Median=7s

Total number of data ='7 + 5 + 9 + 7 = 28


,R+l
Middle position =
--
= 14.5th Position

Median = Mean of 14th and 15th values


41 +13
2

(d) Total number of dat^ = 3 + 4 + 5 + 1 + 4 = 23

23+l
Middle Position = -- ^
= 12th Positlon

Median = 44.7 mm

(e) Total number of apples = 3 + 2 + 4+ 3+6 = 18

li{ + l
Middle position = )-
= 9.51h Position

Median = Mean of 9th and 10th values


84+86
=2
=8sg

(0 Total numb€r of cals = 9 + 17 + 12+8+4+1=51


51 +1
Middle position = 2-
= 26th Position
Median = 2 people

t*- r"
@ "".*",",
The Mean
I - The mean of a set of data is obtained by dividing the sum of all the dafa by the total
number of dcta.

Sumofdata
Number of data

2. The mean of a set of r data, .rr, .rr, -ri, .... .v,,, denoted by t (read jr bar) is given by

rt +r2 +:r? +... +"!,

WORKED EXAMPLE 1:
(a) Find the mean of the following set of numbers.
8,9,4,5,7,12
(b) The mean of five numbers is 36. Tltee of the numben are 28, 3I and 39. If each of
the other two numbers is equal to r, find the value ofr.

(c) The mean of six numbem is 21. lf another number is added, the mean of the seven
numbers is 24. What is the number added?

SOLUTION:

(a) Mean =
8+9+4+5+7+12
45

(b) The sum of the 5 numbers = 5 X 36 = 180


28 +31 +39+-r+.r = 180
2r+98 = 180
\ =82

The sum of the 6 numbers = 6 X 21 = 126


The sum of the 7 numbers = 7 )< 24 = 168
.. The number added =168 126 = 12

Chaetrrr:SEthrs
@
(E> WORKED EXAMPLE 2:
The mean of a at\d b is 24. The mean of d' b, x ar,d ) is 3t Find the mean ot
and )-

SOLUTION:
Thesumof .tandb=2 x 24=48
Thesumofa'D,-IandY=I I3l = 124
'/6
.. The sum of .x and ] = 124 48 =
16
The mean of r and Y = ;1 = 38

3. Given a set of data, t!, ,t2, t3, .. ' r,, occuring wjth corespondjng ftequencies'i'i'-i
...,/,, its mean, t is given bY

v",", o =4rlfiifrf
_ t"ft
- r.f

(@ WORKED EXAMPLE 3:
A survey was carried out to find the number of hours a group of 50 students spent on Ih
jntemet in a particular week.

Find the mean number of hours spent on the internet per student-

SOLUTION:

0x8+ I x 12+2x9 +3x ll +4x6+5 x4


Mean = 50
-1A
50
=2.14h

r"*"'",n.
@ ^.. 'u
Altemative m€thod:

tft
Mean = -7
107
50
= 2.t4h

WORKED EXAMPLE 4:
A survey was conducied to find the number of siblings each of a group of students has. The
rcsults are shown in the table beloq

(a.l If the median is 3, find


(i) the largest possible value of .x,
(ii) the smallest possible value ofr.
(b) If the mode is 2, write down the smallest possible value of r.
(c) If the mode is 5, write down the Iargest possible value of-r.
(d) If dre mean number of siblings is 2.9, find the value of r.

WW
soLuTtoN:

(4, gj4' l' ;' I' 2. . 2.S. .@.4. ....4. s. .... s


3'7
(i) 3+7+r=3+6+8
l0+x=17
.. The large(l possible \alue of .r = ;.

Chaeler lr: Statistics


@
(ii) 3+7+r+3=6+8
13+'I =14

.- The smallest possible value oft = 1

(b) Ifthe mode is 2,


the smallest possible value of -t = 9. - Sffiii ..--S

(c) lf the mode js 5,


rhe larsesr possibte vatue of x = ?. .-iii.*ifffi,Sl

(o) Mcan number of siblings = 2.9 (Given)


0x3 + Ix l +2tx+ 3 x4+4 x 6+5x 8

3+7+.r+4+6+8
83+2r
28+i
83+2{=2.9(28+r)
83 + 2ir = 8l.2 + 2.9.r
1.8 = 0.9-r
1.8
'' 0.9

WORKED EXAMPLE 5:
The number of goals scorcd during 30 soccer matches in an inter school toumament
season is shown in the table below

(a) Showthatn+)=8
ibj t{ th" In.on nu.ber of goals scored is 2 9, show that t + 5) = 20
(c) Find the values of; and 1.
(d) Hence, state the modal number of goals scored

soLuTloN:
(a) Total number of matches = 30
3+i+7+5+2+l+5=30
22+r+l =30
x+J =8 (Shown)

t*- tu
@ "*n".",*.
Mean number of goals
0x3+l xr+ 2x'7 +3 x 5 +4x2+5x]J+6 x 5
30
0+:r+14+15+8+5)+30

67+r+5)
30
6'7 +r+5!
t+51 = 20 (Shown)

(c) r+) =8 _fl)


r+5)=20 _ (2)

(2) - (1): 4y = 12

Substitute ] = 3 into (1)'


i+3 =8

..r=5 and )=3


(d)

Modal number of goals scored = 2

Further Worked Examples


WORKED EXAMPLE 1:
The table below shows tle scores obtained by 40 players in a cefiain game.

t 2 3 1 5 6 7

8 7 5 10 3 2 5

Find
(a) the modal score,
(b) the median,
(c) the mean.

SOLUTION:
(a) Modal score = 4

4n+1
(b) MiddJe position = 2::
= 20.5th position

a*r*,, ,*,*u*
@
Median = Mean of 20th and 21st values
3+1
2

1 x 8+2 x 7+3x 5 +4x l0 +5 x3+6x 2+ 7 x5


Mean =
40
139
40
= 3.475

(K> WORKED EXAMPLE 2:


The diagram illustrates the number of occupants per flat for a sample of 100 flats iq
particular HDB block.
(a) State the modal number of occupants per flat-
(b) Calculate the mean number of occupants per flat.
(c) Find the medjan number of occupants per flat.

:10

35

l0

flats
20

l5

10

0
012345
Number of occiLpanh per flat

SOLUTION:
(a) Modal number of occupants per flat = 4

(b) Mean number of occupants per flat


0 x 5 +1 x 10+2x 15 + 3 x 25 + 1x40+5 x5
100
300
100

@ ""*..*"'^',tu
lno+l
(c) Middle position =
-j
= 50.5th Position

Median number of occupalts per flat = Mean of 50fh and 5 1 st values


3+3
2

WORKED EXAMPLE 3:
The stem ard leaf drasram below shows the number of minutes each time Caroline chatted
lsi online in January with her sister who is stltdying ovelseas.

I 5 5 8 9
2 I 3 3 3 4
,7
3 2 5 6 6 8 9
4 5

l{:.", ,i51"!1"*o 1511 I

Find (a) the mean,


(b) the mode,
(c) ihe median.

soLuT|0N:

Stem Leaf
5 5 6 8 9
0 1 3 3 3
,7
@ 8
2 5 6 8 9
5

a3+162+292+45
(al Mean =
2l
582
.21
- 27.7 min (colrect to 3 sig. fig.)

rb) Vode = 2r min .-'6ffi1"$#-.


d! :Lr".'1l(4. ii.

Chaeterlr:statsdcs
@
(c) 2l+1
Middle position =
2
11th position

Median = 24 min

@ Comparison between the Mean, Median and Motle


l. Although the mean is the most cormronly used average, it may not always be
most appropdate choice.

(D 1rygp659 gx66p1g 1'


The daily eamings, in dollars, of Mr Tan in a week are:
80, 88, 86, 89, 87, 470, 80

(a) Find the


(i) mean,
(ii) mode,
(iii)median
of the ditdbution.

(b) Which avenge gives the best picture of Mr Tan's daily earnings?

SOLUTION:
(a) (i) Mean d?ily eamings
$(80 + 88 + 86 + 89 + 87 + 470 + 80)

$980
1
= $140

(ii) Mode = $80 *-ffi


(iiD 80, 80, 86, 87,88,89,4?o
t
Middle losiiion

Median = $87

(d) The median gives the best picture because six ofthe daily eamings for the week
between $80 and $89.
The mean is $140, but six of the daily eamings were less than $90.
The mode is $80 but five of the daily eamings werc morc than $80.

(O ""*"."0- ^..
o
WORKED EXAIIIPLE 2:
The sizes ofjeans sold by an apparel shop on a certain day are shown below:
25, 28, 28, 26, 25, 28. 28, 26, 28, 26

(a) Find the


(1) mean,
(ii) median,
(iii) mode
of the sizes ofjeans sold that day.

O) Which average gives the best picture of the sizes ofjeans sold on that day?

soLuT|0N:
25+28 r28+2b+25 | '8 r28'2b- '8 -26
(a) (i) Mean =
10

264
t0
= 26.8

(jD 25, 25, 26, 26, 26, 28, 28, 28, 28, 28

Middle oosition= :#
= 5.5th Position
Median = Mean of 5th and 6th values
26+28
2

(iii) Mode=28 .---W


(b) The mode gives the best pictule since it shows the most popular size ofjeans sold.

WORKED EXAMPLE 3r
The table below shows the number of people living in each of 50 flats in an HDB block'

(a) Find the


(i) mean,
(ii) mode,
(iii) me.dian
of the disdbution.

(b) Which average gives the best pictue of the number of people livinB in each flat?

chaprer 12: sdiinics


@
soLurl
TION:

l\2+2<5+1.8r4 15 +5<q+6 . 1- 1. 3+ 8 |
(a) (i

,,r,r"."==."1;;""ffi
(iii) Middle position = I|
-:o = 25.5rh position
Median = Mean of 25th and 26th values

= 4 people
(b) The mean gives the best pictue since all the values in the data were used in tl
calculaLion.

2. The table below gives the comparison of the strengths and weaknesses for the mea
median and mode.

Mean Uses all the values Affected by exfteme Is the most


in the data. values, reliable
representatrve
of numerical
data Fovided
there aie no

Median Not affected by Its calculation uses It is a better


exfteme values. only one or two representation
middle values. of average tha
Does not use the the mean wh€!
total quantity thete are
represented by the extreme valu6
oata.

Mode Not affected by There may be more Useful as a


exteme values. than one mode or it measurc of
may not exlst, opinion and
Does not use the popularity.
total quantity
represented by the
data.

Tutor 28
Mean for Gmuped Data
1 . To calculate the mean for grouped data (where data arc grouped into inteflals). use

where r = mid-value of the class interval


f,A"un, ; = # and / = frequency of the class inte$al

WORKED EXAI',PLE 1:
The table below shows the scores obtained by a group of students playing the latest
computer game.

10 '74 '75'79 80-84 85-89 90 94 95 99

8 l2 l0 l6 8 6

Estimate the mean of the disdbution.

SOLUTION:

c{
idd
no

Mean. .i = +LJ
5030
60
= 83.8 (correct to 3 sig. fig.)

.. the mean score of the group of sfudents is 83 8.


ia
of
ld
ty-

Chaeter 12: Statisti6


@
(ffi> WORKED EXAMPLE 2:
A survey was caried out to ftnd the nomber ofhours a student used the internet in a cert:r'-
month- The table below shows the disftibution

0<i<10 l0<n<20 20<h<10 30<h<40

(a) Write down the modal class of this distribution


(b) Calculate an estimate ofthe mean number ofhours a student used the internet in ||.

soLuTloN:
(a) Modal class is 10 < h <2o.

(b)

0<&<10 5 8 40

10<h<20 t6 240

20<n<30 25 t2 300

30<ft<40 35 9 3t5

,t0<i<50 5 225

Lfx = 1120

tfr
Mean. t =
-
r 120
50
= 22.4h

. . the mean number of hours a student used the internet that month was 22 4 hou:i

*',*.,,", r*.
@ 'u
WORKED EXAMPLE 3:
The speeds of cars passing a certain road are given in the table below.

40<v<45 45<v<55 55<vs70 70<v<80

(al For this distribution, find


(i) tle modal class,
(ii)the class interval where the median lies.
(b) Calculate an estimate mean speed of the cals.

soLufloN:
(a) (i) Modal class = 55 < t < 70
(ii) Tota! no. of cars = 50
50+1
Middle position =

= 25.5th position

Median = Mean of 25th ard 26th values

The median lies in the class interval 55 < v < 70.

40<r<45 42.5 t0 425

45<v<55 12 600

55<v<70 62.5 20 1250

70<v<80 8 600
r/= s0

Mean, ; _ tfr
28'75
50

Chaeter 12: Stansri6


@
Tutorial

A. Dot Diagrams
l Complete each dot diagram to .epresent the set ot' data given below.

(a) 1 (b)

| + | F __+_
0123:1 '7 a 910u t2
5
- 13

(d)

+ lrlrrlil
36 37 38 39 40 41 42 45 50 55 60

(e)

l"l"l"l lrr | !r-!!]


'i 3 o ,l.o 5.0
1l

Mathenatjs rulor 28
@
The following dot diagram shows the waiting fime of some patients at a clinic

a
aa aa aa rraaa a a
| | | ! | I | | rr 1:
o 510 15 2a 25

Wajting time (nin)

la) How many patients were there?


rb) Find the maximum and minimum waiting times-
rc) Name the clusters in the distribution of the waiting time
Find the percentage of patients who had to wait more than 15 minutes to see the dodor'
'd)

The following dot diagram shows lhe number ofbooks read by each of 15 students in a cerain
month.

.........."
1234561
. _-+ +
Nunber of books read

ia) What is the most common number of books read?


tb) Find the minimum and milximum number of books read.
rc) Find the percentage of students who read less than 3 books in thnt month.
rd) Describe briefly the distribution of the number of books rcad by the students m that
month.

The following dot diagram reprcsents the marks of 30 students in a Mathematics quiz.

fl 80

ra) What is the mosf common mark?


Find the highesi mark obtained by the students.
'b)
c) 507, of the students obtained t marks or more Find the value of t
'
rd) Comment briefly on ihe distribution of the data.

cheler 1r: snxsrics


@
5. The following dot diagram shows the masses, in gams, of each of the pebbles in a bag.

aa
aaa
a a -r
o taaa a a a
lrr I rrrrl rrrrl r.r rrt rrrrl
30 35 40 45 50 55
Mas' G)

(a) How many pebbles were there in the bag?


O) What was the most common mass of the pebbles?
(c) Find the mass of the lightest and heaviesr pebble.
(d) Find the ftaction of pebbles which have masses less than 40 g.
(e) What can you say about the mass of the heaviest pebble?

6. The lengths, measured- to the nearest centimetre, of 24 lobsters in a falm are measued
recorded below.

(a) Complete the dot diagram below.

2t 22 23 24 25 26
t ngth

(b) What is the most common length?


(c) Write down the minimum and maximum length of the lobsters.
ld) Find the percenLage of lobsters rhal are tess than 24 cm long.

(,D **".",.,^..,"
Marc rolled a five-sided die 20 times. The number appearing on the die each time is recorded
below.

(a) Construct a ftequoncy table for the data.


(b) Draw a dot diagam to reprcsent the data.
(c) Find the percentage of times the number appearing on the die is even.
(d) What conclusion can you draw from the distribution?

In a fish farm, a study was conducted on the Srowth of 30 fishes, fed on two differcnt types
of fish food. 15 fishes were rcleased into a pond and fed with fish foodA. 15 other fishes were
rcleased into another pond and fed with fish food B. The increase in the length of each fish
affer a week was measurcd and presented in the dot diaglams below.

rllrr
2345678 ---r--r+
2345618
Increase in length (cm) Increase in length (cm)
(Fed with fish food A) (Fed with fish food B)

(a) Find the percentage of fishes which had grown at least 5 cm longer after being fed with
(i) fishfood A,
(ii) fishfood B.
(b) Analyse each of the distributions.
(c) Wtat can you conclude from the two distributions?

Chapler 12: Slalislics


@
9. The following data shows fte heights, in centimetres, of 30 plants recorded in a nursery.

(.,r) Represent the data in a dot diagram.


(b) Find the number ot' plants taller than 30 cm.
(c) What is the most common height?
(d) If the same data is to be represented in a pie chan, calcuiate the angle of the secratr
rcprescnting the heights of plants which are at least 28 cm.

10. The History marks of a class test are given below.

(a) Draw a dot diagram to represent the dala.


(b) Write down the lowest and highest mark.
(c) What is the most common mark?
(d) Ifa distinction is awarded ro each student who scores above 85 marks, find the perceruug:
of students who scores a distinction for the test.
l
(e) Thetof. ofthe srudents ffe selected ro reprcsent the school in an inter school Mathematici
Olympiad. Find the lowest mark of these students selected.

@ "".".",",.". ru
Slem and leaf Diagrams

,W
Complete each stem and leaf diagram to represent the set of data given below.

Stem Leaf
, --T--"
' , 3
4
5
i o
5 1
8

5
5
6
6
7
'7

Chapter 12: Statisncs


@
12. The stem aad leaf diagram shows the speeds of cars, in km/h, rccorded passing a clock towq.
along a road.
Stem Leaf
3 58 9
4 0011
5 02 2 3 3489
6 00 0 0 223
'7 l3
8 8

(a) Find the number of cals recorded.


(b) Find the most common speed recorded.
(c) Find the maximum and minimum speeds recorded.
(d) A speed camem is Iocated along that stretch of the road. The speed limit is 60 km,41. Find
the percentage of cals that exceeded the speed limit.
(e) Describe bdefly the disrdbution of the speeds of the cars.

13. An entomologist measured the wingspan, in mm, of 20 butterflies. The rcSults are displaled
in the stem and leaf diasram below.

Stern Leaf
5
5 6'7 9
00 2 4 4
6 55 6 8 88
1 t2
7 8

k?.4n:Y$@
(a) Find the most common wingspan.
(b) Find the fraction of butterflies with wingspans less than 60 mm.
Find the percentage of butterflies with wingspans at least 65 mm.

ton"-",n. t*- ru
@
The following stem and leaf diagram shows the lengths' in mm' of 40 worms.

8 r23 3
8 5566"t899
9 0001722333 344
9 66'.77',78
10 0013 4
10 67 8 8
l(Pv 8 | rePre'enl' 8l mm
I:_,
(a) What is the most common length?
(b) Find the length of the shofiest and longesf wolms.

(c) I of the woms arc shorter than t mm. Find the value of ,t

A group of students took part in a selection race to qualiry for the 100_me[e lace during sports
day. The stem and leaf diagram shows the time' in seconds, taken by the group of studenfs to
complete the mce.
ll 34
ll 56 6 8 8 9
t2 00 01r 34
12 JI
l3 0l
5

E:'-l lF;"**:'! llf :"{: i


(a) Find the number of students who took part in the selection ftce.
(b) Find the time taken by the fasrest student
(c) The top 8 students will be selected to compete on spofis day Find the range of tlme a
student needs to clock to qualify for the 100-metre ftce on sports day'

Chu*.. r:, Sturi.ti""


@
16. The stem and leaf diagrarn shows the montlily wages of the employees in company 48C-

I 0t 2 2 4 6 7 7889
2 00 0 3 5 ? 8
3 24 5
4 57
5 9

llo reDresents sluuu.

(a) Find the number of people employed by Company AiC


O) Find the highest monthly wage.
(c) Find the percentage of employees whose monthly wage are less than $2500
(d) Describe briefly the distribution of the data.

17. The stem and leaf diag&m below shows the masses, in grams' of each mango in a box'

J) 89
36
37 .0 I
38 00 2 2 2 6 6 8
39 03 4 6 6 6 6 8 899
40 00 5 7 ? 8 8
I
rP':J518 represents Js8 g
I
(a) Fird the number of mangoes in the box
O) Find the mass of the heaviest and lightest mango.
(c) Find the most common mass of the mangoes
(d) The grades of the mangoes ale detemined by theh masses Use the table below to
the percentage of manSoes which are
ti) Crade A. A 400 <,Y < 500
(ii) Grade B, 380<x<400
B
(iii., Grade C.
(e) Describ€ briefly the shape of the distribution. C 350<r<380

(,D r**.*",r.,,"
The following back-to-back stem and leaf diagnm shows the marks of the students from two
different schools who sat for an inter-school Mathematics competition.25 studedts from each
school took Dart in the comoetition.
Leares for School A Stem Leaves for School B
5l 5 2',l9
74 3 5688 89
3110 ,7
4 67789
996 644 3 8 o 255
988 887 530 0 9 o 37
t0 00
I rer: 511 rcpresenrs 5l marks.

(a.l Which 5(hool had


(i) the highest scorer?
(ij) the lowest scorer?
(b) Which school perfomed better in the examination?

The following back to back stem and leafdiagram shows the duration of each call, in minutes,
made by Bobby and Anne using their mobile phones. Each of them made 20 calls on a cerlain
day.
Leaves for Bobby Slem Leaves for Anne
40 I
5l 2
8 65 32 20 3
76 65 3l 4
3 5
2446 88 8
4
,7
1223 17
'7 8 556

(a) Who made the longest call that day?


(t) Who made the shoflest call that day?
{t) Compar€ the distribution of the duration of calls for these two people.

Cbaprer 12: Slatjstics


@
20. The masses, in kilograms, of a group of 20 children are recorded below

(a) Represent the data in a stem and leaf diagram.


ab) What is the mass of the heaviest and lightest child in the goup?
(c) How many childrcn have masses less than 50 kg?
(d) What percentage of childrcn have masses more than 36 kg?

21. The heights, in cm, of 30 students in a class were measued and shown below'

(a) Represent the data in a stem and leaf diagram.


(b) Find the difference in height between the shortest and tallest student
(c) Wtat is the height that occurs most ftequently?
fi) Find rl" frn"tion of students whose heights are between 140 cm and 155

Matherolics ru!o! 28
@
the stem and leaf diasram below shows the mathematics marks of 15 eirls in a class.

5 001
6 0248
7 t2'7
8 055
9 29
The mathematics marks of 17 boys in the same class are shown below.

(a) Construct a stem and leaf diagram to represent the mathematics marks of the 17 boys in
the class.
o) Construct a single ordered stem and leaf diagnm to represent the mathematics marks of
all the students in the class.
For the whole class, find
(i) the most common mark obtained,
(ii) the percentage of students who scored a distinction, given that a distinction is a
score of at least ?5 marks,

chapler12,sratistics
@
C. Mean, Median and Mode
23. Find (i) the mode, (ii) the median and (iii) the mean of each of the followins sers of

(a) 3, l, 2,5,6,2,5, a,2,8 (b) 7, 5, 3, 6, l, 11,4,6,3

(i) Mode = (') Mode =


(ii) Median = (iD Median =
(iii) Mean = (iii) Mean =

t4,2,6,8,9, 4,7 (d) 92, 80, 63, 90,'7s, 88, 80, 92,
82,'7 5,92

(D Mode = (i) Mode =


(ii) Median = (ii) Median =
(iii) Mean = (iii) Mean =

(e\ t8.'7, 14.2,12.5, 13.8, 12.0, 15.8, (f) $49, $38, $46, $29, $48, $3s,
12.9, t'7.2, 15.8, 13.2, 16.9, 13.3 $38, $45, $23

(1) Mode = (D Mode =


(ii) Median = (ii) Median =
(iii) (in) Mean =
End the mean, median and mode of each of the following diagrams.

(al
aa
aaaa
+ +
123456 8 910 11 t2 13 t4
Time (hours)
cost($)

(cl
aa ao a
aaaaaaaaa a
-|_]_++...l+
20 30 40 25 26 27 2a 29 30 31 32
Age uedt H€isht (cm)

*i
(e) Stem Leaf
2 t2
3 6'7 '7

4 03 55
5 29
6 0

Ke): 21represents2l.

ChJeErlr.Sr.r\rA
@
9 0 22 7
10 o2 26 I 015
l1 89 2 33 3 3 9 9
t2 01ll 3 15 6
l3 89 4 00 0 8
fey: 913 rcpresents 93 rnm. r€y: 012 reFesents 0.2 g.

(i) rrcquencY
,7

3
2
1
10 11 12 13 14

56789
Ma$ (ks)

5 5

3 3

z 2
1 1

20 21 22 23 24
Lensth (cm)
E) The mean of eight numbers is 34. When another number is added, the mean of the nme
numbers is 35. Wlat is the number added?

it) The mean of five numbers is 9. The numbers are in the ratio
largest number,

(c) The mean of nine nDmbers is 12. If one of the numbe$ is removed, the mean becomes
13. What is the number that was removed?

rd) The total mass of six boys is 291 kg. When the mass of another boy is added, the mean
mass of the seven boys indeases by 1.5 kg. Find the mass of the new boy.

(e) The mean ofa set of numbers is 21.5. Given that eight of the numbers are 12, 16, 18,20,
23. 24. 33 and 35. find the mean of the other four numbers.

{fl The mean of a set of 5 numbers is 2.3 and the mean of a different set of 17 numbers is
4.5. Find the mean of the 22 numbers

Chaete. 12: Stalislics


@
(g) Ama[da needs at least a mean score of 85 on five exams to receive a distinction.
grades for the first four exams were ?0, 92, 86 and 89. Wlat is the minimum score
needs on the fifth exam to receive a distinction?

(h) The mean of nine numbe$ is 16. The mean of seven of these numben is 15. Find
other two numbers if the difference between them is 3.

The mean of a set of five numbers is 6 and the mear of a differenl set of seven
is r. Given that the mean of fhe combined set of twelve numbers is 7.75, calculate

(j) The mean of 15 numbe$ is ),. If each of the 15 numbers is increased by 30, find th.
inuease of the new mean in terms of

(k) The mean of six numbers is 28. Three of the numbers are 19, 27 and 35 and each of tlE
other three numbeN is eoual to ,r. Find the value of .r.

(l) The mean of three numbers a, b and c is 24 and the mean of five numbers a, ,, c, r atrd
) is 27. Find the mean ofr and ).

(rr) *"."-"'", -.. *


(a) The mear of the numbers 13, 8, r, 5, 11, 7, 17, 10, 6, 2r is 9.5.
Find
(i) the value ofr,
(ii) the median,
(iii) the mode.

rb) The median of ten numben is 6.5. If nine of the numbers are 7, 5, 9,2.9,1,2,'l andg,
find
(i) the missing number,
(ii) the mode,
(iii) the mean.

(a) The numbers 2, 5, 8, 9 and r ale arranged in ascending order_ If the mean of the numbers
is eqDal to the median, find the value ofr.

r-b) The mode and the median of the numbe$ below are 126 and l2l respectively. Find the
values of -{ and J, given that r < ).

{a) Given that the mean of the above numbers is 8, find the value ofr + ).
(b) Given also that .I > ] and the mode of the numben is 9, find the values ofr and ).
tc) When another number, ? is added to the above numbers, the medial is ?.75, find the value
of a.

Cfraoter fZ: Statisrics


@
29. ( the mode, (ii) the median and (iii) the mean of each of the frequency distributi
Find (i)
below

(a) ffi|;-f,TtTt
l;# ETil I 9 7 !
L.%ff2 il7 el8l' 6 7 9 l 9

(D (i) Mode =
(ii) Median = (ii) Median =
(iiD Mean = (iii) Mean =

Elru
mIeTtT,o frr EIl3l

(i) Mode = (i) Mode =


(ii) Median = (ii) Median =
(iii) Mean = (iii) Mean =

(1) Mode = (D Mode =


(ii) Median = (iD Median =
(iii) Mean = (iii)
l''er ]
l 2 3 1 5 38 39 10 41
:f' 9 l6 p l3 I5 52 19 36

Given that the mode is 2, Civen that the mode is 38,


find the largest possible value ofp. find the smallest possible value ofp_

(c) 5 l0 20 25 12 l3 l1 l-s
2.1 )9 32 11 t1 8 6

Givcn that the median is 15. Given that the median is 13,
find the smallest possible value find the iargest possible vahe ofp.
of p.

l0 20 30 40 9 IO t1 I2
,7
26 32 l1 16 l0 5

Given that the mean is Gilen that the meaD is 9.5,


find thc value ofp. fiid the valuc of/.

Crr,rc,rz,StutlrU*
@
Given that the mean is 2,
find the values of a and ,.

, @ ""*".",", ^.. r"


The results obtained in a survey of 100 schools on the number of drinls vending machmes rn
eacb school is shown below.

Calculate
fa) the total number of vending machine.
(b) the mean number of vending machines per school,
(c) the modal number of vending machines per school,
(d) the median number of vending machines per school.

The rable shou \ rhe number of mobile phone\ in some households.

(a) Find
(i) the mean,
(ii) the median,
(iii) the mode
of the distribution.
(b) w1lich average gives the best picture of the distribution?

Cr,"Ot., lZ. St"r,s'""


@
restaurant
33. The numb€r of hours of wolk of each of the 15 part-time workers in a fast-food
given below

(a) Find
(i) ihe mean,
(ii) the median,
(iii)the mode
of the distdbution.
(b) Which average gives the best pictue of the data?

of a
34. A suvey was carried out to final the number of books bought at a book fair by each
of 200 itudents from a school. The results are shown in the table below'

(a) Calculate
(i) fte mean,
(ii) the mode,
(iii) the median
of the distribution.
the
(b) When the similar survey was carried out on 100 students of anothel school'
300 students
number of books boughi per student was 'x. Given that the mean of
the
these two schools was 1.78, calculate the value ofr'

@ t*"'"*' ^'""
Five coins were tossed 24 dmes. Tbe table below shows the number ofheads obtained in each
of the throws.

(a) Find, for this distribution,


(i) the mean,
(iD the mode,
(iii) the median.
(b) The coins were tossed thrice morc. The mean number of heads obtained in all the 27
throws was exactty 3. Find the number of heads obtaned in each of the extla three
throws.

Two factodes, each with 25 worke$ assemble toys for export. The number of foys
employee can assemble in 20 minutes is given below.

Factory A

ns$t{ihisff l.. o
4
8
5 6

7
7

2
>8

^4ffihnl -1

Factory I
N <3 4 5 6 7 >8
,7
N 4 2 t0

l^) Is it possible to calculate the mean number of toys produced by the workers in each
factory?
o) Find, for each factory
(i) the median,
(ji)the modal
number of toys that each employee can assemble in 20 minutes.
(c, Which average is best used to comparc ihe workers of both factories?

Chaeter 12:slansdcs
@
31. Eachschoolsent5sltlclentstoparllcrpareuranintel-schoo]Mtthematjcsolynpiad'ThelNh.:
shows the scorcs of each student fiom thc top 3 schools

mean scorcs' which school \lould wiD thc Mathenetr'


(a) Ifthe schools arc ranked by their
Ol)n'pi. Jl
schools are rankcd by lheir nredian scores' which
school would uin L:'
(b) iii"tla,ft.
Nlathematics OlYmPi.ld?
-
(c) iiui" ,t. "tt" "i,rtJ from school B, would you choose the Inean \core or tl
".,npetito$
medirn scorc to rank the schools?

May erc given bclow


-{1. The nunber of absentces of 5 classes frotn Jatuary to

2A 3
'1 l 3

2 4 I l I
2B
2 6 2
2C -l
5
)D 4 -l

2E I 5 2 ) 3

bY compa ng
(.l) Find the class with the most absentees per month
(i) their moctes,
(ii) thef nrediaDs
r"
(b) lJ\e their mftlns to rank tle classcs ln order fl{n the mosl number of absentces nt
least numbcr of absentees.

@ *,,n"*'"''*-.u
,A group of students was asked how many music CDs they had bought during the prcvlous
weekend. The table below sbows the resulls.

(a) Wdte down the laryest possible value of j!, given that the mode is L
(b) Wdte down the largest possible value ofr, given that the median is l.
(c) Calculate the value of n, given that the mean is l.

The table below shows the number of hours spent doing community work of each of a gloup
of people in a week.

(a) (i) If the mode is 3, write an inequality satisfying x.


(ii) Using the largest possible value of .x in (a)(i), find the mean and the median of the
distribution.
(b) If the median is 4, write down
(i) the largest possible valoe of -t.
(ii) the smallesl possible value of .d.

Ch.eter12:slatistrs
@
41. A survey was conducted to find the number of books some students read last month.

(a) Wite down the largest possible value ofr, given that the mode is L
(b) Write down ao equality that r must satisfy if the median is 2.
(c) Calculate the value ofr given thaf lhe mean is 1.72.

42. The frequency table shows the number of pets in each household.

(a) If the mean number of pets is 1.7, find the value of -r.
(b) If the distdbution is bimodal, state the possible value(s) of .r.
(c) If the median number of pets is 2, find the largest possible value of .x-

43. The number of pens that each studenl in a class has in his pencil case is shown in the tabl-'
beIow.

(a) Given that there are 20 students in the class, calculate the mean number of pens pe
sludenL
(b) If the mode of the distribution is 3, find the range of values of .n.
(c) If the median is 3, find the largest possible value ofr.

t",0"."0*^,-r"
@
The table shows the distribution of the number of siblings of a group of students.

(a) Find the smallest possible value of .x if the modal number of siblings is 3.
(b) Find the value of ,I if the mean number of siblings is 2.5.
(c) Find the Iargest possible value of .r if the median number of siblings is 2.

The table below shows the number of guests in each of the 100 chalets on an jsland resort
during a particular day.

(a) Find the value ofr + ).


(b) If the mean number of guests per chalet is 2.65, show that r + 4] = 135
(c) Find the values of i and ) by solving the appropriate equations.
(d) State
(i) the median number of guests per chalet,
(ii) lhe modal number of $re.ts per chdlel.

Chaeb 12: Sratistics


@
46. The table below shows the number ofnovels purchased by each ofa group of 100 studenls i!:
d monlh,

(a) Sbowthrtr+]=48.
Given that the mean number of novels purchased per student is 3.5. show tha.
3r+5-r=196.
(c) Solve the equations jn (a) and (b) simultaneously to find thc values of .r and I.
(d) Hence. state the modal number of novels purchased per student.

47. The table below shows the marks obtained bv 40 s$dents in a Mathematics quiz.

(a) Given that the mean is 7.2, calculate the values ofr and ).
(b) Hence, state the modal mark and the median mark.

48. A fair die is thrown 25 times. The results are recorded in the table below.

(a) Show that -r + ) = 10.


(b) lf the mode is 5, find the maximum value of.I-
(c) If the mean is 3.44, find the value ofr.

@ t"""-",,* ^.. r"


The nuftber of goals scored by a hockey team in each of 30 matches played during the pasf
year was as follows:

(a) Complete the table below.

O) Draw a histogram to represent this information.


(c) Find, for this distribution,
(i) the mode,
(ii) the median,
(iii) the mean.

(a) Use the given text to complete the freqDency distribution of the number of letters in each

(b) Find, for ftis disffibution,


(i) the mode,
(ii) the median,
(iii) the mean.

cr'rnr",LZ,Suirti.,
@
51. A group of students was asked the number of watches they owned. The histogram illu
the rcsults ofthe survey. Fi[d
(a) the number of students in the group,
(b) the modal number of watches per student,
(c) the median number of watches per student,
(d) the mean number of watches per student.

The number of goals scored in a hockey match in each of 36 matches of a hockey team i
shown in the bar graph below.
(a) Find, for this distribution,
(i) the mode,
(ii) the median,
(iii) the mean.
(b) ff tiisinformation is to be represented in a
pie char| calculate the angle of the sector
represenling 2 goals scored per malch.

@ "",0"."u* ^.. r"


The following data shows the shoe sizes of a group of 24 students'

(a) Complete the dot diagram below

l0

(b) Find, for this distdbution,


(i) the mode,
(ii) the median,
(iii) the mean.

and leaf
The ages, in years, of 21 patients in a hospital ward are reprcsented in the stem
diagram below.
I
4 ]J 88 9
5 6
6 33 id 9
'l 24
Find, for this distdbution,
(a) the mode,
(b) the medlan,
(c) the mean.

chaprer 12, sratistics


@
55. The heights, in cm. of 20 children are given below

(a) Draw a slem and leaf diagram to represent this infonnation.


(b) Find, for this distriburion,
(i) thc mode,
(ii) the median,
(iii) the mean.

56. The stem and leaf diagram shows the Mathematics marks of a class test of the boys and girl\
of Class 2A.
4 1 3
33 t 2 02'7
l 3 003 389
u5 0 4 225 57',7',l8
3 5 02
Boys cirls
(a) Find the number of
(i) boys,
(ii) girls
in Class 2A.
(b) Discuss the disdbution of each gender
(c) Which gender perfonned better?
(d) Find
(i) the highest and lowest mark,
(ii) tle modal mark,
(iii) the median mark
of Class 24.

r'r",r,".",i., r".. :r
@
Mean for Grouped Data
The table below shows the ages. in years. of 30 people at a beach carnival

(a) Complete the following table.

f+

-9
'7

10 t2
13 l5
16 18
19 2l
22,21

(b) Hence, calculate the mean age of the people :lt ihe camival

The lenqth of 40 lishes. measured correct to the nealesl mm, luc glven l]clow

l-encth (mm,l l-10 1I 20 2.r l0 31 40 4t -50 51 60 6t 70

uency 2 3 12. 9 5

(a) Complete Ihe table beiow.

-lld-**-" I FrequencY !f r t Ir
I l0
1l -20 -
21 -30
31 40
,{l ,50
5r -60
61 ',70

(b) Calculale the mean lenglh of these 40 fishes.

O.u*.'
"n.0.,', @
The tabLe below shows the mass. in grams' of each of the peaches in a box

I10 lI1

Complete the ibllo$iDg table.

ll0 Ir4

(b) Hence, calculate the nean mass of each pench in the box'

60. The lifespaDs, in days- of 60 ameobas in a colony are shown in the table below

0<{s10 10 < l <2t) 20<f<30 30<l<40 40<t<50 50<{<6f


I 2
,7
lrJ l3 I

(a) Conpletc the following lable

0<{<10
l0<t<20
20<t<30
30 < { < ,10

:10<l<50
50<l<60

(b) Hence. calculate the mean lifcspan of the colony of ameobas


(c) Write down the modal lifespan.

@ r**..,..r,. 'u
The times, in seconds. taken by a group of people to walk acrcss a pedestrian crossing are
liven in the table below.

30<r<15 35<r<40 40<r<45 ,15<r<50 50<r<55 55<r<60


'] 2 l5 12 9

a) Complete the table below.

Mid-value t.x I

30<1s.15
35<r<,+0
,10<r<4-5
.15<r<50
50<r<55
55<r<60

h) Hence. calcuhte thc mean time taken by these people-


Witc down the nodal class.

The masses of 40 letters. in 8ramsl ar€ given below.

25<,n<35 35 <, <45 45<rr<55 b5<n<75 75<rr<li5


ll 1 3 8 6 5

r) Complcte the table below.

Frequency (/)
-25<rr5-l-5
35<n145
45<r <55

65<n<75
75<n(85

b) Find the nlean mass of the :10 lettcrs.


Wdte dowD the modal class.
d) Wrilc down the class inteNal whcriJ tbe median lics

"n.*.,,a.,*. @
6J. I h( rable belo$ \hor. lhe mo rhly \a\ ing. (q) ot J0 .rudenr:

0<.r<20 20<r<40 40<\<50 50<s<80 80<r<

fa) C,)mflete the table below.

0<s<20
20<s(.10
40<s<50
50<r<80
80<.r<100

(b) Hence, calculate the mean monthly snvings ofthe studcnts, giving your answercorrect ti
(c) Write down the modal class.
(d) Which class intcrval conlains the nredian?

The tablc bclow shows the heigbts, in cm. of 120 plants in a nursery.

3,+<r<J8 38<h<12 42<h<16 ,16<h<50 50<ft<58

(a) Completc the table bebw-

(b) Hence. calculate the mcan height of the plmts.


(c) State the modal class.

r..-*,'., r,,-'u
@
A group of,l5 scouts took paft jn a fund raising project. The amount of money collected are
shown in Ihe table below

aa) Complete the table below.

0<.r<20
20<r<40
40<r<60
60<5<E0
80<s<100

(b) Estimate the mean amounl ()1'money collected by each of the 45 scouts.
(c) Find the percentage of scouts who collected morc than $60 each-
(d) State the class interval which conlains t}le medial.

The heighls, in centimeffes, of a group ot' 30 children in ar enrichment centre were measured
and recorded below

(a) Complete the table bclow.

l
J
Ileight lcm)
91

Ill
- 110

120
Thlly Mid-value (r) nrequency ft | ;x

l 12l
13l ll0
130

l
J
14t

151

l6I
150

160

170

(b) Esdmate the m n heighl of the children.


(c) Eslimate the pefcentage oI children whose heights arc less than or equal to 150 cm-

Cr,ekr ,'\ {ic.


@
67. The lengths of a group of 30 leaves, measued corect to the nearest centimete, are
below

(a) Complete the table below.

(b) State the modal class.


(cl Estimate the mean lengft of the 30 leaves.
(d) Estimate the percentage of leaves which are at least 35 cm long.

ru"tr'"*ri* r"r- ze
@
The rcsults of a mathematics test of 50 candidates are shown in the table below.

(a) State the modal class.


(b) Write down the class intewal where the median lies.
(c) Calculate an estimate mean of the mathematics marks of the 50 candidates.

The masses. measued to the nearest sram. of 40 tomatoes are shown in the table below.

.niffi Si zs zo 30 34 35 39 10 44 45 49 50 54
.q{ffi.t&trffiffiffi 2 t2 5 8 '7
6

For this distribution, find


(a) the modal class,
(b) the class interval where the median lies,
(c) the mean.

Chaeter 12: Slatistics


@
?0. In a survey, 60 children were asked the number of hours they spent usrng the computer
week. The results are shown below

Calculate an estimate mean number of hours each child spent using the computer per we€L

71. The table shows the heights, in cm, of the gymnasts taking part in a competition.

140<h<145 r.l5 <ll < 150 150<r< 155 r55<n<160 160<l<165

(a) write down the modal class for this distribution.


(D.l Calculate an estimate mean of the heights of the gymnasts.
If illusffate the above information, calculate the angle of
a Die chart is to be drawn to th
sector where the median class of the distribution lies.

r",n..**, r,*
@ 'u
The nasses, in kilogmms, of 80 alhletes are given in tle table below.

(a) State the modal class.


Calculate the mean mass of the 80 athletes.

'13- The distance travelled. in kilomeffes, by each of 40 taxi drivcrs in a shilt is given below.

(a) Construct a frequency table using classes 200 < d < 240, 210 < il < 280,...,
400<d<140.
(b) Write down the modal class.
(c) Estimale the mean of lhe distribution.

Chaprcr 12: Slaristics


@
L If the mean of the two digit numbers Ur', 1V8 and NN is 28, find the value

2. If the mean of a. D and c is 12. find the mean of 3a;3b and 3c

3. Marc has talen 5 Mathematics tests this semester If he obtains 72 marks on his next test, ir
will lower the mean of his test scores by 3 marks What is his mean mark now?

The mean of four consecutive even intege$ is 29. Find the larBest of these integers

(ri) """"*u*-..,u
{fthemeanofthleeconsecutivemultiplesof4is32'whatiSthesmal]estofthesemultiples?

lf a + 2b = 8d arrd,
c_d=ltd
find the mean of d, b, c anal /, giving your answer in terms of d'

the mean of the three digits is equal


Find the number of positive tlree_digit numbers for which
to 2.

a recent test she gave to the studentsbecause


tb€ lest tumed
A teacher adjusteal the marks of half the
a t".,jt" than she intended it to be she drd this by deducting only
aifff"tlt
""i before the adjustment was r'
numt"r of.art. u .tuO"nt misseal. If the mean mark of the class
find rhe new mean mark afler lhe adJusment in lerm' ol J'

Ctrurc. rz, sturi.ti.,


@
the mean is.6l kg lf the mass
9. The median mass of seven boys is 62 kg the mode is 68 kg and
neavrest boyl
of the lightest boy is 50 kg, what is the greatest possible mass ot the
(Assume that the mass of each boy is an integer)

10. The median of ? consecutive integels ls :'


Find ill tems of t
(a) the mean of the 7 htegers'
of fhe squares of the integels'
iUj *t" Oft"."n"" t"tteen the mean and median

are given below'


11. The ages, in years, of a group of people arranged in ascending order
10, 12, 12. 14. Io. lo. lb. d. 21. 24
ftnd the value of d
Given that the median age is 0.1 gletlter than the mean age'

@ *"*".",., *.. r"


Given that the mean, the median and the mode of the set of ordered numbers below are 84, 85
and 88 respectively, find the values ofp, 4 and r..
p, 81, 81, 82, q, r, 88, 88, 89. 93

The median of seven diflerent positive integers shown below is 8.


r
(21 + 1), 6, 4, 9, 8, 1s,
Find the value of .I.

l0 students each fiom two schools, A and B, took part in an inter school Mathemari,js
competition. Their sco.es are displayed in the stem and leaf diagram below.

SchoolA Stem School B


531 '7 024
8654 8 5x6
2vo 9 023
The sum of the scores of all the 20 students is 1669.
(a) Find the value of.I and the value ofj',
(b) The school with the higher mean score won the competition. Which school was the
winner?

a"*,,'
"n"Oo,,r @
Probability

l Probability is the branch ot' statistics which allows you to work out how likeiy rl
unlikely an outcome or rcsuk might be. lt is used in areas such as predicting sale.
rnaldng investmenls decisions and planning political campaigns

2. Some everyday probability conveNations are given below:


Whal is thc prcbability that my school will win the Science competition?
There's a 50 percent chance that it will not rain today.

ES oetnitio.,
l Probabilities are ratios. It is usually written as a proper fraction. lt can also b!
expressed as a decimal or as a percentage,

2. A probability experiment is an aclivity where the result depends on chance.


E.g. Tossing a coin and drawing a card from a pack of shuffled playinS cards.

l. Each o[ the possib]e results of an experiment is called an outcom€ When ea.:


outcome has cxactly the same chance of happening, then we say that the outcom:!
arc equally likely.

1. A possible outcom€ is a result fhat could possibly occut even though it lnay n:
occur this time round.
E.g. I The possible outcomes of tossing a coin are 'Hftds' and 'Tails'.
E.g. 2 The possible outcoDres of rolling a fair die arc l, 2, 3, 4, 5 and 6

5. A favourable outcomc is a result thal we arc intercsled in.


E.g. I If we want to get a 'Head' iD a coin toss, then getting a 'Head' would be .
favourable outcomc.
E.g.2 If we want to ge1 an even number in rclling a die, then the favourab':
outcomes would be rolling a 2, 4 or 6.

v"rr..."ti., ruto.:t
@
6. The sc! of aII possible outcomes for an expedment is called the samlle space lt is
denoted by S and the outcomes are put in braces { ).
E.g. 1The sample space oftossing a coin, S= {Head, TailJ.
E-g. 2The sample space of rolling a die, S = { 1, 2, 3, 4, 5, 6}.

1. The total numb€r of possible outcom€s is the same as the total number of elements
in the sample space. It is denoted by ,?(S).
E.g. 1 If S = {Head, Tail}, then n(S) = 2
E.g.2 If S = { 1, 2, 3, 4, 5, 6}, then '?(s) = 6.
8. An event is a set of outcomes in which we arc interested in. Any subset of a sample
space is :m event.
E.g.l when we toss a coin, we may define getting a head as an event.
The outcome of the event is 'head'.
E.g.2 When we roll a die, we may define getting an even number as an event.
The outcomes of the event are 2, 4 and 6.

rE> WORKED EXAMPLE 1:


An experimenf consists of drawing a card fiom a j ar containing I 2 cards Each card has a
dilferenl monfi ol lhe )ear wrilren on il
(a.) List the sample space for the expedment
List the following events.
(b) The event A consists of drawing a card haling a monlh beginning with I
(c) The event B consjsts of alrawing a card having a month beginning with M or N'
(d) The event C consists of dnwing a card having a month that has 4 letters or less
SOLUTION:
S = lJanuary, February, March, April, May, June, Jult August, September,
October, November, December J

(b) A = {January June, July}


(c) B = {March, May, Novemberl
(d) C = {May, June, July}

9. Probability is a measure of how likely it is for an event E to happen. It is denoted by


P(r).

The probability of an event E, P(4, in an experiment with equally liLely outcomes is

Nurnber of favourable outcomes for event ,E

Told' number of Po.tible oulcome'


n(E)
_
n(s)

jn event E and
where n(E) is the number of favoumble outcomes
,(t
is the total number of possible outcomes in the sample space' J^'

Chapter l3: Probability


@
E.g. Find the probability of rclling a number which is more than 3-

Possible outcomes = {1,2,3,4,5,61

Favourable ourcomes = {4, 5, 6}

Number of favoumble outcomes


P(rolling a number more than 3) = Total number of possible outcomes

I2
10. The probability of an event caD be a number from 0 to l(both inclusive).

0<Probability<l

Bvent: It will snow When a coin The sun will rise


in Singapore. is tossed, it will be everYdaY.
a head.

unlikely . likely
Impossible 50-50 chance Certain

Piobability: 0 1 I
2
0.0 0.5 1.0
oEa 507o 1004o

The closer the probability is to zero, the more unlikely (he event is to happeni
closer the probability is to 1, then the more likely the event is to happen.

11. If the probability of an event occurdng is 0, then it is an impossible event, i.e. it


never happen.

E.g. The probability of rolling a 7 on a die is which is 0.


;
12. If the probability of an event occurring is 1, then it is a certarn ev€nt, i e it
definitely happen.

E.g. The probability of drawing a red ball from a bag of 5 red balls is which is
I

r.r"."o*
@ ^..""
13. The probabiljty that an event does not occtr is the same as I minus the Fobability
that ihe event occurs.

P(Event does not occur) = 1 - P(Event occurs)

E.g. The probability that it will rain tomorrow ls ! . What is the probability that it
5
will not rain tomorrow?

P(notrain) = 1-P(rain)
_1
5
1
5

WORKED EXAMPLE 2:
The diagam shows a spinner having 9 equal triangles. The pointer is spun. Find the

'lll
probability rhat it will stop at a tiangle conraining
(a) a 5,
(b) a 6,
(c) a whole numbet
(d) a number that is less than 10.

SOLUTION:

No. of favourable outcomes


(a) P(a 5) =
Total no. of possible outcomes
2
9

o
(b) P(a6)=-

(cJ Pta whole numberl=


i

r{a no. < tol =


(d)
}

Chaptef 13: Probability


@
(ffi> r t6pK59 6xan plg 3'
A box contained 20 coloured balls, where 8 are green, 10 are red and the rest arc yello$.
A ball is selected at ftndom from the box. Calculate the probability of getting
(a) a green ball,
(b) a ball that is not green,
(c) either a green or a yellow ball,
(d) either a green, a rcd or a yellow ball,
(e) a blue ball.

SOLUTION:
Number of yellow balls = 20 8 10 = 2

(ar P(u sreen ballJ


' =
R
::2t)
2
5

(b) P(not a green ball) = I - P(green ball)


.2
5
3

t0
(c) P(a green or yellow ball)
20
I
2

(d) P(a green, red or yellow ball) =


fi -ffi
0
(e) P(a blue ball) =
20

t'- r"
@ ""*".",,".
g> WORKED EXAMPLE 4:
A box contains 10 cards numbered 1' 2' 3' 5' 6'
8 9' l0' 13 and 15 A card is drawn at
random liom the box-
total number of possib]e outcomcs' n(t)-
{a) Write dowll the sanple space' 'S aDd the
on the card drawn is
iij plni trt" p.ou'tilityihat the nr.rmber
(ii) an odd number' (iji) a prime oumber'
(j) r 15.
(iv) a composile numbe! (v) a perfect square'

SOLUTION:
(n) .! = {l' 2' 3,5,6 8' 9' 10' 13' lsi
,r(S) = I t)

I
lbl lir P(a l5J = to

6
(ii) P(m odd numtrer) = l0
3
a

4
(jii) P(a prime runber) = t0
2
5

(i\,) P(n composite number) =

(v) P(a pertect square) =

cl[pLer l3: fiobabill!


@
WORKED EXAMPLE 5:
A six-sided die is rclled. Find the probability of getting
(a) a 4, (b) an even number, (c) a pnme numoer,
(d) a multiple of 3, (e) a number less than 4, a positive integer,
(g) a 7.

soLuTtoN:
Sample space = { 1, 2, 3, 4, 5, 6}

I
(a) P(a 4) = -

3
(b) P(an even number)

I
2

3
(c) P(a prime number)
6
I
2

2
(d) P(a multiple of 3)
6
1

(e) P(a no. < 4) = I6

1
)

(f) P(a positive integer) = 9


6
t

(e) P(a 7) = -

Marhemalics rtitor 28
@
WORKED EXAMPLE 6:
Acircleis dividedinto4 sectors as shown.If a dart is thrown and hits the circle shown. find
the probability that it will land
(a) inside sector A.
(b) outside sector A,
(c) inside sector 8,
(d) inside sector C or D.

(a) P(inside sectorA) (b) P(outside sector A)

= 1-P(inside sectorA)
90" .1
360"
3
1

(c) P(inside sector B)


Area of sector B
Areaofcircle

360"
I

(d) P(inside sector C or D)


Toial area of sector C and sector D
Area of circle
120" + 105'
360"
225'
360"
5
8

Area where E occurs


Totai area

qwter I r, rrcutiritr
@
(D WORKED EXAMPLE 7:
TWo farr coins are tossed together
(a) List the sample space.
(b) find lhe probabilily of gertin8
(i) 2 heads,
(ii) t head and 1 tail,
(iii) at least I head.

soLurloN:
(a) S = {HH, HT, TH, TTI where H = Head and T = Tail'

Q) (i) r(two head$ =


]

2
(ii) P(1 head and 1 tail) =
n
r
2

(iii) P(at least t n*ol = ]


*ffi
Ca> WOBKED EXAMPLE 8:
A two digit number is chosen at random. Find the Fobability that the dumber chosen
(a) is smaller than 30,
(b) is odd,
(c) is a multiple of 5.

SOLUTIONI

Sample space,,t = {10, 1l' 12, 13, 14' ,99}


n(s)=99 l0+l
=90
z\
la) Pla no. < 30) =
90
,7

30

(b) P(an odd no.)

Mathenatics rltof 28
@
18
(c) P(a multiple of 5) =
90
I
5

WORKED E)(AMPLE 9:
A card is drawn at Gndom from a pack of 52 playing cards. Find the Fobability of drawing
(a) aD ace,
(b) a red card,
(c) a pictule card,
(d) a card which is not a spade.

soLuTtoN:
4
(a) P(ar ace)
52
I
13

(b) P(a red card)


26
52
I
2

12
(c) P(a picture card) =
52
3
13

(d) P(a spade) =-


I

P(not a spade) = I P(a spade)


1_f4
1
4

Chaptq 13: Prcbabilfty


@
C@ WORKED EXAMPLE 10r
A bag contained 50 marbles coloured red' blue and green When a marble is drawn a
rand;m from the ba8, the probability of getting a red marble is 36% and the probabiliB d
getting a blue ma{ble is 0 42.
(a) Find the probability of getting a grcen marble'
(b) Find the number of
(i) red marbles,
(ii) blue marbles,
(iii) green marbles
in the bag

SOLUTION:

tal P(a geen marble)


= I - P(rot a Feen rnarble)
= 1 LP(a red marble) + P(a blue marble)l
=1-[364o+0.42] *_ s@Pffi
=1 0.78

(b) No. of rcd marbles


=36%x50
::1
= to0 ^50

(ii) No. of blue marbles


= 0.42 \ 50

(iii) No. of green marbles


= 50 (18 + 2l)

ru",r,...,""r**ze
@
WORKED EXAMPLE IIl
There are l0 red pencils and i yellow pencils in a box. Apencil is selected at randorn fiom

the box. If the probability of selecting a yellow pencil is


i, find the value ofi.

soLuTroN:
Total number of pencils = l0 + ir
3
P(a yellow pencil) = : (Ci'"en,

"Jl
.."+ j" 5 ' c,os-mulriply.
l0
5r = 3(10 + r)
5,v =30+3,r
2x =30

WORKED EXAMPLE 12:


The table shows the number of siblings 80 students have.

A student is selected at random from the group- Find the probability that the student
selected
(a) has no siblings,
(b) has at least I sibling,
(c) has more than 2 siblings.

SOLUTION:

(a) P(studcnt has no siblinqs) = l14


I7
40

P(student has at least 1 sibling) = I P(studefi has no siblings)


11
1--
23
40

(c) P(student has more than 2 siblings) = 19


80

!5

Chlprer 13: Prcbability


@
@ Using Set Notation to Describe Probability
l. We can use set notation to describe gobability.

2. The set of all possible outcomes is called the sample space. It is denoted by S ad
the outcomes are put in braces { l.

3. The total number of possible outcomes is the same as the total number of elemeG
in the sample space. It is denoled by /'(t.

4. Here, each outcome is an element of the sample space and each event is a subset d
the sample space.
E.g. The sample space of rolling a die, S = 11, 2, 3, 4, 5, 6l
I € S, 2 € S, 3 € S,4 c S,5 e Sand6 € S
Let t be the event of rolling ap me number
E= {x : r is a prime numberl
= {2.3. sl

n'
6es

6"

Eventofrclling a pnne number

5. t
The probabitity of an ev€nt, in an experiment with tr equally likely outcomes atd
if m of the outcomes favour the occulrence of event 4 then

Probability of 6 happening. PaD - Tolal number ofpossible outcomes

_ n(D
- ,1(s)

wherc ,r(t) is the number of favouable outcomes in event E and


r1(S) is the total number of possible outcomes in the sample space, S

6. The probability of the even! -d, the complement of event E is:

P(4=1-P(t)

(a raa*enatics ruld 28
IF WORKED EXAMPLE 1:
A number is chosen at mndon frorn I to 25. Find the following plobabilities.
(a) The event A that an even number is chosen.
(b) I
The event that a number greater than 25 is chosen.
(c) The event C that a Dumber less than 26 is chosen.
(d) The event, that a prime number is chosen.
(e) t
The event that a number both even and pdme is chosen.

soLuTtoN:
s = {1,2- 3,4,5, ...,251
,(s) = 2s

(a) A = {2.4,6.8, 10, 12, 1,1, 16. 18,20,22,24]t


n@) = 12
r(A)
,(Jl
t2
25

(.b) B=g
n(B) = 0
n( B)
Pf.B) = ,1)l
-
t)
25

(c) C=S
n(C) = 25
,(C)
4r)
25
25

"n"0,.,,,, "'"O"OU*, @
to) D = \2,3. 5,',7, 11, 13, 17, 19, 23 ]
n(D) = 9
n( Dl
P\Dt = ^

9
25

(e) E= l2l
n( E)
P(A = n(s)
I
25

WOBKED EXAMPLE 2l
30 cafds are numbered I I to 40 and placed into a box. Acard is then chosen at random from
the box,
(a) Write down the sample space.
(b) Let E be the evenl that at least onc of ihe two digits on the card chosen is a l
List E.
(c) Find
(1) P(E), (iD P(s).

SOLUTION:
(a) Sample space, S = { 11, 12, 13, 14, ..., ,+0}

(b) E = { 13, 23, 30, 3r,32,33,34.35, 36, 37, 38, 39J

n( E)
(c) (i) P(E) = ^
t2
30
2
5

(it ytL t= | ylet + r/ rtE:qs meevenr r.


- does d6toe.1tr : .:

3
5

@ t"*"-,,o,',.. r"
Tutorial

Match each event to the probability that describes it


I
. 999
I An impossible event . 1000
l

I
2

A very likely event .0


I
An unlikely event 1000

' A 507o chance

Wdte down the sample space, S and the total number ofpossible outcomes, n(s) for each of
the following.

I (b) Drawing a card from a box


l
conIaining 8 identical cards
numbered 12 to 19.
I

Drawing a ball at random fiom a bag (d) Spinning the pointer shown below.
containing 3 red balls and 4 green
balls.

Chapter 13: Prcbability


@
3. Calculate the probability of each of the followiltg.

(a) A six-sided die is rollcd. Find ihe probability of getting ar even


numbef.

(b) Ther€ arc 4 rcd balls and 6 gfeen ba1ls in a bag. A ball is dfawn
at random frcm the bng. Find the probability of getting a red
ball.

lc) \.uoe] $.,.r:, fi.d oul on 3 lroup or 200renior.'ri,,en..z0or


them arc women. Find the probability that one person chosen at
rnndom will be a man-

(d) A=12,4,7,8,9.111
Iflln element is selected at random fromA, find the probability
that it is odd.

(e) A six sided die is rolled. Find the probabi lity of getting a numbcr

(f) Find the prcbability ofchoosing a number from I lo l0 that is a


multiple of 3.

(g) Find the probability of getting a nunbcr more than 6 on rolling


a six-sided die.

(h) Find the probability of getdng Deither t he:d nor a tail when a
coin is tossed.

(i) A letter is choscn from the wod 'MAIHEMAIICS'. Find tic


probdbility that lhc letter chosen is a vowcl.

0) A bag contains four couitersi one marked with dle letter t, one
with thc letter B and two with lhe letter R. A colrDter is dnwn at
rundom ftom the bag. Find the prcbabilit) ofgetting the lctter R.

(k) On any h ip to the malket, the probability lhal Agnes will buy a
l
chicken is Find the probability that Agnes will not buy a
;.
chicken on one particular dp to the nrfiket.

(1) The probnbility that i vending machine will accept a panicular


one dollarcoin is 0.8. Find the probability that the mrchin; will
nol rccepl ., frrli.ulrr one doll.r c.in.

@ nr.n. -,i. r,.,:e


A fair die is rolled. Find the Fobability of getting
(a) a 2, (b) an odd number, (c) a prime number,
(d) a multiple of 2, (e) a factor of 6, (0 a natural number.

A fair die is rclled. Find the gobability of getting


(a) a 3, (b) an even number, (c) a number greater than 4,
(d) either a 2 or a 6, (e) a perfect square.

An experiment consists of selecting the last digit of a telephone number at random.


(a) List the sample space.
(b) Find the probability that the last digit selected
(i) is less than 4,
(ii) is odd,
(iii) is not 8.

12 cards, each numbered 1,2,3, 4, 5, 6,7,8,9, 10, l1 znd 12 are placed into a box.
A card is then dmwn at random from the box. Find the probability that the number on the card
drawn is
(a) a 9, (b) an even numbet (c) a prime number,
(d) a composite numbet (e) a perfect square, (f)
a 3-digit number

Chapter 13: Bobabilily


@
8. A box contains l0 cards as shown below- A card is druwn at random ftom Ihe box. Find thr
probability that the number on the card drawn is
(a) an 8, (b) greater th:m 5, (c) cn odd number.
(d) less than 3, (e) a positive integer
a t a--.] --u l. )
tsEsE 3t
trL!trEitr

Each of the letters of the word 'PROBABILITY' is w.itten on a card. The cards
shuftled and placed jnto a box. A card is then drawn at random from the box.
Find the probability that the lettcl on the card drawn is
(a) a B, (b) aP,
(d) a coosonant,

I0. Each of the 6 cards shown below has a letter of the alphabet wfitten on one side of the caJd
The cards are then shuffled and laid face down on the table. A card is chosen at random lrom
these cards. Find the Drobabilitv that Ihe letter on card drawn is
h)aP. (b) an A,
(c) an E, (d) either an A, a P or a l.
Hffi8ffiffiffi
11. A bag contains ,l gold medals. 8 silver medals and 6 brcnze mcdals. A medal is selected ar
random from the bag. Find the probabilily thal the medal selected is
(a) gold, (b) nol gold,
(c) either silver or bronzc, (d) either gold, silver or bronze.

r",n..",n.,*-,u
@
the can
sweets A sweet is dmwn at random from
12. A can contains 10 red. 8 white and 6 yellow
Find rhe probahility tbiLt the sweet Llrawn i\
rar rellor.r. lb) not ]ellos'
ici irot a colour of the rainbo*, (d) (0
red or yellow'
(ej red, white or yellow, green'

He co-unted. the number


13. Ma{c conalocted probabilily experiment using a box of 3O.ma$les
a
" ;;ttbtl'fe;'"oloo' *i '"iotata tn"rnrhat ii the table below He then draws one marble at

random from the box. Find the probability


(a) a yellow marble is alrawn from the box'
6
(b) the marble drawn is not green 15

a 20'l' chance that it will


l:1. (a) According to the weather report' there's
Find the probabitity that it will not lan
tomolrow'

green marble from


that the probability of dmwing a
(b) There are 49 narbles in a can Given

calculate rhe numt]er or malt)le' in


lhe can fial i\ nol green
the can 1s
;,

rt arnves
at a station is 0 3 The probability that
(c) The probabilitY that a train arrives early
on time is 0.52 Find the probabiiity
that the train is late-

@
(d) A manufacturcr makes switches for electric circuits. The probability that a switch t
faulty is 0.05. Mr Lim buys 200 of these switches. How many ol lhese switches can be
exDected Io be fhultv?

(e) A manufacturer produces microchips. ln a sample of 100 microchips, it is found thaI8 o:


these microchips arc faulty. Find the probability that a microchip chosen at random frorE
the samDle is not faultv.

(1) A printing company fbund that the prcbability that a parcel will be delivered wirhin
'7
2 working days of being posted is Find the probability that a parcel will be delivered
t.
nore than 2 working days after being posted.

1-5. Two fair coins are tossed together Find the probability of gettilg
(a) exacdy one tail,
(b) at least one lail,
(c) at most one tail.

Three lair coins are tossed logether Find the probability of getting
(a) 3 heMs,
(b) 2 heads and 1 tail,
(c) at least 2 heads.

u"tr,".,ri* r"o. ze
@
17. There are 50 children at a games camival. 35 of them arc boys 28 of the children wear
spectacles. A child is chosen at random from this group. Find the probability thir lhe child

(a) is a girl,
(b) wears spectacles.

18. A box contnins 30 chocolates. 9 arc milk chocolates, 15 are mint chocolates.and the rest are
dark chocolates. A chocolate is picked at randon from the box. Find the probability of
(a) picking a milk chocolate,
(b) not picking a milk chocolate,
(c) picking either a milk or dark chocolate,
(d) picking either a milk, mint or dark chocolate.

!9, The diagram shows a spirmer with 8 equal secton. The pointer is spun Find the probability
that it will stop at a sector containing
(a) a 5, (b) a 9,
(c) an odd number, (d) ejthera2oraT,
(e) a multiple of 3, (1) a factor of 35,
(g) a prime number, (h) a composite number,
(rl neilhef a prime nof a compo\ile number

chqter 13:Prcbabiliiy
@
20. The diagam shows a spinner in the shape ofa regular octagon. When the pointer is spun, fitd
the Fobability that it will stop at a triangle conlaining
(a) an even number, (b) a multiple of 4,
(c) a number less than 5, (d) a number more than 8,
(e) a number no less than 9.

21. The diagmm shows a spinner having 9 equal ftiangles. When the pointer is spun, find the
probability that it will stop at a tdangle containing
(a) a 3, (b) a positive integer, (c) a whole numbet
(d) eithera2ora3, (e) a number less than 8.

22. There are 500 students in a school. Each of them bought one ticket for the school anniversary
p.rze draw. The 500 tickets, numbered I to 500 will be put into a barel and the wiming ticket
drawn,
(a) Marc has the ticket numbered 288. w}lat is the probability that he will win the prize?
(b) Find the probability that the winning ticket number will be greater than 325.
(c) Esther says that either a girl or a boy will win the prize, so the probability that a 9tu1 will
. i\ t_T\ \he correcrl
win lhe Dfi/e -.

23. A lette. is selected of alphabets. Let E be the event that the letter
at nndom from the set
selected can be found in the word 'SUNDAY'.
(a) Li\l lhe sample space.
(b) Find P(t') and P(t').

@ r",n..",", ^,",,u
.1.!- Sandra chose a month of the year at mndom.
(a) Write down the sample space.
(b) L.t A b" ,h" .u"n, lhat the name of the montb begins with the letter M Express A usrng
the listing method. Hence find P(A).
Express B
(c) Let be-the event fhat the name of the montb begins with the letter A or '/-
I
using the listing lnethod- Hence nnd P(B)
(d) -
Let be the eveni that the name of the month contains the letter R' Express C
using the
listing method Hence find P(C).

the
:5. A bag contains 30 balls numbered 1 to 30. A baU is dlawn at nndom from the bag and
number is recorded,
(a) List the sample space.
(b) Let be the event that the number on the ball drawn is a multiple of
I' 5 List all the
possible outcomes of the event
(c) Find P(F) and P(F').

3' A 3 digit number is fomed at random using the numbers 0, 3, I and 5 The digirs cannot be
repeated.
(a) List the sample space.
the listing
(Uj I-et C Uc tle event lhat the number formed is an even number Express C using
method.
(c) Find P(C) and P(C).
iaj l"t O tt" ttt" "u"nt that the number forried is at least 350 Express D using the listing
metnoo.
(e) Find P(D) and P(D').

Chaptei 13: Prcbabilitl


@
27. Trckets are numbered I 1() are then shufflcd and placed into a bag A licket is the:
50 They
drawn at raodom liom the bag. Find lhe probability that the ticket drawn is
(a) an even number, (b)
a multiple of 7.
(c) a nunbef less than 33, (d)
a pedeci cube,
(e) not a perfect cube.

28. A bag cortains some btlls numbered 30 to 43. A ball is drawn at random from thc bag Fin'l
the Drobabilitv Ihat the number on the ball drawn is
(a) odd. (b) a prime nuDber, (c) morc than 33,
(d) divisible by 8, (e) a pefect cube.

29. Car{ls with the numben 3 to 102 are placed into a box A card is then drawn aI random fioE
the box. Find the probability of dnwing a number that is
(a) even. (b) less than ll,
(c) a prime number less lhan (d) a mulliple of 11 greater than 100
30,

30. There are 4 cards in a box numbered 2' 3 and 4. Two cards are drawn at random from lh:
l'
box to a fo.m a 2 digit number Find the probability that the number forned is
(a) less than 20, (b) grcater than 30, (c) a mulliple of 12,
(d) a pdme number, (e) not a prime number.

Mathematics fttor 28
@
31. A two digit number is selected at random. Write down the probability that the number selected
rar i. a pedecr \quare.
(b) is greater than 80,
(c) has at least one digit which is 2,
(d) is a muttiple of 11.

32. A card is drawn at random frcm a deck of 52 playing cards


Find fie probabilrly thal lhe card draun is
la) an ace, (b) an ace of clubs, (c) a spade,
(d) a picture card, (e) not a pictue card.

-i3. A card is dmwn at rardom from a deck of 52 playing cards.


Find rhe probabiliry lhat the card dra$ n is
(a) a red card, (b) a 3, (c) eitheraTora9,
(d) a king, (e) not a queen, (0 either a pictue card or a diamond.

34. All the picture cards are removed frcm a pack of 52 playing cards. A card is then drawn at
random from fhe rcmaining cards in the pack. Find the probability that the card drawn is
(a) a rcd ca.rd, (b) not a heart, (c) not a 5.

ch.pler 13: Prcbrbilily


@
35. A card is drawn at random from a pack of 54 playing cards. Find the probability that fhe card
drawn is
(a) a red king, (b) a black card, (c) eithef , jrck or a queen.
(d) aioker, (e) not a joker

36. The diagram shows 12 cards. A card is selected at mndom. Find the Drobabilitv of selectins
(a) a heart, (b) a club, (c)
a picture card,
(d) an 8. (e) either a 4 or an 8, (1) a queen of hearts.

The diagram shows a circle divided into sectors with different colours. A point is selected ar
random inside the circle. Find the probability that the poinl
(a) lies inside the blue secfor, (b) does not lie inside the blue sector,
(c) lies inside the orange sector, (d) lies inside t}Ie red sector,
(e) lies inside the red or yellow sector

r",*.,,., on*,u
@
In the diagam, O is fhe centre of the larger circle of mdius 14 mm. ZAOB = 90" and the radius
of the smaller circle is 4 mm. A point is selected at random inside the larger circle. Fi[d the
probability that it lies inside
(a) the smaller circle,
(b) the sector AOB,
(c) the shaded region.

The pie charl shows the propol'tion of T-shirt sizes wom by the employees of a company:
Small, Medium, Large and Extra Large.
(a) Find the value of,I.
(b) Ar employee is chosen at random fiom lhe company. Find the probability that *re
employee chosen wears a T-shirt of size
(i) small,
(ii) medium,
(iii) large.

The diagmm shows two circles of radius x cm and 4r cm. A point is selected at random inside
the larger circle. Find ihe prcbability that it lies inside the shaded region.

Chapter 13: Probability


@
4l . The diagram shows a circle of radius 20 cm and a scmicircle- centre O and radiu s 8 cm. A ooiT
is selected at randorn inside the circle. Find the Eobabilitv that it will lie inside
(a) the semicircle,
(b) Ihe shaded region.

12. The diagram shows 3 concentdc circles, centre O of radius 2 c1n,4 cm end 8 cm respecdveh:
A poinl is selected at random inside the largcst circle. Find the probability tbat the point rl:l
lic inside
(4.) region A,
(b) region C,
(c) region A or B.

The diagram shows I squafe ABCD. P and 0 are the midpoints ofAD and AB respectivel)'. r-
point is selected at nndom insidc the square. Find the probability that it lies inside rriangl:
APQ.
AQB

@ r.,*.",", r*- ru
The diagnm shows a dalt board. Each quadrilateral in the diagram is a square. A dafi is thrown
at random and hits a point on rhe board. Find ihe Fobability that it will land inside
(a) region A, (b) rcgion B, (c)
region C.

y and Z When ball is selected at random ftom the bag,


A bag coltains some balls labelled X a

it is labelled x is
t7and the probability that it is labeued f is
the probability that i Ls
.

(a) If a ball is selected at random fro-m the bag, find the probability that it is labelled Z
(b) If there are 86 balls labelled Z in the bag, find the total number of balls in t}le bag.

There are 20 grcen balls and r brown balls in a bag. A ball is drawn at random from the bag.
(a) Write down, in tems ofx, an expression for the prcbability ftat the ball drawn js brown.
rbl Given thdl lhi. probabilily i' t9.
find lhe \alue ofr'

Chaprer13:Prcbabirily
@
rcd marble and
17. A box contains 120 red, green and blue marbles The probability of drawing a

x green marble Irom lhe bo\ clc 5 irnd : le'nccli\el]


(a) Find the number of blue malblcs in thc hox-
itl wnen blue marbles nle removed fiom the box' the prcb:rbility of drawing a blue
"
m:uble frcm the rcmainirg marbles in the box becom"s Find tht value ot
ft ''

48. There are 27 boys and r girls in Class 24


3
(a) Find the value ol i if the probability of selecting a boy from Class 2'A is l'
2
(b) Fiid the value of ir if the probability of selectiig a girl from Class 14 is
t1

puryle clayons ani


49. There are 32 puQle creyons end 28 brown crayons in a box Alter 'rt
(l + 2) brown irayons afe removcd fron the box, the probilbilily of sclecting a brown crayon
'I
fiom the box becomes Find the value ofr-
t.

t^,n"-,'., tn.. tu
@
Abag contains 50 balls. n ofthem are black and the rest of them are red. When 6 more rcd halls
5
are added to the bag, the prcbability of&awing a fed bcll ftom the bag becomes Find the
t
\ tiue o1\.

There are 30 studeDts in the hall. t of them are girls


(a) Write down in terms of .r, the probnbility that a student chosen at random from the
grouP rs
(i)
a girl, (ii)
a boy.
(b) 8 more girls and 14 more boys entered the hall. The probability of choosing a girl lrcm
3
the hall becomes Find the value ofn.
|3.

A bag contains 8 red balls and 4 blue balls.


(a) A ball is drawn at random from the bag. Find the p.obability of getting
(i) a red ball, (ii) a blue ball, (jii) a grcen ball.
(b) One red ball is removed liom the bag. A ball is then drawn at rundom fiom the bag. Find
the probability of getting
(i) a red ball,
(ii) a blue ball,
(iii) eitler a red or a blue ball.

Cbrpd 13:Probabilriy
@
53. Each of the numben from 000 to 999 is written on a ball and ali the balls are p]aced insrde z
conlainer A ball is then druwn at nndom from the container. Find the probability that the bBl
drau n
(a) bas 3 digits that are the sane,
(b) hes the lasr digir 8.
(c) has the last digir which is no1 8.

54. A packet contains 30 lollipops. 12 lollipops are strawberry flavoured and have red w.appeE
I lollipops are vanilla flavoured and have white wfappels.s lollipops arc strawbeffy flavoural
and have white wrappers. ,+ lollipops are vanilla flavoured and have red wmppers. A lollips!
is picked at random from the packer. Calculale the prcbability that rhe lollipop picked
(a) is vanilla flavoured,
(b) bas white wrappers,
(c) is strawberry flavoured and has red wrappers,
(d) is vanilla flavourcd and hns green wrappers.

55. A roulette wheel hrs 38 slots around the rim. The first 36 slots are numboed 1 to 36. Halfd
these 36 slots are red and the other half are black. The remaining 2 slots are numbered 0 asd
00 and are green. As the roulette wheel is spun in one direction, a small ivory ball is rolled
along the rim in an opposite direction. The ball has an equally likely chance of falling into alrl
one of the 3E slors. Find the probabiliry that the ball
(a) lands in a red slot,
(b) lands on 0 or 00,
(c) does nol land on a number from 1 to 16,
(d) lands on an odd number.

@ t'n.-",,.,,*-,"
There are 18 red balls and 22 blue balls in a bag. 10 of the red balls and 4 of the blue balls have
a star painted on it.
(a) A ball is drawn at random from the bag and placed into a box. Find the probability that
rhe ball in lhe box is
(il red.
(ii, has a qlar painted on it.
(b) The first ball dlawn is a red ball with a star painied on if. It is not retumed to the bag. A
ball is then drawn at ra[dom from the remaining balls in the bag. Find the probability that
lhe ball drawn
(i) is a led ball witi a star painted on it,
(ii) is a blue ball with a star painted on it.

A bag cortains 5 bl;ck cubes and 13 ihite cubes.


(a) A cube is dr.iwn at randam from the bag. Find the prcbability that it is
(i) blnck, (ii)white,
(iii) rcd, (iv)either black or white.
(b) 7 green cubes are then added to the bag. The cubes are thoroughly mixed before a cube
is drawn at random from the bag. Find the probability that the cube dmwn is
(i) black, (ii)greer.

The number of pets each of a Broup of 20 children in a kindergarten has is given below.

A child is chosen at random frcm the goup. Find the probability that child chosen has
(a) no pets, (b) 2 pets, (c) 2 pets or more.

Chapter 13: Probability


@
59. The table Derow shows the number of poted plants each household has in a block r{
apartments.

.Numbei o 0 I 2 l 1 5

er o[ housefioldd -t2 21 8 6

A household is chosen at random from the apartment block. Find the probability tlat rh
household chosen has
(a) 2 potted plants,
(b) more than 3 potted plants,
(c) no potted plants.
(d) I potted plant or less.
(e) more than 5 potted plants.

60. A die is rolied 200 times and the results are rccorded in the tablq, below

(a) Find the probability of getting


(i) a 2.
(ii) a 4.
(Give your answcr in decimals.)
(b) Is the die biascd?

61. The tnble shows the masses, in kilograms, ol a group of 60 students.

45<r:<50 50<r<55 55<i<60

A student is chosen at mndom flom the group. Find the probability that the student chos.r
werghs
(a) over 50 kg,
(b) morc than ,15 kg but less than or equal to 55 kg,
(c) not morc than 60 kg.

t.,n".",'* t*- ,u
@
sNG s't'-
r@1
Be^ eoRNg=
A box contains 865 red marbles, 529 gleen marbles and 324 white marbles.
(a) Find the number of white marbles that must be added to the box so that the probability

or drau ing a u hire mrrute become' j .

(b) Find the number of grcen ma$les that must be rcmoved from tbe original box so that the

probability of drawing a green marble becomes


f,.

On a biased six-sided die, a score of 2 is twice as likely as a score of 6. A score of 6 is twice


as likely as a score of 1. The probabjlities ofa score of l, 3, 4 or 5 are allthe same. Calcrrlale
the probability of a score of
(.a) 2,
(b) 6,
(c) less than 4.

Ar intege.js chosen at random from 1000 to 9999- Find theprobability ofchoosiqg an integer
having 4 different digits such that the difference between the first and last digits is 2.

Cnapt€r t3: Eobability


@
4. The diagmm shows a circle of radius 14 cm- A square is inscribed in the circle A-pomt
selected at random inside the circte. Find the pmbability that it lies inside the shaded regi(
22
I Take lt = 7.]

5. 3 red carals are numbered 1 d 3 Another 3 green cards .tre numbered 2, 3 and 4' Two
,2 a
are d.rawn at random, one red and one green and the sum of the numbers on the two cards
recorded. Find the probability that the sum is
(a) 5,
(b) even,
(c) odd,
(d) a prime number.

6. Two unbiased alice are rolled together Find the probability that they will show
(a) the same number,
(b) two different numbers,
(c) two even numbers,
(d) two odd numbels,
(e) one odd and one even number

t",n"-*", r,.. r"


@
Statistics
Probability

The dot diagram shows the time taken, in min, by 30 children to fix a jigsaw puzzle.

5.0 6.0 '1.\) 8.0 9l) 10.0

Tnne (m'n)
(a) Find
(i) the mode,
(ii) the median of the distribution.
(b) Write down the shortest time taken by a child to fix the jigsaw puzzle.
(c) 20% of the children took at least ,I minutes to fix the puzzle. Find the value of r.

The hourly wages of 20 employees in a company are given below.

30 15 40 45 50 55

Hourly wases ($)


(a) Find the mean, median and mode of the dat4.
(b) The data is to be represented in the stem and leaf diagram below. Complete the stem and
leaf diaBraln.

Stem Leaf
3
3
4
4
5
5

RdisionErerc,se6.@
3. A box contained 24 packets of peanuts. The number of peanuts in each packet are given belou.

(a) Complete the dot diagram to rcpresent the number of peanuts in each of the 24 pack€lr

80 83 84 85

(b) Write down tbe modal number of peanuts.


(c) Find the fraction of the packets which contains less than 82 peanuts
(d) A packet of peanuts is chosen at random from the box.
Find the Fobabiljty that it contains
(i) 85 peanuts,
(ii) at least 85 peanuts.

ron"."o* r*-,u
@
The stem and leaf diagram below shows the masses, in grams, ofa shipment ofpapayas from
a plantation.

,7
35 I 3
36 U 0 4 5 8
31 2 2 5 5 6 8
,7
38 U l I 2 4
39 3 1 5 8
,10 0 0 2

(a) How many papayas are there in the shipment?


(b) Find the mass of the lightest and heaviest papaya.
(c) What fraction of the papayas weighs less rhan 375 g?
(d) The papayas are gmded according to their mass from the lightest to the heaviest with A,
B, C and D respeciively. Wlat is the minimum weight of rhe rop 2570 of this shipment?

The stem and leaf diagram shows the ages, in years, of 27 people who took part in a Birgo
competrtron,

Stem Leaf
2 8 9
3 5 1
4 I 3 6
5 0 0 4
6 2 2 3
1 1 3

(a) Fjnd the mean, median and mode of the disfiibution.


(b) Find the percentage ofpeople who arc more than 65 years old.
(c) Write down the age of the youngest and oldest competitor
(d) Describe briefly the shape of the distribution.
(e) A person is selected at random frcm the group. Find the probability that the pe$on
.elecred i\ le\\ than 50 )ealc old.

RwisionErercise6
@
6. The diagram below shows the heiShts of the employees in a company'

Men Women
t4 8 9 9
,7 '/ 1 6 0 0 3 3 5 6 6

9 8 5 5 2 2 t6 l -+ 5 5 5 8 8

9 6 6 6 5 1',7 2 3 3
3 1 0 18

(a) Find the number of people oL each gender in the company


(b) Write down the modal height of the employees'
(c) W.ite down the height of the tallesl and shortest employee
(d) Find the ratio of the number of male employees to those of female employees who
above 150 cn but less than 170 cm

are 52500 each


7. A uomoanv has 7 emplo)ees lhe monthly salary of three of the employees
The ..,|lry ol lhc |e\r dre \2800 \'1000 \5'lU0 rn'l $8000
'nonrtrtl
(a) Find
(i) the median,
(ii) the mode,
(iii) the mean of the monthly sahy per employee'
new mean is Xj36m_
(b) When the salary of a new employee is Mded to the ebove set the
Calculate the salary of the new employee'

@ r**.**.. ,u
The mean of the numbe$ 14, 1 2, 16, 15, 11,lT,randyis 14. Given that the ratio ofr to ] is
4 : 5, find
(a) the values ofr and ), (b) the median, (c) the mode.

t- (a) Given that fhe mode, the medran and the mean of four numbers are 88, 86 and 85
respectively. Find the mean of the smallest and largest number.
(b) The numbers, 2, 9, 10, 15, 18,22,25,26,28 and r are arranged in ascending ordei. Find
the value of'I if the mean is eoual to fhe median.

t0. The table below shows the number of days each of 40 students jn a class was late for school
in a ceftain month.

For this distribution, find


(a) the mode, (b) the median, (c) the mean.

In an experiment, each of the students in a class planted 50 seeds. They then counted the
numbea of seeds that germhated.

(a) How many stndents were there in the classt


(b) For this distribution, calculate
(i) the mode, (ii) the median, (jii) the mean.

RdisionErercise6
@
12. Asurvey conducteal to find the numberofpets each sludent has is rccorded in the lahle helrt.-

(a) Write down tbe largest possible value of-t. given that the mode is I.
(b.) Write down the largesl possible value of .t, given that the median is I.
(c.) Calculale the value of r:, given that the mean is l.

I3. The table shows the numbel of hours spent by each student to complete an asstgnmeDt.

(a) If fie distdbution is bimodal, f-ind the possible value(s) of


(b) Find the possible values ofr: if the median is 4
(c, ll rhe mean i- {.34. ilnd ihe \rlue of.r

14- A fair six-sided die was rolled 100 times The number appearing on the die each timr 15

rccorded below

(a) Showthati+l'=32.
(b) If the mean is 3.5, show that 3r + 4) = Il4
(c.) Find the values of't and 1,.
(d) Hence, state the mode and median of the distribution.

,*- ,u
@ "",*-,u*
In a suvey, a group of 150 people was asked the number of watches each of them owned. The
re\ulls tu-e recorded in lhe lable below.

(a) Given that the mean number of watches per penon is 2.6, find the values of and
(b)
', ).
Hence, wri!e
{ir lhe modal number of \ atcbes per person,
(ii) the median number of watches per person.

The masses, in kg, of 50 students are given in the table below.

(a) Complete the table below.

(b) Estimate the mean mass of the studenls.


(c) Find the percentage of students who weighs at least 50 kg.

RelisioiExdcise6
@
17. The time taken, in seconds, by a group of students to send an SMS are given below.

(4, Write down the modal class.


(b) Complete the table below

40<t<45
45</<50
50<f<55
55<r<60
60<r<65
65 < t <70

(c) Estimate the mean of the distribution.


(d) Calculate the percentage of students who took I minute or less to send the SMS.
(e) A student is chosen at random iom the group. Find the probability that the
chosen takes
(i., more lhan 55 \econds Lo send lhe SMS.
(ii) 50 seconds or less to send the SMS-

r",'"-ou*
@ ^.,,u
40 light bulbs are tested and their lifespans, in hpu$, are recorded below

(a) Complete the table below.

(b) Calculate the mean lifespan of the 40 bulbs.


(c) State the moalal class.
(d) Write down the class interval where the median lies.
(e) A light bulb is selected at random frcm the group. Find the Fobability that its lifespan is
shofter than 160 hours.

RdisioiExdcise6
@
19. The table below shows the waiting tines, in minutes, of 30 people to ride on a roller

(a) State the modat class.


(b) Calculate the mean waiting time of the people.

20. The amount of money spent by 40 shoppers at a store are recotded in the table below.

Calculate an estimaie mean of the amount of money spent, giving your answet conect
the nearest cent.
(b) Write down dre modal amount of money spent.
(cl Shoppers uho spent at leasr $60 were given a free gin. Find lhe percenlage of
who will receive a free gift.
6 red balls and 12 green balls are placed in a bag. A ball is then drawn at random from the bag.
Find lhe probabiljly lhal lhe ball drawn i<
(al green,
(b) red,
(c) blue,
fd) either red or green.

A fair die is tossed. Find fte probability of gefting


(a) a 3,
(b) not a 3,
(c) an odd number,
(d) a factor of 12,
(e) a number smaller than 1,
(0 a natural number

Each of the letters of the word 'TRIGONOMETRY' is written on a card. All the cards are
shuffled and placed into a box. A card is then drawn at random frcm the box. Find the
probability that the card drawn is
(a) an R,
(b) either a I or an M,
(c) a vowel,
(d) not a vowel.

RdisionErrcise6
@
24.6cardsnumbercdl,3,4,5,Tand9areplacedintoabag.AcardisthendrawnatrandomftE
the bag. Find the probability that the number on the card drawn is
(a) a 5,
(b) a prime number,
(c) an odd number,
(d) greater than 3.

25. A traffic survey is caried out at busy T-junction for one hour The probability of a car
a tumiE
dght, going staight aiead and tuming left are given below
(a) Find the value of,rr.
(b) If 450 cars used that junction
during that hour, find the lumber
of cals that tumed left.

26. Two coins are tossed at the same time.


(a) List the sample space-
(b) Find the probability of getting
(i) two heads,
(ii) a head and a tail,
(iii) at least one tail.

@ ""*"-",n, t".. ,u
Three coins are tossed at the same time
(a) List all the possjble outcomes.
(b) Find the probability of getting
(i) three heads,
(ii)two heads and one tail,
(iii) no heads,
(iv) at least two heads.

Some balls numbered 30 to 59 are placed into a box A ball is then drawn at mndom from the
box. Calculate the probabilily that the number on the ball drawn is
(a) less than 42,
(b) is grcater than 45,
(c) has the digit 3 in it.

There arc 6 red balls, 8 green balls and I 0 yellow balls in a bag. A ball is &awn at mndom from
the bag. Find the probability of drawing
(a) a red ball,
(b) not a red ball,
(c) either a rcd or yellow ball,
(d) a blue ball.

R*isionErercis"6
@
10. r- { -1. 0. t. t.s. 2l
An element is selected at random from the set I and is denoted by i. Find the probability
each of lhe following.
(a) .,r < 0,
(b) l- r.
(c) r is a natural number.

31. Marc selects a day of the week at random to play badminton.


(a) List the sample space.
(b) Let G be the event that he selects the day of the week which begins with the lener
Express G usiag the listing method.
(c) Find P(G).
(dr Find rhe probabitiry lhaL lhe day tiar he \elecrs contarns
(i) tie lener n.
(ii) fte letter I,
(iii) 6 letters.

32. A card is dmwn at random from a deck of 52 playing cards.


Find tle Fobability that the card drawn 1s
(a) a jack,
(b) not a jack,
(c) a l0 of hearts,
(d) a black card,
(e) either an ace or a picture card.

@ r****, ^.,,,
A card is drawn at nndom from a pack of 52 playing cards. Find the prcbability that the card
Jm$n is
fa) a queen of heafl\.
(b) a 9,
(c) a red ace,
(d) not a red ace,
(e) a picture card of clubs,
(0 either a 2 of spades or an ace of hearts.

A number is drawn at random from 2l to 50. Find the probability of drawinB


(a) an even number,
(b) a pdme number,
(c) a composite number,
(d) a pedect square,
(e) not a perfect square,
(t) a multiple of 6,
(g) a perfect cube more than 30.

R4isio,Ererciw6
@
The diag4m shows a spinner having 9 eqoal triangles. When the pointer is spun, find
Fobability that it wil stop at a tdanSle containing
(a) a 5,
(b) a natural numbeq
(c) a pdme number,
(d) an integer,
(e) not an integer
(0 a number less than l.

36. (a) A hat contains some copper and gold coins of the same size. A coin is drawn at
from the hat. If the probability of &awing a copper coin ftom the hat is 0.65, find
Fobability of dftwing a gold coin from the hat.
5.Tbe probability
rb) Tbe probabitiry rhal a cheqs playerwillwin any particular game is

the player will end up ir a alraw in any particular game i. l Find th" p-bability that

ptayer will lose in any particular game.

M"6ena,ics ftto. ,B
@
The diagram shows a spinner divided into sectors having different colours. The pointer is spun.
Find the Fobability that it will stop inside
(a) the red sector,
(b) the blue sector
(c) the yellow sector,
(d) either the gleen or yellow sector,
(e) the sector containing the colours of the rainbow

The diagram shows two squares of sides x cm and 5.n cm respectively. A point is chosen at
mDdom inside the laryer square. Find the probability that the point lies inside the shaded
reSron.

The diagram shows a circle, centre O of radius 20 cm Two smaller circles or radius 2 cm and
6 cm rcspectively are dmwn inside this circle. A point is selected at random inside the largest
circle. Find the probability that it lies inside the shaded region.

RdisionErercise6
@
:10. (a) There were 36 red balls and 19 green bells in a bag After r green balls werc rcmoret
fiom the bag, the probability ofdrawing a green ball from the rcmaining balls in the b4S
becomes I.Finrl the value of -t.
(b) Abag conLined 49 blue papcl clips and 7'1pinl(paper clips After ,I blue paper clips a,nc
(x + 6) pink paper clips were added to the bag' the probability ofdrawing a blue paper clif

frorr rhe bdg become. :.)- Find the vdlue ol '

41. A bag-Acontains 12 gold balls and 9 silver balls-


(a) balt is dra*n at Bndom f-ron the bag Find the probability that the ball drawn is
(i) eold,
(ii) not gold,
(iii) neither gold nor silver'
(b) i gold balls and I silver ball are rcmoved from the bag A ball is then drawn from ft.
re;aining balls in lhe bag Find the probability that the ball drawn is
(i) gold.
(ii) silver,
(iii) either gold or silver'

t.,n"-n". t*-rn
@
The number of senior citizens employed by a group of 80 companies is shown below

A company is selected at mndom from the group. Find the probability that the corrpany
employs
(a) 4 senior citizens,
(b) at least 2 senior citizens,
(c) less than 3 senior citizens,
(d) not more than I senior citizen.

The arnount of pocket money received by a group of students in a certain week is given in the
table below.

10<r(20 20 <.r < 30 30<r<,10 40<x<60

A student is chosen at random from the group. Find the Fobability that the student receives
(a) more than $30 in that week,
(b) not morc than $60 in that week.

R.\r"r,nErcrclG6
@
Final Examination
Specimen PaPer I
Part I (so marts)

Answer ALL the questions.


A11 working must be clearly shown.

2r+5
L Civen ihal)+.jr='i-". g1p1655 v in lermsofx

In the diagram, AABE = AEDC, AE - 8 cm, BC = q cm and AAAE = 90o. Find lhe
of ED.

r"*"^",", r*- r"


@
Given that ), is direcdy proportional 10 the squaxe root of j and if ] = 2.5 when r = 9" fui
(a) an equation cormecting _{ and },
(b) the value of-r when v = 131.
'1

(a) t2l
(b) {11

The diagram shows the Mathematics marks of a class of 30 students in a quiz.

:.::.
0 510 15 20 25 30 35 46
Mathematrcs mdks

(a) Find the modal mark of the students.


(b) Wriie down the mark of the stlrdent who topped the class.
(c) The passing mark for the quiz ras 25 marks. Find the fraction of students who failed the
quiz.

t1l
(b) tll
(c) tll

Fidar Exminatioo sp€cinen p"p". r @


5. A map is drawn to a scale of 1 : 200 000.
(a) The distance between two fire stations on the map is 6.4 cm. Calculato the actual
between the two fire stations, givirg your answer in kilometres.
(b) The actual area of a reren oir is l5 km:. Calculate lhe aria on the map u hich
the reservoir, giving your answer in square centimetres.

Answer: (a) tU
(b) tz'l

6. In the diagram, A,{-BC is mapped onto MB'C'by an enlargement with centre A and
factor ,t. If AB = 6 cm and BB' = 9 cm and CC' = 6.3 cm. find
(a) the value of l,
(b) tlrc length ofAC.

(a.J t1l
(b) Ql

'7. A card is drawn at Endom from a pack of 52 playing cards. Find the probability that the card
dmwn is
(a) a red picture card.
(b) an ace or a 10,
(c) not a diamond.

Answer: (a) tl1


(b) IU
t11

tu,*.u,*, ru
@ ^..
(a) Factodse 2(r': 4) x(x + 2).
(b) Expand and simplify () + 3Xy - 5) - (y - 4)'.
(c) Solve6z:- 1= llz+9.

(a) I2l
(b) t1l

t2l

The diaglam shows a metal solid which consists of a cylinder and a cone.
(a) Find the volume of the solid.
(b) The solid is melted and recast into a hemisphere. Fhd the mdius of the hemisphere.
lTake 7t = 3.142.\

(a) t2l
(b) t2l

Finar Exanination specimen p"r.. r


@
10. Solve the following equations.
(a\ 3(2r + t) - 2(a - 7\ = s(x - ll)
/l \2 l
(br -2.J =-
l:
,^, 2t-l - .r+l +l=0
,., 4 l

Answer: (a)

(b)

(c)

11. Cotn bought r plastic ftles and y ring files.


(a) He bought 17 files altogether Wnte an equation in terms oft and y.
(b) Each plastic file costs $2 and each ring file costs $7. The total cost of the files was
Write down a second equation in terms of .I and ).
(c) Solve these simultaneous equations to ftnd the number of ring files he bought.

Answer (a)

(b)

(c,

(.i) "*0"**r*- *
(a) Three of the interior angles of a pentagon arc 68", 84" and 122' while the other two are
5x and 9x. Find the value ofr.
(b) The size of each interior angle of a rcgular polygon is five times the size of each exterior
angle. How many sides does the polygon have?

(al t2l

(b) I2l

12 students from differcnt families were asked how many of milk were usually
consumed by their families per month. The answe$ are:
5, 11,1, a, tt, 6, 4, 10, 3, 5, 11, 15
Find
(a) the mode,
(b) the median,
(c) the mean
of this set of data.

ta) tll
(b) tll
(c, t2l

Final E\amination Specmen Pt" ,


@
14. Tho curve ] = l 3r - 10 cuts the r-axis at the points A aod B, and the:).axio at the point C-
(a) Write down the coordinates ofA, B and C.
(b) Write down the,cootdinates of the lowest point on the gaph.

Answer: (a) A =

(b)

@ ".*..,0*
r"',,,
II 1so martsl Time: t hour 15 minutes

ALL the questions.


working must be clearly shown.

A (22 marks)
(a) Facrorise l5p.r - 44I - 5qx + l2py. t2l
(b) Express as.a single fraction in its simplest folm.
5_2+l t3l
2r-8 12-3r r-4
If I + y'z = 34 md ry = 7.5, find tho value of 5(r ])'z t2l

The diagram shows a cardboard consisting of 3 semicircles and a circle. The radii of the
semicircles are 14 cm, 14 cm and 28 cm respectively. The diameter of the circle is 14 cm. A
ribbon is then glued onto the edges of the cardboard.
(a) Calculate the area of the cardboard. tzl
(b) Find the lenglh of the dbbon used. tzl
22t
i'

Finar Exmination specimen P"r" t


@
pyramid with a square base, ABCD of side3
3. The diaqram shows the net of a right
i+ fr i, p"tp"nai"ular to '4' and is of length 25 cm'
"-.
Calculate
(a) the total surface arc4
(b) the volume
of the pyramid formed.

m OY n8 tl-water to a height
4. A rcctangular tank with a base measuring-1 6 T lt l]tfl--
into th-e^water
Ld ,n*s6 .J;l
"i"iii"lt. ;J. ualls of diameter I2 cm each are dropped
in cendmetre-q
in lhe uater level in rhe tank giving your anqwer ^.
the mass of all
", or materia ot aen-sitv i 25 g/cmr' calculate
' ' il"";;;;iil;"maae
i;j
metal balls, giving your answer correct
to the nearest kllogmm'
22.

@ ""*"*u*^.. ru
Section B (28 marks)

5. (a) On the Venn diagram in the answer space, shade the set A' n B'.

ttl
(b.)€= [ r :.:r rs an integef and 2 <xS 14 ]
A= { r : ris aprime number }
B={x:risaoddnumber}
C= { -r :ris amultiple of 5 }
(i) List the elements of A' a C. tll
(ii) Find ,?(B u C). tzl
(iii) Describe in words, rhe set B'. I1l
(c) If z(P) = 54 and ,(0) = 89 and P c O, find
(i) n(P tll
(ii) n(P ^u 8),
8). tll

Final Examinarion specimeD Papd I


@
t'. (a) The table below shows some corresponding values of) = r' 2x L Calculate the valur
of a. Il_

(b) Using a scale of 2 cm to rcpresent I unit on the r-axis and 1 cm to represed


1 unit on the l, -:Lxis, draw the graph of l, = x': -2x-l fbr-3<r<5. [-r
(c) From your graph, lind
(i) the value of) when x = 3.4, tr
(ii) the values ofr when ) = 0, tr.
(iii) the equation of the line of symmetry. tl,

7. The diagram shows a right-angled triangle ABC where AB = (r + 2) c.:t,


BC = (L\ + 3) cm and AC = (3r - 1) cm.
(a) Use Pythagoras' theorcm to fbrm an equation in x and show that it reduces ii
2r' 1lx-6=0.
Solve the equation 2r' llr - 6 = 0. ;-:
(c) Calculate fhe area of AABC.

r",'"..u*
@ ^.,,"
The lengths of service of 150 employees of an advertising agency are recorded in the table
below.

0<s(4 4<r<8 8 <s( 12 12<r(16 16<s<20

(a) Calculate the mean leng1h of service of the 150 employees. t4l
(b) Write down rhe modal class. t1l
(c) An employee is chosen at random from the agency.
Find the probability that the length of service of the employee chosen is
fi) more than 12 )ears. tll
(ii) 8 years or less. tll

Final E\dination Speinen P"*. t


@
'Final
Examination
Specimen Paper 2
Part I (so ln-t t Timei t hoe

Answer ALL the queshons.


Al1 working must be clearly shown.

l. Factodse the following.


(a) - 50),'z
18r"
@) 12p' '24p - 1s
(c) 13+s r-rts

(a) tq
(b) tq
(c) g
2. (a\ Expand and simpfiry (2ry + lt3x'zyz - 2, + 1)
(b) Evaluate by factorisation the value of 385 x 615 + 385'?

tal
(b)

t"*"*o*
@ ^..ru
Given rhar tr =
Eh+3 . e\pressll in tenns of l
{ /i _ 5

t3l

Solve the simultaneous equations:

3x+2r=13
7x+6y=41

t3l

Finar Exdniialion specinen Paper 2


@
5. If y is inve$ely propoftional to the square of t and the difference in the values of ] when

x = 2 and r = 3 is 5, find the value of] when x = ]

6. (a) Solve the equation 2t (5x + l) = 25x - 12.


,5
(b) brmpury - -.

(a)

(b)

t""*",", ,".. r"


@
A motorist travels a distadce of 40 km at an average of 80 km,1h before stopping for half an
hour's rest. He then continued his joumey at an average speed of 60 km/h for 20 minutes.
Find
(a) the total distance travelled,
(b) the average speed for the whole joumey.

Answer: (a) tzl


(b) t2l
In the diagram, BC // EF, AD ll FG, LABC = 90", AE = 3 cm, BC = 16 cm, EF = 4 cm and
FG=6cm. D
Calculate the lengths of
(^) BE,
(b) A4
(c) AD.

(a) ttl
(b) ttl
(c) I2l

Final Er.mrnchon Specimen P"*,,


@
9. The diagram shows Plramid A with a square base of sides 5 cm and a vertical height of 12 cm'
(a) Find the volume of drc Pyramid.
(b) Pyramid B also has a square base. It has the same height as
Pyramid A but its base length is twice that of Plramid A.
Calculate the total surface area of PFamid B.

Answer: (a)
(b)

10. The area of a plot of land is 5 km'z. It is represented by an area of 1 25 cm'7 on a map
(a) Find the reprcsentative ftaction (R.R) of the map.
itlOn ttre ploi ot tand, the distance between two coconut trees is I km 40 m apart Fin4
in cm, the distance between the two trees on the map.
(c) The area of a lily pond on the map is 0.025 cm2 Find the actual area of the lily pond

Answer: (a)
(b)

(cl

@ ""*"-u*.- r"
The table below shows the marks of a spelling test by a group of students.

Find
(a) the smallest possible value of x if the modal mark is 6,
(b) the possible values ofr if the median is 5.

Answer: (a) tll


(b.) t2l

The heights, in centimetres, of 16 men in an office were measured. The informabon 1s

displayed in the stem and leaf diagram below.

l1 8 9
15 0 5 t' '7

16 I 4 5 9
t'7 U 6 7 8
18 3

(a) Find the mode. median and mean of the distribution.


(b) Calculate the Dercentase of men who are shorter than 157 cm.

(a) t1l

(b) tll

Final Exanination Specinen P"t".,


@
13. If € = {11, 12, 13, 14, 15, ,6, 17, 18, 19,201,
P= {r : r is a primenumber},
0= {i:-r is an odd numberl and
R = {r : ,v js a multiple of 61,
find
(a.) P u R,
(b) P' Q,
(c) ^
n(.Q' R').
^

Answer (a)

(b) 11,

(c) tr,

14. A box contains 9 red balls, 6 blue balls and 5 green balls.
(a) A ball is &awn at random ftom the box. Find the probabiliry that the ball drawn is
(i) not red,
(ii) either red or green.
(b) 4 red balls arc then rcmoved from the box. The rcmaining balls are thoroughly mixdi
before a ball is dftwn at random from the box. Find the probability that the ball dras:!
is blue.

Answer: (a) (i)


(ii)

(b)

r*r"."0* ,,,.. ,u
@
4. The diagram shows the graph ofl = 9 - t.
(a) Using the same axes, draw the gftphs of
I ._
)= .rand)y=Jx+l)
(b) Fmm your graph, solve the simultaneous equations
(ii) 1= 9 a
I
5r=3r'+25 t2l
(c) What is the figure formed by the three lines and the line ,I = 0? t11

r"*"."n* r*- ru
@
I
Section B (28 marks)
5. The diagam shows a solid cylinder inscribed inside a hollow cone with a base mdius of42 cm.
The height of lhe cone ic 70 cm and lhe beight of the cylinder i\ 52.5 cm.
(a) Find the numerical value of the mtio
volune of cylinder
t3l
(b) Calculate the lateral surface area ofthe cylinder,
giving your answer conect to 1 decimal place.
t1l
(c) The cylinder is melted down and recast to
form a spherical solid. Find the radius of the
solid. t3l
ITake n =
=.1

rilal ennimtiu srcimo raner z


@
6. Mr Tan bought r pocket dictionaries that cost $180 from a supplier
(a) If he wants to sell the dictionaries at a pmfit of $3 for each dictionary, write down atr
expression, ir tefins of.{, for the selling price of each dictionary t2l
(b) H; sold 28 dictionaries at a profit of $3 each while the rcst were sold at $7 each write
down an expression, in tems of i, for the total amount of money he would rcceive for
.clling all the Jiclionarie\. If
(c) If he made a profit of $l00lor selling all the dictionaries, form an equatjon in 'I and shoc
that it reducis to xt 56x +'720 = 0- Hence find the number of dictionaries Mr Tan
bougrrt. t3l

@ '.*".,,o,^..,u
The time, in minutes, taken by a clerk to type each of 30 repofis are given below

(a) Complete the table below. Iit

t0- 14

15-19
20-24
25 29

30 34

35 39

(b) Find the mean time taken by the clerk to type each report. t2l
(c, Write down the class interval where the median lies. t1l

Fiial Exmiianon specinm Papd 2 g?


8. Answer the whole of this qu€stion on a sheet of graph papet
A ball is thrown from the top ofa building.Its position during its flight is given by the equaticxl
y = 72 + 6x I where y metres is the height of the ball above the ground and { metres is iL(
horizontal distance from the building.
(a) (i) Solve the equ a|Lon 0 = 72 + 6x - | . Ill
(ii) Descdbe briefly what the positive solution of the above equation represents' [I]

The table below shows some coresponding values ofr and ).

(b) Calculate the values of d and D. -Ill


(c) Using a scale of 2 cm to rcpresent 1 metre on the x-ilxis and 2 cm to represent 5 metre''
on th-e y-axis, draw the graph ofl' = 72 + 6t xzfor0 <r< 8 tll
(d) Frcm the graph, find
(i) the geatest height reached by the ball and the value ofi when it occurs tll
(ii) the distance the ball travelled hodzontally while its height was more than
75 metres. Itl

@ t",*-",", "*- ru
(e) Volume oflrnmid
= |t eu."-"u r n"ist't

A- P.vramids -; x 111 xl7


r. (a) Volune of pyramid
(f)
= I x Base area x Heisbt
= i Base eea ' Heighi
= ],, 1ro,.s1 xrz
- 320 cn3 =;.(6 11) 15

(b) VolDne of pyramid

x
2. (a) Basedea=8 x 8=64cm:
=1x Base alea Heisht
Area of 4 trieguld faces

=;x(6x6)xro =4. lt/Base He'cht]

= 120 ml
=4 lr"","J
Tolal slrface ate! of !'rmid
= Base ma + AFa of 4 trianguld faces
=64+160

o) Bsea@=6x4=24cm'
AFa of 4 lrimguld f@es

=z^ = 6.11.81+2^l; 4 121


AB=18+2=9m
Using Pythagorat th@rem on AY,AB, = 118.8 cm'
vB1 + 92 = 15'
vBz = 15'1
q Total surface dea of pyramid
= 144 = Base area + AJ€a of 4 tlianguld faces
= 24 + 118.8
VB = ,\^
(c)
Volume of pyranid

=; x Baxe rea x Height


=;.rr8.l8r '12
= 1296 tn"

(d) Volune of pyranid

=a xBasereax H€ight

_ .I Y)Y/.)J. r.6

Answer with compiere worked s.r.ttom


@
AB =4 +2=2m 3- Volum€ of let.ahedon
Using Pythagoru' theorcm on AYAB,
vBl-2'+6'1 =j x Basedeax Height
=40
vr=aftmm =j,,r:'+
-16m"
Base a.ea = 4 x ,1= 16 Imr
tuea of 4 tianguld faces

=4 lt 4.J4o]
=8'[o *'
Total surface ma of pynmid
= Base area + A.ea of 4 trianeultu faces
=16+8\,kJ
= 66.6 Im' (3 siC. fiC)

Height

= 107.8 cmtr

5. (a) Volume of pylmid

= j >< en,"-"o,. r"rgr't

= J t {r0,, rot ,. rs
= 1280 cm'
PB =28+2 - 14 n
Uslng Pythaeorat $eorem on AYPA.

= 480
vP =
"[so
.
QC=20+2=IOm
Using Pythagoras theoren on AVQC,
vQl + t01= 261
ve, = 261 tol
_ 576
Ler the sldt heighl be ta'cn.
vO =
"'Ea
Using PythagoEs' theorem on AYNX,
Base rea = 28 x 20 = 560 n' rT' = 15'+ E'
A.ea of 4 tdangula iirces
= 2lJ9

=2 [tY'6Y1146!']+2 < (! *zo'z+) vx = .ElJg


\2
= 1093.45 n' .. The slant height of thc pymid is 17 cm.

ToLal sLrlace area of pyaftid Total sutlace arca oflyamid


= Base dea + Arca of 4 trianguld faces = Base dea + 4 x Area ol triangb vac
= 560 + 1093.45
= r6s0 lll (3 sig. fie.)
=rt6^ lot+4^ l: . 16 ril

,"
@ "",n"'",'"' ^,o,
Volume of lymid = 48 cn' (Given)

a x Bale area x Height = 48 ,4X = 16+2=8nn


Aredoi 4acD=2 l' r0 8J
ax(6x6)rvA=48
48:3 = 80 mm'

=4cm . voln e ofpyramld = 880 mm'(Given)


.. The height of the pyramid is 4 cm. i . B6e rfl Hciehr = 8ri0
(b) AB=6+2=3cm i x 80 x Height = 880
380^l
Usjlg Pythagoi$' theorem on Avr{B. HeiSht =
vE =4'+! | = 33 tnm
_- i\ -
VB =rG The heighl of the pyrmid is 33 mm.
\ ..
=5cm etr]m :'"
(d)
.. The slarr height of the pyramid is 5 cm.
=ax Bdse aiea x Helght
Total suface area of pyramld
= Base a.ea + 4 x Ata of AYQR

=(6 x6)+4 x l;x6x5j


Total surfde afta of pyramid

r. = Base area + 4 x Area of triangula. face


(a) Volume of pyranid = 112 cm' (Cjven)

i . Bse e€ Hershr = ll2


=(9 x 9)+4x l: x9x 1r]
i; x?r x Height = l12 (0 tf,t the length of a side of t\c square base be i cm.
ll2 Total rurface dea of pyramid = 96 cn' (Giver)
HeiChi =
- Bate area + 4 x Aea ofttansuld face = 96

(rrr)+4x /12| xrxiJ=e6


_\
.. The heighl ol Lhe pltanid ls 16 cln.

(b) Let the length of a side of the squde base be r n. rr+ loa 96 =0
volune of pyramid = 405 mr (Gi!en) (r 6)(r + 16) =0

ax Base dea x Height = 405 '.i 6=0


or r = 16 (rejeded since
i; x (r x r) xJ'=405 lenglh is losilive)

,. = 19r
-'5 . The lenSth of a side of Lhe squee basc is 6 cm.

=8i
, = ./8 t

.. The len8th of a side ofthe squarc base is 9 m.


Answen with compreLe worted s"r,lt.'" \143)
Let ih€ sldr height of the pyramid be n m. 8. (a)
rorar sunace ,rea or pyrmo = ruu m (urveo)
Bale area + 4 x Area of t.ialguld face = 200

18^8r+4^ l:.8.hl=2fi)
64+l6h=2N
16, = 136
l=8.5m
.. The slant height of the !,mid is 8.5 m.

(h)

Using Pythagorat theoem o. A,4AD.

AD _ ,\t_ .n
Total sultace area oI t€trahedrcn
= 4 x Area of AAAC
Il ,
=4.lrYoY\ll/J-\
- 62.4 .'f (3 sic. ficJ
Let the iengrh of a side of the sqraie base be r mn.
Volume of pyramid = VolDme of cnbe

I Base eed Heishr = 8'

\t x (x x x) x7= 5r2
512
'
8cm ' rcfr

, = .,66 9. (a) (i) rectangular pymnid


=16mm

. . Trp length ofa side ofthe sque baw is 16 bn.

C) tf,tthe height of the pyrmid be /, cm.


Volume of cube + 6 x Volume of py.amid

:'-ox []'o
27+18h=l),1
18t = 90

.. The heighr uf edch p)rnid i\ 5 cm.

v*r'"-"'i" r"' zo
@
(ii) Total surface dea of pyramid 11. Volume of pyramid
= Area of base + A.ea of ,1 tnansulaf fac€s
= - t l44X 15
=(8x6)+t2x(+ x8x9)
+2x(;r6xe.4)l Volome oI water in the tdk
= (30 x 30 x 15) 720
(b) (i) = 12 780 cm3

Let the height of the water level afief the pyramid is


removed be, cm.
30x30xn=12780
l2 730
'' :10 r 30
= 1,4.2.m
.. The required height ls 14.2 m.
12. (a) Tota] surface dea of pymnid fomed
= Base &ea + 4 x Arca of AYCD
(ii) trt the sldt height of the pyranid be I cm
=(14 x 14)+4 )< (t x 14 )< 25)

___+r-+
BC= 10+ 2 =5 cm
Using Plthagor ' rheoren on AdaC,
h1 + 51= t31
h1 l31-5'1
- 1.14

h= Using Pythagoras' theorem on AVPo,


'\^
=12cm
Total suface dea of pyranid
= 576
= Base a.ea + 4 x Area of AAAC vp = ]s%
=(10 x 10)+:l x (t x 10 x 12)

= 34O cm' Volume of pylmid tblrned

10. Let the height of the squatr pFAmid by ,'? cm. = J Base area Height
. . Height of rectanguld plnmid = 2h cm. I
let the length of a side of the squde base be r cm. =ax(14x14)x24
1\plramrd,lpyramro _Il = / Volume of souare I
/ Volune ot recaneuldl
I

*.(9 8r.Zt=t lx ,) J

r = {44

.. ftre lengiuld'ideot'he'ouarebr\eot,5ep)ranid

Answers with comprere worke,i sorutios (a


13. (a) 15. Heigbl of pyraDd = 21 - 12 = 9 cm

= Volune of cube + Volumo of !).lmid


= rt0 6 l2l+ ti lt0 . 6r \ 91

' Total mas of soud


= Densiry x Volume
= 9.8 g/cnr x 900 cmr
= 8820 c

(b) (i) Total surface e€a of Pyrmid


= Base dea + AJea of 4 tneCuld faces

= (42).30) +12x (+ t!'x25)


+2x(;x30x2e)l 16. (a) Height of!)tmid = 52 16 = 36 cm
= 3180 m': Volune of pyrmid

(i0 =i x Basedea x Height


=ax(25x14)x36

Voiume of Pnsm
= Area of cross{@tion x Len8th

-(;x14X16)x25
py=42+2=2lcm = 2800 cnr

Using PYthaSoEs' the@m on AOPI, volume of solid


oP + 2t'z= 24 = Volune of !)mmid + Volume of lrism
oP'1=291-21'z = 4200 + 2800
= 400 = 7000 cml
Op= .[4[n)
= 2Cr cm
(b) DeDsily = volufre
.. Th€ height of the Pyrdid is 20 cm.
56 000 g
= ?ooo;;l
(iii) volume of lyramid
= 8 c/dn"
= J dea Hei8hr
Bae

=i .112 /l0r .20 17. (a)


= E400 cm'

14. Let the slanl height of the pyramid be I cm


Total sDrface aJea of D'ranid = 965 ctr': (Civen)
Base area + 6 x Area of AOAB = 965

260+6 lJ L0 hr =965
260+30l'=965
30n = 705
h =23 5 cm
-. The sldi height of the Pyramid is 23.5 m.

u.rr'eoatics r.rnr zn
@
Usiry Pythagoral th@rem on AYAD. (c) Toral surface area
. /t Y - = 3 x AEa of AYAC + Aroa of ADEF
vD'+l1r)=r + Perimete! of ADEr x CF

r.'t.8 ffi,-,; t ,ot'


(;'l +t(8+8+8)x201
#
3,
it B. Con€s
= 669 (3 sig. fis.)

=1*
19. (a) {i) Volume of cone
vD =
tr;r = anth
- '4. -- 122
=ax 1x7.x24
Base dea of t€rrahedrcn
= 1232 .m3
I
=rxABxvD (ii) Total surface trea of cone
t - ua' = 1Lt" + 1ttl
= t'"' )
=; 1'+; - 1" 2s

= 704 cn'
(b) (l) Volum€ ofcone
(b) volume of ietatEdror = 60 ctrl (civen)
= 7E'h
: ^ Bde ma ^ Hersh! = 60
22-.,-'r
| -/',1
r:: . 10=60 -1:1 7
J . 2-
60x3x4 = lr4; m"
"" - J3xlo
= 41.569 (ii) Total surface area of cone
r = a[116, =#+ftt
= 6.4s cn {3 sic. fic)
=; xs'+; x5x13
18. (a) Using Pyftagoras' thorem or Ar44C,
= 2421 n'

= 180 (c) (i) Volune or cone


VM = ..lrso
= 13.4 n (3 sig. ng) = anlh

=\xlxzr'xzt
= 12 936 n'
(b) Using PythagoEs' theoren on ADEN,

Dlr' = {!8
= 6.93 n (3 sic. fic)
._-8n_ B

Using Pyliagoms' thmrem on A,4rc,


BC =21'1+28'z

Answes with conprere worked sorudds (3D


BC = -tE22s Total suface dea of @ne

=35m
22^2
. . The sldt heighi of the cone is 34 n = x 3.5'+ X 3-5 x 9.1
- -
= 88.6 cn'
Tol2l surface @a of cone
(t) (i) circumferonco of lop = 88 m
22.22 2Er = 8a
= x21'+i x2lx35
= 3696 n'
2t=tt=aa
88x7
(d) Le1 lhe base ndius of the cone be I mm.
Using Pylhagoras' theorem. = 14m
Let the height of cone be 11 m.
I = 1,8.21 1,6.81
168on Using Pythagorat theorem,
=4g
r= $t
= 110.25
(D h = .tflcr?J
= 10.5 m
= a"th
=i.;.r ro.8
= 1ltth
= 862.4 nmtr

(it
=r.;.r4',10.5
Total suriace area of cone
= 21s6 f

=; 1')+;-1 t8.2 (ii) Total surface dea of core

=; 4'4'+; t4-t7.s
(e) (t
. Base tuea Heisbr
= i 20. (a) votume of cone = 675 c'f (Given)

= . 38s ! 84 da
i i Bse Heisht = 67s
= 107,8 crn'
- ^ 15O^Heieht=675
(ii) Base a.ea of cone = 38.5 cm'
tu' = 3a.5
= l3-5 cm
;xl=38.5
.. The heidt of the cone is 13.5 cm.
, 38.5x7
'- 22
Let the base radiu of tho cone be / cm.
Volume of coDe - 2l12 cmr (Given)
t = .lr2E
= 3.5 cm l tih = 2112
Let the datrt height of the cone be I cm.
.;.t.14=2112
Using Pythagoray theorem,
11=a.4'+ 3.51 . 2lI2\3xj
= 82.81 8.4cn '_ )?']4
I = .l8z.u = )44
| = 1\44
=12cm
.'. The base radius of the cone is 12 cm.

t",n..,".t*-ru
@
Base radius of cone = 24 + 2 = 12 m (lr) Radius of cone = l8 - 2 =q m
vorume or cone = r/ob m (brvenl

!,sn - rst
x3.112x12:xh=3i6a
. 3764x3

= 25.0 m (3 sic. fic.) Let the heighl of the cone be , m.


. . The beight of the core is 25.0 m. Using Plthagoras' thdem,
(d) cured sudace area of cone = 275 c# (Given)
?tl = 275
= 144
_YrYt2.5=275
22
h = .t144
215\7 =l2m
' = 22 , 1L5
' =]cm volume of core = t Drn
Th€ base radils of the cone is 7 cm.
..
=- / 3-t4/,9'^12
Cufled surface dea of cone = 1l7r n' (Gven) = 101736 m'
rti = 1l1l
9xt=1t7 (t Base circumference = 88 cm
\t7 2nt = 88
'9 2x;x/=88
=13m
.. The slant beight ofthe cone is 13 m.
88x7

Total slrface area of cone

a^14^rF
'--ifi;---*
Let the slant heiehr of the cole be I t) Volune oicone - 24r cm' (Civen)
Using Pythagoras theorem, 1 x Base area x Heigh! = 24,'
3

= 625
t='Gt ix8,'xHeight=24,
Heisht = --
Total surface rea of cole =9cm
The heighr of rhe cone r\ 9 cm.

ffi
= 3.142 x.11 + 3.142 x1x 25
= 703.808 mm' 21. {.a)

G) Let tle slart heiehl of the cole be I cm.


Total suface arca of cone = 5500 cm': (Given)
rl+ft1
= 5500
3.142 x 28'+ 3.142 x 28 x I = 5500
2a63.328 + 87.9761 - 5500
81.9161 - 3036.672
I - 34.5 cm
(3 sis. fis.)

.. The sldt height of the con€ is 34.5 m.


Answes wirh compiere worke,i sorutiom (a
't'r-M = Volume of original cone Volume of cone ffnoved

= ;h8r'r2a)- idli24- l5r


= 512n 27r

2,4. tFt the heighr of the cone fomed be ft cm.


Volume of cone = Volune of cylindef

3
i9)'h = Aq'e4)
, 6'.u.3
22. (a) Let the base iadius of tbe cone be / cn.
Base rea ofcone= 38.5 cm'(Given)
'=n-=32cm
.. The heighr of the cone fomed is 32 cm.

1 38.s r'1
22

, = .ti,,:
= 3.5 cm

Total suface area of cone


25. (a) Let the base radius of the cone fomed be / cn
22"22 Volurne of cone = Volume of pymid
= - x 3.5'+ - x 3.5 x l2-5 1/
*.ar /, =
1/
I Btue ara . He,sbr

22
(b) Let the height of the cone be n cm. 7' rzx14 = (.27 x16)x33

A 22x14

r = 1Zr1
=18cm

ry_
Usirg Pythagoras' th@ren,
.. The base Edius of the coDe fomed is

(b) L.r the slant heighr of the cone formed be / cm.


l8 .m.

h' = 12.5' 3.5'

h-'l*
=12cm
Uling Pythagoras' theorm.
1

=- x Base dea ! Heisht = 520


I = ,6zo
=-X38-5X12
"-
Total surface dea of cone

22.22
= x1E +-xllJX J52o

= 2310 cn: (3 sic. ficJ

Mathenatics ruto 2B
@
26. Let the depth of waler in the cylindrical tank be I n. Total surface dea of solid
/c,*.d *.r*" -* )
=/curued sufme em\+
/ v"r,." "i *"1., ) = lv"tume or water in \ \ ofc)linder i \ orcone I
\rn c)lindrical tanl/ \ conical tank /
=12 ; 1-a2i+2 \; 1 251
lx t.5'x h = r x y'x o.9r x 3.5 .

'r".ilffi Llk
l

n.
(b)
= 294E o"

= Volume of cylider +2

t' ctt*t t1 ' ; '1'


x Volume of cone

.-. The depth of water in the cylindrical is 0.42


=ri 24\

- 8932 cn'

,t-l
1,.-

Height of cone = 16 - 1l = 5 n
LEt the s]dl height of the cone be I h.

Using Pythagoras' theorcm,


f-*+t21
= 169 29. (.) Sldt height of cone tbmed
I = ,iies

Total surface dea of tent CuNed surface aea of cone forned


=/cuned Nrfrce ro\ + /curued $fee dea\ = Area of sector oABC
\ ofcylindei / \ or cone )
= 12 t,t x 12 x 11)+ (r x 12 x 13)
= 1320 n' (3 sic. ficJ
=;xnx18'
2E. (a) Radius of cylinder = 14+2=7cm = 162n
Radius of cone = 7 cm = 509 m'z (3 sis. fic.)
Height of cone = (90 42)+2=24cm
Lel lhe base ndius of the cone fomed be n cm.
Curved surface dea of cone fomed = l62t cml
tRt = t62l
n(18) = 162
-
x= 162
rr
=9cn
Using PythagoEs' lheorefr, .. Tlle base radius of the cone fomed is 9 cm.

30. (a) Ler the base mdius of the cone fomed be r cm.
= 625
t = 'EE Cn umference of base ofcore = 16jrcm .

.. The base mdius of tbe aone fomed is E

Answe^ with cobplete work* *t'.* (rD


(b) Sla!! beight of cone fomed = 12 cm

Irt the heiSht of the cone fomed be t cm

UsingPythagons lh€orem,
(a) (i) tf,t the base radius of the cylinder be I cn
Bde area of cylinder = 8l n cm' (Giver)
/b;= 81,
l;=r5o ,_,ti
= 8-94 cn (3 sic. figJ =9cm
Base radiN of cone
.. The height of the cone fomed is E.94 cm. = B6e radius of cylinder

(c) Volune of cone fomed


.. The bde radius of the cone is 9 cm.
=;x,'x8'x J8o
(ii) Le1 ihe height of tbe cone be li cm.
= 594 cnoj (3 sis. ffg,)
Using Pytlagoraj tho€m,
31 (a)
tt + 9'= rs'
h'1=15'-9'
= 144
h = ,Eqc
-12cm
.-. The height of the cone is I 2 cm
L€t the bde radius of the cone forned be r cm
Volune of sold
UsingPythagoras theorem. of cone
= Volume of cylinder + Volume
r)122- q \r:l
-t-; r-24t,,i.7
= 8l cn
= ?124
,="EI
(in) Tolal surlace @a of solid
aBase ma I fcDNed surrace )
tuea of circle lsod to fom base of cone = [ofcyuaer)+ [dea ofcYlinder,
cwed surfee )
+f
= 81,11 \rea of mne ,
= 254 cnz (3 sig, ficJ
=r; e')+(2'7 o z+'
(b)

tu"u oi
+(;x9xls)
I . ) rco^.a,o.r"ce \ I Base area I
rccrms-ord
tuea ofcone lofcone
- sneer oi I lfomed 4.
2o?'6i .m'
I Lfomed l =
tpaper )
=(65x41) (rx9x41)-81,' (b) Mas = Densiiy x Volume -
- 1250 cn'z (3 siC. fiC.)
= 7.5 s/cm' x 7128 crn'
:160 C
= 53
= 53.46 Lg

Malho-atisrutd2B
@
c. spbrs (e) (i) VolNe of hemisPhere
2]
33. (a) (')
=at3.142tt2'
- 3619.6 nr (1 d.p.)
=Jx3.142r4.5',
(ii) Suface dea of hemisPhere
= 38r.8 cnr (1 d.p.)

(ii) Surface dea ol sphere =3 t 3],42 x Lt


= 13s7.1 In' (r d.pJ

= 254.s cln'(r d.pJ (0 (t Voldne of hemisphere


?-s
(b) (t
- \*' =]r:.t.+zrts'
= 7069.5 mr (1 d.p.)
-{r:.t+ztto'
- 41893 mr (r d.p.) (ii) surrace tuea of hemisPhee

(ii) Surrace dea of sphere =3x3.1:12xL5'


= 2120.9 cln'(r d.p.)
=4x3I12xlO1
= 1256.8 cn: (1 d.p.)
34. (a) volume oisphere= 960 cn3(civen)

(c) (i) =t =960


. 960x3

=;x3.142x8',
= 2144.9 nlm' (r d.pJ - 6.12 cn (3 siS. fig.)

(ii) Surface dea of sphere .'. The radius ofthe sphere is 6.12 m
volune of sphee = 7328 mmi (Given)
= 804.4 Inrn (r d.p)
- nl = 1328
(d) (i) Volume of hemisPhere . 7328x3
?-t
.llzzs v. t
=; x 3.142 x 6' \4n
- 12.0 mn (3 si8. tic.)
- 452.4 cln' (r d.p)
.. The radnB of $e sphere is 12.0 !m
(ii) Surlace area of hemisphere
volume of sphere= :1500u nl (Given)

fr' = 45AOl
=ix3.142x6- =
cm (r d.p.)
= 339.3
t. =__4
4500x1

.l45oo _r

=15m
.. The radius of tbe spherc is l5 m

AdsweB with conprele work",l sor'ttos


@
35. (a) Suface area of spherc = 530 cm'? (civeD) Surface ma of spherc
= 530
530 =4x3.1,1x151
= 2826 cm1

1530 Lel the height ofthe cone fomed be, cm.


' Volume of cone = Volume of sphere
= 6.49 c6 (3 sig. ficJ 1-4,(
.. The radius of the sphere is 6.49 cm. i 8'.h=i n 6

(b) )ufface .rea ol spncLc = i)o4d m tur\en]


, 4x6r
It'
4rl
= 664a
= i3.5 cm
, 6648
.. Thc height of tne conc ibmed is 13.5 cm.
i4618
Let the base radius of fte hemisphere be r cm.
Volume of hemisphere = 24 x Volune of spb.r.
= 21.0 m (3 sig. flc.)
1.4.
.. The radius oirhe sphe.e is 23.0 b. tt=u ; r t5'
Surhce ma of sphere - l69u cn'(Given) r 2,1x4!351
4il = \691
2

, t69 - 2058
, = $058
= 12.7 cm (3 sig. fig.)
/=i, ha, .. The radius of rhe hemispher€ fomrd i$ 12.7 cn
= 6.5 cm
.. The radius of the sphere ls 6.5 cm.

16. (a) sun(e ma ot spnerc = rr44 cm (urven)


22^
4 /.
= / t'=5541
. 55'14 x 7
Let the slot heighl of the cone be I mm.
=441 Using Pythagoras theorem.
/=1[4,
=21 cm
= 1369
I = ./i369

Let the mdius of the sphe.e be r hm.


422 Surface ara of sphere = CNed sudace ea ofcG
\t;=y't12x37
= 38 80E o" . \2\ 31
'4
Volume ofsphere= 14 130 cm'(Given)
= lll
-x3.l4xl=14 130
, = ./in
= 10.5 Im (3 sic. fi8.)

t=,1t:ll4 .. The radius of the sphere is 10.5 mb.

' - lTrai

r.,n".",". ,"'* ,u
@
o Volume of henislhere Let the hciett ofthe cone be t cm.
volume of cone = Volume of sphere
= t x Volume of cylindef 1^
- .. r1.5' ^ h = r543.5f
=;x(rx8'x24) , 15,1:1.5 x 3

= 15.l2 cn
Let lhe iadius of the hemisphcrc Lre / cnr.
.. The heieht of the cone formcd is 15.12 cm.
2.
1/t = 1t52/t
t) Volune of 70 hemlspheres
1 ll52xl
=70^-^,'r\1.5
/= 1/i7rs
Let the rise iD thc ratcr lcvol be /? cm,
/\52xh=151.5/
Total sudice area of henisphere . t5?.5

.. The nse ir the water level ls 6.3 cm.


= 1360 cn'? (3 sic. fic.)
(k) Let rhe height ofthe cylinder be ft cm.
Volumc of 200 br1l bearings
Volume of cylinder = volLde of sphere
= 2OO t 1fl/
y'\8-4xh= a ty'x4'
=200^:^z^0.8'
, 4x4r
= 428.93 cm' 3xn'
I
Ma$ = Density )i Volume =13 cm
= 7.85 g/cn: x 428.93 cnl
= 3370 e (3 sic. fic)
.. The heisht of the cvlinder is I - cm.
Volme of cyllnder = 3 x Volume ot sphere

Ern =r - ;tu 31.

lh =41
lr'
; = ..,- ,,:,@1''*
.l

(i)
= volune of hemislhere + Volume of cone
(i) surface dea of sphere = 141z cn'
4y't = 44\t
,r = r 10.25
=l " i,o'- \ " ] "s''rz
r= 1ri loli = r54s; ctu'
= 10.5 cn
(b) Total surface dea of solid
volume of sphere fcurved surracc I lcurved surfacel
= Jtr' = larea of hemislhercl
+
l"."or"o* .l

=axrx10.5' =1uxfx11*tf;rsxr:t
= r:r I cm'

AnsqaN wirh conplere worked sorutio*


@
3E /---- -116:\
(ii) External surface area of container

,r*W =/s!rfac€ a.ea\ + / cdred sr&e \


\ of sphere / \ de. of cylinder/
=(4 x
22.22
x 2.1')+(2 x
t 7' x 2.\ x lLa
= 221.8 m'(1d.p)
Volume of rcmaining solid
= Volune of hemlspherc Volume ofcone
222^t22- (; x 7 x14'x 14)
=(a x 7 x14')
= 287s cn" (correct to nearest cb")

39. Honronnl ,urtace ma ot $ arer in conLin-

i Area or circre r+ fA-Es


or.rcctaicr')
= lof.adius 2.1 mJ
\&o wrdrn 4., m /
)). 21)+(126 . 42)
=(

Lat the height of cylinder be n cm. 42. (a) Volume of each conical cup
Volume of solid = 3850 cmr (Civen) 1 :2...'.o
-l
fvolume of I /vulune or\ fvolume or) ^^-
l"*i.pt'"*J '[,.r'"a" J'l i"* .1=tssn
,,))22t22
., ,t'ti 1'Yh\-t t / tr1 No. of conical cups filled

x 10) = 3850

718;+15ar+513i=38s0 = 103.896...
l54h = 2618 .. t03 coni@l cups cd be filled.
2618
'' - 15.1 Volum€ of waler left in the container
-17cm = 12000- 103 x r15.5
.. The height of the cylinder is 17 cm. = 103,5 cm'

40. (a) Radins of hemisphere = 2 + 4.5 = 6.5 cn


trtthe radius of the conlainer be / cm.
Votune of conlainei = 12 000 cnr
Volume of caddle holder
= Volntrof hemispheF Volmeof cylindricalhole

=ti ir ' 65'r-(r. 2'. 1.5r


l"|"t= 12 000

12000x3xt
= ss6 c# (3 slg. fic.) 2x22

(b) Total surface @a of candle bolder ,= ltqztzt


frotal surfee area) fcu^ed surface deal = 17.892 cm
= lofhemrsphe€ .l + |\ofc]lrndricalhole J
External surfice area of container
=(3 x rx 6.51+(2 x zx 2 x 1-5)
= 4r7 m? (3 slg, fig.)

41. (a) (i) Volune of container = 2010 m? (3 sic. fig.)


= Volume of sphere + Volume of cylind€r

=t;' i . 2.1't+t; /2.1'/ t2.6\

= 213.4 mr G d.p.)

Mathenatics rutd 28
@
43. (a) Yolme ofhemisphere = 354.68 cn (Given) (b) Snrfee Na of sphere = 312 m' (Given)
4tt? =3r2
;rf = 3s4.68
. 354.68 x 3
nr- Total extemal suiface trea of open cylinder
2
= nt + 2B(2r)
- 532.02 cm'

= s(78)
= =
t*(6r\ = 390 rtr

= 2(532.02)

(b) Total suda@ area of hemispherc = 180 @2 (Given)


1n; = $o
nt = ffi.tt
Lel rhe slant height of the cone be I cm-
Using PythagoraY th€orem. (a) Using Pythagorat theorem,

i = \]'1 - r5'z

I=[;7 x =!A
= 15t' cm

Total surface dea of con€

-Et+d"En =8+17
=25m
= (t + ,E)nf (b)

= (l + .v5tx6o)
=7nh
= 42s cn' (3 sig. fig)
=axzx15'x25
- sE90 crnr (3 sic. fic.)

UsingPythagom theorem.
t'=25'+15'
= 850
I = 1650 cin
trt the radius of the sphere be . m.
Cufled surface dea of cone
.. The radius of the cylinder = / m and
the height of the cylinder = 2/ m.
=rx15x1/850
Volume of cylinder = 204 m' (Given) = 1370 cn (3 sic. fic.)
df)(2t) = 204
2rd' = 204 46. (a) Volune of 50 marbles
# = 1,wm1 /Volume ofhemisphencdl I ...
= | containei
Volme of sphere - lD2 = r: x 1.14 ^ 12'r- 2910.78

- loort = 706.5 cmr

Answes with conpiere worke,l sol"tt"* \.l47)


(a) Reqlired surtace area
Let the radius of each maJble be r cm
706.5 I i surr*. m!
- *"'r'
vohme of each mtrble = =;- ldh..,.ph."
-6
= 1413 cmr -; x t2'(r8t + 2z(r8X52) + 418)(30)l

'1^rra^l=14.13
. 1413x3
r= (b) Volume of Nater in container
4 \ai I rVoumeot \olLmof voludeot'
- '[hetuspr.e .\linder - cone

= ,, I;dlsl + ,dr8)'(52) + i'r118)'(24)l


. = t6i?5
= 1.5 cm
.'. The radius of each marble is 1 5 cm 49.

(b) Extdior sDrface arca of 200 containds


=mOxQx3.14x12)
= lS0 864 cm'?
180 864 , 1"
.- _
n' - -ind11.25cn -
= 18.0864 3,?5 cn
Cost of painting the exterior of 200 conB €rs volume of henisPhere
= 18.0864 x $96
= $1?3629
(cored ro nearest Gnrl =ix3.142x8'
= \072.469 .d
47. Volune of nodel = Volume of cone
Volume of frustum
l\iorme of Jr (u9lYT:_:fI u.r,."
!\ spnet / \ LtrINq I = "r-* r5,-'; Jri2 2'<l7r'
-,ir't.ra:>8
ifr - tfoatl--
. r',"rtst a = 989 73 cml

Jl*rr"=trol W volune of solid forned


fvolune of ) fvorune ofJ
+ (frusun /
= [hemisphereJ
= 1072.469 + 989'13
/'(jf 45)=o - 2060 cn3 (3 sic. fic)

4s =0 50. (a) Volune of ball


l/
1/ =4i
r=3375mm 1.21kg
=:,----
12r09

Irt the slanl beight of the cone be I cm'

Using Pythago€s' lh€orem'

t = r&o

Madema,ics rutu ,B
@
If,!the height of sdd in the cvlindrical Pait be r cn
volume of bal] = ll0 cmr
volDme of smd = 5.85zcm-
I volume ol ) | volume of ldd l
lhemsphencal part,+lin cvlrndrical
pan l
r 110x3
= 5.85n

.1t-10 3
" ti I t s l +y' - r r' " r = : t:r
l1 4Ytr 225 + 225x = 5.85
= 2.97 cn (3 siC. fig.) 225t =36
. . The radius of ihe bal] is 2.97 cm
r=l6cm

Volume of conical contdner


of ball = 1.5 + 1.6
= Volme of water + Voiume
= 3.t cm
= 1500 cmr + 110 cmr
= 1610 crn'
..:xnx8rxI=1610
. 1610x3
h -.^
- 24.0 cn
.. Th€ heiSht of the container is 24.0 o.
r 3cn '

'-"{@ 2 x 2.5
1._ Base diametor of cYlinder =
-5cm
Using B^nagor6 theorch,

[, ps=,6
volumeofbox=4 x 3 x 6
/volume of i + fvolDme of ) - 72 crn'
= [conicat partJ lcylindica] PanJ

=(; x rx 1.s'x 2.4)+(zx 15:x 18)


= 5.85,r
= 18.4 cnr G sic. fig.)

",rk,
Ler x cm be ihe length of each.side of the cub€
Surface dea of cube = 384 cm-
6l = 384
t'=64
,=,E4
=8cm

Answef with complete worketr ****


@
Using Plthago.as th@ren,

=8'?+81
= 12E
y - ii:a cm

UsingPythagoEs theor€m.
t'l
=8'+128
1
: = .6s,
"-
3

.!,2
2
Sldace area of sphere

=4\nx L I

- 603 .n' (3 sig. fig.)

UsinC simild tdaElcs,

3 12

6 .. ^

Volume of water in tbe container


Lrt the length of erch side of the cube be 2f cm.
1..
Radiusofspher-/cm
Dimetei of cylinder = ) cm
= - a4tt6l
Using Pythagoms rheorem,
r')=l2r)'+l2rl1

) = 1is7
= 2r5f cn

Radius of cylindef =;
2\Er 3cn
2
Using sinilai biangles,
= .6'
"' 312
R=8x I 12 l
=;i12 ,)cm

r"."*u*^., r"
@
VolNne of test trbe = 29.25zcmr (Clven)
/ Vll,ne.f I / volu de of I ^-:".'t
-,
=:n8r1l2r lhemFfhencrl prtl+lc) lind.ical pdJ =

lrir'+ ,'h=29.zsn
Volume of the top empty portion of jnletcd
:i h=29.2\
+t 1l r

= Volume ofcontainer Volume of watc.


= 256n - 32r / Volume of I I I Volude of l
I ne.,'pt'.... t pcnJ
= ll ' lcyl, dricclprnl
2-1
a rR'H = 221ft ati= i t lt'h
h =8r_(2)
r ,12.rr hr]
r12 /?r = 2t4l
Substilure (2) inlo (l):
I (.t2
3 ^t^ (12 h)i = 224
;,:+l(8r=2e.25
nt = i512
12 h = 1r.478 ;f'+ 8r' = 29.25

,r = 0.52 cm (2 d.pJ
81r'-29.2s

r = .N3.3r5

= 1.5 cm

Subsiltute /= 1.5 inlo (2):


rr = 8( 1.5)

..r=l.5cmandn=Ucm
U\inCP)thJgomt fteorenr.
OA =AB'+OB1 . 4d=12 t)cm (i) Let ihe heiehl of ratcr level in lhe cone be
(2+,1:=12,r+12 /r OB=f2 t)cm
(2+ rl =2Q i)1
2+, = j:(2-4 < 'ac squrerNl 2.5' t =2q.2il
on totb sjdos 1 /
2 + , = 2./, - 1|'r
r+ ..lt r = 2,8 -2 ' 29'25 x 3
41+ .l:j)= 2.E 2
2.5_

= 14.04 cm
.. Tnc h€ight offte water level ln the cone is
= 0.34315 cm
(ii) Lct the gant height of cone in contact wit!

=; x zx 0.34315',
= 0.u nj (2 d.p.)
Using Plthagoral theorem,
|
|
=2.5'1+ t4.04'
= 2t)3.37l6
-.,120jl1t6
= 14.2608 cn
v
1
V
/t

Surface arca of cone in contacl with saler


= Cunod s!.fa.e dea of cone
=rx2.5x14.2608
= 1r2 cndr (3 sic. lic.)

AnsweN wnb conplete Wort* *t,.r" @


I 0

A. G.aphs of Linear Equations 0 -l


1. (a) :i I
I=r+2 ii l ::-a ::.:
1:l , :::::: r,ill
i::

:z 0 2
:r' |:l :aa:
2 4
rlr
0
:
, i::i1
::i it
.::a
iril il i'i :t::I
'-J
r)
i., | ,r l
l'ii I \ ::t
i,
t'-:,1
l:i (:!-jj .il lr
rl
'i,l'
::t ttl,
I
Iti:l \ l,i
fi:1:
I
I :
,l
i i: ^.,
r,il ,l;) z: il tl,

ii Ijrt i ,-.
,i
:ii 'y'.. r i!:
.:: Ii
a:, 1
l:lii /' lr :lI- fr
il
l:t:i
,iij :i::l
:t ): (d) r= s -;r
6
i: ti
.!

i'iL1
l
(b) r=2t
:l ,' j, 111
:'n:i !.fl
-2 0 2
I
:iI :
'_!
I i :t ii , tL il
0 I: Ii ij
]i] i :::l :i r.:r , L:i
r+t:i
+l1l i rl ,l i :il r l:l:
:]i r:i
-, il irri +r +
l
.:
xi i:
:
it ,i 'f
,l,1
:ii i
ti
rrl \ Il
:)' irl
,l|
I
lrE ltf--l
::::
lix 1-: ;- -nJtl .ll i ril
ir-ri
:l
ii:il i r:l . i !,t' il
ii i:tl l I Li t'l l l',t
tl:+ ll ,-+ jril
:li l= '. ( 1:r:,
l
li:t,
i
li il'
F: 1,1.1 i:l:l
i
il
t],
:: iI
irl liii
i
'ttl :L:l

l:lr r
il tt l iin
F. t'l l :]

lrl F:l : iri 1 'L 11


T
jj::i :i: I

r'a"t'".u* ruo, uo
@
2r+5J=10
51 = 2J+ l0
]=-a+3 l,lvic mn
0 2
r=-ir+, * tsrm bv r.

3 I -1 5 0 5

I lll.- rliil
4 2 0

ll' [11.1 i it' ir,. rl'I


irri
i.l 'f, ltt
I :::l
.: i:il i i:
.'. i,

I r:l
\.,
i1\
,l- u.,
I' '.
,=.i :i::: \
::::::.
i
Itd
,,,
:::::: l
ll:: ::: ) - r tl
l

lr,
:\ f': N '+,
'1
l-:
q
x
'l-
::
::::.
t. NI
t \ il' \
l_r:

r,"il
:::
(t) 2r r=+
(h) 2)-3r+6=0
2 0 2 2r=3x 6

-8 , = 1"-'

jl : ::::: il 0 2
ir' ::i::! i:
i]. i::.:i:i
0 3
;l -3
'.',),!,
ltl ,/ lrl
J,i
rt:l::i
r::r:
!. l,i
l
,i;, l/ :f.
_: iI
rl lt,it. ,ti.
rti rl:
i:i l

I iir
:-:i l::i:l:l
li rtl [+'

Answers wilh conllete workeo *t,.* (,D


1.5-l=0
)=15

(e)

(rD r*- r"


""*"-"u*
3. (a) (c)

7.@
@
@
@

8.

(d) a<0..=0 (a) Equalioi of line@ :r= r


Equationorline@:t=-2
Gndient of line @ := ?
=2
.. Equatioi of line @ J
: = 2x

{-}= r+2
(b) P=(1,-2)
Equaiionofline@:l=xt
!=zr+3

2(y + 1) = 3(r 4)
3_
..Q=Gr,-2)

Answers with conplete wo*ed sohtio"


@
"il 6 0 (i) wher t = 15,1=2'
,5 (ii) dhen)=6E,t=0.9.
7 I
Yes, (2, 9) n a solution to the equation J = 2t +:_'

ll. (a)
3
3)= r+6
l^ 3 2

(b) Area of lndgle fotmed

=;xBasexHeighr
=j,.s'.:'
= r2.5 units

10. (a) r="'+5 3 0 l


-t 5 tl
(i) shen r= 21,J = 1.2.
(ii) whe. I = 2.7. x = -2.1
(d)
wh€nr=3.6,lt=0.8

12. (a) 3x - 4t1- 12 =O


4! =31 \2
3^

-3 3

@ r**'** ",-
,u
whe! ) = 1.8.1- 6.4.

=:xBaseXHeight

ll. (a) 3r+)=0 = XBalex Height


-2 0 2
)= 3r
4 =lxlx9
2
= 13.5 uDits'z

15. (a) tu+b+6=0


Ar 12,4),
h(.2)+k(:4)+6 =lJ
2h-4k= 4
h-zk= 3- (.1)

(c) F.on tne graph, when r = I,


P-l=3 Al ( 8, l),
ft(-8)+t(l)+6=0
14. (a)
8n+l=-6 (2)
),=r+ I
0 QJx2l r6h+2k=_t2_(3)
(1)+ (3) :
-3 I
SubsntuEn=linto(1):
| 2k =-3
0 1 2k=1
3^
)= tr+r -3 3 9

Ans*es widl comlrete work"d s.rlttom


67)
:z 2
Whenn=landl=2.
r+2)+6=0 -2
=8x3

2 6

3
0 4
(c) Froft the graph, the r,'i ercept=-l 0

when)=-5..=4. 0

(a) )=2r
16.
2 z
8

0 0

-r2 0
l=2x+8 0 2

8 12

tu*"-",", ,*- ru
@
(b) Kite

Coordinates of the verti@s of the kit€:


Q; t, (-2, 3\, (2, -i) a\d (6,3)

x-?!=7
(-1)-2y = t
-2Y=8
l=4
.. (-1, -4) is a solution of the equation r - 2) - 7. (a) P = (-3,0)
(b) o = (0,4)
(c) x = (-3r 4)
(d) (i) Arca of APoR
3(10)+s]=15
30+5t=15 =t'8'se'HeisbL
=!x+x:
. . (10, 3) is not . solution of the equntion
Xr+5r=15-
(ii) U8ing Pytnagors' theoren,
PQ' = PR" + QF
=4'+3"
Po =
'!8
= 5 uDits
2+!=O
v =2 Perimeler of APOn
=PQ+QR+PR
.. r-1.2, is a solu$on of$e equation
=5+3+4
2
= 12 un s

(d) tu-81+3=0 2t. (a) SDbstitnte ( 3. p) into 2r + 3) = 9.


2(-3) + 3p =9
7(3)-8y+3=0
21-8)+3=0

.'. (3, 3) ts a solution of the e4uation O) sub;dture (2q - 3, -l) inro 2r + 3t = 9.


fu 8]+3=0. 2(2q -3) + 3( r) =9
r 4q 6-3 =9
Requir€d equanon: =3 4s-18
q=4,5
(c) - 3/) irto
Substitute (4., 5 2r + 3) = 9.
2@t)+3(5-3r)=9
8r+15-9r=9
t =6

Answ6 with comptek worked soludos


@
22. (a) Substitute (d, 0) into 51 4) = 8 B, Solving Sinullan@us Linear EquatioN Graphi@!
5d 4(0) = 8
25. (a)
5d=8 -!=r-5 3 6
.I = 1.6
5 -2 I
Substitute (0. z) lnto 5r - 4] = 8.
5(0)-4"=8
1e=8
e=-2
SubstitDte (4, /) into 5r - 't) = 8
5(1) 4/= 8
20 4J=8
4f= 12

(b) 5r-4i'=8
Wher r = -2,
5(-2) 4) = 8

-10 4)' = I
It= ta
,l

.. (-2.
'2 -41) fies on the lin€ sr - 4J = E-

2l Subsdrute 12.l + I) into -r + 4r = tl (b) The solutionisr= 3 udt ='2


l(2) + 4{i? + 1) = -t I

L5 + 4t +,1= l1
26. (a) r- 1r-3 0 3

4h= 16.5 3 2 1
h = -4.125

(J,
At thea irtercePt, ) =
r+3(0)=10
t= 10

.. A = (r0,0)

3r- 5), = 15
Atrhe)intercept,r=0,
3(0)-5J=15

..8=(0!-3)

=l^Base Heiqhr

=lxror:
2

Mddrematics rutor 28
@
(b) Thesolutionisr= 9 and, ={r. 29. (a) !=u+3 3

3 9 15

n, r-2f -6
3 I

The solution isr = 2 andJ = -2

0
lhe \olurron sr = z andl = /.
5 7

(ii) )=-r+4 0 3
0
5
8 0

-l 0 3

14 tz t0

(c) The solution is x = -2 and) = 6.

Answers rjtb Conplete work.d S"1"1i-,


@
Zr+r+6=0 8 0
| ='Lt 6
10 2

l 0
I= 2t
8

The sotution is r = -2 and J = -10.

0 2

l^ 2 -t

0 2
J=-3i+5
The solution ,\r = -4 dd.'' = 2

(e) -t=r 2 2 0

2 0 2

t 2.y+2=0 ,2
2J=r+2
I
r=lx+1

Tbe solution isr = 2ddl =-1.

The sohnon is x = -2 and I =0

r.,n".",n, t"
@ "*-
(i) 3x- 101= 0
10] = 3, 0 8 0 2

j= 3
ro"
0 t.2 2.4 7

x+2r=12
2
2r= '+12 0 8

3 2.5
2
)=;x+1

The solution is : = 7.5 and J - 2.25

G) 31 2J-9 0 3
2'-3t-9
}= :J.4
'22
0

Tha solution is x = 2.4 dd J = 2.8.


0 3
11
(D
{.5 2.5 1
t : 0 I J
3

2r-])+l=0 5 2
3J=2.+l
2l -3 -l
33

The solDtion isr = 4 and)' = 1.5. The solution is r = -2 and J = -1.

Answes with conprere worked sorutioN


@
0)

0 2 4

12 1

J+;)=-i
l^
;)=-r+r
l-4x+12

(l) 3r- 8l = 12
Therc n ar inflnite number of solutions 0
8t=31 12
r= I'
r': -1.5 1.5

8
5r=r+:
ti 0.6 2.2 3.8
-55
3)-r+6=0 0 3 6

l^ ,2 1

3 6

3.5
;u= f'*zs

The soluiior is r = 12 andJ = 3.

(,D r"*'.,",'.,,'
(m)
I 0

6 2

5.5 6.5

The $ohlion is r = -2 and J = -3.

(o) 4r+5)=20
2
5r=4x+20
4, 2.4 0.8
]= ;r++

0 2
4f=Jt+24

"= f'*o
6 2.5 I
: 0

-5 -l
2j+l+7=0 2 0
t=2x 1

I -3 7

Ans$eF with Conplete Worked Soludons


30. (a) l=2 r -1 0

3 2

=:xBasexHeight
=!rzxz
2

1= l*++ 0

(i) wh€nr=2.5.1 =-.0.s, 5

(ii) wh€n'--1.8,.t=3.8.
The solution to the simultaneous equarions ir
(c) )=-1I+3 x=6odt=6.
-t 0 2

5 3 -l 32. (a) I+2)=i0


5
=-t+

--ffi
2f l0

*-ffiffili
,, The sobtion to the simultaneons equations $ r =

31. (a) 2)r 3r+6=0 0 8


2! =3x-6
3" -3 .] I
'2

r",n.-"u* ,",. r"


@
(b)
0 9

l 0 2 3
_3

5 10
The solution isr =6ddJ =2.
l4
(r)

=f^sase^nereht
=fxsxo
2

33. (a) (i) )'=xt 1


0 3

I 5 1t

(ii) 3)+r=24
0 l
r- lr+s 8 7 6

The solntion to the simultaneous equation is r- E

. . Pauls is 8 years old dd Maft is 12 years old

35. (a) The totat cost of tickets for 2 adults dd 3 childrcn


(b) (i) The solution isr = 3 a;d) = s. is $48. (Given)
(ii) The solutior is x = 4.5 aldl = 6.5. . . 2($r) + 3($r) = $48

itr:i=;ffi
2! + 3J = 48 (Shom)
14. (a)
(b) The rotal cost of tickets for 1 adull dd 2 childrcn

ffi
is $28. (Given)

l(tu)+2($))=$28

Answers with comprete wo.ked sor"tt.* (O


(c)
0 9 l5

1- 1r+16 16 10

x+2!=24 0 8
2t=-x+zz
l4 l0 1

The solution to the simultaneous eqDalions is r = 85

His narks for Mathematics was E5 and his ma.k fo.

The solulion to the sinrulianeous eqlations is

. . The cost of aD adult ticket is $12 and the cost

of a .hild ticket is $8.


When.=160,1=4.

'' lli=ji'.' qflffiffiffiffi#qi-"i' 160=r+41


..r+4J=160.
When.=190.d=5,
190 =r + 5J
(b) a+l = 150 ''.t+5J=190
0 50 100
(b) -. + 4l = 160
50 40 80
150 100
4)= r+ l6C)
I 40 30 20
x ),_ ;a+40
t- =20
0 50 100
) =r-20
a+5)r=190
:o 30 80
5)=-t+ 190
0 40 80

I __ ]IJ 30 22

vrr'"-"t* n o, zt
@
srmultareous eqDatons's

...=40+3(V +-

.= 40 + 30(10) (a) she needs to sell5 vases to break even

. . The cost of a chest wilh 10 drawes is $340.

2. (a) tFt x be the nunber of vases sold and )' be the

j.l=&r+30-ffi
prcduction cosL for mating x vases.
(b) she n@ds to sell at least 6 vases to nake a prcfit

t€r r be the nunber of vases sold and I bc ihe


(c) when 3 vses were made.
.'r=r.r".ffiffiffiffiffi
r=8r+30 tnss - Production cos! - Tot l sales
= $54 - $42
7
= $12
30 E6 112 .. M^ Wory's loss when lhree vases were ma.le

(d) when 12 vases $erc made rnd sold,


goduction cosl = $126
7

0 9E 196
ftofit = To€t sales Prcduction cost
= $i68 $126.
= $42
.. Nrfts Wong\ profit when twelve vases were
nade md sold was $42.

Ais$eA $rh Cnmplete WoiLeO *,,** 6i


(ii) Let the slan! height of the cone be I cm.

1. (d) (i)
= Volume of pyranid + Volune of cuboid

= (! x 32 x 32 x 30) + (32 x 32 t 36)

(ii) Total surlace dea of solid Using Ptthagoras' th@rcm,

rLrrnlsrrre
= \mnotpym al
I - /BNm!\- fluDlqud.dl
lor' ruo,tl \rs ordboid , = 676
I = .Gro
=4 x (l x 32 x 341+(32 x 32)+4 x
(32 x 36)
Total surface area of solid
- 780E cm? /C!nuradnel lroprE! I r fcm'dr{d*
= lJrtiorcl!nd{l + lot(r irtrJ \tuoor.oit
(b) (i) volume of ntid
= Volune of benlsphere + Volume of corc = 12 >< 3.142 x 10 x 8) + (3.142 )< ldr -
(3.142x10x26)
= ?ot !
33 * nfn = 1633.E4 cm:

=, I.tlr2 J0'-''.rl42 10r'dn (d) (,)


= Volume of sphere + Volume of cyliode.
= 94 260 cm"

= ij x 3.142 x 91+ (3.1,12 x 9'x 25,

= 9416,574 m',

(ii)
= Surlace ma of sphere + CNed surf{E
Let the slaDr height of the cone be I cm.
Usiry Pythagorat theorem.
11 =30'z + 101 = (4 x 3.142 x 9r)+ (2 x 3.742 x 9 x :,
- 2500 = 2431.908 m:
I = .{6500 (e) (i)
= 2 X Volume of core
Totai suface dea of solid

=
/ Lurve{l \drtace ) -, rCuded edeel
lr,o.t ncmspnereJ [doorrcre ] =2 x (J x 3.142 )i 6'x 8)

= (2 x 3.142 x 30') + (3.142 x 30 x 50)

= r0 368.6 mz (,il Ler rlre \lrnt heiChr of Lhe coDe be / cm.

(c) (i) l'=6'+8'


= Volune of cylindcr + Vo[tme of cone = 100

I = lrioo

= rl.l42 l0 8)+r; ll42vl0'"241 TLtrJl surface area ot sotid


6cm

cni =2 x Clred luface dea of cone


= 5027.U

=2X3.142)<6X10
= 377.04 cm:

r,,n"."'* ,'- ,u
@
(t) (i) Ler the heighr or the cone Lct tte drop in the water level be ft cDr.

UsinS P)-thagoras lheorem- /,


1rt8')<h-: x6!6x 12

h'+12'=20' , 6!6!12
h'-20' 12:
= 256 = 0.72 cm (2 d.p )
h .rr.s6
= .. The diop l! thc waler level is 0.72 cfr.

e. rhenerFli nr' prirSul-rbo.c"f rl'rp)n. dbc


= Volune ofcone + Volume of cylinder +
Volume ofhenri\pher
1,,,2. n+
= 1nt h+ nt 1Et
=t- t3-112x l2?r 16)+(3.142x 12'x

l0r+r; 142 l2l Using PyrhaSoras theoreh.


= 19 606.08 Imr
(ii) Total $'iface arca of solid
lLLNdqnftr. rcmrdafN ,lcuqL *tk l I = 1]T92 dm
=
iE0.'r r - lr iorqrnd"l - id0rh{m5!h.rl
= (3.142 x 12 x 20) + (2 x 3.142 )1. 12 t
l0) + (2 x 3.1,12 121 x = t 16 ilq2
= 3921.216 mttr: = 110.85 mnrl

L€t the sla.t heighl of Lhe pymid bc t/ cm.


Total sdrface arca of pyramid = 780 mm' (Given)
Ba\e arca + Area of 3 inangular faces = 780

110.85+3 )< (t x 16xrl)=780


I10.85+2411=780
24H = 669 15
d omm l.iB ligr
.. The slart height oilhe pyranld is 27.9 mm.

(a)

(b) Total sudace dea of pyfamid


=Arca of .{Imgular basenaCD + Area ol 4 A locn B

P0=r0+2=5ctn
=(8x6)+12x(txEX r1)l+t2x(+x6 using PythaSont theoren.
x r4.25)l

vP = J141

.. The height ol the pyramid i$ 12 m.

.{ns}e6 lvith complete worked soludos (3;)


(b) volume of pyramid Let the base radiN of lhe soLid be / cn.
volumc of solid = 3465 cm' (civen)
=:xBasedeaXHeighr voiume of cone + volume of cylirder = 3465

=:x(10x10)x12 L Elh + nlH = 3465

(; ; ?1 xt xt:t+r? x I x 8)=3465
(c) Tolal surface dea of rhe py.anid
= Base area + tuea of 4 r.iancular faces
^l-, ,. 1..-.,-.
= (10 x l0) +:l )< (; x l0 x 13)
3r; t = 3465

6. vollme of pyranid
I 3465

:t]1
=:xBasearcaxHeight
= 110;
=:.c6.16) 6i)

= 25 920 cn" /= .lt1ol


ll 4
= t0.5 cm
= VoluDe of cone + Volume of henlsphere
.. The base radiLs of the solid is 10.5 cln.
l"_2.
33 Dimeter of hemisphero
=ti - 2.1 lt+t: :=.2t1 =26 4 4

.. Radius ofhenisphere
No. of solids that can be fomcd
_ 25 924 Total surfa.e arca of naterial used
/CuNed $rf{c I + /Areofcncb I aAtaolcirle .

= 500.928
= la,erorhenispheEl loi ndrus ]] cm] lofndiusecn
.. The naximum lumtEr of solids that can be formed =(.2 t 7t x 91) + (E x 131 (z x 91
is 500.

7. Radlus of cylinder = 12 - 2= 6cn 10. (a) Ler the radius of the semicncle be r cm.
Volume of 2 metal spheres
e.u or,".i"i."r" = tz j,,
".'1ciu"n)

=2 - ,r'25'
= 130.8997 cm' .n!j-J
Ler the incrcase of the water lelel bel1 cm.

4/rn = 130.8997 cml t


rx6rxr=130.8997
, B0 8997

= i.16 cn (3 siS. fig.l


.. The dl.merer oltbe scmicircle is l0 cn.
.. New depth of waler in cylinde.
=8+1.16
= 9.16 cn (3 siC. fi8,

r'n".",* ,*. ,u
@
Lcngth oi dc ,{f Lenglh oia sidc ofthe cubical box
I
=tx7nr =z t 5.25
= 10.5 cm

Extcrior surface dea ol box


Lct the base radius of Lhe cone ibrmed be R cm =6Xl05l
= 661.5 m"

(b) Lel the radns of the sphcre be R cm.


. . Lenglh of a side of rhe cubical box = 2,{ cm

.. The base diameter ofihe cone fomcd is 5 cm


\o ume o' -noc.uprcd p-r a bo^ - ') /i c
(G \en)

(c) Let the height of lbo .one formed by , cm Volune of box Volufre of sphere = 35.28
4 )),D
,-,-,1
,^^,1
t ^, "."
8r' - 4tn' = 35.28

3
t X'= 35.28

_r _Tf
35.28
^=
Usnrg PylhaSoEs' rheorem. 2l
= 9.261

1E.75
n = 1i9'261
=
= 2.1 cm
l' = .i1sl5 ."
.. The radius ofthe sphere is 2.1 ch.
Volumc of cone fomed
12. (a) Volumc oi sphere = 2 x volume of cllinder

.|it'=2 t ttR'(R)
= )< 2 x 2.5'X 1/18.75

cn' € iig. ric.)


x 15' = 2R'
= 28.3 3
4500 =2R'
d = 22so
n = iD25o
- r3.r cm (3 sic. fic)

. /Turxlsudace L ='
rDr fsurfrr aRal
1,. rur or,rt'n,t.r
l.r'pher
2nk +2nRE)=3 t 4nr'
(a) 2nl9)1+2n19)19)=12'';
Surface area of spherc = 3,16.5 cml (Gven)
4nl - 346.5
32ll'f = tut-
'}.1
a /. ?Z
1
x; = 346.s
= 5.20 cn (3 sic. fisJ
: r,16.5 x ?
'= 4't
= 21 .5624
lt. (d) Required surfacc a.ea

,=.1tr.s625 ,
"
*" d,"
. f' | 'd rr 'r I ,(or,rd
A'c,or '.,
daor."' 'e-r",n.or 8. l
= 5.25 cn \rddi!\ l0un I md d.6(m

- /. '0 2- | ldr' ,n 8 18'-r,r'R:r


= 2s80 cn' (3 sic. ficJ

ANucF urrh Cornp de Worl'"d S"l,I'm,


6)
(b) 16. (a)

Let d cm be the diaineter of tbe sphere.


Using Pythagoras' theoren,

Using Pythagoras' theorem.


= 100
ni+8?=l8r
ti = 18, 8:
,I = 1/tao

= 260
i \,540 cm
=10+2
Using PylhagoEs' theorem,
h,'+2d=451
h: = '+5" - 20'
= 1625
it' = t/rOZS
"' 3

Amount of water in pail


3
fvotuneoi
Icone of
II fvormeor )
conc of
= 524 cln3 (3 sic. Iic.)
- l*a..zo".J
"- l
I'aai.s a Sudace arca of sphere

-,ll:l o :o ul625, ,1 .n #
1560 )
- 314 m' (3 sig. fig)
= 15 800 c# (3 sic. fic.)
).1.

14. Volune of conrainer


fvoruneor ) /\,oluneol \ ...
= lcone or loneot
lrad,6lo(ml I'J,ri6 6.mJ \mns!rere /

-( t n lO 20) r- u.6' l2'-


11 x rx
6')
= tr90 cln] (3 sic. fic.)
(a) VolDme of solid formed

15- Let the mdiu! of each tennis ball be r cm. + ,, v.r"-"


. . Radius of cylird.ical can = r cm
= volume of henispherc ] "r "o*
Height of cylindrical can - 2r + 2/ + 2r 2.11,.
- 6r cn
=(1 .E.21 t+tl 1.n 21" t6)
_ Voluhe of 3 lennis balls
- 54 965.3
Voime of cylindical can
= ss 000 crt' (3 sic. nc.)
3x xnNf
=* ,,_" 6n (b) Mals = Density x Volume
= 0.84 g/cm' x 54 965.3 cm'
= 46 200 g (3 sis. fic.)

2
=a6.2kg_={{!iW

6o) r"r"."o*r*-r"
(c) Ler the slmt beight of tbe cone be I cm. (b) The figure formed is a trap@ium.
UsingPylhagoras theoren, Aea of trapezium

= j x Sum of parallel sides x Heisht


I = \q.AE =tx(1.5+6)x3

Total surface a.ea of solid


=;x7.sx3
= 1125 unitsr
led.und l,l,lAF,ocrck
-_,.1
lffiaorhem{pheft.l 2 lufddius 27cml
l
I _ lcutred surfxel , i Arca or I
2 l&eaolconc / ltrir8ulrrr-"J
=,2.1t 21 r j tft.21 t-2 ,n.

27 . 45r+ r; 51 16l
= 8606.0?
= 8610 cn (3 sic. fic)
No. of lins of paint needed
t 8606.07 cn'
5|]O
= 4oooo;;-
= 107.58
19. (a) (i) r=xr+5 3 3
. . 108 tins are needed.

18. (a) (i) 2t ]=0 -l 5 lt


2 2 6
!=Lx
12 (ii) r,=-2r- I
-l 0 3

(il) a+)=l 5 -1
: 2
)=-r+3
5 3

ADswe$ with comprele workeo r"*.* (,D


(b) 21. (a) J+I=2
=;xBarcxHeieht
0 12
-jrtot,r l0

20. (a) r= ;i+r


2

3 2,5 2

(b) The solution isr = 6 and, = -4.

!= r+4
2 0

h )= t
)=2I+l
9

(c)

l"
)=-;r+J
Frcm the emph. when ) = 2.1,
2p = 1.6
r'=1.8
(d) lr-2t=l
2)=31 1

.,- t, 1

{.5 2.5 5.5

lhe \olutron rs r=| dno1= r.

t"*".",", ru
@ "*-
(b) J=-2r+6
0
41 5)=10 5 3
5l=4j 10
2
'5
1^
0 3
2t- 10' = 35
,3 loi =2.t-35 -l 0
-2
ll 4.1

The solution is r = 4.8 dd J = -3.6. The solution is r = -2.5 and I = -4.

2 0 2
23. (a) Substiture (1 2*, 6) inro 5), + 26 = &(.
',= 3r+6 8(1 2e)
5(6) + 26 =
12 0 30+26=8 16t
i6k= 48
5r j,+14=0 l=-l
2 2
)=5r+ 14
(b) Required equalion:, = s
l4 24

(c) A = (0,8)

The solurioD is r = -l od, = 9.

AnsweN with conrplete worted solutims


@
)= +r+3

3t=Lr 9
2

I
r=tx 3

(c) The solution to the simultaneous equalions is


r = 4.5 andl = 6.
2- .. The p.ice of a mug n $4.50 and the price of a
plare h $6.

2^ i,e,3J=2x-9
@
4' + 3! =36
@ ty=i'-1 1,e,2!=r-6
@
@ :r= tx+3
2^
@

25. (a)
16.50 0 1

5 ll
(b) 2j+)=15 4x+3t =36
0 3 3t =4x + 36
/= 2r+15
l5 9 3 t =-1.*n
t+2/=16.5 0 3
0.5 2.5
2y=-x+16.5 tz
y=-|x +*x 7

@ **"."0"" r"
^'.
(c) The solulion to the simultaneous equalions is

.. Marc liles 3 km ftom Pe!e\ Pizza while Estber The solutio! to the simultoeous equatons
lives 8 km fron Pete\ Pizza. r= 15 dd)= 15.

??.ra,Sr+6.r_-:t:. Nunber of Brand ,,1 bulbs

ffi
ffi
=8x/-t
= 120

Number of Brand B bulbs

(b) 8r+ 6) = 2i0 =6 x 15

6)= 8r+210 =90

)=_Jr+35
0 15 30

35 r5 5

t5 30

0 15 30

Ai\we6 wirh Complete Worled SolurloN


@
substitute r = 1tuoy= ; r+6.

r!,=(;)'(;)+6
.l

tu-3)=0
.. i=0 or r 3=0
a=0 or r=3
2. (a) J =.+4r r - P = (3, 0)
=(r 1)(a + 5)

(x- l)(r+5)=0
..x-1=0 or r+5= 0
r=l or r =-5
...P=(-s,0)andq=(1'0)
When r = 0,

...x=(0,_s)
0+3
'2

=-2 . . Eq;non of line of sylmetf : r = 1.5


Substilute i = 2ilto)=j:+41 5,
r = 1.5 inio y = r: - 3r,
Substiiute
l=( 2),+4( 2) 5
) =(1.5t-3(1.5)
=9 = -2.25
...s=(_2,_9)
.. Coordinates oflowcst point, i.e. minimu poin
= (1.5, -2,25)
= (-r + 2)(r + 3)
(d)

( r+2Xr+3)=0
.. -r+2=0 or r+3=0 r(, - 4)= 0
x=2 ot r= 3

.. A = (-3, 0) and , = (2, 0)


._. e=@,4)
.. c = (0, 6)
5. (a) r=2.t+3r-20
i+2 ../-(0,_20)

r,,n".'o* r*- ,u
@
(b) /=2r:+3r-20 Substitlte x = 2, ) =, and d = 5 into )r = aar.
= (2r s)(r + 4) b= 5(2f

(2r-5Xj+4)=0
..2r-5=0
r= 2t q
I
.. a = (-,1.0) Jntr C= (2 t,0)
...t=-.q^'t"=z!2

.. bquJUon
^'
rne or ()nmerJ: r = -i
]-(i+l)6-3)
O+ 1)(r 3)=0
.. r+1=0 or r 3=0
r= I o! r=3
..A=(-r,0)andB=(3,0)

...c=(0,_3)

-l+3 (a) wheDr=2.7.J=3.1


z whe! r = 11.9, J = 2.4.
(b) whe! ) = 4.r = -{.4 orr = 2.4
=i (c) the greatest value of) is 6.
.. Equalion of lire of symmeq: : = I (d) the equation of the line symmeny of the curve is

Substitute a = 5 and) = d i.to -! = (1+ l)(r 3).


d=(5+1)(5 3)
= (6X2)

Subslitute r = I dd)=/intoJ=7+3r r:.


/=7+3(l)-l'=9
Substitute I = s od) = 3 into l = 7 + 3}ll.
3=7+3(r) s:
l-3s-4=0
(r + l)(r -,1) = 0
.. r+1=0 or r 4=0
r= I or r=4
... /=9ands=-lorr=4

(b) Substitute r = landJ=5intoy=dr.


s= a(l)'

Answers wirh conprete worke,i soruriom


@1)
(a) To solve a' + r 3 = 0. tind the loints where the
graph cuts the r-sis, i.e, ) = 0,
. . The solutioN de r = -2.J or r = 1.3.

(i) Fron the graph. the snallest value of


J = -3.25
(ii) Equatio! of line of symmehy: x = -0.5.

10 (a)

(i) whent=-1.2,J=-2.4.
whenr=2.4,J=-1.6.
(ii) when J - 3,r = -2.35 orr - 3.35.
(iii) tbe smallesl valDe of I' is -5.2 and the
conesponding value ofr is 0.5.
(iv) the equation of the line of symmetry is

12. (.)

(c)
(i) whenr=1.5,t=2.2
(r) wneny= u,r=5.r5
(iii) the coordinates of Lhe minimlm point is
(2, 0).
EquanoD of the line ofsymmelry:r = 2.

11. (a)

t",'"."u* t*- ru
@
(c) Fron the 8raph. 14. (a) J=(1 r)(r 3)
(i) when:t'= 0, t: -2.?s orr - 0.?s
(ii) when,= l s,.)' - -325
(iil) the equation of line of symmetrv: r = -l
(iv) the coordinates of the marjmum poinr 1s

(-r,3)

13. (a) I=2l'+3' 1

(i) when r = 1,x=0.6orr=3.4


(ii) whenr=4.5.1 =-s2s.
(iii) lhe greates! value of ) is 1 and the
Equation of line of symmety: x = {.75 conesPonding value of r is 2
(d) (iv) the eqnaiion of the line of symmelrv of lbe
(i) whenr=ls,]=8 cufle: r = 2.
(ij) j l.rs
whetr ) - 5,r = -2.6s or =
(iji) 2f 3, 1 is -2.1
the smnllest value of +

Answ4 with comprere worked s.rudotr (a


15. (a) )= ztr+7r+a
d= 2(r)'+ 7()l + 4 -9
b= 2(2)'+7\2)+4-ro
c= 2(5)':+7(5)+4=-rr
(b)

(i) whenr=4.3.J=12.s.
(ii) when ] = 8.r = -1.4 ort = 3.9.
(lii) the smallest valu€ of ) is -6.25 and the
coreslonding value ofr is 1.25.

17. (a)

d= 1f5l l) l-tr':t = -3

b=;1512)-2')=3

(c) Equation of line of syftmelry: r = 1.75


From the graph. the maximum value of
2;+tu+4is10.2.
16. (a)
a=2(0.51,5(0.5)-3=J
b=2\5)1-5(5)-3=22

*"*".",., t*- ,u
@
(c) Equaton of litre of symmety: r = 0.85.
(i) whenr=4.7.t-0.7 (d) From the graph.
(ii) wlen)= 2.5,r- r.4 orx = 36. (i) when i= 2.5,J = -13.25.
(iii) the greaESI value of)
is 3.1. (ii) when) = 15.x =-0.45 orx=2.1s.
(iv) lhe eqDalion of the line of symmetry; r = 2.5
20. (;) ),=2;+ar e
18. (a) a = t2(0) 3t(5 0) = -ls
D = t2(6) 3(5 - 6) =-9

(i) wben)= 10,r=0.i1


(ii) whenr=2.5.t*s.
Eqnation of line of symmeLry: x = 325.

19. (a)
a= 18 +5( 1) - 3(-l)'= 10
,=18+5(3)-l(3)r=6

(i) when I = 10. r = -4.25 or r = 2.25.


(ii) whcnr=-2.5.1--6.s
(ili) the equation of the line of symmerryi r = -l.
(lv) the solutions arer 3.35 ort = 135.
=

2L (a) ))=t -\-s


a = \,0.5)' 21 0.5) 5 = -3,75
b=1'-2(l) 5=-6

AnsweN with comprere work"d s.r'ttds


@5)
(b) Equation of line of synmelry of cune: r = 0.
(c) The soludons are r = -1.7s or r = 1.7s.

From the graph. the least vaiue of ) is -6 and it

Equation of llle of symmotry:r = l. 23. (a) r=3-sr r'


-6 -5 -3 : 0
whe!j=1.8.S--5.35.
whe!I= 5.35., = 0.2 orr = 1.6. 3 3 7 9 9 3 l
: P=o2
i.p=0.2md4=-5.35

22. {.n)

r.,n".,,". ,'-,u
@
(i) 3 5r _l=0arcr=-5.55
rhe solutioDs 1o
orr = 0.55.
liil .he .ot | .n. o , ., i .5 Je x - _4,S5
or r = -0.45.

&) A--l;+3fl+q

(b) Frcnr tbe g.aph,


rrt rtrc m lrmum rort .uix.r ore, u. th..one l
(ii) wher,1=6.'=18.
... When nD to&l surtirce area is 6 cn,
r- l.ti cm.

25. (a) A= t x Sum ofpd.llel x


sides Heighr r N max'nn'n Nhen I = 3.5.
I
-txl(r 2)+([ zr,x(r+2) AD=3.5-2=l.5cm
I ,c=ll-2(3.5)=4cn
= 2(e-J)('+2) AE=3.5+2=5.5cm
= t(er + 18 _l xr)

=t(r,+tu+18)
... A= I :r.+ Jir+9)cm, (Sho$n)

Answe$ wirh comptere worked sol,it.m (a


26 (a)

Let the length of the enclosure bel m.

Perimeter bf rectangular en.lorut = l0 n (Given)


2(r+l)=10

r=(5 r)m

A=LenglhxBreadth

A = (sr -i nl] (sho\Yn)

' (i) when / = 2.4, Y= 23.


.. The \olume ol rhe tanl r.2J m' trheo iE
radius is 2.4 m.
(ii) when Y= 60. / = 3.9.
..lr'"d.').irherark \ l.q m { \en rlF
vohtue of the lank is 60 mr.

(i) whenA =s,r- l.4ora= 3.6.

.. When thc dea of the enclosnre is 5 m',


x=1.4nofx=3.6n.
(ii) the maximum area oi rhe enclosure is
625 m:.

r"r"."u* r"
@ ^..
28. (a) r=l-14'+80

(b)
(i) nininun, r
{i) whera=2.5,}=:13.75.
rhen ) is = 7. .. The fircworks rcached a height of 43.75 d
.. Mr Lim needs ro order a batch of7 cbairs after 2.5 seconds.
for the cosr per chair to be minimum. (ii) rhen)=20,t=0.75.
(ii) whenr < 45. r= 4.5,6,7.8.9 or 10. .. Tbe shortest tine taken by the fi.eworks
. . For the cost per chat to be less thm $45 to.each a lieight of 20 n is 0.75 s.
he rceds to order the chai$ in batches of (iii) the ftaxinun heighl rerched by tbe firesorNr
4, 5, 6, 7, 8,9 o.10. abole the ground is 45 n.

29. (a) 30. (a) LeL Lhe breadL\ of the rcctdSle be t cm.
Perineler of iectaryle = 12 cm (GiveD)
2(x+))=12
r+)=b
)=(o_r)cm
AEa of rectargle = Leng$ x Breadth
,4 =r x (6-r)
A= (tu-l1 m" (sbom)

Ansve6 virh conprete worke,i sorudom


@e)
31. (a) )= t+l8r+168

(b)
(i) the stop needs to sellt bottles of rvine a dat
to haxinise ils daily p.ofil,
(ii) the ddily na{imun prolir is $249.

32. (a) h=3+\ 1l

(i) whena=1.8.,4-5.75.
.. The.rea o rhe rec.dB.e rs5.?5cm rhcrl
r = 4.8 cm.
(ii) whenA =4,r= 0.75 or j= 5.25
whe!J=0.75, I
Lensth = 0.75 cD
Breadih = 6 0.75 = 5.25 c'n I
I

Leng$ = 5.25 cm
Bddlh = 6 5.25=0.75cm
.- The dimensiors of the rectangle ar€
5.r. cm Dy u,/5 cm.
(iii) the marinun dea of the rectdgle is 9 cmr.
(i!) when A is ndinun. r = 3.

Breadth = 6 3=3cm
. . The dlnensions of the rectangle when its
area is mdimum ls 3 cm by 3 cm.
(v) when its aEa is maxinun, thc shape of rhe
recongre s a squar.
(b) Frcm tbe graph, Marc buches the waler whe!
t- lJ5 s.

r"*".",., ,u
@ "*-
ll. (a)
(t jc aFs n rhe cemenreo pan. 4 r. r-d\rmum
whenr=1.5m.
(iit the dea cemerled pon 4
ol r' e him uhPn
-- 0.65 m orr - 2.35 h.

EF = (3r 2) Lt
-(r 2)m
Fc = (s 2x) zx
=(5 4r) m
Arca of shaded reeion, ,'l
= (5 - 2I)(3r 2) (5 -ar)(r 2) lt. q)
=(l5r 10 6l+4r)-(5r 10 4l+&)
- -6r'?+ l9I 10 - (-4i + l3I l0)
- 6r' + l9r l0 + 41- l3r + 10

r = 4 is the ii.e of symnetry of the cnfle. the


Slnce
poDJJ.q and r<.41{iI ha\e rhe.de, coordrnares.
Sub\.irule!a.4)"ndr(../li.ro)-/ |p -l roBe.
q=9+3p+21 +q-3p=30 (D
q = 25 + 5p + 2l
= q - 5p = 16
(2)

Solve (l) dd (2) simultanoously to obtain the valuo

(l) (2): 2P = -16


P=-a

When r= 0, ) =..
...=3 +
. r=a.l+bt+3
( 1. 0) and (2, 3) into I = al
Srbslitute + ,a + 3.

0=a(-l):+r( 1)+3=a D= 3
3= a(r1+b(2)+3 .1\a+2b=o-(2)
(t) x 2:2a -2b = 5-(3) -(l)
(2) + (3): 6a = 6

SDbstiiute d =-l into (1):


1 , =-l

.. a= -l, b =2 =3
^nd.

Answe$ virh Conplete Work€d s.ldds @


Let the ienglb of each simild recr2nguld stalh be

Lenglh ofwle fencin8 = 150 m (Gi!en)


lr+2j=150
2) = 150 _ 3r

r=175-tlrlm
'2

A =2 x Arca of each rectdglld stalls

= zdrs - l-Lx)
2
-a = {1s0.! - 3l) It' (shown)

(b) Equation of line of synmetry: r = 2.5

whenr =:1.2., = _1.7.


whenty=-1.7,a=0.E.
..4=0.8md,=-1.7

r=Px.+Qx-(t)
Choose dy two poifis fron the table and srbsliluLe
tlem into the equatio! above.
Subsntute ( 1- 3) into (1):
t=p_a-i)t
Slbsnture (1. -2) into (1):
1=P+Q-Q)
(2)+{3): l-2P
P= !2
substitute P = .l7 into (2):

2'
-2
.. P= ! ando=a!
2-2
r.i Fmn rhe gapL1. Lhe mar,mum valre o',he ola
enclosed dea is 1875 m'.
WhenA = l8?5, r = 25.

u=75-
'2 1(25) = 17.5
L

.. The dimen\ion. ot e,cl' \llll de 17.5 m b'


25 m when tle totrl elclosed dea is mdimun

t"*".",", *.. r"


@
(a) ,1 = {a : r is a pribc nunber}
(b) B = {r : x is a pGitivc square numbo < 25}
nt.A) = 7
(c) C = {r: x is ar intcger and 5 <J < 9}
(d) , = {r : : is a mnlripl€ or 4}
(e) E = {r :: is a mtural Dunber snallo then 6}
A = {Tresda}, Thursdayl
nlA) = 2
/ r = lx:r is a po\iti,e odd number smsller thatr
r0l
/o\ G = Jr : r i\ lhr lellcr ut the trnuli.h llpl'rberl
(c) a = {r,2,3,4,5t 6,...} (h) fl = {x : x is the month ol the year starting \dth
,(A) = inrinite
the letter './'l
A = {r,3, s,7,9}
,(,1) = s L G) L.{ l
Th , .e i, nol s cll.detimd . n.e { e do noL knor
,\ = 12,3,5,7. |.13,11,191 how 'ge.t is measured.
,{,4) = E

(i) A = {r,2,3,6,9, 18}


(b) [z
| ..er
, . $ell-defined
n is r prihe nunber g.eaLer rfian :10.

tg) A = {6, 12, 18, 24, 30} (") l{l


TILS,- - nol $cll-delined, n.e {e donoLLnos
how 'sman is neasu.ed.
,1 = {2Ll) + 1. l(2) + l, l(l) + l, 2(l) + 11 "
A = {J,5,7,9} (d) 7l
This ser is wel.defined since we c.n list rhe subsets
of {d. r}.
(r+2Xj 5)=0 subsets of {,. D} are o. {,}. {} }. {d, D}.
..r+2=0 of r 5=0 (e)
]= 2 or r=5 aa-i
A = {-2, s] This set is sell-d€fined. lL is the eopty set.
n(A) =2
3r l<9
irl
B= lb c a q tl ser !4 ha6 1 more ctmrni

a U,2,l)
=
r = l, 2. I' ;tiF
numbei
a uaiw,
:
ti l

dA)=l

nn both sided.
a = {_r,1l
nlA) = 2

2'++'=20
3'+ 2'= 13 3' + 3'= lE lr+4r=25
(.) la
r1 = {1, 2. 4, 5. r0. 20J
,{ = {4, 13,18,20,25} u = lt_ 2,.1, 5.201
,da) = s

An\{( \ $ irh conrf crc u.rl",i s,,l,t.* (O


(d) E Q= tl,
li) P=Q
e. a, p)

0t nco
(a) 2E {2.5,7}
. A={3,3} (b) oE)a
Lt+1-1
-r@r".,,rt
.. B= {3} {r}8ir,3.5}
A+B
9. (a) 2E{0.2.4,61
l t-l
A=11.2,3lu{4.5} (b) {a}g(a.r..l
A = 11.2,3.4,5l
a={1.2.3,4,51 o fq-l {appre, pa, orange}
..4=B
l8.18lE{ls.8}
(0 ltl
A = ld..t.
A =A
el.\ lb.cl 10. (a) E
5r- 1= 3 E t", ee
({a.,'t is a lroper subset ofA, i.e. {a,rl cA.,

5 fl, Z is a sutser or every seL.

(d) ct
..B-A
(e)
ld,,@i+a( ) =a)
G = 12,3, s,7)
a= (5,6.7,8,9, ...1 (0 E]
C=A s.3l
D=tal
t=
r = 17,2,
= {2.3,5,7} G) E
{01 "r = 15,6.7.8,9, ...1
1t
(") E
C = r, C and a de equal sets.
G = 1. G and 1 ue equal s€ts.
1l = .r. Il and ./ de equal sets.
G) [tr ({r}
l4€8
is prcpera subset ofB. i.e. {t} c a-)
(aJ n(A) = 4
(c) E, Z is a subset of eveq, set.

(b) No, rub! e d.


(d) E z(8) + 10, z(B) = s
(c) No,lstaplerl ed.-ffi (e) E {a. r.i... il d B
(d) € = [All stationery itebs] ({a, t, t, ., r} is not a prcper subset ofd
since it contains the sam€ number of
(e) B = {p€ncil, stapler, eraser} €lemen$ 6 A. Her€, {4, r, t, c, rJ is a slbsr
ofr. i.e. |a. r, t. c. r] E 8.)
1. l^J

R = lP,e, tl
r(R) = 3

r,,n .o", ,"


@ ^.,
(l) E] o is a subsel or elery set.
(.rE
...P=1_1.tl (n) E {bN}g I!, u, b, n. a. \ i, n.
"l
c=trl
f = {0, l,2l
s={ l, 1}
(.) E
(a) QcP 'r' [] tt' aa 1. lvond.D. lue d.').wedne\dry.
Thmday, Fnday, Saturday,
(b) PER Sundayl

(.) QcR E] {7t E {1, 2, 7. ral


(d) PEs or scP 15. a,I2l
A = {1.3,5} a, \al, IbJ, Ia, bJ
a= {1,5}
c = 1r,2,3.4.51 z, {ant}, {bee), {ant, bee}
D = {1,51
(a) (i) n(4) = 3
a, {tl, I2J, I3t, 11, 2t, 1r, 3}, {2, 3}, {1, 2, 3l
(11) n(B) = 2
t6. o, {4}, {6}, {8}, {{, 6}, {4, 8}, {6,8}, {4, 6,8}
(b)
(b) (l) Bca z, {4}. {6}, {8}, 11,6}.
= 14.8}, {6, 8l
ltl) ACC .. e 7 proper subsets of.1.
There
(ni)B=D
l7 AC6 l^
14. c)E (a)

.. Mdimum ,(A) = 49
(b)E
(c) EJ m e e {r, tu. r, r,
"}
(d) E
(e) f{] 2 € {2,3,5.7, ll, ...] IfCEDandDqC,thonC=D.
(i) n(O = 20 (civen)
(t [fl, 1i..,.] e {r, s,,,..., rl ''' n(Dl = 2o
({,,.. el c {t, s, o,.. e.l})
(ti) c-D
G) El.5 e l-5.61
18. (a) (i) a,t,h, i,., sl
E = |n, e,

61 fl. o.zs x 16 € lr : is a factor of 121


1€ '
11,2,3.4,6.121

(D E. t:t e {-r, :t (:t c {-r, :lt


(b)'
E.ze {r,I is a !e'fe.t square}
(O c {r:a ls a perrect sque})

rtr I

Answes wirh conpiele workt *t,... @r)


(c) (i) ,(€) = E (c) (i) €= 11. 2. 3,4, 5. 6, 7, 8, 9)
(ii) r(A) = s A= 11.3.5,7,91
(iii) ,(A') =3 B = 12,3.5,71

(a) .
19.

ffi/-l
v"t/ \-/
I ll t*'u I

(i) A' = {apricol orange, suaYa} (i0 AoB=11,2,3,5,7,91


(il) B'= {sfrawberry, plun, ap.icot, orang€J A^A={3,5,7}

They de disjoint s€ts. (d) (i) €= {1. 2, 3. a, 5, 6, 7, 8. 9}


A = {1.2,3.6J
20. (a) (i) € = {1, 2, 3, 4, s, 6, 7, 8, 9, 10, ll, 12, 13, B = It,2l
14, 15]
(ii) A = {s,10,1s}
(iit B = 1r,2,3,6,9I
(iv) A'= {1, 2,3' 4, 6,7, 8,9, ll, 12' 13' l4l
(v) B'= {4, s, 7, E, 10' 11, 12, 13! 14, ls}

(b) (i) ,(s) = ls


(ii) ,(A) =3
(iii) tr(r') = 10
(it ArB=11,2,3,61
21. (a) (i) s = 11,2,3,4,5,6.7.8,9l A
^R
=Ir,2l
,,1=11,2,3,4.s1
I = {2.4.6,8}
(e) (i) €= { 1, 2, 3. 4. 5, 6. 7. 8. 9l

B=11,2,5)

I o \/' ' \

(it A u a = 1r, 2,3, 4, s,6, 8]


\-7r." /
A^B=12,41
(iD Aur={r!2,4,5!8}
(b) (i) € = {1, 2, 3,4, 5. 6. 7, 8, 9l A^B-A
A = {1,3.5,71
f = {2.4,6, 8} O (1) s= { 1,2, 3.4,5,6.7. E,9J
A= \21

i,--\ /---{ B = \2,1,6, El

\r!y
/ ' 5 \/

9
, 'i \

(it A u A = {r,2,3, 4,5,6, 7' 8l 13 s 7


A^B=A
(ii),1uB={2,4,6,6}
A^B=I2l

ru'*-,,n, r*- ,u
@
22. (.n) A U B = {7,8, 9, r0, llJ 13} (b) d.d = {r0,20,30,40,50,...}
A^B=T9}
A. a= F :r is a nultiple of 101
(b) Au B = la, b, c, d, e, f, sl
25. (a) (i) € = {r,2, 3,4, s,6, 7r ...,,{9}
(iD A = {4,8, 12, 16! 20! ...,,r8}
(c) AuB={3,6,9,12,15} (iii) , = {16,30,44}
A^B=I6,t2l
(i) A^B={16,44}
(d) Ar) B= Ip,q,t,v,w,r,zl (ii),(,,{.6)=2
A^B=A
26. (a) A = {2,3,5,7, rr, n,...]
A u a = {pecil, pen, eraser, ruler, stapl€rl n = {4, 6,8,9, r0, 12,...}

O) A^B-A
(i) A=la,o) (ii),(4.a)=0
B = la, el

(h)
A = {1.2,3.61
B= (1,5)
AwB=11,2,3,s,61
Ana={1}
AUB=\k,e,!l
oo
A^B=A
(i) A=ll,ar,s,a.l
Aw B = Ia, d, e, s, t,n,o, sl
A^B=Ie,t,nl
(j) A= {-4.1}
B- {1,2,4,5. 10.201
AoB= .4,1,2,4,5,10,m1
A^B=l4l 2'7. (^) (i) AnB={3,s,71
(ii) ,{ u B = {r,2,3,4, s,6,7, 8,9, rr]
(a) A ua= {3,4,6, 8,9, 10, rr] (ili) ,(A B) - 3
(iv) ^u a) = r0
(b) A u C= {4,6,8,10,12, 16}
"(A

(c) , u c = {3,4,6, 8, 9, rr,


(i) [r] (ii) E
I-1. ,
12, 16} (iil) A, Not an elements in D are in A
d
(d) A.a={6} (i')lrl
(e) A^c=14.8J 28. {r, 2. 3,4, 5. 6.7. 8, 9, l0l
hlA^c)=2 {2,4, 6. 8. 10. 12. 14, 16, 18, 20}
C= l\5. 16, 17. t8. 19, 20. 2t. 22, 23, 24, 25]'
(t B^c=A (i) na=
,(B.C)=0 (a) A {2, 4, 6, 8, 10}
(ii) a. c= {16, r8,20}
(a) (i) €= {1,2,3,4, s,6,7'.,.1
(iit)A^C=A
(ii) / = {s, r0, 15,20,25,...} (nr,(A^a)=s
(v) ,(O rr
(iit , = {2,4,6, E, r0, ...} (vi),(A^O=0
=

Answes vith conplete work"d sor.Lio$ (A


(t 8.7 etr r- 11.2.l.4. \.6.7.8.q. 10. l.12. 11. 14. 15. l6)
(il) A = {2,3.5,7. 11, 13)
E,A uB = {1, 2, 3,4, 5, 6, 7, 8, 9, 10, 12, a = {1, 3, 5. l5l
14, 16. t8,201, 16 e,{ u
(iji)[tr ' (a) A u d rs}
= {1,2, 3, s,7, 11, 13,
(iv) [tr (b),a.8={3,s}
29. (^) € = {i. 2, 3,4, 5, 6. 7, 8, 9. tol
(i) A = {2,4,6, 8, r0l ,c, 4'^a-11.4.b.8.o l0 l'. 14 la lol . rl.
() B = {3,6,91 3,5, l5i
(iii)c={4,8} = {r, 15}
(iv) A'= {1,3, s,7,9}
(v) a'= {1,2,4, s, 7,8, r0} td, A B =.2. t.5.7. ll . ll .
12. 4. o. '. 8. o. lr.
(vt c, = {1, 2,3, s, 6,7,9, 10} 11. 12, 13, 14, 16] ^
= {2, 7, rr, n}
(i) .{ u a = {2, 3,4, 6, 8,9, 10}
(ii) auc={3,,t,6,8,9}
(iii)A.a={6} 32. (a) sinceA =8,2 -; and, = 1.
(i!)A^c-{4,8l
(!) B^c=A (br sin.e,4 _ A - A, rhe r{^ |ne\,. -or I re6a|'
(vi) A'^ a'=
U, s,7l i.e. they @ pdallel.
(vii)A'u C= {r,3,4, s,7,8,9} 1
a = t md , cd be any value ercept l.
l0 (a) (i) r(a) = 3
(il) (r) (i) P = {r€ctansl6, squares}
Q\ Aw"(4=a
B = la,b,c,d,el
33
(ii) 0 = tsquaEs, .hombusdl
(iv)A^B={dl (iii) a - {paralldosrams, rhoinbuses,
Ectangls, squares]
(b) (i) ,(r)=6
(ii) n(C) = 2 (b) € =.{All quad.ilateEls}
(in) r(cu D) = s
(iv) c n D= {r,,} lr, r. r} (c) (i) P^0={squares}
= {r}
^ (ii) 0.x = {squares,rhonbus6}
(c) (i) n@^n=2 (d) PcR -
(11J n(E \) F)'=z
(iii) (E. 4'= {1,2,3, 9, 10, l 34. €= {1,2.3,4, 5.6.7}
(iv) Eu F = {1. 2, 4, 7} u {1, 2, 3, l0} C= 12,41
= tl,2,3,4,7,10) {1,2,4,6}
(v) E 10, lll-{1,2.3,r0}
^,c={3.9,
= {3, r0}
(vi) True
(vii)False,3e(Euo
(viii) True

(d) (i) G'= {3, s, 6,9, roL


{li) r(Gu14'=3
(iii) d d= 13. 5, 6, 9, 10j . 12. 6. 8, 101
^
(iv) u 11=
= {6, 10}
8l u 9l
(b) lt) C^D=12,41
G 11. 2,,1, 11, 3. 4, 5, (c {1j 3, s,6,7}
= {1,2,3, 4, 5, 8, 9} ^D)'=
(v) dug= {3,5.6.9, l0} u {r,3,a, 5.9} (ii) c u, = {1.2.4,6l
u,3, 4, 5,6,9, 101
= n\Ct)D)=4
(vi) lialser(5€11but5 € /t')
(vii) false,2€G^It Yes, C c D.
(viii) Fals€ ({1, 2, 4, 8J is not a proper subset of
G since iI contains the sme number
of elenents as C.)

r"r"."u* r"
@ ^.,
q. 10, 11, t2. BJ r'
.1r € = {t, ,1, 5,6. 7, 8, A' u = {6. 9, 101 u {6. 8l
A = 13,5,6, 101 = i6, E, 9, r0l
a = 15,7,9, 10, i1, 13)
4' . d'= {6, 9, 101 . {6, El
(d) (i) A !.r l] = {3, s,6,7, 9, 10, 11, t3} = {6}
(ii) A .8 = {s,10}
{iii) a'= {3, 4,6,8, 12} Altemative neihod: Drar a Vem diag.am
{iv) (/ u 8)/= {4. 8, 12}
'(,4u8)'=3
(v) A'^ A = {4, 7, 8.9. l r, 12, r:l} . {5.7.9,
i0. u, 13]
= {7.9. I l. 131
h(A'^B)=1
(b) (i) False (ii) False (lii) False
(iv) True (v) Tne (vi) Falsc (a) (4 !.r d)'= {61

16. r.= \a. b. c, d, e,l, L h,...1

(b) (4 . Bl' = 16, 8,0, rrrl

(r) ,{ - B = {a, I, I, i}
(b) B.\ C = \a,t,i)
(c) A.c={a,r!/,i}
(d) (A . 6) u {A . O = 14. s. l. i}u {4, /, l, l} (c) .1' Lr B' = {6,8,9, r0} '
= la, t, a s,tl
(e)
rgr '(A^B)=4
,rd^tl ='l€) ,rd.tl. i,iia):26
(d) ,4'- 8'= {6}

37. (a) 1'= {r,3,5}


B = 12,4,5l

(c) (')'=A 39. (a) M. N= {1.ll


= 12, 1, 6l n(.M^l!)=2
,1u ll'= {2.4, 6} u 12,4,5l (b) (i) MLrN= {r,..d.c,/}
= t2, 4, s, 6l (M u ,\)' = tel

rl,. a= {1.3,5} . I1, 3,6l (1i) ,w. N= {.,.,8} . {..4..J}


= u,3' =\c,ej
(l r ! 6,= 11.3, 6l ! i2,.1,5) (iii) ,u u Ar = 1., ., s) u {r, s}
= {1,2, 3,4, s, 6} = lb,c, e.sl

G) R . B',= Ir, l, 6l . 12. a. 5l € = 12, 3, 4. 5. 6, 7, 8. 9. 10. I l, l2l


=O (a) f, = {2,3,6,9}

18. (a) A u A= {5,7.8,9. l0} (b) 5r l>14


(ArB)'=16l 5r> 35

A {5.71 .. r = 8, 9, 10, 11. 12


(,,1^a=
. B)' = {6, 8,9, r0} s = {8, 9, 10, rr, 12}

Ansse, s wirh conprete worrre,j sorutios


6Ee)
(c) R'. s= {4.5.7,8, 10, I l, 12}. {8,9, 10, I l,l2l 42. E - { 1 , 2. 3. 4, 5, 6. 7, 8, 9, 10, ll. 12. 13, 14. 15}
= {8,10,11,12} A - {5, 6.7,8,9. 10, ll, 12. r3]
B = {5,7. ll, 13}
(d) R uY= 12, 3,6.9) U 12.3,4. s.6, 7l
= {2, 3, 4, 5, 6, 7, 91 (a) A' = U,2,3, 4, 14, rsl
(e) n'^ y = {4, s.7.8. 10, 11.12). {2.3.4.5.6.71 (b) B' = {r,2,3,4, 6, 8,9, 10, 12, 14, l5}
_ {4.5.71
(c) ,4 B = {s, 7, 1r, r3l
'(r^s')=3 ^
L/ f s={4.5. .E. l0. ll.l'lL ' . 'r .6 I (d) A'ua=11.2.3,,+, 14, 15| u 15,7, ll. 1ll
= (2,3.4,5,6,7,8, t0. l l. r2l = {1, 2,3, ,{, 5,7, 11, 13, 14, l5}
a(Ru,5)=r0
(e) {1. 1.7}
^8=- {1. 1.7}
^ 15.7, ll. lll
E= Vjr.i,g,o.n.n,e.ll I7l
A = ls,e. a. n,t. r, Jl
43. (r) .

(a) (i) (A B)' = {s, o, t, n, r,.}'}


^

(ii),1'.d={t}
(b) €= 17, 8,9, 10, 11. 12, 13. 14, tsl
P = {9. 12. 15}
-
0 {7, 9. 11, 13, 15l
R= {10. ll. 12, 131
(iii) (,a u 8)' = {D}
(i) P \, 0 = {7. 9, 11, 12, 13, 15}
"nl
t, t,t,rl
(iii)
n(PQO=6
(ii) x' = {7,8, 9, 14, 1s}
O
^
R'= {7,9. ll. 11. 15} . {7.8.9, !4, 15}
lirJ A w B' = Ie,s, o, h. = {7.9. 15}

44. (a) A ua= {squaresl u {rhombuses}

(b) (i) A/. a'= {1. nl .{8,,,n.^tr'l


= lnl c u D= {parallelogrdms} u {rectanglesl
nlA'^B')=r = { paralebgran$ }
-c
(ii) A' !, B'= {,. ,l u {8, ,. r. /, )l
= t8, t. o, h, r.r) ,,1 I = {squlresl - thonbsesl
n(A' w B') = 6 ^

r",*.",". r*- ru
@
(d) A. D = {squarcs} {rectanetesl (b) Since C c ,, set C is contained inside set D
= tsquaJesl
^

X= lb.c,dl r = |d.b,c..1, el
(^) z-Ia,el
X r) Z - lb,c,.tl u Ia, ej (i) ,c^D)=n(i:)
= la. b.c, d, el

(ii) h(cwD)=n(DJ
lb) z = [b,., d) - =10
Y = Ia.b, c..1,el 1b,., dl
^z ^

G)
E
trl
tf t
@.4'na=a
G)
ffl
(d)
(08
(h)
E.Acg

E (b) P^Q=P
@

(b) 5'+ l2'= 169


(d) PL)Q=Q
.. The trimgle wilh sides 5
is a right-angled tridgle. 50. (a)
A = {1, 2. 3,:1. 5, ... }
B = {1.2,3.4,5,...1

G) [fl
A = 12.3,5,7, 11. 13, ...1
a = {2,4, 6. 8, 10, 12, ...1
A^B=12'
..A^B+A

48. (a) Since A


^
B = O.,4 and a de disjoint sets. c)E
lalgAbsaeA.

o
'(A
uB)= n{A) +
=E+5
"(r)
{d E]Pt /- l.zlanda ll.J.

c) [L]
(r [L
l e,RbutI+n.
efd,o

G) ElrA =8. theiA E B andB E,1 .

Ansses wirh conplete worked sorurtos


@)
(h) E E.s.IfA = { i } and B = {1,2}. then 4 CB (c) Ana

(i) E
l) Tit

(d) A'.8

IfACA,thenAud=4.
l-r I

o
tf A c B aa.l B
^ Q= A. $en A
^Q
- |

flln ta t = a, rrterto a +a. (e) u B)'


A (A

(m)

(r)
(1) Aud

IfA n, = A, tlen A !r I - A.

ffi
51. (a) A'

ffi
(b) AUB
G) (A . a)'

(A Mathemdaruo,28
A'QB'

(i) A' B' (l) (Aua)^(A^a)'


^

:n; 1

(A.B)UA,
52. (a) AUB

(b) /t^,

{nswe6 wi!h complere !'vork"d s.r't


"", (O
(,,r. o' (gl 4'. c

(d),4'uB

(h) (,4 u 8). c

ffi (i) (A^a)uc

(t A^B'
B

@o i

x:
(j) (A^B)u(r^O

r"*"."0* ,"
@ ^.,
(k) (A u B)'^ c (c) acA'

(d) A^B=A
oo
(t) (8.\ c)' .\ A'
5.1. (a) A^B=A A AQC=A
v"

t
@o
a c,4, r(d. c) =0and c.A - c

53. (a) A CB

(c) B c 4. a.c=o cGA Jotl,r4. cr+0

(b) A^B=A

oo t,lr (4 uC) c A, CeA cndr(4


^O+0

An!sc,\ s rh conprete worked s.ruLroN


6.i)
There ae other Possible answeB
(c) f4uq.(r.O
(a) Ant
(b) ,'
(c) AUB

(d) A'^B
(e) (,a na)'
(D IAJ B),
(d) (B u C).4
(s) A.r'
(h) A'u B

(t (A.B)u(B^C)
(j) auc
(k) (,4 c) n B'
^
(t) (,{^r)uc (e) (A u c
^a)

r. (a) tB. O. A',

(l B'.rC/]A

Q) (Aua)^c

2. Answers mY vdy
(a) (auB)nc
ft) (a^a)uc

@ r.*..**',-
,u
a la. bl la. b. cj L€a-st possible value of ,(A u a)
la. d = n(A)
Ibl la. dl =24

Idl lb, dl
\.,.t|
4. (a) Greatest possible value of r(A n B)
= n(B)

5. r = { l,2, 3,4.5,6,7. 8, ...}


M*N=(Metr^(M^M'
tf,tM= {1.3,41 andN= {3,4,5.6}
Mu N= 11.3,4,5.61
Mn N= 13.4)
(Mn M'= {1.2,5.6.7. E,...}
(,uuMn(M^^')' = {1,3,4,5,61 {r,2,5.6,7. 8,...}
(b) Lqst lossible value of,(d . ,) . M* N= Il,5,6l ^
=0
.. { 1. 3,41 * {3, 4, 5. 6} = U, s,6}

6. (a) (i) {Pupils who like apples bur not grapes}


= {Pupils who like apples} lPeople who
do not like grEp€s] ^
=A
^G,
(ii) {Pupils who like gapes btrt not both apples
and pears I
= {Pupils who like grapes} .
{Pupils who
do not iike both apples dd peaB)

(c)
=c^(A^n'
Crcatest possjble value of,(A u B)
= r(.{) + ,(8) (iii) { Pupils who only Like two of the thrce ftrits }
=20+8 = {Pulils who like apples and grapes bul not
peml u {Pupils who like grapes and pears
but not applesl u lPupih aho like apples
and p€ars but not grapesl
6^G)' P .\G' n. A t^ P)^tJ'

(b) Dnw a Venn diagram to dswer this section.

LeI l be the number of pupih who like all three

AoqweF rrLh Complete Worked S"lr,l""'


@
There are 4 pupils inA G-
There re 7 lupils in C ^ P. .
8 pupils llke ody apples.
=2
No. of pupils who like otrly Srapes .. Eqmllon of llne of symmery : r =2
=19 (4 I) j (7 r)
=19 4+r-r 7+r (c)
r'=(5-2Xr+2)
=9
No. ofpupils who like both apples and pears btrt

=2 3+1
2 (a)
No. of pupils who like ody pea6
-5
=20 2 r (7 r)
.. Equation of line of symmelry :r =5
= 1l

..8+r=li j
Substitute (3,0) and (7,0) into ) =r+ d+r.
0=3'+a(3)+,

o=11+a\7)+b
]a+b=49_12)
..,(,4.G.P)=3
12) - l1): 4a - 40
.. 3 pupils like all three l'tuits.
S$sttute a = l0 into (1):
3(-10)+D= 9

.-.a=-lOandb=21

) = 5' 10(5) + 21

. . The coordlnates of the midinum pollt is (5, -4).

lar nre rd,,ge.l va ue.


^l ' lo, shich
,te gadienr oi
(a) 1= (5 - rxl + r) tlE curve is positive is.r > 5.
When jr =0. . 1=5 l-ine uf\rmmflry
(5 J)(1 +r)=0

- (-r, 0) and B = (s, 0)


..1
Wheni=0, .
v=(5 0x1+0)
,,r 'lffl,rygM.
craaicir; 1
c=(0,s)
ff"ii",
- 1',;.1 s'10*t 9t',
-Nr9)6
r'1
@ra.

t"*".",", ,*- ,u
@
3. (a) l=J: 4r+3 :1. (a) ]' = 2r' si + I
d=( 1t 1( 1)+3=8 a=2(t): 5(l)+r--2
,=3' ,l(3)+3=0 , = 2(,1)' 5(4)+r=t3

(i) when a = {.8.J = 6.65-


(ii) whenl = 2, x =0.25 orr = 3.75.
(iii) the coordinates of the nlnimum point or the (c)
curve is (2, -r). (i) when J - 10.: =-1.2 orx - 3.?.
r.\l J'e equaioo olJ'e Ine ot jme,D i x=2 (ii) whenr=2.6,t=1.5.
{iii) the minindm point is (1.25, -2.1).
(iv) the equalion of the line of rhe symmotry is

5 (a)
o=2 3(4.5) (+.5)'= -4.7s
b=2 3(-2:) (-2i = 4

An,sers u,rh compek wurr€d sorm,o", (rE


(i) when J = 15,r=-2.ss
(ii) the mdimum value of I is 3.5 when r = li
(iii) the equatior of the line of symmery: r = a5
7. (a)

1..irrL:.
Equalion of lire of symmetry:r = 2

(i) whe! r = -4.2.) =-3.0s- Coordimtes of minimum point = (2! 0)


(ji) when ] = 2.5,i = -2.8 orr = -0.2
(iii) tbe ertatest talue ofJ = 4.25.
(tr) wnen y =ru,a =-r.r5 orr =5.r5.
t=3+Zt-21 (iii) whon,= 3.5,1 =4.s.
a=3+zGi)-2(3)7=-2r
8. (a) ) -t(1 1)(r + 3)
=

p=-;{ 2 l[ 2+]r=r.s
4=-;d r(1+Jr=0

*"*".",", ru
@ ^..
i:, ()
(i) ) = 10. x = -2.75 orr =3.25.
when
(ii) r = 1.5,1,=-2s.
when
(iii) the smallesr value fo 2rr a - ?8 is
app.oximately -28.25.

(c) Fmm the eraph, the greabsr value of ] ls 2 and i!


(d) Equation ofliDe ofsynmeLy: r = 0.25

28+r=t
EquatioD of line of symnetryrr = -1. 2,r' a 28=0
.. The solutions a.e x = -3.5 or x = 4.

whenr=0.7,r=0.55. L0
wnen I = u.:\),r: r./ or r= u./
(c) Areaoftnangle=
t x Bare x Hcighr
.. a - l
-2,7,
A-tx(r+2)x(s r)
.. a = -2.7 and , = 0.55
t- r + 10
= 2(5r Zr)
e (a)

= ,(-j +lj+ l0)


r = (-tr'+ I + 5) cm'bhosn)
2r

(b) il= ;;+ r;x+5

Answes wirh conprere work"d s.r,t.N


@)
(i) wben I is gleatesl, t = 2.
.. The ball is at its g€ates! heighi when

(ii) the Breatest heighi reached bv the ball is 16 m


(iii) I
when = 10. t= 0 8 or 1= 3.2
.. The requircd rarye is 0.8 <, < 3 2'

12. (a) A=(a+8)5 {r+2)


.{ = (40 + 3r - x?) cnra (Shown)

A is maxinum when r= 15.

base= 1.5+2=3.5cm
heiCht=5 1.5=3.5cm

ll. (a)

(c) From the 8!aPh,


(i) whenA isnaximun.r= 1 5'
.- The area of the remaining sheet of
aluminiun is ndimum when t = 1'5 m'
(ii) whenA =41.r=04orr=26
. . When the dea of the iemaining sheel of
aluminium is 4l cnf x=0'4cm ori=2'6.n

13. (a) (i) €= {r,2' 3,4! s'...}


(ii) .4 = {4 4, 6,8, 10, -}
,
(iii) = {3, 6' 9' u' rs' - }
(iv) A'= {1, 3' 5' 7, 9, . '}

r"
@ ""t"..,o,...
14. A = {2,4,6,8, 10, 121 (lii)E,geobure€0.
a= {4,8, 12}
c = 1.1,2.3,61 0v)E]Pn9={2,3,s.71
D= {5,7, 1tl ..3eP^Q
E= {1, 3,5.7,9,...}
F={1,2,3,6} t9_ (a) (i) Pun={7,9,rLrsl
G = {2. 3, 5, 7. I L, 13, ...) (ii) 0^x={11}
(jii) ,(0 x) = r
(a) aEt (iv) n(P u^ R) = 4
(v) (Pu0).R={7.9, 10,1l} {7, ll. 15)
@) B [q.4 = 17, rrl ^
(vi) (P . 0) u R = {9. 111 u 17. 11, 15}
(c) c El. = {7, 9, r, 15}

(d) DEG G) (i) EP-{7,e, 1r}


15.
0 u n = {7, 9, 10, 11. 15}
..Pc(OuR)
(") E
(b) E 1i1 fflrap=1e.rt1
.'.9ee^Ql
(c) bl {vl 4,1but l}} CA
1ii1 fl 9 un= 12, r, ro, ti, t:1
(d) ffl ..10 € (q un)

G)E 20. (a) 3

(o E, {r} dA

(b) (i) A' = {4, s,6 8}


(ii) Aua={3,4,6,7,9}
(iii)14^B={9}
G) []l (iv)(,auE)'={5,8l
nIAUBJ,=2
Ff
{b)
(c) No,A€t.
G)E
(d)
21. (a) B^C={5!t}
ffl. o is a subset of eve.y set.

(a) (i) A^B={8,20} (b) A' . a= {4, 6. 7, 9, 101 n {3,4, 5. 8}


(ii) = {4}
A u a - {L 3,4, s, 8' 12'14,16,2I,241

(b) Ul
(c) B' c - It, 2, 6, 7, 9. r0l. \1, 2, 3, 4, 9. 101
e = 17,3, 4,5,8,12,74,16,20, ^ = 11,2,9,1Ol
i8. (a) (t P^O={2,3,s,7} (d) a u C= {3,4.5,6,7. 8}
(ii) P v Q = 11,2,3,4,5,6,7,8,91 (B u C)'= {r,2,9,10}
(ii\) nlP . Q) = 4
(iv) {1, 51.P= {1. 5} {2. 3, 5,?} A u C= {1,2,3.5,6.7,81
= lsl
^

O) (D E.PeCbutPcO. € = {1,2. 3, 4, 5, 6, 7. 8, 9, l0}


M = 11.2,',1,9)
(ii) E N= {2.5,6}
P = {6.7,8. 10}

Ansves wilh comprete workeu r"*.* (oD


(r) M^N=l2l 26. f- [7. P.']. 10.
Ll. 12. ll. 1,1. 15. 16. l-. 18. la.2c.
2t,22]'
ff'^ N = 13, 4. 5, 6, 8. l0l - {2, 5. 6} A = {8. 16l
= {5, 6} d = {9, 16l
(N^ c = {7, 11, 13. 17, 19}
M \.r P) = {1.2,7.91 u 16}
= 1r,2,6,7,91 (a) A^d={16}
NUP= {2.5,6,7,8. l0l (b) A^c=a
(NuP)'=11.3,4,9) r(A.O=0
(c) (]={8.9. 10. 12, 14, 15, 16, 18.20.21.22}
ll (r) (i) E= {r,3,4, s,7,8,9} rlc) = rt
(it a = {3,,r, s, 8}
(iii) B = {3, s, 7} .u 4 'c q, t6l . {8.o. t0. t2. t4. 5. 6. P

{iv) B'= {r,4,8,9} 20.21.22)


(v) /Ua=13,4,s,7,E} = {8. 16l
(vi) A.a={3,s}
(vii) (Aud)'={1.9}
n\AuB)'=Z 21.
(vili) ,1' . B = { l, 7. 9} . ll. 5. 7l
= tll Q) ru. = {s. r, u. r,,}
n(A'.8)=\ (K r L)' = 1.r. tj
Y6, {3.7} c B (c) (./ u L) r= {4, r, r. r, !, rI .ls, u, r, v}
^ y, yi
c= 13.5.71 = tr,

28. {/.,.1, r} .
24. (a) .

(a) 4
BL) C = ld, e. t, a, s, tl
n(BrC)=6
(b) (t) B'=Ia,b,e,Il (c) A=B
(11) Aw B = 1.,d, f,s, hj
(ili) (lr u,.1)' = {a, ,, 29.
li\) {A
"l e,l,
ts)'= |a, b.d, hl
h(^
^B)'=6
25. (d) (i) € = {pdallelosrans }
A = {rhombuscs}
a = { reclangles l
A.Zr={squaresl
30. (a) .
(ii).
a-).]
,-'-:\ C

\"/'".-
(b) € - {1.2, 3,4,5....}
(b) P= lisosceles bianslesl M = { l. 2. 3.4, 5, 6. 7, rJ, 9l
0 = Iequilltern1 rri.ryles l N= {3. 4. 5. 6, 7, 8, 9,...}
(i) P
0 = {equilateral trianslesl (l) M. N= {3! 4, 5, 6, 7, 8, 9, ...}
(ii) ^
u 0 = {isosceles t.ianeles}
P
(ii) M = {r.21

r",*-,,.,
@ '*-,u
31. (a) (b) Aug

(c) (A^4u(Aua)',
Y x
(b) € = {2. 3,4,5,6,7.8,9, 10. 111
(i)
(ii)
P =12,3,4,61
zx 1<l * H *
2r<8 (d) (,4^a).(Aua)

... 12,31
(iit) P ,Q =g = 12. 3. t.6) {r.5. 6. -. 8. a. 10.
^ 11)
= 14,6l

32. (a) Aw B'

l.) A'^ B
AB

(.trw
/-\ ffi

(1r) €= {1,2,3,4.5,6,7. 8,9. 10. ll, 12, 13}


A = 11,3,5.7.9, 1r, 13) (0 B'^A

@Nffi
B - {2. 3, 5, 7, 11. t3l
c - 11.2.3, s)
(i) A.c={r,3,s}
(ii) d'u c = {1,4.6.8,9, 10,12} u {1,2,3.51
= {1,2,3,4,5,6,8,9. 10, 12} G) (,,r . a)' u c
n(B' w c) = 10

(iii) A a'= {1, 3, 5,7.9. 11, 13} . {1.4. 6. 8.


^ 9. 10. 121
= {1,9}

w- x (h) (,4 u 8). c


R c

n 4

Answes vilh cooprete wo.k"d sor'ttods (e


Most comnon mdk = 85

(b) Highesi mdk = 98


A. Dof.Diagrams
1. (a) :
l{;x30=15
0l2 3'l 5 Fron the dot diaerum, 15 siudenLs scored 85 ndks

7 8 910 ll 12 13
The distiiution shows two chsters. at 69 lMks
(c) . md at 85 marks. The marks !a.y from 60 to
98.

36 17 38 39 ,lO 41 42 s. (a) No. of pebbles in the bag = 16

(b) Most comnon mass = 39 g

45 50 s5 60 Mas of lightest pebble = 30 g


Ma$ of heaviest Pebble = 55 g

{d) No. ofpebbles ofnas less than 40 e = l0


80 85 90 95

=t0
3 0 4.0 5.0
5

2. (a)
The nds of the heaviosl pebtrle is an extrene

(b) Mdimun wainng time = 25 min


Minimum waiting tine = 0 min

(c) 9 min and 16 min

(d) No. ofpatients who had to wait more than 15 mir

2A 21 22 23 24 25 26
=itrcfr% L.ngdr

{b) Most comon lergth = 24 .m


=411%
Minimun length = m.m
3. (a) Most conmo! no. of books read = 3 b@ks Maximum length = 26 cm

(b) Mininum no. of books read = 1 book ' (d) No. of lobste6 tha! de les than 24 cm long = i2
MaximDm no. of books aad = 7 books

(c) No. of studenls who read less thd 3 books = 6 =;tr0o%


- SOAa

= 9 ,. roogt

= 40E

The data clustef around 3 The dala

r,t"rr..mrtc.t,rozt
@
7. (aJ (b) Disaibutio. (Fed with fish food Al:
Tle rcrea'e in lenSrh o'e"ch f.l'\a,) t,^a .m
ro6 cm. Tl'e in*ase ir lerglh i. taidy \ymrelricdl
with a peak at 4 cm.

Distribulion (F€d with f,sh food r):


tternc,ed.e,n lengthole"chfi.nraD lrom4cm
to 8 cm. The data cluster dound 7 co.

Tbe fishes fed wi$ fish food A shows a faster

e. (a)

l-l--t- rr 2u 25
Eeighl Gn)
30 35

(b) No. of plmb talle. than 30 cn = l0


(c) Number of limes the score is even
(c) Most common heighr = 29 cn
(d) No. of plants wbich are at least 28 cm = 20

Required dgle
=-^100% 2A
=-^160.
= 2q"
The .lata clDster mund 3, This shows tbat lhe
10. (a)
die used is a biased die.

8. (a) (i) F€d wiih 6sh food d 60 65 ?0 ?5 80 85 90 95 r00


No. of fishes which had grown at lealt 5 cm
(b) Lowest luk = 60
=5 Hiehest rnaik = 99

Requned percentage
(c) Most conmon mdk = 7r

{d) No. of students scoring nore thm 85 mdks = 5


Requircd percenlage

=-^100%
rii) FeLlwith fish food,
No. of lahes which bad grcwr at least 5 cm
(e) No. of studelts selected
-14 l
Required percentage

Lowest mdk of students selecled = 92


I

Answe$ wiih conplete work€d S"l.tt.* 6?)


B. St€m and leaf Diasram. Mdimm speed recorded = E8 lcn/Ir
MinimDm speed recorded = 35 kr lr

(d) Number of cds tEv€llirg nor€ thd 60lG/h


I I 3
2 ; 5 5
3 0 '7 8
3

- x
5 9 lU)Ea
=
(b) = 24%

l The speeds of the cds clusler dou.d 50 kn-/h to


I 5 8 60 kinh.
,7
5 0 0
2 2 3 13. (a) Most comon wingspan = 6E mm
'7 5
8 0 (b) No. of buuedlies with wingspes les thd 60 !m
(c) sten Lerf
2 27
3 00 0
5
20
35 5
I
5 01 =7
6
7 3
No. of botterflies with wingspm at least 65 mm
(d) Sten If,af - t{)

5 0 l
5 5 7 99
6 2 = _: ^ ru)Ea
6 6
1 0
l 5 14. (a) Mosi common length = 93 mm

(e) Sten Leaf (b) Lengft of shonest wom = 8l tm


Lenglh of longest wom = loE tm
12 38
t3 01 7
00 0 (") ;
i5 2
16
.- 30 woms m shorter thd r tDm.
(f) sten L€af From the stem and leaf diagran, 30 of the {voc
tre sboner thd 98 Dm,
15 0
5
't7 3
1A 2 7 15- (a) No. of studenb
'79 t
=2+6+7+2+2+1
=20
12. (a)
TTT::TffI1'ffi (b) Time taj(en by fastest studen = 11.3 s

Qualiftins tine < rr.9 s


(b) Mos! comon speed ftcorded

(C tu,n*u.".
"'-
ru
16. (a) No. of pcople ehpioyed 19. (a) Bobby made Lhe longest call thal day. (87 nin)
=11+7+3+2+l
=24 (b) Bobby made rhe sbolresl call that day. (10 niD)

Highesr monthly wage = $5900 (c) Tbe duration of cals made by Bobby cluster around
30 nin to:10 min. Hc nade ver) l€w calh lasdng
(c) No. of employees whosc monrhly wage < $2500

The duEtion of calk hade by Anne clnster dound


60min to 70min. She frade lery few calls hsnng

= t x 10070 ThDs, the average duration of call made by Anne


= 62.5E ls longer than the duFtion of cau ftade by Bobby

The monthly wages of the employees clustcl


a.ound $ I 000 ro 52000. Very few employ@s eanr 20. (a) 2t0 6
no€ than $3000 per month. 302 6 6
412 t 5
17. (a) No. of nangoes h the box 50r 5

=2+2+2+8+Il+7
Mass of heaviesl child = 58 kg
Mdss of lighrest child = 20 kg
Mass of heaviest ndngo = 408 e
Mas of lighlest mango = 358 g (c) No. of chil&en havhg mdsses < 50 kg = 14

Most common md\s = 396 c No. of childeD havjng ndsles > 36 kg = 13

(i) No. of G.ade,a mansoes = 7


Pe.centage of Grade A maneoes
=;r100%
= - t 100E0
= 65E
= 2l,a1s%
21,- (a) 12 89
(ii) No. of Grade B margoes = 19 tl r355 889
Percenlage of Grade B mangoes 0:1 45 667 9
l5 0001 35889
= ; x i007. 16 0241
= 59.375Ea
(b)
(iii) Pe.centage of Grade C nangoes
= taoqo 21.875% , 59.375Ea
- 14.75E
Required heiShl = 150 m
The mases of the mang@s cluster dound 390 g to
No. of studenh whose heiehts de betwen 140 cm
400 g. The distdbutior has a thinner tail towards
the lighter mases. This means that thEre de only
a fes maryocs with mases les thd 380 g. =12

18. (a) (i) School, had Lhe higbesl scorer (100 na*t
(ii) SchoolA had the lowest scorer (51 marlt 12

(b) School A lerfomed better in the examination s,nce 2


the ndks of rhe studerts from School ,4 cluster
dound 90 Narks ro 100 ma*s while the marks oi
fron Sch@l B clustef alound 60 ndks
the students
to 70 mdks. Tbls meds that the average hdks of
the studenh fion School A is higher than that of
the students in School d.

Ans,es with compreb worked sorutro*


6D
22. (J) 057 Middle losilion
tt38
34599 {d) 63. 75. 75. 80, 80, 82, 88, 90. 92. 92. 92
238
(i) Mode = 92
(ii) Median = 82
(b) 000157
0112 3 4 88
12345799 = 82.6 (3 sic. fis)
023558
2t399 Middle position

(c) (i) MosL conmon mtuk = s0 12.0. 12.5. 12.9.13.2, 13.3, 13.8, 1.1.2, 15.8, 15.8-
(iil No. of studerts who scorcd > 75 ma*s = 14 r6.9, 17.2. 18.7
Required lorccntagc
(i) Mode = r5.E
- ^ rno,;
= 15+17 12+l
(ii) Middle pn\lti.n =
= 43.757a
-
= 6.5th Position
C. M@n. Median and Mode Mcdian = Mean of6lh and 7th values
l3 8 + 14.2
Middle posiLion =,2
(a) 1,2.2,2, 3,.1.5.5,
- 14.0
23. 6, 8

(,)
= r4.7 (3 sie. fic)
(ii)
Mlddle position

(iii ) Mean
s23, $29. $35, $38. $3E, $45. $46, $48.1j49
_ l+2+2+2+l+4+5+5+6+8 (i) Mode = $lE
lo (il) Median - $38
38 $351
=lo (iii) Mean = e
- 3.8 = $39

Mlddle posilion
24. (^) Mean =
lxl+2x2+4r2+5r4+6xl
(b) l, 3, 3.4.5.6, 6,7_ tl
39
(l) Mode=3and6* =ro
(li) Median = s

(iii) Mean =t
l0+l
= s.r, (r sra. fle.) Middle Dosltion =

= 5.5th posillon
-
Middle posi!ion
Mcdian = Meu of 5th and 6th values
(c) 2,,1,6.7, E, Ij,9. 14

-
(i' ) Medicn =-

r",n..,n'",
@ ^.,,"
(t) Med 202 + 188 + 281+ 168 + 91 + lol
ll xl+12x2+13t3+ (i) Mean =
_ $(3:1+9:2+l0t I+ l,l x l)
1135
$1,16
= 70.9375
= $11.23 (oNct io nearest ent)
16+1
i3+l Middle position = ---
Middle positjon= 2 = 8.5th Posilion
= 7th Position
Medid = Mean of 81h and 9th values
Median = $11 10+71
=2
Mode = $rr = 70,5

750 Mode = 50 sDd 71


(c) Metu = 2,a

= 3125 years 282+4tA+231+443+271


Mem=
24+1
Middle losition - --- 1689

= l2.5tn Position = 112.6 Im


Mediu = M€an of l2th dd 13th vaiues
15+l
30+32 Middle posinon = ---
=2
= 8th Position
= 31 Years -

Mode = 121 mb
539
(d) Mean - r9 1.1 + 3.6 + 15 + 10.2 + 16.8
= 2$.4 m (3 sic. fic) (h) Mean =
l9
46.7

Middle FDsition=
19+1
--- =1,
= 2.46 c (3 sig. frgJ
= 10th position
19+1
Mediatr = 29 cm Middle losition = ---
= 10th Posilion
Mode - 26 o tud 29 cm

Medid = 23 g
43+ll0+218+lll+60
]J Mode = 2.3 g

=13 5x3+6:5+7x7+8
= 41.7 (3 sic, figJ
(i) Mem =
3+5+7+4+l
135
13+l X
Middle posilion = ---
= ?th position
20+l
Median = 43 Middle position = --
= 10.51h position
Mode = 45

Ansves wirh comprete wdked soruriom (4D


Median= Mean of 10th and llth valoes 25. (d) Sum of 8 ndnbe6 = 8 x

Sumof9nunbeB=9x35
=7 ke.
= 315
Mode = 7 kC
Nunber added = 315 - 2?2

C) Meai
10 :,1+11r6+ 12t3+13x2+14x6 (tr) Sum of 5 nunbeN= 5 x 9
- ,l+6+3+2+6
252

=t2h x45
l+2+l+4+5
l
2l+l 5
45
Middle posinor = --
= llrh position
Median = 12 h Sum ofg Nmbers =9 x 12
= loE
Mode = 11 h and 1'l h
Sumof 8nuftbeN=8 x t3
3x4+4x5+5t3+6;4+7x2 = l(t'1
(k) 4+5+3+4+2
85 Number renoved = 108 104

= 4,72 years (3 sic. fic)


18+l 291
MidJle @sirion =
-
= 9 5th Position = 48.5 kC

Mediar = M€an of 91h and 10th lalnes

= 48.5 + 1.5
2
=50k8

=7x50
=3509
0)
20x 3+ 2l x i + 22x 5 + 23 x 3 + U, 2
3+l+5+3+2 - 291
= 350
=59kC

= 22 CIrl Sun of 12 numbe6 = 12 x 21 5


= 258
14+l
Middle msition=
-
= 7.5tlr position = 12+ i6+ 18+20+23+24+33+ls
= 181
Medid = Mear of 7th dd 8th varues

Sum of 4 other numbe6


--2
= 258 l8l -

uarr-"tics ruto zo
@
Mear of 4 other rMbes Sun of 12 nmbes = 12 x 7.75
=93
.-.30+7I=93

(0 Sun of5 number= 5 x 2.3


= I1.5 (j) Sum of 15 numbes = 15 x:r'
Sutu of 17 nunbes = 17 x 4.5
Sun of I 5 nunbes {each in@sed by 30)
=15)+15x30
Sum of 22 numbeB = 11.5 + 76.5 15) + 450
=
=88

Mean of 22 numbers =
88
:: _ l5r +,150
15

,1(}' + 30)
=____
Let the scorc for fie fifth exam be x. =J+30
Mean > 85 (k) Sum of6 numbds= 6 x 28
19+27+35+r+r+r=168
70+92+86+89+r 8l +3i= 168
>85
3r= 87

x+337 > 425 0) Sumof3numbeG=3x24


J >E8 a+b+c=72

Minimun valDe ofr Sum of5 nunben= 5 )< 2',7


= 88 a+b+c+r+!=135
'12+'+!=135
.. The ninimum scoe she n@ds on the fifih exam
is 88 to rceive a disnncdon.
r+t=63
(h) Let the two numbes be r dd r + 3. Meanofrddl,=-63
Sun of9 numbeB = 9 x 16
= 144
26. (a) (i)
Su of7 number= 7 x 15
13 +8 +r+ 5 + 1l + ? + 17 + t0 + 6 + 2r
= 105

Sun of oth€r 2 ,urnbers - 144 105


,+(r+3)=39
7r =36 71 + 3x =95< |
r = 18 3r=18 ]

a+3 =21

.. The two numbers d€ lE rnd 21. Middle position

(i) Suln of5 numbers= 5 x 6 5.6.6.7. 8, 10. 11. 12,13,17 <-,-,1

Sum of? numbeB= 7


=30

xx
Medid

(in) Mode = 6
=
3+10
2

W
A$*es *i1h conprete worked sor!'toN (e
(b) (i) Let the missins lunber be r. 28. (a) Mem =8 (civen)
Middl€ losirion
7+12+r+2+9+]+16+3+8
l
\,2,2,5. x,7, /.9.9.9
, ="
Medim = 6.s (Gven) x+ t + 57 ='12
r+v =15

2 -- (b)
t+1=13

.'. The missing numbe! is 6.

(ii) Mode = 9

(iii) Meo
l+2+2+5+6+1+1+9+9+9
I
2,3, 6.7. 2,8,9.9. 12. 16
51
t0 Median = 7.75 (Given)

Middle position
z+8 =15.5
27. (a) 2,s,8,9,r
29. (a) (i) Mode = 3

Med (ii) t/=2+3 +7+9+8+ 1=30


2+5+8+9+r l0+l
5 Middle position - ---
llr 5
= 15.5tb Position

24 +.x = 4Q Median - Mem of l5tb md l6th values


r =16 3+3
=2
(b) ,<y (Given)
114, r15, 115, l16, r, 124. 126,126,r,l3O

Medim =
11121 1,26
0r2+1x3+2x7+3x9+4x8+5xl
=- 30
Sirce node = 126,.. )}= 126
81

Mediatr = 121 (Given)

2' (b) (i) ,.0"=u""0*-ffi


(ii) V=8+6+7+9+3+9-42
..r=rr8andr-126 M,d.lle Nsitron =
- posiion
- 2l.5th
Median = Mean of 2l sl and 22nd values
5+6

*"*"."0.
@ ^..ru
Med (i) (i)
3x8+4x6+5x?+6x9+?x3+8x9
--- 4, (ii) t/= 52+ 38 +42+ 30+ 18 = 180

230 130+l
Mid.Ue losilioD = 2

= 5,48 (3 sic. fic.) = 90.5th posilion

(c) (i) Mode = 8 Median = Mean of 90th dd 91st valu€s


30+35
(tt) 2f= 7 +6+5+4+2+1=25 2
25+l
Midde Position = -- .

= 131h Position
(iii) Med
Medid =9 28 x 52 + 30 x 38+ 35 x 42 + 38 x 30 + 42 r 18
r80
(ni) Meu 5962
8 x ? +9 x6 +10 x 5 + ll x 4 + t2 t 2 + 13 t
= 33.r (3 si8. ffc,
241
2a 30.
w
(d) (i) Mode = 39

(ii) x/= 12+ 13

Middle lositjon
+ 8 + 12+ 15 =6tr

=
60+l
---
= 30 5th Position
W
Largest possibie value
t

ofp = 15

(b)
Median = Med of 30th and 3 I st values
31+37
2 38 39 40

52 49 36

(iii) Mem
35 \ 12 +36 x 13+ 3? x 8 + 38 t 12 + 39 x 15

60
2225 Smallest po$ible value ofp = 53

= 37.1 (3 sic. fic.)

(e) (i) Mode = 30

(i1) >f =7 +16+12+8+2=4s ffiffiffi


ffi!ffi!ffis fiffii&&w
'15+l
Middle Position = -- 13'qasE!;acrr-issi !,?"6ffibs]etssfr*rl
= 23rd Position
5,..., 5, 10.... 10, 15,..., 15, 20...., 20. 25,...,25

21 29 32 t7p
(iii) Med 2l+29=31+1,7+p
25 x?+ 30 x 16+ 35 x 12+ 40x 8+ 45 x2 50=48+p
= --_ 1s
1,185

Smallst possible value ofp = 2


=33

Anses with Complete work* *n'o-, @


(d) 12 13 t4 substirute b = 8 into (1):
15
a+8 =22
tl

ffiW
6

(h) v =10
8+7+a+5+b-40
JJ a+b =20
12, ..., 12, 13, ..., 13, i4...., 14,15,:-., 15

Pll 86
5x8+6!7+7x4+8x5+9xb
P=10+8+6
'ld+9b+ 122
4r =t^
Iagest possible vatue of, = 24 7a+9b+122=zaO
70+9b=158
Men =24
r0 x 26 + 20 xp+ 30 x 32 +,10 x 17 a+b =20 _(1)
26+?+32+11 7a+9b=158 _ t2)
20p + 1900 11) x 7: 7o + 7b= 144
^_
(2) (3): 2b = la
20p+19ffi=24(p+75) b =9 -Q)
20p + 19ff) -Up + laOO
100 = 42 sub8rituler=9inlo(l):
d+9=20
a= ll
(t Mean - 9.5
I x 16 +9 x l0 + l0 xp+rl x 7+ 12 x 5
16+10+p+7+5
4 =7O
18+a+16+b+ l=70
a+b=29
l0l,+355=9.5(p+38)
ljp +355 =9-5p+361
2x18+3xa+4x16+5

m = t:t
3a + 5b+ 142 =245
*=30 3a+5b = 1,03
u+5+r+3=30
d+b=22 a+b=29
3a + 5D= 103
Mean = 2 (l) x 3: 3a + 3r= 87 _ (3)
lxa+2x5+3x6+4r3 (2) - (3): 2b = 16
30 b =8

Substirute, = 8 into (l)l


a+3b+22=60 a+8-29
a+3b=34 a=2\
a+b=22_(t)
a+3r=38_(2)
(2)-\1):2b=t6
b =8

r*o*"0*
@ ^,.r"
U] x/ = 100 Median = Med of 15th and l6th vaiues
15+10+d+/,+ 19 = 100 2+l
a+b =56

Med
0r 15 +1 x l0+2xa+3xr+4x 19 (iii) Mode = 3
100
=2.3
2,+3r+86 ^" The mead giles rhe best plcturc of fte disrrlbutior
r00 = -' since i$ calcdallor lnlolves all fte dala dd therc
2d+3b+86=230 arc no exlreme valucs in the drla.
2d+3b=144
Middlc position
4 +, = s6 _(1)
2d+ 3b = 114 (.2) l1 (r) 5, 36, 37, 39, 40, 40, 42, 45. 45, 46, 46, 48, 48, 48, 65
(1) 2: 2d + 2b = ll2
x {.3)
(2) (3): b =32 - 630

-
(1):
substitute, = 32 inro = 42h
d+32=56
d =24 15+l
(ii) Middle oosinon = -_:-

= 8th Positbn

31. (a) Tolal number of vending machines Median = 45 b


= x 29 +2 x 22 +3 23+4
\ t x l8+5 x 8
(iii) Mode = 4E h

Mean no. of vending machincs per school (b) Themedian givei the best alerage since the data
254 have two extreme laldes. 5 dd 65.

= 2.51 34. (a) (i) Mean


0,35+l r64+2 x68+3\ lE+4x15
Modal no. of lelding nachines per school = r 200
314
100+l 200
Middle posinon = ---
= 50.5th posnion
(ii) Mode=2books
Median do.of verding machines per school
= Mean of 50th and 5lsL values 200 + I
2+2 (iii) Middle posilio. = 2
2 = l00.5th posilion

Median = Mean of l00th dd Lolst lalues


32. (a) (i) Mean 2+2
0x3+l!5+ 2tl+3x9+4x1+s\2
= 2 bmks
'12

30 (b) Total rc. of books bought by 200 studerts = l14


Total no. of books bought by 100 studenls (lion

30+l =l00xr
(ii) Middle position = 2 - 100r
= l5.5th posinon
Total no. of books boughr by 300 studenls
= 314 + l0or

Answes wirh comprele worked sor'tr.".


6r)
Nlei .o oi bn,'r.\ houehr b) 100 \ruilenL. = l.7E FacLorv a
31.1+ 100r ro]\
--{xr = r 78 NLeoLJn =5 Medrd = tjth vatue

3l'1+ l0(L = 53+ (ii) FactoN n


l00r = 220 Modal no. ol Loys = :l toys
, = 2.2.
fict(w A
3s. (!) (i) Nledn \'lodal no. of loys = 6 tols
0x I+| r6+1 !5+1x.1+.1x 2+5 !6
The mode is best uscd otcohpare fte so*es of
both laclodes sincc tnen medirns are equal !trd
Lheir eds cinnot be calculated.

Mean scorc (School,4)


rii) Modc = 1 and 5 _ h+86+91+85+93
5
2.1+l .132
liiil Middle Dosltion =
-
= l2 5ih Positnnr = 86..+

Mcdia. = Nle.n of 12tb and lllh laloes Ilean scoie (SdroqL9)


2+3 rJ.1+ u2 + ?6 + 91 +33

= 2.5 431

(b) Irt Lhe no. othe.ds oblained nr each ofthe cxL,!


= E6.2
3 rhn)s( be,. , .nd ..
Mcan score (ldlorc)
Nlean of27 throqs = 3
85+95+90+75+92
66+d+r+. .
t ='
n6+!+r+.=Nl The mNnun
= 81..1
..,r=5.1=rJnd! =5 eJrb tiLox i.:
lf lhe schools re ranked by ficj. medn scores.
.. The no. ol heads obtaircd in exch of lhc .l School C {,ould win the Marhematics olynpird
throws dc 5, 5 aDd 5. since it hrs lhe highesl mean scorE,

Middie positlon

(b) School A: 75. 85, 86,91,9:l


Mcdian s.o.e = E6

Middlc Po\itnnr

SchooLS: 76- 8.1- 88. 91, !2


(bl (i) E&lo+l Medirn scorc = 8li
2i+l
Mlddlc posilion= r Middle posior
= rr; posilion
S.hooi C:15. 85.90.92,95
Median = 5 (ors Median score = 90.

lr .h- .Ll"'1 e Jll,c'q) . med.. .nrc


'
School C would \rl! thc Mathedrtics Oly'npiad

r",n..uo, t*., .u
@
39. (a)

I would choose rhe m€die soE to rank the


schools.B) conpdjng the medid scores, School
t would be ranked secod. If tbe mean score n . Lr+est posible value.'fr = 7
used insrsd, School B would tE.mked third.

38. (a) Class


Middle position

24 : 1, 3. 3.
Clas 28: 1. 1, 1.2.4
Clas 2C: 1.2,2. 1.6
Clas 2D: 1.3,4,4.5
Clas 2t: 1.2,3, 5.5
4.7

0. ...
682x
0, t,
ffi
._.. t, 2,2, 3. ....3

6+7=2+t
1,3 =2+r

. . Ldgesl possible valuo ofr = rr


(c)
0x6+lx8+2x2+3xr
Class t
has the mos! absenlees per month if
their modes a.e used to conpre the clasles.

(ii) Class, har the mosr absentees per month if


tbelr medians ae used io conparc the classes.

:10. (a)

Requjrcd jnequality:r < 29


Alte.native answer: r < 28

(ii) Ldgest posible value of t = 28

Ra*:Class 2,4, Cl6s 2D, Class 2E Clas 2C, o t 2:1 + | x 13 + 2 x r'7 + 3 i 29 +4 : 26 + 5 x 28


Chss 2A ?1+ t3 + 11 +29 +26+28

=2.7h

Answ€N wi,n conprete worrr.d sorurio^


6;)
110+l (c) Mean = t.72
Middle position = -- 0x6+1x7+2x5+3r2+4xf, l.'12
= 70.5th posinon

M€dian = Mean of 70th aDd 71st valu€s .+20 - ''


3+3
4x +23 =1.12(x+20)
=3h 4r+23 = l.7Z\ + 34.4
2.2k = 11.4

42. (a) Mean = 1.7


0x3+1x5+2x8+3xj+4xl 1.7
3+5+8+r+1
0, ....0, 1, ..., 1, a ...,2, 3, ...,3.4.....4.5, ....5,
27 13 1729 26 x
3r+25=1.7(r+1?)
+25 = 1.1x + 28.9
O n+13+17+29+25=x 3x
1.fr
a=lll = 3.9

.. Ldgest posible value ofr = lll.


lii) 21 + 1,3 + 17 + 29 = 25 + t
86=25+r

Smallest possjble value ofr = 61.

41. (a) Ldgest possible value of r =6 (c)

0.
WW
...,0. r,..., l. 2,...,2,
6',]52
3,3.
0.0,0,

35811
1. _.., 1, 2, ...,2, 3, .._,3, 4,

3+5+7=r+1
6+7+4=2+x x- 14
.. Ldgest possible value of r = 14.
.'. Ldgest possible value ofr = 15
43. (a) Tolal no. of studenls =20
6+7=4+2+x 1+3+r+6+4+3+0=20
l7+t=20
.. smalest possible value of r = 7
Mear no. ol pens per studerl
Requiied inequality:7 <.' <15 0xl+ix3+2x3+3x6+4x4+5x3
2A
58
20

(b) r <5 jf the mode is 3.

@ "u,n.o",t*-ru
'ffi 0,1,1.1.2, ...,2,3, ...,3,4, ..., 4. 5, 5,5. > 5
(c)

(2)
r+t=45
r+4)=135
(1): 3] = 90
)= 30
-(1)
-(2)

13x6430 Substitule) = 30 into (l):


x+30=45
1+3+r=5+4+3+0

.. t{gest possible value

44. (a) Smalest possible value of 2J =


. Smallest possible value
ofr

ofr
=

=5
E

10 '''m+Fm
(b) Man = 2.5
+2x9+3x2r+4x2+5t1 100+1
6+2+9+2t+2+7
= rn 5ri n^(irlnn

Median = Mem of 50th and 51st values


6x + 63 = 2-5(zx + 26, 2+3
6i+63=5r+65

(it Modal no. of guests = 2

46. (a) Total no. of studenrs = 100


27+r+25+y=lU)
0,....o. r,r, 2, ...,2,3. ...,3, 44 5....,5 r +J = 48 (Sho$,n)
6292a27 (b) Mean
6+2+a=2x+2+1 2\21 +3xt+4x25+5xr
100
3r+5y+15,1 --
l0o = t'
.'. Ldgest possible value of x = 35
3r+5.t'+154=350
45. (a) Total no. of glests 100
3i+sJ = 196 (Shorvn)
=
j+35+20+)=100 () r+ J =48
'+f=45 3r+5J=196
(b) (l)x3: 3r+3)-144 -(l)
Mm = 2.65 . (2) (3): 2t =52 -(2)
t r+2.35+I.20+4\y ^-- ! =26 -(3)
r + 4) + 1.10 ..r Suhritule ) = 26 into (1):
r00 r+26 =48
x+4]+130=2.65(100)
r+4r+ 130 = 265
J + 4J = 135 (Sbown)

Answss witb comprete wo.k* *ro-. (.i)


2 5
l:6 + 2:r + I x2+ 4:3 +5 ! ! + 6: 1

21 22 25 16 15

l,+tr+ 1ll --,


-15='*
Modrl no. of novcls rurchased = 2
2t+51+4lj=86
1r+5) =38
.17 (a) T{)l.l no. of (udcnLs = 40 1ry-" I pro. pgn
ar+r+9+8+)+5=40 }t+5)=lli (i)
r+J=12 (l) x 2: fr+2t=20 - []
(2) f3): 3r = tti -
Mcan = 7.2 )=6
5.ii+o^!+7 ,+\'8+'r')r+ln 5 --
Subsrirure] = 6 inlo (l):
6r+9-r+207 -- r+6=10
- 4r = t'
n\.r, Flu 188
cd.h 'r-l =ll
D,viLre
rem bY l

r+r =12 (1)


\+3i=27
(!) x 2: 2r+2)=2'1 -
12) (3): |=r -{.2)
-(3)
snbsLiLure )'= I into (l ):
r+l=12
-!=9
..'=9andt=3
6 7 9

9 l 5

Modal mark = 6 and 7

Middlc lroenon - 2-
= 20 5th tositron

Median nrrk = Mexo of 20th and 2lst


ldues

=7

18. (d) Tolal no. of thrcss = 25


6+r+?+3+)+;1=25
r+r + 15 =25
x +J = l0 (sholvn) 30+l
(ii) Middle Postuon- 2-
= l5.5dr posixon
nrinimum laluc oit = 7
.. m.xinunr value of, = l0 7" Itedian = Mean.l l5tb rnd 16th lalue\

= 3 sodls

r,,n-,,".,*. ,u
@
(ii) Mean (ili) Mcan
0x3+lx5+2x6+3i8+4t7+5! | lx2+2x8+3x5+4rll+i/9
30
126

= 2.47 coals (3 sic. fis) = 3.5 Coals

50. (b) Angle of sector rcpesenting 2 goals scored pel

=-x360"
= 80'

(b) (i) Mode = 3letters 53.

(iD r/= 31
ll+l
Middle P\iLion = :
= 16th Posilion

Medid = 4 letters

1,18
(iii) Mean - x
= 4.77 l€tt€rs (3 siC. IiCJ

51. (a)
=3+4+6+2+4

Modal no. of w ches per student = l (iiJ Middle Dosition


24+1
=
19+l = 12.5ft position
-
Middle tosnion = 2

= l0lh posinon
Medlan = Mean of 12Ih dd 13th values
1+J
Median no. of watches pd student - 3
=7
Mean no, of waLches per studonl
lxl+2\4+3:6+4!2+5x4 (iii) Mear
=--1, 4 ! 5 +5 x 2 + 6 x4+ 7x6 + 3!.1+ 9r2 + l{r : I
24

t9 156

= 6.5
52. (a) (i) Mode =l soals
36+l 54. (a) Mode = s6 years

= 1E 5th Position (b) Middle position=


-:
Medio = Mean of 18th ard l9th values = lltb posihor

=2 Median = 55 yearc

ADsReB with comprete worled sor!1io*


@
7, + 235 + 180 + 123 + l,1o .i ..Since lbc mean ma r! ofthc gids is hisher than the
2l mexn ndk of the Lrors, the girls pertbrncd better in

'"iT*"*llll-
(d) (i) Highest mdk = 53
= 55.5 yeaN (3 sis. fic.)

55. (a) (ii)


.10+1
5 2. (iii) Middle posidor = --
6 24
1 l2 5 1 = 20.5th positron
5 6
Nledian ma.k = Mcar of20th and 2lstrdlrcs
9 z4 5 7
38+39
l0 =2
= 3E.s
(b) (i)
20+l D. iUea! ibr Grouoed Data
(i) Middle position= -l
= l0.5th positton 57. (r)

Medirn = Mern of ioth and l lLh v.lttes


ee.,l*;lMd*.r* * 'l r*q"*"v v
'l 7,

= tr.i* 8.: 21

l0 t2 11 5 55
52 + 126 + 371 + 606 + 378 + 100
(ili) Mean = 2A 13 t5 l4 l0 140

l6l5 16 18 t1 102
=ro
= 81.75 cm
19 21 20 80

22 24 2l 2
(i) No. ol boys in Clas z4
,ft=47
=l+9+1+8+l
hr
:n
(ii) No. ofeirls in Ch$ 2A
Meao age = t/
=l+3+6+8+2
= ztl
= 14.9 yeals
The narks of the bols dre clusteied dound 20t
and .10s. The marks of 1he Sirls are wcll spread 58. (a)

C)
14+213+:ll+356+53
=----
669 110 5.5.
=n lt 20 15.5 :16.5
= 13..15
21 3t) 255 8 20.1

13 + 6r + 201+ 36:l + 102


ll :10 35.5 I2 126

= -- tIJ ,ll 50
,109.5

750
51 211.4
'n 60 55.5

I 65.5
65.5

tr4 = 1440
'f=40

@ t,.".".* tn...u
61. (a)
abl Mee leneLh =
=
14,14

=36mm 30<r<35
59. 35<t<40
40<t<45

80-84 82 8 656
45<r<50

85-89 87 12 1044
50<r<55
90-94 9Z 2l 1932 55<rs60
95 99 28 2',716

100 104 1,02 t6 t632


{b)
105 109 lo7 10 1070
215
ll0 - ll4 112 5 560

V= lfa = 45.5 s
100 = 9610

t6 Modalclass=40<r<45
lb, Mean mas! =
=; a. 6)
9610
tm
= 96.1 c

60. (a) 25<m<35 30 ll 330

35<n<45 40 7 280

45 <mS 55 50 3 150

0<f<10 55<n<65 60 480

10<{<20 65 <d<15 70 6 420

20<{s30 75<n<45 80 5 400

30<t<40 tJ=40 zj, - 2o6t)


40<{<50
50<4<60 (b)
:.f
Medlras!=:r
2060

tn = 51.5 g
(b) t{e4 ulespan =v
(c) Modal clds = 25 < ,1 < 35
2370
60
4n+1
= 39.5 days (d) Middle losition = 2

(c) Modar lifespd = 40 < l<50 < l = 20.5th Position

ffi I
Medim = Mean of mth and 21st values

. . The class intenal where th€ median


45<D<55.

Ans*e qith conprete worked s.1"1t"^


@5)
63. (r)
Fre1::iq] Nlid-vilue
(" $t (') U) fT
0<r<20 i0 5 50 0<r<20 l0 2 20

20<r<40 30 l l0 1 t10
.10<r<50 .15 I lIi0 50 tl 600

50<r<80 65 6 190 l9 l3l0


3 270 80<r<100 90 450

:/- lo :/r = 1250 tli =26t4


Nlean mo.rhly savi!.ss
Mcan anourl of trev collecred =
lli =
tl $2610
$1150
=itl = $slt
- $41.67 (correst to nearest c€nt)
No. of sconls who collcdcd > 560
N{odalclass=20<r<:10 =19+5

30+l
Middle posiLion = .
= ts.iLh posirion
=- lt)l),i
Mcdiaf = Mcan ol lirh and l6th val'es

.. Tle class interv{l which conlains the medianis


20<!<40.
(d) Middle posnion =
(a)
- posnion
= 23rd

.. Thc clars inte,lrlwhlch conl.irs the medianls


60<s<80.

(r)
3E<l<42
42<h<16
Taly Mitlal!e

9t |l0 tl 100.5 2 201


50 <,t < 58
in,t20 115.5 1 Iiou.5

12r Ll0 t25.5 6 753

t/i 131 1.10 [55 5 617.5


(b) Nlear height =;
l:tl 150 6 873
5054
- lro t5l 160 155.5 I r55 5
= ,12.1 cn (3 sic. ficJ
110 165.5 l
(c) Modal chss =.18 < l, <42 :/=30 :li = 1965

r",*.,,'.,,*-,u
@
t/l Mcdia. = Mean oi 25th dnd 26th values
/D Mc.m hcighr = -t The class inteNal sherc thc mcdiln lies i\
1965 40<nS60.
= 132 cn (3 sig. fig.)

(c) No. ol children whosc hcigtts < l50 cn


=10 4 av,
=26 0<n<20 t0 2 2t)

Required p.rccnlage 20<,r<40 l0 ll 330

=;xr00% 50 1.1 700

65 9 5ri5
= a6: q.
70 < r': 100 it5 l:1 90

:/- 50 :/a = 2E:5

l,englh Tally fxequency


(x) f) (b)

l5 t9 l7 l 5l 21125

20 21 22

25 29 )1 21
Modal clas = 30 - 34
30 3.1 32 256

15 l9 37 148
Mlddle posiLion = --;-
:10 4.1 ,11 7 294
= 20.51h position
,15 '19
3 l.1l
Medl.n = I4can ol 20th and 21sI valL'es
:/t = r 005
Th.class inLe,ul$hee the median liesis 35-39
th) N,lodrl clas = 30 - 14

(c) t/i
Mem lengLh = :/ Mid-relue Freqtreml
l00i (r\
2a -29 21
= 13.5 cm
30 3,1 32 t2 38:l
No. o{lcaves shich are > 35 cm
35 39 31 7 259

12 5 2t0
8 376

50 5.1 52 6 312
= + x looc,
tf = 1t)
.2.
=40i /r

68. (a) Modal class = 40 < m < 60 lnd 70 < t < 100 l5L)5
= .10

i0+l = 19.875 e
Middle losilio! = .
- 25.5th positior

\n$eb qlrlr Complete \\brhed Sorml.",


@
12. l^) Modal .las = 60 - 64

- ;;*
o<t<2 9
{kg) UI
2<t<1 3 54
55 59 57 1026

5 l2 60
60 64 62 tt 1.188

6<r<8 1 15 i05
65 69 6,7 t9 1273

8<r<12 6
72 13

Y=60 l = 2li8 l5 -19 1',l 385

:A E0 E,1 lJl 82
Mean = !f :/= 80 tli = 5190
28E
= 6t) Mean na$ = :f
:/J

=4.8h
5190

71. (d) Modal clas: l5s</tS160


= 64.875 kg

{x) 13.
'112.5
l:10 <, < l:15 142.5 5

t4?.5 16 2160

1270
200</<2,10 l
150<n<155 152.5 28

l0 240<d<?80 5
155<'<1C0 157.5

.1062 5 280<d<120 7
t62.5 25

2680 120<l<i60 l2
r67.5

:/\ = l! 810
.l
:/\
(b) lvloddl cla$ = 120 <d <360
l8 810
= rro
-Ti"' VJ
120+1
Middle position = ; 100<d!240 220 l 660

= 60.5rh po$tion 240<d<2110 260 5 Lt00

<./ < 320 l0L) 1 2100


Nledian = Mean ol60tl and 61st rxlues 21J0

t), .1080
320<l<360 340
Cla\s intenal whefe median lics = 155 < rt < 160
360<l<400 llJo 3:120
Angleofscctor=; x 360'
420 1680
= 90"
:/= 40 tF = 13 2,10
:/i
Mean = :/
rl2.10

= 3ll krn

@ r'*'"*.r.-,u
a+2b= 8.t
c b=tr.r (2)

L the numbeN
The sum of the is 3 x 28 = 84
(l)+(2): a+b+c=t9d -(r)
-
..lN+^ts+NN=84 a+ b +c+d =1,9d+ d
dd eror, N =
By trial 3
13+38+33=84

.. The mean of a,, c dd dis tt.


Since the mean is 2, the sum of the 3 digits is
M@n of 3a 3, and 3.
2t3=6.
List the 3-digit numben wherc the sum of the digils is
3

At:-l::l
-z r05, l14,123. 132, 141, 150 (6 nos. in 100 s)
204, 2t3, 222, 231, 240 (5 nos. in 200 s)
= 3i6 303.312,321,330 (4 nos. in 300 s)

Ir! r rcpresent! the med aier 5 tests. The sum of the


From the patrem, we can conclude thal there are
.-.
The mean afler 6 tests: 3 nos. in 400 s.

I no. in 600 s.
6(x 3)= 5r + 12 ..tor.]=6+5+4+3+2+l
6x 18=5t+12
r=90
8. Since the adjusment was done to each students mdks.
.. His nean now is 90 marts- the etrect on the mem will be exacdy the same.
If a student emed p mdks, he mi$ed ( 100 -p) ndks
Lf,r r. r | ).' + 4 dd r + 6 repR.enr the'odr /t00-r\ mdks.
consecuhve even f teSers. After lhe adjustment, he will miss
| , J
r + (r+ 2) + (., + 4) + (r+ 6r ^^ .. The new grade aier adjustm€nt

4t+12=116 -
' 4t = |U
= 100 [---J
x-26
r = 100 50+2
.. The laigest of ihe four consecutive even numbers
26+6-32.
=,U+)
Le_ rhe three cor\eculi\e muhiple. ot,1 be 4r.
4(, + 1) and 4(r + 2). -. The ftem mark after the adjustnent is 50 + ;.
4t+1('+I)+41'+2)
9.
1U + 12 =96 lighlest nedBn
l2x = 84 ll l, 68,68, L
50. l, 62.
.-. The smllesi of the tblee nulLiples is 4J = 4 x 7 = 28.
Let the nds of the heaviest boy be t ke.

5,nc $e _ean N s,ven as 6l \8. rhe muss of'he


heaviest boy will be greatesi when the two lnknown
ms.\e. be o{ roe n edirn hr\e rhe .mallr'r po.,ib -

Answers wilh conprete worked sor'rr..s (e


fhe rqo unlno$n. u\. oe unequal . ,.e,te ode '
68. .. r= 8u
Hence, these nasses must be 51 kg and 52 ke. Middle posjtion
Given thar lhe mean is 61 kg.
50+51 + 52 + 62 +68 + 68+r p. 81, 81. 82, 4.'/, 88, 88, 89, 93

Given that the nedian is 85.


x+351 = 421
r ='76 ''. !lt2 a<

.. The grealesl possiblc mass of the heaviest boy is 4+88 a<


76 kg.
q+88=170
q =42
r (Given)
10. (a) The nedlar of the inteeers is Given thai the ned is 84,
r 3,r 2.r l,r.x+l,r+2.r+3 . . The sum of the l0 nos. = l0 x 84
= 840
1

Median
p+El -81 a' 82+88+88{88 8(:) o1- 8.10
r" ! 7r2 = 840
r :l+r 2+ r - I+r +] +I+ r +2+ r+ 3 P=68
..p=6E!q=82and/=88
1 13. since\2r- I N!re,lP |f)'.,.ndr]', rllmrJbF
oD differcnt sides of 8 as 8 is the median.
Mediar
(b) O-3t.(r 21, (r 1)1.1.k+ l):, (r + 2)'. (J + 3)'

r,4,6, @. e, 15,2r + r
..r<8and2.r+I>8
- (' n: +(r 2)' +c-r)' +"+(,+r);.(r+21 +('+rt L\>1
'r + + l zr+ l+;+;+2!+1
+q+ r>3-
,lJ ,1 I
= (.1 - o'
+l+4a+4+r:+6j+9)+7
I
..3t<r<tl
Slncer is an integer. r= 4.5,6 of 7.
7(.1 + a)
- r = ,1 and x = 6 are rcjected sincc all 7 integers .e
rI=5,2r+l=11
Reqtrned difTererce = Mean - Median Ifr=7,2r+ I = 15 (rcjected)
=l+a-|
14. (a) Given that the sum ol the scofes of nre
ll. Mean age 20 sludenrs is 1669.
l0i /r12 lr{lo+"r b , 2 1669 = (80 +r) + (90+)) + 1494 < t
=-rl '
, iitddii.'ii
r _ robo r4{4 _ qo "o io6"l i6ls^
=10
Middle losition rte posible values of r arcAoi 6.
'rhe possible values of) are\0/, 1 or 2.
10. 12, 12, 14,16, t6,16.a,21.21
By trial md enor, x = 5 andi = 0.
t6+16
Medianase=-=16
(b) ToLal sco€s ibr School,l = 834
- l4l+d -.
ro--l-=ul Mern score for sshool A= ff = i:.+
l4l+d_1<o Total scores for School a= 835

l,1l +a = 159 Mea! score for school d = T =83.5


a=la .. School A was the winner.

t*n*..* r** r"


@
(0 P(a multiple of 3)

999
L An imlosible event
looo (g) P(a no. > 6) -
=0
t
A very likely eveDt 0
(h) P(neiher a head nor a lail) = -
An unlikely elent
looo

1
!ll
(a) s=U,2,3,4,s,6l
=6
'(s)
(b) s = {12, 13, 14, ls, 16, 17, 18, r9l
r(s) = I 2

s = {R,, R, Xj, G,, G:, G3, G4l


n(s) = 7 I
=2
(d) s = lA, B,c,D, E, F,G, Hl
=8
'(s) P(win buy a chicken) = ;
3. (a) P(wiU not buy a chicken)
No. of flvourable outcon€s = I - P(will buy a chicken)
Tot€l .o. of possibLe outcones l
3
5
I
0) P(will a@ept the coin) = 0.8
P(will rot acaepl fie coin)
I
ab) P(rftdball) = - = I - P(will ecept the coin)
=-
2
= 0,2

{1,2,3,4.5,6}
(c) No. of men - 200 40 = 160

^ 160 (a) P(az) = tI


vra mdr= 7n0

1
(b) P(d odd no.)=
;
3
(d) P(an odd no.) =

I
2
1
6

I
2

=l

Answm with conprete wortt *to-. @


5
3 (ii) P(last disit is odd) = Jt
(d) P(a multille of 2)= ;
=1
I =t 1

9
4 (iii) P(last disn is not 8) = id
(e) P(a factor of 6) = e
2
'1. J = {1,2, 3.4, 5,6. 7, 8,9, 10 11' 121

.t
(a) P(a e) = 12

(0 P(a naturar rc') = 6' 6


O) P(ai even no ) = t
=tI
5
P(a pdne no.)= 12

(d)

5. (a) P(a 3) =?
1
3 =t
(b) P(an evei !o.) = ;
I
=2

2
(c) P(ano.>a)=;
I
(e) P(a lerfect square)
3
2
(d) P(eitle.2 or 6) =6
I
1

0
2 (0 P(a 3-digii no.) = l'
(e) P(a perfect square)= ; =0
I
(6i P(m 8) = li
1
(b) P(a no. > 5) = i0

6. (a) s= {0, 1, 2. 3, 4, 5, 6, 7, 8, 9} (c) P(an odd no)= lo


3
4
(b) (i) P(l4t disit < a) = 16
2

(+D u",l".u,i* ru". za


(c) \o ot.ilvrr dd brcoa medal. = 8 - 6 = 14
{d) P(a no. < 3) - lo
r(s,vn or Dmnze,=
li
7
.10
f'apost|velnRger)=li
=l
ld, Pleold- elver or broDe) = -
e. (a) P(leuer d) = i
12. Total no. of sneete= 10 + 8 +6= 24
1
(b) P(lener P) {a) P{a vellow swet)
= ;i = -
I
(c)

(b) P(no! a yellow 3weet)


(d)
= I P(a yenow sw@t)
=l P(not a consonant) I

{cr Plnot a colour of tle rainbow) =-


10. (a) P(letterP)=; I
t

2
(b) P(letter,4) = ;
1
f (d) P(a red or yelow sw@t)

24
P0ettert)=; 2
=o 3

24
(dl P(letterA.Po.L'=: (e) P(a red, while or yeltow sw@o = ,;

ll, Total no. of medals =4 + 8 + 6 = 18

2
6
1.1. (a) P(a vellow meblel=
-
(b) P(oot sold) I
=I - P(cold)
2

Answers with comprere worked solurtds


@
(b)
1 P(a sreen mble) (b) P(ar least on€ tail) =
=

=1,-: q
(c) P(al most one tail) =
2l
30
7
=ro 16- S = {HHH, HHT, HTH. THH, HTT, THT, TTH. TTT]

(a) P(3 headt


14. (a) P(it will not rain lonomw)
= t
= I P(it will min tomorow)
=l-20% (b) P(2 heads ald I tail)
= a4%

(b)
(r) P(ct least 2 beadt
=-x49
8

No, of mbl6s lhar ee nol green 2


= 49 -35
17. (a) No. of girls = 50 -35 = 15

P(late) = I - P(not lare) PrA oirlr = :


-50
=I tP(edly) + P(on time)l
3
=l (0.3 + 0.52)
=10
=I 0.82
= 0.lE
2a
(b) P(wem
(d) No. of faulty switches
stectacles) = :;
= 0_05 x 200 14

(e) No. of nicrochips which m not faulty I 8. No. of ddk chocolates = 30 -9- 15 = 6
=100-8 (al P(a nrx chocoiater - ];
3
92 t0
P(notfauln,)=:-
(b) P(nor a milk chocolat€)
= 1 P(a nilk chocholate)
3
(0 P(more thm 2 workins days) =l- lo
= I - P(wilhin 2 workilg days) 1
'1

2 9+6
(c) P(either a nilk or daik chocolate) = 1n
'15
15. s = lHH, HT, TH, TTI
(a) 7
P(exac y oDe tail)=
=t

*",n".",", ,u
@ ^..
(d) P(either a milk nint or ddk chocolate) =;
I
2

19. (a) P(a 5) = 1


8 1
2t. (a) P(a 3) = t
(b) P(a e) =
I
0
8
(b) P(a losilive inteser) =
:
9

4
(c) P(tu odd numbet = (c) P(a whole no.)=
R
t
I
=t 2

td) Pla2orl)=:
1

(e) P(a mulliple of 3) =


;
(d) P(a 2 or 3) =t
I I

8
(0 P(a factor of 35) =
;
(e) P(a no. < E)
9

(P) P(a nrine no.) = : 22. (a) P(Mtrc winwin)= 5;


I
{b) P(winning ticke! > 325)
175
=5oo'
l]') 1
20
(i)
I
No, tbe probabilit) that a girl will win the lnze
cannot be detemined sirce we do not know the
20. (a) I
8
number of students in the school thal do girls.

23. G) s = la, b,c, d, e,f, s,


-, zl
P(a multipte of a) =
; (b) E=ls,bn,d,a,rj
n(E)
(c)

(d) P(a io. > E) - s


3

I
2

AaweF with complere work* *t, -, (o*)


P(t) = I P(E) (a) s = {r03, 10s, 130' 150, 135, 153' 301' 30s, 310,
3 J50, Jl5, 35t, 50t, 501, 510, 5J0, 5tJ, 5Jl;
=1-=
10
(t) c = [130, rs0,3r0,350, sr0, s30]
l3

u. (a) s = {Janu{y, February, March, April, Ma&


June, July, August, September, October, 6
Noeembe., Decemberl

(b) A = {Ma.ch, May}


3

P(C)=1-P(C)
2 1
=a
I 2

(c) a - {January, Apdl, Jmq Jury, Angust} (d) D = [3s0,3s1, s0r,503, sro, s30,513, 531]
P(A)= -:;-
5
12 8

(d) C = {Janua.y, F€b.uary, March, April,


September, October, Novmber, De(:mberl

!s) P(D') = 1 P(D)


",^
8
12 5
t
3
n. s= {1, 2, 3. 4, 5. 6,.... 50}
25. (a) s= {r,2,3,4 5,6,..,,30} ,(s) = 50

r' = {s, r0, rs,20,25,30} 25


(a) P(d even no.)=
-
I
(c)
2

(b)
I 50

32
(c)
P(r) = I - P(t:)
I

(d)

@ r**'",",.., *
(e) P(rct a perfect cubo)
=I- P(a perfect cube) 8
100
2

=so
(c) P(a prime no. < 30)

= roo

s = {30. 31, 32, 33, 34, .... 43J (d) P(a mulriple of ll > 100) - l6d
,(J)-41-30+l =0
s= ll2, 13. 14,2t, 23, 24, 3) . 32. 34, 41. 42, 43j
n(.5) - t2
14

l2 io.
P(a < 20) = t
I
3
(b) P(a pnme no.) = ra
6
(c) P(Do€ rha. 33) = 1 P(< 33) P(a no. > 30) = 12
4
,- 14
2
l0
P(a mdlliple of 12)
5 2
1 =n
I
(d) P(a no. that is divisible by 8)
2

I 5

(e) P(not a pnme no.)


(el fla penect cuDer =i; I P(a ?rime no.)
=
=U 5
=1- =
s= {3,4, 5, 6. 7,.... 102}
tr(J)=102-3+l
= 100

50 s=
la) P(an even rumben =
31. {10, 11, 12, 13,14....,991
- n(S) = 99 10+1=90

2 (n) P(a perfect squde)


6
=ro
I

Ansvs wirb conprete wuk€d s.rtrrtds (e


(b) P(a no > 80)
l9
90
13

8
52
{c) P(a no. having a disit which is 2)
2
l8
=ro

I
(d) P(a nultiple of ll)
90

10
I

r(s) = 52 P(not a queen) = I- P(a qu€en)

t2

(f) P(a picrw cdd or a dimond)


(b) P(d ace ofclubt - s,
22
l3 52
P(a spade) =- ll

(d) .11
34. n(S)= s2 - 12
P(a picture cdd)= 52 =40
3
t3 ?l
l2
t0
(b)
(e) P(not a picture caJd) = I - P(a licluie .dd)
3 L

l0
P(not a heart) = 1 P(a heart)

33. ,(s) = 52
3
ral Praftd ced) = - =7
i
=t

uatrrematics nrtor ze
@
(c) P(a 5) = {d) P(an 8)
- = ;
1 I
=ro

P(Dotas)=l-P(a5) ' (e) P(a 4 or 8) =D


I
9
t0
(i) .0
35. n(t - 54
=0
(a) P(a red king) =;
37. (a)
=t P(inside blue sector) =

60'
= 360.
{b) Plablack ctud) = - 1

t3
(b) P{odside blue sector) =I - P(itrside blue seclor)

(c) P(a jack or a aueenJ = -


5
4

Area of oMce setur


ronsroe ordse s4tor) =
ldl P{r iokerr = :
90"
_ln 360'

(e) P(not ajoket = 1- P(ajoker)


(d) An8le representing red sector

26
-
180" 6ilP (adj. zls on a sh. line)
=E
36. r(D = 12

(a) Pla hean) = - 120.


360'
I
3

(b) Pla.lub) = (e) P(inside rcd or yellow setor)


- Afta of red ed yellow sctos
1
_
- Area of ciicle
120" + 45'
360"
3
(c) P(a picture card) =
n 165"

i ll

Arcwds with comprete work"u .t*"^ 6)


38. (a) P(irside smaller cncb) (iii) P(ldse sizc)
Area of sfr.ller cn.b AEa of sector repesentine hrge size
= A,* t,,c...*t" Afta ol cncb
"f
= i6rr.
35' + l0'
r5,f 360"

40. Arca of shaded region


= A*" lfger circle
"it"rg.""l*t" = Area of AJea of snallef ci.cle
90'
= 36rr

P(lnside shaded region)


Area of shaded region
P(inside shaded region)
=1 P(unshaded reeion)
- -I [P(inside smal]er circle) + P(inside sector
AOB)l
15

Area of circle = r(20):


131 = ,100t cm':
t96
l^
Area of semicircle = - x tl8)"
39. (a) r + l0'+ 190'+r + 90' = 360'(1s ar a poino
1+ 290P = 360" = 32n cm'
2.t = 10'
Area of semicnch
la) P(insiJe senilircle)=

(b)
Aed or;aor rep€sc.rine snrall size $oy'
-- AEaolcircle 2

= 360.
(b) P(shaded region)= | - P(unshaded resio!)
35"
2

(li) P(medium size)


42. (a) P(insjde rcsion,4)
Aiea of s4tor represenring medtum size
ArcaofrcgonA
Area of cncb _
- Area of ldgen cncb
9CP

360' ,(2)'
,nl8r

I
16

r"*"."0....r"
@
= Area of ldgest chcle - Area of middle cilcle are ofmid,Cle squaE Area or $mlest squm
_
-
= d4)' AEa of Larsest squde
'/.8)'
P(inside region O
-:-
Area olFgion C
- Area of hgesr circlc

3 8

(.) P(iDside region A or B)


= I - P{not iNide roCion ,4 nor a) 1 P(not inside reSior O
=
=1 P(inside resion O = I tP(inside recion A) + P(inside resion n)l
3
=t_i
I I

16
43. Let the tength of AP = r mits
.. L€ng1h ofAD = 2r uits At Q a

a5. (a) P(a ball labelled Z


"1;ffi"
P(inside AAPo)
Area ol AAPO
= 1 - P(a ball not labelled Z
= I tP(a ball labelled 11+ P(a ball labelled l)l
. (1 rl
-' 15 rll
2
,:.- 2

l
I (b) Let the total no. of baUs in lhe bag be i.

(a) P(inside resion A)


r=86x3
Area ol snall6t squde
_ = 25E
Ar€a of ldgest sqnde

.. There are 258 bals in the bag.

a6. (a) P(a brown ball)=

I = n*t

I
rbr Pra brown ball) =
=
x t.r'
)A+] / 19
19r = 9(20 +a)

lox = 180

Answds with Complete wort* ,t*"* @


47. (a) P(a blue narble) Total no. of crayols left = (32 !) + (26 x)
=1 P(!ot blue) =58-2r
=I - tP(a red mdble) + P(a sreen frarble,
ri . r) P(a brown cnyon) =
3
t
No. ofbrcw. dayoN left 3
_
30 Tolal no. ofcrayos left - ?

l9 26 tr.r3
53 - r.xt
7(26 =3(58 ?jt)
-
182 ')
7* = I'74 - 6.
t9 -r =-8
-;.frx12O ..x =E
-76
No. of balls in bas after 6 red balls are added
7
-50+6
(b) P(a blue marble) = j?
No. ofblue nrbles 1
Toral no. of Mbles 18
'16
- ,\,. 7
Do ls
18(76
-,n= 7(120 -')
-r) P(a red ba!) = t
5

1368-18,=840-h
528 = 111
7(56 -r)= s(56)
''-48 392-7r=zaO
112 = ]x
1
48. (a) P(a boy) = i
No.olboys _ 3
Tolal no. ofchilde. ,l 51. (a) (i) P(a sid) = ;
n \y'3
,J *a'i (ii) No. ofboys = 30 -r
27(1) = 3(27 +t)
108=81+3a
3x =27

(b)
l
P(a grr , =-
(b) P(a sirl) = ; =
30.8. r4 r3
No. ofeirls , 2
Toral no. oichildren - ll
5' *n
13(r+ 8) = 3(52)
,?*'ttxu1n l3r+104=156
=2(21+t) 13x = 52
1,1,x =54 + Zr
9x =54
x =6
s2. G) (i) P(a red bal) = 12
49. No. of purple crayons left = 32 a
2
No. of trom dayons left - 28 -(r+ 2)
=28 a 2
=26-r

r"**o*'",. r"
@
(i i) P(a blue bJU)= !\2
l
3

(iii) (a) Plvrnilla ilavoured) = -:-


P(a gMn ball) =
it
=0 l3
30

1
(b) (i) P(a red ball) = ll
(b) P(white wralpes) = a-
(ii) P(a blue bax) =
_tl
30
7

(c) P(stawbenf flavoured md has red sappers)


(iiir Pr! cd u! blue ball) : - 12
30
2
53. (a) ,(S) = 999 - 0 + I= 1000 5
P(3 digits are the sue)
l0 (d) P(vdina fiavoured and has sreen wrappe*)
t000
I 30
100 =0

(b) P(tast disit 8) =


# 55. (.) P(o red slot)= .:3
33
I 9
l9

(b) P(0 or 00) = L


38
I
t9

(c) Plx no. aom I to 16)= --


8

P(nol a no- ftom I to 16)


= I P(a !o. from 1 to 16)

tl
=1,
(c) P(ldt digit not 8) = 1 P(last digit 8)

=l- lo (d) P(an odd nubeO =


*
t0 t9

Adswes with complete worked sorltios


@
56. Beibre: Total no. of cubes now = 18 + 7 = 25
5
ri) Prr black cubel =
-

7
riir Pra crcen cube) = -ii
Total no. of red and blne balls
58.
=18+22

(a) (i) P(a red ball)


q I
= (r) P(io p€ts) = 20
: 2
m
5

(Ljl P(a \td on ball) = f


40 (b)
7
3+2+l
20 (c) P(2 Des or more) =- -
6
=20
3

59. Toial no. of households


= 32+?4+ 8 +9+ 6+ I
=80
Total no. of red dd bluo balls left

rar Pl2 ootted Dlets) =


=
I
'9(i) with sta) =
=10
P(a rcd ball a
]'
3 6+l
rbr P(> 3 Do ed olanlsl = :-
7
4 =80
(ii) P(a blue ball witb a sttr) =
:'
32
5'1 Total no. of cubes = 5 + 13 = 18 (c) P(no potted plmts) = i0
5
(a) (i) P(a black cube) = is 2

13
rir Pra white cube) = = 32+U
0 (d) P(l poned plmt or t€ss) =
(iii) P(a rcd cube) = it 56
=0 80
l8 7
(iv) P(either a black or white cube) = rs
l0

Math"-'tics ruto. 28
@
(e) P(> 5 potted tlanls) =
q (b) Let the no. of grccn marbles remoled by ).
P(a Ereen mrnlel=;
0
519 r ,16
r

60. (a) (i) P(a2)= nn


l0 -;.-l - + Cros$mutrbjv.
:-
57(529 l) = 16(1718 - ))
= 0.05 30 153 - 5b - 27 488 16)
2665 = 4t)
(ii)
58 l-65
P(a a) = ,00
- n.29 .. 65 green marbles nust be removed.

Y€s, the die is biased. The probability of getting r


4 is a few dmes that of t}e probabiiily of getljng a Ler rhe probability of eetting a score of 1 be r.
2.

61. (a) P(> 50 ks) = --:.


Pla scorc of 6) = 2r
60 P(a lcore of2) = 4r
I
10
'., +L-\-lrr' l" TOidl probittil'l;6
l0L = | r, eqdal ro J.
36+4 ''
rbr Pl> 45 k! but < 55 k!) = -:- t0

40

(c) P(< 60 kg) = 60 2

2
t0
t. Total no. of marbles in the box now
= 865 + 529 + 324
I
5
= 17r8

(a) Let the no. of whi1e marbles added be r. (c) P(a score < a) - P(a score of 1) + P(a score of2)
+ P(a score of 3)
5
P(a whitc marble) =t 121
=lo*a*io
124 +1r r i
=:- ! =r crcs-ruIriDh. 3

22(324 + x) =5(Ul8 +r)


7128+22a =8590+5r
l7t - 1462
t =86
. - E6 wbite narbles must be added.

A.swes wirh comprere workeu r"*.". (*)


,(5) = 9999 1000+ l-9000 )t- l1'd
\,cJ ofr rcle= ;
The pn\sble l\t r.d lr\t
digi6 (DittereNe = ?) = 616 c#
Arca of shaded region
(2. 0)
(1.l) = ;(Area of ci.cle Arca of squarc)
(3. l)
I
=;(616 392)
{4.2)
(3. 5)
(5. 3)
P(inside shadcd rcgion)
.
;rtei.]!* 1!ea;l u
(,1.6) tt;t r,
(6. 1) _ Arerolshded Fsion
(5. 7)
(7. 5) 56
(6. 8)
(rJ. 6) 1

(1.9) =rl
19.7)
we Jr..s J po. ,b ti.) d cSr. rn r. N|L. fu*ro .

rsr drg,ts rasr drg s


onlcones ol $e sainple $pace.

For each paif, iind the two niddle digits.

Since 2 dign\ are alreldy used in cach paif lbr the tst
and lasl digils. thc middle djgits can be iilled by 8 of
the rmrining digits.

So for e.ch prir, Ihe.e are 8 )< 7 numberr thal can be

a(J) = 9
.. The no. of favourable ontcomes = l 5 x (8 x 7)
=15x56 (a)
= 840
Prsurn = 5)= -'
3.10
riequireLl frohrhilir) = ii66
7

.,(7't 75

\,/"* )
=1 P(srn = even)
-7
Dianeler olcncle = 2 x 14=28ch
Let the len81h of a slde of the sqnare be J cm.
Using PyLhago.ai rheoren. 5

:' = 392 (d) P(sun = prime no.)"


5

= 192 cnl

rur,"'u,'", ,u,o. r,
@
6. (d)

2 5 6 I
I (1,2) (1.3) ( l, -ll (1.5) 1 (t t) ( l, .r)
2 (2, t) (2 ll (2..1) (2. j) t.2.6) 2 t2, r) (2, 2) (2. t) (2.1) tl i) (.2,6)
3 (:1. r) (1.2) (:1..1) (r,5) 3 (l ll
(.1, 1) (,1,l) (1 r,r (.1.2) 3) 5)
(5, 1l (5, l) (i .+) (5, (:,2) (5. 6)
(6.2) ) (6. 4) (6. 1) (6, 2l l) 5)

P(t,\uudnn mbe^r =
j l*",*
'o I fxlou.ihle
ontcones
=:
P(sanrc DUD$cr on both di.e) (e) POne odd ard onc eren)
=; ThercN6fcvnutuhknurlonc\. = 1 tP{both even) + P(both odd)l

I = ij.:l
lb) P(dillircnt numbeB on borh dicc)
=r l
= I P(srme rnirbcf otr both dicel I
I 2
=l 6
5
6

2, 5
Ir ) (r ) (t, -l) (t,5)
= I5.5th |osition
2 {2. r) (2, ) t2,5)
3 (1 l) (1,2) (3. 3) 5l (l. a)
N{edian = Med oi l5th and l6th values

(,1. l) (4.5)
2
5 (5, l) (5,2) 15. tl (5,.r) i5. a) (5.61 = 73 nnn
6 (6, t) (6.ll (a i) Shorresl tiDe k&cn = 5.2 min

P(L\ '.'cn nu nli!Ar = a


I .. 6 chrldren t@k at l..st Jmh to iix Lhe puzlc
Fon thc dor diaeftm, 6 childrer look aL lclst 9.l

\nsl.r\ lvitl Co.rflclc W.&t *t,.* @


(a) Medn=130 x 2l + 32+(34 x 2)+37 +le + (c) No. of papdyas $ith nass < 375 g = 12
2.
(.10 r 3)+42+('15 x 2) +47 + (49 x 2)
Relulrea frrction = =
+(52x3)+551 -20 *! ]
20
= $42.20
(d) # ).36=e
Minimum nass of top 9 PaPaYd\ = 395 g
Mrddtc P'^irion =
5l + 110+ 116+431 + t+222its
= l0
- 5th Position 5. (a)
21
Medlan = Mcan of l0Lh and I Ift values

=2
40+42

= $41
Mode = 940 and $s2
21
= 53.5 years (3 sig. Iig.)

Mrddle Do)iLion= :j
W
= 14ih posiro!

3 00 _1 1.1
3 79
00 02 No. of people who arc > 65 YcaB = 5
55 199
5 22 2 RequLtd p{renra e= - lu+ir
5 5
= 18.59. (3 sis, fig)

(c) ABe ol youngest cohpetitof - 2E vea6


Age ofoLdcsL comPetitor = 78 Yea.s

(d) The distribution varies lion 28 veaa to 78 vea$'


: The dataclustefaroud the stem eqdal io 50 years
.4.{.4.+...
,r:
iq so gr tu 8r 84 8j 8b 37 8R 80
Nunber olPeaDuE
(e) P(< 50 Yeas old) = ; 3

Modal lunber of Peduis = 85 6. (a) No. of nen = 20


No of women = 20
No. of packets conlaining < 82 Fanuts = 4
Modal heieht = 165 cm
Rcqrired naction =1
Ilcighi of $lLest enployee = 183 cm
6 Height of sholten emPloYee = 1'18 cm

=: No. of nrle chployees above 150 cm bul tess


r,lr il PrconuLn\ 85 l.einur.)
I No. of fenalc enployees abote i50 cm but Le$
1
6+3+2+l+2 Required ratio = 10 : 12
(ii) P(.ontains > E5 Pearuts) = 11 =5r6
24 12 Middle Position

6Fb o-4'-r
4 ldr \u.ol f'oa)a. 4 7. 2500 , 2500 . 2500 , 2800 . 3600 5400 8000
tu *o*
(a)

(i) Median = $2800


hlY**'
\b/ \4d.o lsnr'r paPdr-=JsoI ercl (ii) Mode = $2s00
N4r ' I hed.rr. pdp ). - 40e s :ltm

@ r"*..'. ',* 'u


$25ir0 rI + $?800+ $t600 + $5400 + $m0
q. ,o, N'lode = 88. Med dn b6. \4ern = 85 ,u,\enr
(iii) Mean =
Median = 86
$27 300
I
1 r, ), 88 88
= $3900

(b) Tolal sala.y of 8 employees = 8 $1600 r


= $28 800
Medid = 86
Salary ofnew enplolee = $28 800 s27 300
= $1500
=86

8 (a) ")'+88=172
Mean
14+ 12+ 16+ 15+ 1I + 17+J+t

- ir5
3
j+)+85=l12 J+260 ^_
r+l-21
i:)=4 5 (given)
r+260=340
r=80
r=1
y5 .. The four numbeA are 80, 8,1, 88 and 88.

'= ;) Mean of smallesl and largest number


80+88
x+J=27 2
\=:l-l2l =E4
j -(|)
substirute (2) into (l)i (b) Midde position

':!+!=21 2. 9. 10. 15. 18. 22. 25, 26, 28, j


t3+22
) -r =)1 2
)=15
Substiiute y = l5 inlo (l): Mean = Median (civen)
1+ 15 =27 2+9+ l0+ 15 +18+22+25+26+28+r

... : = 12 andr'= t5 155+r


to -_'
(b) 14, 12, 16, 15. ll, r7, 12, 15 . ,T;ifu&rxll: 155+t=200
Middle posinon
I
11, 12, r2. 14. 15. 15, 16, 17 ' ;;*'slli
,*t#',ii'
10. (a) Mode = 1 day

Medirn = -_ ,5#,f,: (b) Mrddle DorLron =


2

= 14.5 - losinon
= 20.5th
(c) Mode = 12 and 15 Median = Mean of 20th ud 21si vahes

=lday

Answers wirh complete workeo *t,.* @r)


tb)
0t9+l!12+l:4+lrt+4r5+5:2+5xl

=lt
= 1.925 days

11. (a) No. of studcnts 1. ,1, 2, .2, 3.....3, r1 ....4, 5,....5. 6....,6
l_aJ\Y ]\r ]\] ]
=3+ l0+.r+6+8+9+,1+ I
=45
59r856
5+9+J=7+5+6
(b) (l) Mode = 44 sceds

lii) Middle Dosinon= 1 Largesl poslble va['e of r =4


= 23rd posilion
Median = 46 seeds
5+9+r+7=5+6
r = 10 (Imposslble)
(iii) Mean .-. SmallesL poNible value ofr =0
ri r i + 11 r 10 I 15 r 4 I ri r 6 + l1 r ! I rl r 9 I 19 x,r I i0 r l
.. Posible lalues ofr = 0, l, 2,3,4
2019
Mean = 3.3:1
I i 5 + ? r!r ir r 4 i 8 | 5 x i + 6 r i
3
j.9+!r815ri
12. (a) Ifthe node is 1, r < 9. lr+ t6 .i,
-. Lnrge\t possible value ofx = !i r+33
3J+il6=3.34(r+33)
lr+ 116 = 1.34x+ 110.22
when thc imdiah is ngq : 5.78 = 0.34r

0, ...,0, l. ..., l, 2, ...,2. 3, ...,3 14. (a) Total number of rolh = 100
16+19+'+)+16+17=100
896r i+r+68=100
8+8=6+t r +) = 12 (Sho\vn)
Mcan = 1.5
(c) Mea. = I i 16r :{ l9r lirrlrr+5r 16+6r 11

0r8+r:9+2r6+lrr
3r+4y+236 ^-
100

r+21 3J+:h,+236=350
3r+21 =j+23 3r + 4J = ll4 {Shorn)

(c) r+l=32 (1)


= _ 3a + 4_! 114 (2)
(l)xl: 3a+3,=96 (3)
(2) (3): ,=18
Substitulcr = l8 i,rro (l)l
i+18=:12

r.,n".",n, ,*-
@ 'u
Nd, on.tie 3 4 5

t6 l9 r6 t7

Mats (ksl F.rcqqan y f)


Middlc Do\itn,n
'2 = -- 42 .. 11
= 50.5th position
.15 :19 752
Mcdian= Mean of50ft ard 5lst lalues
50-54 52 18 916
2
55 64 59.5 t2 ll4
65 69 61 201
15. (,1) Totalno. of people = 150
25+r+30+1E+r+.r=150 t/=50 :li = 2645
J+)+77 = 150
f,+) =73
Mcan
26.15
I t:5 +l :i+ I ! 30 +.1r 18 +5 xr+ 6!,1
li0
= 52.9 kg
2r+5Y+2!l ^-
150 No. of students who weighs > 50 kg
]I+5r +2ll = 390 =18+12+l
2r+5l=179
(l)
'+]-71179
zr + 5r = (2) Rcau.eL-l fttucnt.rre = .- 100?

(l) x 2: 2r + 2r'= 1.16 (l) = 66s'


(2) (3): lr'= ll
- lr. (a) Modalclass=55<r<60
S$sLituLe r = l1 into (1):
r+11 =73
o lr.q*'.1 y1l h
:10</<45 255

No, of ryattheS I 6 11.5 9

a\.l*!"*t- 25 6l l0 l8 l1 I 50<r<55 52.5 1 361.5

(i) Modalro of rvaic}cs l]er person = 2


55<r<60 5?.5 1l
150+I lll
(ii) Middlc position = --- 62.5 2

= 75.5th Position 65<i<70 6r.5 5

Medi.n no ol watches per lerson :l= 10 l4 = rr ll


= Mean of?5th and 76ft lalucs
2+2
2

=T
= 53.625 s

Atrsqc$ $irb complete wofkt *t,.* @


(d) No. ol studenls who tooh l min or less 19. {a) Modalcldss=20<r<30
=6+9+7+ll

Rcqui,ed D(ccnmEe= 100'i


-
= 42.5V.
20<r<30 25 9 225

(i) P(>55t= ll +2+ 5


(e) l0<r.40 35 6 210

40<r<45 5 2r2.5
'10
45<r<50 47.5 190
2tJ
50<r<60 55 330

:l= 10 rl:, = 167 s


l5
Mee warnne nme=
3 =
8
= llltl
30
18. (a) - J8.9 nin (3 sic. fic)
20. (a)

155 - 159 +{' 157 5 ]IJ5

160 164
#,, 162 12 1944

165 169
lnf 10 1670
41
tlt)- ]4 #,, t12 t2 2064

115 119 111 111

:l=40 xJI = 664(

(b)
t/J - l815

66.t0
=qo McaD =
=
t3t5
=166h =10
Modal ch$ = 160 - 164 and l?0 - 174 = $45.38 (coret to nearest cenl)

40+1 (b) Modal amount of money spe!1 = $(60 - 79)


Middle posilion = ---
(c) No. of shoppcrs who spent at least S60
= 20.5th losilion
=12+3
Median = Mern of 20Ih ed 2lsl values

Clas inlenal where median lies = 165 - 16t Reouircd oercentase = l:


'40 )< too'/"

= f,7.sEa
5
(e) P(litbspan < 160 b) =
21. ToLal no. of bdlls - 6 + 12 = 18

l ldl Pld srern b^ll) = :l3


8 -
2
3

r,,*..,".
@ ^.,,u
?4. S={1,3.4.5,7,9}
(b) Pla Lcd ballr = : 4(,t) = 6

=13 (a) P(a 5) = l


Ltr Pld blue brll) =:
;
I
(d) ! rcd uL !reen ball)
;
P(enher =-
=l i
22. r=11.2,3,4.5.6l (d) PGno.>3)=1
(a) P(a 3) = -
(b) P(not a 3) = 1 P(a 3)
,l
6
5
6

L.r P\dn.Jo no.l : n Favo-umble ou(dm6

25. (a) J= 1 0.28 0.54 r


2
= o.lE
(d) P(a factor of 12) =
i (b) No. ol cas $xt tDrned lefi
= 0,l lJ x 450
(e) P(a no. < l) = =81
;
=0 26. (a) S = { IIH, HT, TTI, TT }
(il P(a naruLal no ) = :
(b) r, r Prt$o heads) =1

23 12
riir Pro hcad ard a railr =
;
'(s) = I
2
2

(iii) P{at leasr one lail) = l

(b) P(leter I or M) =
3 21. (a) S= { HIIH,IIH! HTH, TIIH, HTT, TH'I', TTH.
t2 TTT }
I
1 (b) (i) P(tbJee headsl =l
'R
rir Pl2 heads dd I LdrlJ = i
3
I
liii) P('o heads) =1
1r\I PrJL ea\t 2 headsl =

2
2
l

Answen ivith comprete worked sohti.N (e


2E. n(s)=59 30+l=30 lr. (J) ,S = {Monday, Tucsday, W€dnesday, Thursday,
Friday, Saturday, Sud.y)
(a) P(a no <:12)=:n
G = { SatDrday, Sunday }
=?5
uo = 49)
2

=Ll5
rc) P(hrs rhe diln
" lr = : (d) (i) P(lettef,) =
:10 ; "
0
=15 (ii) P(lettef l) =t

29 Total no. ofbaus = 6 + 8 + 10 = 2.r


(iii) P(contlins 6 lcttcN) = ; . l
(a) P(a red ball) =a
1

(b) P(not r.ed ban) =I- P(a rcd bdll)

,(5) = 52
.1

3
4
(a) P(a iack) =1

(cl =1l3
P{eirher a '24 ball) = !-i-.19
nd or vellow
(b) P(nor aiack) = I - P{ajr.t)
24
,t
2
I3
3
=!!l3
ldr Pla blue bllLr = --
=0 =I
(c) P(! 10 of bearts)'s2

?={ 1,0, 1. 1.5,2l (d) Pla black card) = :9


,(?) = 5 i2
=12
(cl H('<ul= . -.i ll
(.) P(cirhcr rn rcc or d DilLu ! !J d)= :
P(i=r)=? 4
l3

ll. (a) P(a queen of hedts) = 1


'52
I
13

r.^".",", r** ,"


@
'52

P(not a red ace) = I P(a red ace)


I
=l
25
26

_ ^._ J

(f) P(either a 2 of spades or an ace oI hearts)


2
52

=l26
34. ,(S)=s0 21+1=30
35. r(l') = 9
(aJ P(d elen no.)= l: (a) l
P(a 5) =
=12 =! 3
.1
(o) P(6 Pnne no.r= !
-+ (b) P(a naturar no.) = 9

(c) P(a composite no.)- I - P(not a composite no.) (c)


=I- P(a prime no.)
P(a Dnme no )
'9 = -
=t 30
=13
7
30
(d) P(an inreserl = ;
(d) '
P(a Dedecr
"quder =
a (e) P(not an inteser ) = 1 P(an integer)
:10
'1

=l 9

,e' P,nor a perle.r squml = | P(a perfecr .out) =?


=t-,1l0
3

lfl P(a nuLtibie of6l = :

=t
G) P(a perfect cube > 30) = ;

Answes with comprere worked sorutiom (A


36. (a) P(a sold cojn) = 1 P(notasoldcoin) 38. Arca of ldgef square - (sr)'
= -1 P(a copper coin) = 25r' cm'
=l-0.65 Area of smaller squde = (r)?
=03s
(b) P(lose) =I- P(not lose) Area of shaded reSion
-l-LP(sln)+P(draw)l = Area of la€ef squa.e Arcd ot smaller squde
. 15 lt
18 5.1
.33
P(inside shaded rcgion) =
7

*}
17. (a)

90' 39. Area oflarycst circle = r(20):


360" = '100n cm'
Area of shaded regior
= Area of ldCest circle Area of two snaller circles
=4AOn ft(.6:)1 n(.2)1

125. A€a ol shaded Esion


P(inside shaded resion) =
160'

12 NM
9
Argle represeniing yellow sectof
10
= 360" - 90" - 125' 45' (Zs at a loint)
= 100" fttt
40. (a) Total no. ofballs at = 36 + l9 = 55
P(inside yellow sectot P(a sreen ball) =1
_ Afta ol yellow se.lor

100' Hxj.-ii1r$iidffiffi
360' 4(19 I) = 1(55 -r)
5 76 '1J=5s r
18

P(inside green or yellow sectot


A€aoleen a.d yeuow sectob (b) Total no. ol paper clips in the bag at lirst
A€a ofctrcle =49+14=123
45. + r00" Total no. of paper clips in the bag afte.
360' =123+j+(r+6)
= 1,29 + ?-a
360'
29
No. of blue pape! clips after = 49 + r
72
Pla blue naper clip) = l
P(insjde secLoa cont ining colous ofthe rairbow)
360" rrq * 2. --i
160" 5(49+r)=2(129+2r)
245+5J=258+4r

@ r'*'*.. ",.,
,*
41. (a) Tokl no. of balls = 12 + 9 = 21
(d) P(dot morc tho l senior citizen)=
12'll
I2

80
7
l0
(ii) P(not a gold ball) = I P(a gold ball)

43. Total no. of studenLs


1
3 =12+36+,1+6+2
=60

rru I PLneiLher sold nor alver) = 1


=0
12

(b) Tolal no. ofeold balls now = 12 2 = 10


Total no. of silverbdlh now = 1 = 8 9
ToLl no. of gold and silver brlls low = 10+8= lE

lr) Pta snld ball) = - (b) P(not mofe the $60)


5 = 1 P(more than $60)
2

aiil Pl, silverball)' = : 29


13 30

=l
Lrr PLenhcr a eold or ahcr ballr = 11

42. (r) P(4 senior c,tizens)= a


80
3 L r+r= _' -

(br Prdt leasr 2 \cnior citia

!1
3
t0 2.A

(c) P(less ihan 3 senior citizens)


32+24+3
=

a
t
A.1,Rc = AEDC (Given)
_.. EC=Ar=8cs * -,.
...8t=9+8=17ctu

Anssrb \irh compete wurked sorur.^ (e


Using PlahaSoras' theorem on 4-448, A!!s4l M4!
AB'+a'=n' 2km+ Lcm
{& 225 , 1kn+ lcm=0.5cm
= iror" oostit. *ooo 7
4a = J225 rcor ol br! sl9::. (l kn)' + (0.s cb)'

15kn' + 0.25 cm' x 15 = 3.75 cn'


ED =AB +
=15m .. The area of lhe reservoir on fte Ddp js

r (J) l=i!r l AB'


Whenr=9.r=2.5. AR

z.: = t.i! 6+9


2.5 = 3t 6
t5
3
25
30

=:
AC+6.3 ^-

, 5- AC + 6.3 = 2.5AC
6.3 = 1.5,4C

13; = -ir
t3: 6
I:- 3 1. l") P(a red Dnrurc c{d) =
-
_; 3
26
r=256{
P(ar .cc or d 10) =
-
Modal ndk = 25 and 32 " 2
l3
Highest nark = 39
(c) P(not a dianond) = I P(a dimond)
(c)
who failed (< 25 mdkt =l:
=io
r4
10 l
30 I
=l ? id:'l'Y':':::-
3
" "; d:i lr$ ?.
5. (a) Map Ae!!d 8 \d 2u -1, .\.' lr.
1 cm + 200 000 cm =2(r+2){]-2)-,(r+2)
i.e. 1cm + 2hn =(r+2) t2(i - 2) - rl "' -
6..1cm + 2 tm x 6.4 = 12.8 kh =(i+2xzr a r)
.,The actual dist nce between the two fire stations
=(r+2Xr-a)
is 12.8 km.

r*n.-'n" ,*- ,u
@
(b) 0 3)O 5) () .r)r
+
=)' 5)+3i 15 O' 8)+16) (b) (; 2)' =
t
=)r ?) 15 rr+EI 16 r ^ ll
=6J-ll
(c) 6:' 1=ll:+9 l^1
6:' I 11. 9=0
6.' 11: 10 = 0 t^l
(3.+2)(2: 5)=0 r -L
3

." 3:+2=0 or 2.-5=0 I


3
\'1
2
r3
3
11
9. (a) Volume olsolld
= Vol ne of cylinder + Vohne of cone

=tt(4)in)+:(4t(h)
= l28z + 32n
= l607t
= 160(3.142) l(2r 1) 4(J+t)+12=0
= 502.12 anj 6t 3 4x 4+ l2=O
2,t+5=0
.-. The vohme of the solid is 502.72 cmr.

(b) Volune of hemisphere = VolunB of soiid .bove


2
1o; = rcao
11. (a) Total files boughr = l7
r =t6A/ - x+J=r/
= 240
,
Tat<e cule root On
(b)
/= 1t40 . bolhides Total cost of files = $59
2(r)+7(r')=59
= 6.2t cn (3 sig. lig.)
u+7J=59
.. Thc radius ofdre hcmislbc.c is 6.21 cn.
r+)- l7 (l)
2,+7J"_59_(2)
(1) x 2: -
2r + 2) = 3,1 (3)
(2) (3): 5)=25
)=5
SubstitureI=5iDto(l)
r+5=U

...r=12.nd)=5
.. FIe bought 5 ring fil€s.

12. (,r) Sun ol inlerior angles in a pentagon


10. (a) 3r1|+ l) 2121 7)=5(r- ll) =(5_2)xt80o
6r+.1 .+r+14=5r-55 =l x 1800
2I+ 17=5r-55
r=24 .- 68' + 84' + 122" + sr + 9r = 5.10'
I4r + 274'= 540'
l4r= 266'
r=19'

Ansse$ *irh comprere wo,ked s"r"d"*


@
(b) Le1 n be the no. of sides of fte reguld polygor'
Size ofeach inL. Z = 5 (Size of each ext. z)
,, -l-lr 10=0
,,
, ^'o, ='t;l
f,on'1
-
2Xr 5)=0
(J +

(, 2)180'= 1800' .' -.r+2=0 or r-5=0


l80r 360 = 1800 r= 2 or r=5
1E0, = 2160
. A = (-2, 0) and, = (s, 0)
2 t60

.. ) = o'- 3(o) 10
= -10
.. The polygon hrs 12 sid€s.
' c=(0,_r0)

\4 dac o. ,.ion 'A"""gd ii


r 1. r. 5.
I
). r,. 8. t0 t. t. lt. ).
4*.":1loq
.oro(nrq Substiluter= l.5lnto)=l 3a 10,
y= 1.5' 3(1.5) 10

lcr N.rure = l1 . n"^Un*r rn"r


the gralh (minimun pom,
...*i -.^.'r*t
.. The lowesi point o!
.: onei.
' : (r.5, -r2,2s).
'

rbr Middle po{rion =


-i
= 6.5th position

Median = Mean of 6th tnd 7th values


w
Section A
=2 l. (a) l5pr 44] - 54' + 12Pl *
-
= l5px 5st + l2pr -4.t

l.' .- .-r
\|edn _
:i'
'-
8- t' | !' -'i,"-;i::!,1'-^
I-

@ *,,*.,*,..,'u
(r) tolal surlacc a.ea oiptrdfrid
= Area of square AACD + :l x Area of A4llt
-t
=14'+1ttt14t25
= 196 + 700
G);+r)=34_lcven

5(r \'t =5(; ae+)') (b) Ler Ex be rhe heiSlit of tho


)')
= 5l(a- + 2lrl
= 5L3a 2(7.5)l - xF= 14 - 2=l cm
= il14 15)
-
Using Pythagoras theorem
= 5rlqr Snbsdrure
=95 l+y'=34
..5(x-r,l=95 dd4=75

DX = ^.[sj6 R
=24cn
Volume of pyfamid

=: Brs ar.-d Height

=jxl41 x24
Arca of rtaded region
cn + AJea of
= 1568 cm'
= Arca of setuicircle of radi6 28
cncle of radlus 7 cm. .. .d, \,, ume of08 rrcr'l h" l.
Reddiu. JlbJl
=!x?xx.+?!:,1
27 1 =o5 I = o1" =l2cn:2
= 1232 + 154 s6 04 cm =6cm
= 1lE6 cm'
1.6m= 1.6 x 100= 160cm? I m= 100.n
Lcnct} of nbbon used 0.E m = 0.8 x 100 = E0 ch
= Pe.imeter of cardboa.d Let thc rise in the water lcvcl be , cn.
rirunt.Hel /ctu'mkEn.tl fc'trumhEnql .- 160x80xh=lilJ70,1
= oi,r.|r.] + of(ituL i +-I ofrirde^l
liJLrrr.m I \todrrr{nr l!!i!\rfi,i l , 3U 704

=t2 1 1)+(2 = r,1)+ = 6.93 cm

-. The risc in Lhe {aler level is 6.93 cm.


1'"

AnsweA with Complete Workerl Sol,il.."


@
(i) P^Q=P
hlP^O=n(P)
=54

(ii) PUQ=Q *
n(PrO=h(O)
=89

(b) Mass = Densily x Volume


= 1.25 g/cmr x 88 704 cml
= 0 880I 6. (a) )=;-zr-l
1r0 880 . a=2'-212)-t=-1
= 110.88 kg
= rn kc (corrcct to nenrcst kc)

Section B

5. (a) 3

2, 3.4,5,6,7,8.9, 10. 11. 12, 13, 14 I


2, 3, 5, 7, 11. t3 )
3.5,7,9, 11. 13 ) (i) whenr=3.4.J=3.7
5.r0) (il) when r = 0.x = -O.4 orr = 2.4.
A' . c= { 4.6,8.9, 10, 12, 14 I . { 5. l0 } (iii) the equation of the line of qannelry is r = r
={10}
(ii) auc={3,s,7,9. ll, 13}u{5. l0} 7. (a) Using Pythagom' tbeorcm,
= { 3. 5,7, 9. r0. 1r, 13 J
AC=AB'+Be
n(B e C) =7 (3r i)? = (r + 2): + (zr + 3)'
(iiD a, = { 2,4, 6, 8, 10, 12, 14 I 9t' 6x + I = I + 4r + 4 + 4l + 12x + 9
a' = { r : J is an even nunber } 9rr 6j+l=51+16r+13
41 22x - \2 =0
(c) Pc O (P is a proper subset of 0) 2'1-llx-6=0(Shom)
o (b)
(2r+ l)(r-6)=0
..2r+ 1=0 or

I
2

r,*".*", r*- ,"


@
,. l
(cr r--tc'r
lcDgth is posi6'e)

lB=\+2 l. (a) l&:50Jr


= 2 o. '... , ir.. _r_ k
=.2lrr,r ,J,lt . i'-l- I
=ta'r ot\4
:
D'
BC=2x+3 - 2i.]x + s]^lx - sr,
=2(6)+3 (b) )2p' 24p t5
= l(4pr 8p 5) -
=3(k + Dl2p 5)
2

2
x8x15
60

8. (,)
=(l rr(, \r " i;iit o,n:*'".""
Iacqlr: Ir,
,. ': ,,
';,d
"bt,a. bt.
.r:::':- _ '-._ ':
2. (a) (2rr' + lxlrl'' - xu + r)
8 <r< 12 '::.-'"-,,"
12<r< i6
= 6ay 4ir + ar"- + 3i. 2a) + I
16<i<20 = or'y' - r'1' + r
20<r<24 (b) lE5 )< 615 + 385r
= 385 x 615 + 385 ! 385
xJ1 = 1352 = 385(615 + 385)
= 385(1000)
Mcrn knsLh ui .e,\rcc = = 385 000
=
tt52
t50
= 9.01 years (3 sig. ffc,
(b) Modalclas=E<s<12
kllh s) - 2h + 1 4 l.i.1ir6pry uotlr siaes
28+9
oy 4-r.
(c) (i) P{> 12 years)
150

l!
150

L
3t+2r=13 _ (r)
1t+6t=41
ll+16 (1)x3:9r+6j.=39 (l)
(3) (2): 2\ =:
67
150
-

\n.sc . $ith (omplee Worke,l Sol"',"'


@
Subsriturer = I into (l):
(b)
25
3(-l) + 2)'= 13
3+2)=13 2(r + 1) 5(r 3)
2t =16
I =8
.. the sohLion is r = -l and, = 8.
= (,_ 3)c. . r)

= (r 3)G +r)
2,
1.
,10 km (20 tn)
80 knl4r | 60 knJh
1i h) | rumri=dn=jh
Distance travelled in 2d part ofjoudey
3.
I

=20km
Total distance travellod = 40 + 20
=60kn
Ditrerence in the values ofr = 5 (Given)

L t_. (b) Tine taten for lst pan of journeY

=
.ft rl - so krttt

5,
_t"
-

v
5
rol,r rme rdr.en= t h + t h+ i h= r; h

-36 Averaee spe.d fo. elrole joumeY


Total dislan.e ravelled
= Tr"l rl.. t"k-
'-. 36

-T 3
3'

l-
36

6. (^) 2,\5, + t) =25x- \2 8. (a) A,Atr is similar 10 A, BC.


l]i +2r =251 12 AB EC
loi-23r+12=0 AE EF
(5r - 4xzr 3) = 0 BE+3 t64
.- 5r-4=0 q L\-3=o
5x=4 ot U=3 BE+3=413)
4 -r BE+3=12
'= 't

Madtmtis rDlor ,B
@
Ib) LAEF = LABC =90'
I-inedscale-.[# .[r.; - squ*,-.
= lcn:2tn
= lcm:200000cm.
Usnrg Plthagoras Lheoren on 44l!. = I :200 000

I linl = 100 000 cm


AF = ..tE
.. Thc R.F. or the ma! is

(c) ACFG is sifrilai to Aal.lD.

63
Ikm.10m=1 1L=1.04k]n
3 Aelldl Map
2km-- lcn
9. (a) volume of pyrmid lkn- lcm=o.5cm
1.04 kr 0.5cD). 1.04=052cm
=1^Baseaea^ugrtrt
l_
.. The distance between the rees on the ftap is
=lx5.xr: 0.52 cm. -
(c) Mip Aead
I cm' * ,1 krr'
0.025 cm' + ,1kDi': x 0.025 = 0.1 kml

.. The acLoal dea of the llly pond h 0.1 km'.

u. (a) If ihe Nodll mdk is 6, then r > 13.


Smallcst possiblc value ofx = 1;l '
:.Mi4q!.4ar*$ ru:iii!.*
r.t!at o4dli mdsf tr.hed.:i:

The sJna bst vaiue The legdt lalqe ot


Pyramid d oI r o(Lus when r occuu whetr rhe
FX=10+2=5cm +i
Using Pythagoras Lheorem on AY!,I, (b) <3..1....,4,5, 5.5, 6-...-6,7,...,7, E. E
vF - 5'+ 12' 0 ll I r 5 2
= 169

0+ l3=2+r+5+2

Totals ace area of Pyramid B .. Snallen posible vrlue of r = 4


= AJea of square,44CD + 4 x Arer of l\v.13
0+13+2=r+5+2
=10'+4x:xl0xl:l 15=J+7
= 360 cn'
.. Ldgest possible value ofr = 8
0. '", nrcr. " '<.m :i \m . Divide sch
- Lm.atrn, Limb) r'i .. Possible vdlfts ofa =,1,5, 6,7,8

{n.ucb {,rh compreLe worr.d s.r"t.','


@
t2 ,a, N40de rrq.m
. t6 ||
- ::o(uB
":
toasr ottt!,
':. .
rE
rhai
!gicl,,
.
W
Se.tion A
- 6 )In Poq rr_ le htrrle
l. (a) (l 2r)(2r + l)(#+ r)
"-;q.. $
Median = Mean of 8rh and 9Lh values
164 + 165
= (l 2rX1 + zr)(41+
=2 = ttr (xrtt(4,, + r)
= 164.5 o
= 1- 1tu'
29?+618+828+r0l+133
,..
(Dl
t
:;
3
' t0
10(a + 2) 10(r- l) = 3(' l)(' + 2)'
= 164 cn (3 sig.lic.)
ior+ 20 10i + 10 =3(.f + j 2)
l0
(b) No. ofnen who arc shorte. than 157 cm = 5
. ll+lx lb=o
Xili,le!-lEb! -r +1- 2 =o
5
x te{6iby 3r' l, j)rr+4)=0
Requiredpercentage = 16
1007,

= 3l.25Ea ..r-3=0 or 1+4=0


r=l or x=4
13. €= {11. 12. 13, 14, 15. 16. U. 18, 19,20J
P= {ll, 13, 17. 191
o= {11.13, t5. 17. r9l
R= {12, 181

(a) PUR = {11, 13, 17, 191 u {12. 18}


= {11,12,13,17, rE, 19}
(b) P.0 = I12. 14, 15. 16, 18.201 .
{11, 13.15,17, 19}

(c) 0'^,r= {12, 14. 16. 18.201 .


{11. 13, 14, 15, 16, r7, 19. 20}
= {14. 16.20} Area of ciEle of ndius 3 cn
nlQ'^R')=3 = At .l
" "i'"r" "f
t",11.,. 12
-t
Toralno. of balh =9 + 6+ 5 = 20
(a) (i) P(noi a rcd ball) = I P(a red ball)

='t 9

I
=i
(ir) P(either a red or green b.ll) = :rl 2A
(b)P(either rcgion a or q
Area of rcgion B + A€r ofregion c
I Area or cir.l€ ol mdius l2cn
20
Areaof-i -leo rJdu,8.n A-aof.u.eJf,diu.1
Ll0 cnct of radius l2.n
Area oI
'

{b) Total. no ofbalh iefi = 20 4 = 16


6

A4,
3
5rt

55

@ ",,n"*,n,^.,."
(c) Pceeiotr ()
Aredofcncholndir'8cm AFa olmle orradiu' 6 cm

Area of ciroie of radius 12 cm

=@
= ."0" (cl Total surface ffia of 2000 hemspheLes
2V = 2000 x 266.0',72
= 512 144 cm')
512144
7 = loooo
= 53.2144 ff
P(not in r€siotr C) = I - P(esion O
Cost of painting 2000 hemispheres
7
=1 - = 53.2144 X $9.50
= $50554 (correct to the n€arut cent)

""'wo'"' 0 2

I 2
Volume of 50 circuid discs
=50x(rx2'x0.5) 5y=3t+25

5 6.8

Let r cm te the base radius of the hemispt'ele

Volume of hemislhere = Volune of 50 discs

!.a = tow
t^3=1$x t
= 150

' = 5.3i
l'iso
cn (3 sis. fic.)
=

.. The radu! of the hemislhet is 5.31cm

(b) Total sudace area of hemisphere


= 3r(i450).
= 226.072
= 266 crd' (3 slg. fis.)

Answes wirh conplete wo.keu rt"u*, 6-r)


(b) (D The solution isr = 6 and J = 3.

The solution is r = 2.5 and J = 6.5

(b) Lateral surface dea of cylinder

(c) The fisurc fomed it a quadrilatenl.

Section B
(c) Let the ndius of the sphencal sotid be cm
'
5. (a) Volune of sphere = Volume of cylinder

;dr = tt L0 5)'152.5)
r lo.5: x 52.5 x
,=-_ 3

= 4341.09375
- = 1,[3rmr3?5
- 16.3 cm (3 sic. fic.) !,
.. The mdius of lhe spherical solid is 163 cm

(
70 5r5
!
N
.l

t\ Cost lrice of each dictiomry


a
-a-
1
180
[\ ^

s
SellinS price of eeh dictionary
/ t80 ^l ilH!
i=+";W =

(b) Total
-frao*r')
!l--a,=
dounl
l

received
H
.. Bse radius of cylinder = 10.5 cm
=28 281(71
.dlto.5)'(52.5)
!,tGtt'0at t040 + 34r -
-l+(r .^-

5788.125 5040 + 84j+ ?r2 - 196r


4t 160
9
=-[

t",*-"u* r"
@ ^..
Profit = $100 (ii) The polidve solulion (i.e. a = 12) is the
horlzontal dist&ce f|om the.buildng when
U + 5040 the ball hits rbe ground.
.l ''= 100
- 280 a =72 + 614) (=Eo
h'- ll2r + 5040 b =72 + 6(6) 6' =72
7; 392I + 5i4o =0 (c)
t'- 56t + 720

r: -56r + ?20 = 0
(i 20X: 36) = 0

.. He boughl36 dictionaries.

(a)

10 14 u 12 2 24

15 19 17 9 153 (i) the greaiest hoieht rcacbed by the ball is


,7 El m and it occun wben t = 3 m.
20-24 22 154

25 29 2J 5 135 (ii) whcnr = 75, r = 0.55 n orr = 5.45 m.


Horizontal dislance tmvelled
30 3:l 32 I 32
= 5.,15 - 0.55
35 39 31 6 222

r/= 30

t/i
lbJ Mean ume Lalen = :';
120

= 24 min

Middle posnon=

- posidon lalues
= l5-5rh
Median - 15th od l6th
Med of
.. The cla$ in1enal where the median
2i -21.

8. (a) (i) o=72+6x-l


r' 6x-12 =0
(r+6)(i-12)=0
or r-12=0
'+6=0
r=-6 or t=12

Answen with complete work, *t'-* @n)


",i.....:..
)) ],:..j :.,,i. ( l'; I I

ril lJJ lJiril,jil Ii


L, U IIi:
ffiW
SECONDARY TWO MATHEMATICS TUTOR Special/Express 2ts
is the second vdume ofa newiy updated complete study and answerguide to your
mathematics texhook. Based on the new l\,4OE syllabus, it is specially designed to
serve as your home tutor for the leaming of mathematics.

This easy-to-use guide is specially designed wih clear and mndse explanations with
stetrby-slep examples to enable you to masterthe subject. Complete worked
solutions prcvided atthe end of the bookwill be useful for your self-revision and seif-
conection.

i:i.,rrr,, ,i'lir ilrr iitL, I:ilr, iiir: lrjr,jr, i irl i,ri

t
1 3 topical chapiers in 24 and 28

* Targets to help you focus on the relevant mncepts


* Comprchensive Notes and Worked Examples
* Tutorials with a wlde range ofquesuor-rs arranged by toplcs
* Thinking Skills Comer containing questions for you
to apply your problem solvng-skills
* Revlsion Exercises
* lvlid-Year Examination Specimen Papers
* F nal Examjnation Specirnen Papers
* Answer Key with Complete Worked Solutions

SGnoxlcou {664o'rstttttntt* "o'

Website: www.sgbox.com Email: sgbox@sgbox.com


@All Rights Reserved
Code: S2BMT

You might also like